Radiology PDF

Download as pdf or txt
Download as pdf or txt
You are on page 1of 585

Radiology for

Medical Finals
A case-based guide

K30031_Book.indb 1 9/6/17 1:34 PM


K30031_Book.indb 2 9/6/17 1:34 PM
Radiology for
Medical Finals
A case-based guide
Lt Col Edward Sellon
BSc (Hons), MBBS, MRCS, FRCR, PgD (SEM), Dip (ESSR), RAMC
Consultant Musculoskeletal Radiologist
Oxford University Hospitals
Oxford
and
Consultant Military Radiologist
Centre for Defence Radiology
Birmingham, UK

Professor David C Howlett


MBBS, PhD, FAcadMEd, FRCP (London), FRCP (Edinburgh), FRCR
Consultant Radiologist
Eastbourne Hospital
East Sussex Healthcare NHS Trust
Eastbourne
and
Honorary Clinical Professor
Brighton and Sussex Medical School
Brighton, UK

Preparation of the illustrations by:


Mr Nick Taylor
MIMI, RMIP, MRCR(Hon)
Honorary Teaching Fellow, Brighton and Sussex Medical School
and Medical Photographer
East Sussex Healthcare NHS Trust
Eastbourne, UK

K30031_Book.indb 3 9/6/17 1:34 PM


CRC Press
Taylor & Francis Group
6000 Broken Sound Parkway NW, Suite 300
Boca Raton, FL 33487-2742

© 2018 by Taylor & Francis Group, LLC


CRC Press is an imprint of Taylor & Francis Group, an Informa business

No claim to original U.S. Government works


Printed on acid-free paper

International Standard Book Number-13: 978-1-4987-8216-6 (Paperback)

This book contains information obtained from authentic and highly regarded sources. While all reasonable
efforts have been made to publish reliable data and information, neither the author[s] nor the publisher can
accept any legal responsibility or liability for any errors or omissions that may be made. The publishers wish
to make clear that any views or opinions expressed in this book by individual editors, authors or contributors
are personal to them and do not necessarily reflect the views/opinions of the publishers. The information or
guidance contained in this book is intended for use by medical, scientific or health-care professionals and is
provided strictly as a supplement to the medical or other professional’s own judgement, their knowledge of the
patient’s medical history, relevant manufacturer’s instructions and the appropriate best practice guidelines.
Because of the rapid advances in medical science, any information or advice on dosages, procedures or diagnoses
should be independently verified. The reader is strongly urged to consult the relevant national drug formulary
and the drug companies’ and device or material manufacturers’ printed instructions, and their websites, before
administering or utilizing any of the drugs, devices or materials mentioned in this book. This book does not
indicate whether a particular treatment is appropriate or suitable for a particular individual. Ultimately it is the
sole responsibility of the medical professional to make his or her own professional judgements, so as to advise
and treat patients appropriately. The authors and publishers have also attempted to trace the copyright holders
of all material reproduced in this publication and apologize to copyright holders if permission to publish in this
form has not been obtained. If any copyright material has not been acknowledged please write and let us know
so we may rectify in any future reprint.

Except as permitted under U.S. Copyright Law, no part of this book may be reprinted, reproduced, transmitted,
or utilized in any form by any electronic, mechanical, or other means, now known or hereafter invented, includ-
ing photocopying, microfilming, and recording, or in any information storage or retrieval system, w ­ ithout
­w ritten permission from the publishers.

For permission to photocopy or use material electronically from this work, please access www.copyright.com
(http://www.copyright.com/) or contact the Copyright Clearance Center, Inc. (CCC), 222 Rosewood Drive,
Danvers, MA 01923, 978-750-8400. CCC is a not-for-profit organization that provides licenses and registra-
tion for a variety of users. For organizations that have been granted a photocopy license by the CCC, a separate
system of payment has been arranged.

Trademark Notice: Product or corporate names may be trademarks or registered trademarks, and are used
only for identification and explanation without intent to infringe.

Visit the Taylor & Francis Web site at


http://www.taylorandfrancis.com

and the CRC Press Web site at


http://www.crcpress.com

K30031_Book.indb 4 9/6/17 1:34 PM


For Louise and Lottie,
for their constant love, support and belief (ES)

To my dear wife Lara and all the children, Thomas, Ella, Robert and Miles,
also to my parents, Ken and Margaret, and remembering fondly
Joanna and Christopher (DCH)

K30031_Book.indb 5 9/6/17 1:34 PM


K30031_Book.indb 2 9/6/17 1:34 PM
Contents

Foreword by Professor Malcolm Reed ix


Foreword by Dr Giles Maskell xi
Preface xiii
Contributors and acknowledgements xv
Abbreviations xvii

1 Overview of imaging modalities 1


THOMAS KURKA AND DAVID C HOWLETT

2 Hints and tips for finals Objective Structured Clinical Examination 7


THOMAS KURKA

3 The normal chest X-ray 15


THOMAS KURKA

4 The normal abdominal X-ray 35


SEAN MITCHELL

5 Thoracic cases 51
HANNAH ADAMS, SARAH HANCOX, CRISTINA RUSCANU, AND DAVID C HOWLETT

6 Cardiovascular cases 175


HANNAH ADAMS, SARAH HANCOX, CRISTINA RUSCANU, AND DAVID C HOWLETT

7 Abdomen and pelvis cases 205


FAYE CUTHBERT, AMANDA JEWISON, AND OLWEN WESTERLAND
8 Musculoskeletal cases 319
EDWARD SELLON AND ANDREW SNODDON

9 Neurology cases 407


VINCENT HELYAR AND EDWARD SELLON

10 Paediatric cases 461


UDAY MANDALIA AND LUCY SHIMWELL

Bibliography 555
List of cases 557
Index 559

K30031_Book.indb 7 9/6/17 1:34 PM


K30031_Book.indb 2 9/6/17 1:34 PM
Foreword by
Professor Malcolm Reed

From the initial discovery of X-rays and their application to medical imaging by Wilhelm Röntgen,
imaging has been an increasingly vital part of medical practice. The modern doctor needs a strong
understanding of the different modalities and their application in the diagnosis and management
of a wide range of medical conditions. While in many situations images are reported by expert
radiologists, the ability to understand and interpret radiological images is essential and the vast
majority of medical schools will require students to demonstrate fundamental skills in this area.
More importantly, diagnostic and therapeutic imaging opens a window to the internal struc-
ture and function of the human body and links the fundamental sciences of anatomy, physiol-
ogy, and pathology to the patient as a whole presenting with symptoms and signs of disease.
The clues gleaned from a careful history and thorough examination lead us to select the most
appropriate investigations to expedite a diagnosis, allowing us to inform the patient about their
condition and commence appropriate treatment. It is the distinction between normal and abnor-
mal structure and function, which is at the core of radiological diagnosis, that provides an illus-
trative basis for learning and a truly patient-orientated understanding of medical disorders. As
such, the use of radiology in teaching and learning facilitates and enhances the understanding
of medicine and is of enormous benefit in preparing for examinations such as medical school
Finals. This textbook edited by Edward Sellon and David Howlett provides an invaluable learn-
ing resource not just for students preparing for medical school Finals but any doctor preparing
for subsequent professional assessments. In addition to the well-illustrated cases and a use-
ful introduction to OSCE-style exams, the real value in this text is in the clearly structured
cases based on high-quality radiological imaging, which span the whole spectrum of medicine.
The book takes a regional anatomy approach with additional chapters on the normal chest and
abdominal X-rays and paediatric cases.
The contributors and editors are to be commended for producing a high-quality, com-
prehensive compilation of cases with clear and concise questions, answers, and explanatory
notes. I would commend this text book to its target audience of final year medical students but
also to doctors in training in a wide range of clinical disciplines as well as those in established
practice.

Professor Malcolm Reed BMedSci, MBChB, FRCS


Dean, Brighton and Sussex Medical School
Brighton, UK

ix

K30031_Book.indb 9 9/6/17 1:34 PM


K30031_Book.indb 2 9/6/17 1:34 PM
Foreword by
Dr Giles Maskell

Radiology is an unusual medical discipline in being able to trace its origin precisely to a specific
event – the discovery of X-rays by Wilhelm Röntgen in 1895. The practice of medicine was trans-
formed almost overnight by the use of X-rays in diagnosis. The development of further imaging
techniques such as ultrasound, computed tomography (CT) and magnetic resonance imaging
(MRI) followed in the second half of the twentieth century and has led to medical imaging occu-
pying a central place in the management of patients with a very wide range of conditions.
Whatever branch of medicine you pursue as a career, at some stage you will find that an under-
standing of medical images – X-rays and scans – will be essential to your work. You will need to
understand not only the principles of interpretation of tests such as the chest X-ray but also their
strengths and limitations and how to make the best use of these tests to benefit your patients.
Although imaging findings can occasionally be so characteristic that they could almost be
called “pathognomonic”, one of the most important lessons that you will learn is that the inter-
pretation of an imaging test depends critically on the clinical context. The classic diagnostic
sequence – history, examination, tests – is as valid today as it ever has been, despite the increas-
ing sophistication of the imaging tests. The doctor who makes a diagnosis based only on imaging
findings without due regard to the clinical context is more than likely to be tripped up.
Radiology is not a discipline that can be learned in isolation from clinical medicine. In this
book, David Howlett, Edward Sellon, and their colleagues, renowned educators in this field, have
therefore embedded the teaching of radiology in a series of clinical cases, which illustrate not only
the specific imaging findings in certain conditions but, importantly, the principles that underpin
the effective use of imaging tests in clinical practice.
Although there are encouraging signs with the establishment of undergraduate radiology
societies in many medical schools, the teaching of radiology to undergraduates has not always
kept up with the progress in medical imaging. I believe that this book will prove invaluable, not
only in preparing students for medical Finals, but also in giving them a better understanding of
the central role of imaging in modern clinical management, which will serve them well in the
early years of their careers as doctors. Maybe some will even be inspired to consider a future
career in this most exciting and rapidly developing discipline.

Dr Giles Maskell MA, FRCP, FRCR, FRCPE


President, Royal College of Radiologists (2013–2016)
Consultant Radiologist
Royal Cornwall Hospitals NHS Trust
Truro, UK

xi

K30031_Book.indb 11 9/6/17 1:34 PM


K30031_Book.indb 2 9/6/17 1:34 PM
Preface

This book has been a long time in the making and is the product of many years of both teaching
and examining undergraduate medical students. Over this time there has been an exponential
increase in the use of all forms of imaging in both acute and elective patient care and this has been
reflected in undergraduate medical school curricula and also examinations. Radiology images
feature prominently in both Finals written papers and Objective Structured Clinical Examination
(OSCE), and whole OSCE stations may be based upon a chest X-ray for example. Various imag-
ing modalities tend to feature, in particular X-rays of the chest, abdomen, and common fractures,
but increasingly CT and MR images. The incorporation of radiology/imaging into Finals reflects
the increasing exposure of both medical students and junior doctors to all forms of radiology and
the requirement for trainees to be able to provide provisional interpretation of many forms of
imaging.
This book is not intended to be an all-encompassing textbook of radiology, and the bibliog-
raphy provides supplementary reading for those who wish to dig deeper. A case-based approach
has been adopted and radiology images in questions have been selected in two broad categories –
those that students could expect to encounter in Finals or, alternatively, to cover key learning
points/educational aspects of radiology. This structure should allow students and also foundation
doctors to approach both Finals and the foundation years with more confidence.
Inevitably within the book there is a strong emphasis on plain film interpretation, as these
investigations are the most common form of imaging that students and junior doctors will
encounter and they will also often be expected to provide a provisional interpretation. Extensive
additional examples are used in case answer sections to explain and reinforce learning points
throughout the book. There is widespread use also of common/important CT/MR images, again
because these modalities are increasingly frontline; for example, CT head interpretation in stroke
care. There is less emphasis on ultrasound and nuclear medicine, as these modalities occur less
frequently in Finals, although an understanding of their use is necessary. Ultrasound does feature
in some cases reflecting more widespread use of this modality on the wards and in the emergency
department.
We hope you will enjoy this book and that it will stimulate and enhance your knowledge and
understanding of radiology, and improve your confidence in image interpretation.

Edward Sellon
David C Howlett

xiii

K30031_Book.indb 13 9/6/17 1:34 PM


K30031_Book.indb 2 9/6/17 1:34 PM
Contributors and
acknowledgements

Dr Hannah Adams BSc (Hons), MBChB Dr Uday Mandalia MBBS, BSc,


Radiology Registrar MRPCH, FRCR
Brighton and Sussex University Hospitals Consultant Radiologist
NHS Trust, Brighton, UK Hillingdon Hospital, Uxbridge, UK

Dr Faye Cuthbert MBBS, MRCP, FRCR Dr Sean Mitchell BMBS, BSc (Hons)
Consultant Urogenital Radiologist General Practitioner Specialty Trainee Year 2
Brighton and Sussex University Hospitals Brighton and Sussex University Hospitals
NHS Trust, Brighton, UK NHS Trust
Honorary Clinical Teaching Fellow
Dr Sarah Hancox MBBS, BSc (Hons)
Brighton and Sussex Medical School
Resident Medical Officer, Emergency
Brighton, UK
Department
Townsville Hospital, Townsville Dr Cristina Ruscanu MBBS
Queensland, Australia Foundation Year 2 Doctor
East Sussex Healthcare NHS Trust
Dr Vincent G Helyar MBBS, BSc, MSc,
Eastbourne, UK
FRCR, EBIR
Interventional Radiology Fellow Lt Col Edward Sellon BSc (Hons),
Guy’s and St Thomas’ NHS Foundation Trust MBBS, MRCS, FRCR, PgD (SEM),
London, UK Dip (ESSR), RAMC
Consultant Musculoskeletal Radiologist
Professor David C Howlett MBBS, PhD,
Oxford University Hospitals, Oxford
FAcadMEd, FRCP (London), FRCP
and
(Edinburgh), FRCR
Consultant Military Radiologist
Consultant Radiologist
Centre for Defence Radiology
Eastbourne Hospital, East Sussex Healthcare
Birmingham, UK
NHS Trust, Eastbourne
and Dr Lucy Shimwell MB BCh, BAO
Honorary Clinical Professor Resident Medical Officer
Brighton and Sussex Medical School Royal Perth Hospital, Perth
Brighton, UK Western Australia, Australia

Dr Amanda Jewison BMBS, FRCR Dr Andrew Snoddon MBChB, FRCR


Specialist Registrar in Radiology Specialist Registrar in Radiology
Brighton and Sussex University Hospitals Leeds General Infirmary, Leeds, UK
NHS Trust, Brighton, UK
Dr Olwen Westerland MBBS, BSc, FRCR
Dr Thomas Kurka BSc, BMBS Consultant Radiologist
Academic Foundation Doctor (Management & Guy’s and St Thomas’ NHS Foundation Trust
Leadership) London, UK
Brighton and Sussex University Hospitals
NHS Trust, Brighton, UK

xv

K30031_Book.indb 15 9/6/17 1:34 PM


Contributors and acknowledgements

ACKNOWLEDGEMENTS
Two people in particular have been fundamental to the successful production of this book.
Nick  Taylor, medical photographer, has worked tirelessly and with great skill preparing the
images, which are such a vital component of any book on imaging. Also Susi Arjomand who has,
with her customary patience and attention to detail, typed up the numerous editing iterations of
the manuscript. Thank you both.
The editors would also like to thank Jo Koster, commissioning editor at Taylor Francis, for her
support and guidance throughout the publishing process. Dr Gillian Watson and Dr Justin Harris
kindly provided some of the radiological images used in the text and Kirstie Leach also helped
with manuscript preparation.
Finally, we would like to gratefully acknowledge all the book’s contributors for their hard work
and enthusiasm, and for finding the time to prepare their cases amidst busy schedules.

xvi

K30031_Book.indb 16 9/6/17 1:34 PM


Abbreviations

AA aortic arch CO2 carbon dioxide


AAA abdominal aortic aneurysm COPD chronic obstructive pulmonary
AAFB acid-and-alcohol fast bacilli disease
AAST American Association for the CPPD calcium pyrophosphate deposition
Surgery of Trauma disease
ABCDE airway, breathing, circulation, CRP C-reactive protein
diaphragm, everything else CSF cerebrospinal fluid
ABG arterial blood gas CT computed tomography
ACE angiotensin-converting enzyme CT IVU computed tomography intravenous
AIDS acquired immune deficiency urogram
syndrome CT KUB computed tomography kidneys
ALP alkaline phosphatase ureters and bladder
ALT alanine transaminase CTR cardiothoracic ratio
ALARA as low as reasonably achievable CTPA computed tomography pulmonary
ANA antinuclear antibodies angiogram
AP anteroposterior (view) CXR chest X-ray
ARB angiotensin receptor blocker 2D two-dimensional
AST aspartate transaminase 3D three-dimensional
AVN avascular necrosis DCIS ductal carcinoma in situ
AVPU alert, voice, pain, unresponsive DEXA dual energy X-ray absorptiometry
AXR abdominal X-ray DHS dynamic hip screw
BCG bacille Calmette-Guérin DJ duodenojejunal
BMI body mass index DIP distal interphalangeal
BNP brain natriuretic peptide DLCO diffusion capacity of the lung for
BP blood pressure carbon monoxide (test)
BPD bronchopulmonary dysplasia DMARD disease modifying antirheumatic
bpm beats per minute/breaths per drug
minute DRUJ distal radioulnar joint
CABG coronary artery bypass graft DSA digital subtraction angiography
CBD common bile duct DVT deep vein thrombosis
CC craniocaudal (view) DWI diffusion-weighted imaging
CDH congenital diaphragmatic hernia ECG electrocardiogram
CF cystic fibrosis ECMO extracorporeal membrane
CFTR cystic fibrosis transmembrane oxygenation
conductance regulator (gene) ED emergency department
CLD chronic lung disease of prematurity eGFR estimated glomerular filtration rate
CLL chronic lymphoid leukemia ENT ear, nose, and throat
CMC carpometacarpal ERCP endoscopic retrograde
CNS central nervous system cholangiopancreatography

xvii

K30031_Book.indb 17 9/6/17 1:34 PM


Abbreviations

ESR erythrocyte sedimentation rate LBO large bowel obstruction


ESWL extracorporeal shock wave LCIS lobular carcinoma in situ
lithotripsy LDH lactate dehydrogenase
ET endotracheal LFTs liver function tests
ETT endotracheal tube LHB left heart border
EVAR endovascular aneurysm repair LMP last menstrual period
FAST focused assessment with LMWH low molecular weight heparin
sonography for trauma LUQ left upper quadrant
FBC full blood count LV left ventricle
FDG fluorodeoxyglucose LVA left ventricular aneurysm
FEV forced expiratory volume MAC Mycobacterium avium complex
FFDM full field digital mammography MAS meconium aspiration syndrome
FLAIR fluid-attenuated inversion MCA middle cerebral artery
recovery MCP metacarpophalangeal
FOOSH fall on an outstretched hand MCV mean cell volume
GCS Glasgow coma scale MDT multidisciplinary team
GFR glomerular filtration rate MI myocardial infarction
GGT gamma-glutamyl transferase MIBG metaiodobenzylguanidine
GH glenohumeral micromol/L micromoles per litre
GI gastrointestinal MIP maximum intensity projection
GORD gastro-oesophageal reflux MLO medial lateral oblique (view)
disease mmol/L millimoles per litre
GP general practitioner MR magnetic resonance
GTN glyceryl trinitrate MRCP magnetic resonance
Hb haemoglobin cholangiopancreatography
HCG human chorionic gonadotropin MRI magnetic resonance imaging
HER2 human epidermal growth mmHg millimetres of mercury
factor 2 MS multiple sclerosis
HIV human immunodeficiency virus MSU mid-stream urine
HLA human leukocyte antigen mSv millisieverts
HR heart rate MTP metatarsophalangeal
HRCT high-resolution computed NAI nonaccidental injury
tomography NEC necrotising enterocolitis
HU Hounsfield units NG nasogastric
ICD implantable cardiac defibrillator NHL non-Hodgkin lymphoma
ICE ideas, concerns, and expectations NICU neonatal intensive care unit
ICP intracranial pressure NPSA National Patient Safety Agency
ICU intensive care unit NSAID nonsteroidal anti-inflammatory
Ig immunoglobulin drug
INR international normalised ratio NYHA New York Heart Association
IP interphalangeal OA osteoarthritis
ITU intensive therapy unit OGD oesophago-gastro-duodenoscopy
IUCD intrauterine contraceptive device ORIF open reduction and internal
IV intravenous fixation
IVC inferior vena cava OSCE Objective Structured Clinical
kg kilogram Examination
LA left atrium PA posteroanterior (view)

xviii

K30031_Book.indb 18 9/6/17 1:34 PM


Abbreviations

PAOD peripheral artery occlusive SCFE slipped capital femoral epiphysis


disease SH Salter–Harris
PCR polymerase chain reaction SIADH syndrome of inappropriate
PE pulmonary embolism antidiuretic hormone (secretion)
PEFR peak expiratory flow rate SOBOE short of breath on exertion
PET positron emission tomography SPO2 saturation pressure of oxygen
PIC peripherally inserted catheter STIR short tau inversion recovery
PIP proximal interphalangeal SUFE slipped upper femoral epiphysis
PKD polycystic kidney disease TB tuberculosis
PPHN persistent pulmonary TFCC triangular fibrocartilage
hypertension of the newborn complex
PPP projection, personal TFTs thyroid function tests
demographics, previous CXR THA total hip arthroplasty
comparison THR total hip replacement
PR per rectum TIA transient ischaemic attack
PTH parathyroid hormone TNF tumour necrosis factor
RA right atrium TNM tumour, nodes, metastases
RCC renal cell carcinoma UAC umbilical arterial catheter
RDS respiratory distress syndrome U&Es urea and electrolytes
RhA rheumatoid arthritis UGI upper gastrointestinal
RHB right heart border US ultrasound
RhF rheumatoid factor UVC umbilical venous catheter
RIF right iliac fossa VBG venous blood gas
RIP rotation/inspiration/penetration VCF vertebral compression fracture
RLQ right lower quadrant VUJ vesicoureteric junction
RR respiration rate V/Q ventilation/perfusion scan
RTA road traffic accident WBC white blood cell
rTPA recombinant tissue plasminogen WCC white cell count
activator WHO World Health Organisation
RUQ right upper quadrant XR X-ray
SBO small bowel obstruction ZN Ziehl–Neelsen

xix

K30031_Book.indb 19 9/6/17 1:34 PM


K30031_Book.indb 2 9/6/17 1:34 PM
Overview of imaging
modalities
1 THOMAS KURKA AND DAVID C HOWLETT

Plain films: chest X-ray, abdominal Magnetic resonance imaging 4


X-ray, and orthopaedic bone/joint X-rays 1 Nuclear medicine 5
Ultrasound 2 Fluoroscopy techniques 6
Computed tomography 3

It is helpful for finals to have an understanding of the core imaging modalities you are likely
to encounter and to have an idea of the relative strengths/weaknesses and indications/­
contraindications for each.

PLAIN FILMS: CHEST X-RAY, ABDOMINAL X-RAY, AND


ORTHOPAEDIC BONE/JOINT X-RAYS
Conventional X-ray remains an important diagnostic tool in medicine and remains the most com-
monly used imaging modality. Plain films are commonly the chest X-ray (CXR), abdominal X-ray
(AXR), and orthopaedic bone/joint X-rays (XRs). An XR is relatively inexpensive, time effective,
and does not require any special preparation of the patient. There is a degree of ionising radiation
associated with X-ray exposure and this radiation dose varies with body part; a lumbar spine XR
entails a far higher radiation dose than a wrist XR for example owing to radiation of pelvic organs.
However, generally X-ray doses are far lower than those associated with computed tomography
(CT). Dose information is included in Chapters 3 and 4. As always ’justify‘ the exposure: does the
benefit to the patient outweigh the potential risk of irradiation?
When a radiograph is taken, the X-ray beam passes through the body part onto an X-ray sen-
sitive screen. Bones, owing to their high calcium content, absorb most of the X-rays whereas
soft tissues absorb a smaller amount, depending on composition and density. As a result, X-rays
from the bones do not reach the screen and appear white on the radiograph, with the soft tis-
sue appearing darker. X-rays pass through the air without being absorbed at all, which is then
detected by the screen and appears black on the radiograph.

ADVANTAGES
• Inexpensive.
• Usually quick to perform.

K30031_Book.indb 1 9/6/17 1:34 PM


1  Overview of imaging modalities

• Painless, noninvasive.
• Good diagnostic tool for many pathologies.

DISADVANTAGES
• Soft tissue, lung, bone resolution much reduced compared with CT/magnetic resonance
imaging (MRI).
• Provides a two-dimensional (2D), single image only.
• Radiation exposure.

INDICATIONS – ARE BROAD


CXR
• Respiratory – infection, septic screen, pneumothorax, chest trauma, inhaled foreign body,
pleural effusion, suspected malignancy.
• Cardiac – clinical heart failure, clinical cardiomegaly, heart murmurs.

AXR
• Abdomen – bowel obstruction, perforated viscus (erect CXR more sensitive), ingested
foreign body, abdominal pain in the emergency setting.
• Pelvic – pelvic fracture, neck of femur fracture.

Soft tissue XR neck


• Inhaled foreign body.
• Retropharyngeal abscess.

Bone XR
• Limbs – trauma, fractures, skeletal survey, acutely swollen joint, osteomyelitis, septic
arthritis, bone pain, tumour/metastasis.
• Skulls – skeletal survey, myeloma, dental imaging.
• Spine – trauma, scoliosis.

ULTRASOUND
Ultrasound (US) uses sound waves of high frequencies, which are emitted towards the studied
tissues and are reflected/echoed back to the probe depending on the tissue density and composi-
tion. This signal is then translated into an US image. US is a ‘live’ imaging modality and requires
interpretation while the investigation is being carried out. US colour Doppler techniques are used
to assess moving blood and are used in vascular assessment, e.g. carotid stenosis.

ADVANTAGES
• No radiation, noninvasive (some US is performed using endocavity probes, e.g. transrectal,
transvaginal, transoesophageal).
• Real-time assessment and interpretation of results.
• Relatively inexpensive.

K30031_Book.indb 2 9/6/17 1:34 PM


Computed tomography

• Useful for imaging of soft tissue and muscles, extremities, testes, breast, and eye, plus
abdomen, pelvis, chest, and vascular colour Doppler applications.

DISADVANTAGES
• Requires a skilled practitioner with US interpretation skills, operator dependent.
• No use for bone imaging as sound is attenuated/absorbed by bone.
• Images are degraded by gas and fat, and this restricts US use in the abdomen/pelvis in
some patients.

INDICATIONS
• Abdomen – trauma, malignancy, abdominal aortic aneurysm (AAA) surveillance,
gallstones, suspected hydronephrosis.
• Chest – assessment of pleural spaces.
• Musculoskeletal – assessment of muscles, ligaments, and tendons.
• Scrotal – assessment of testicles, epididymis, and scrotum.
• Obstetrics – growth scans, placental sighting, anomaly scans.
• Gynaecology – transabdominal and transvaginal imaging of ovaries, uterus, and Fallopian tubes.
• Baby hips.
• Breast, eye assessment.
• Vascular applications – suspected upper/lower limb deep vein thrombosis (DVT), carotid/
peripheral vascular assessment.

COMPUTED TOMOGRAPHY
CT uses X-rays, which are emitted from a rotating X-ray source around the patient with mul-
tiple detectors to produce a series of 2D axial images of the studied body part. This can then be
­computer-reconstructed to obtain axial, coronal, sagittal 2D, and three-dimensional (3D) images
of the studied body parts. There are other imaging modalities that make use of CT imaging such
as positron emission tomography (PET scan).

ADVANTAGES
• Provides 2D cross-sectional images of the body, which are rapidly acquired with the
potential to reformat in multiple planes; 3D reformatting is also possible.
• Provides a detailed image of the studied body part and the surrounding tissue.
• High sensitivity and specificity in particular for assessment of the lungs, mediastinum,
bones, abdomen/pelvis structures, the brain – especially acute blood.

DISADVANTAGES
• CT scanners are expensive.
• Moderate to high dose of radiation, depending on areas scanned.
• May require intravenous (IV) iodinated contrast use – risk of contrast reaction (allergy,
anaphylaxis) and nephrotoxicity in those at risk.

K30031_Book.indb 3 9/6/17 1:34 PM


1  Overview of imaging modalities

INDICATIONS
• Head – trauma, brain imaging (ischaemic/haemorrhagic strokes, calcifications,
haemorrhage, malignancy).
• Chest – detailed imaging of the lungs to detect abnormalities not seen on CXR, used
in diagnosis and surveillance of malignancy, pulmonary embolism (CT pulmonary
angiogram: CTPA), emphysema, fibrosis. Cardiac – CT to image coronary arteries.
• Abdomen and pelvis – diagnosis, staging, and surveillance of malignancies, bowel
obstruction, AAA, pancreatitis, renal calculi (CT kidneys ureters and bladder [CT KUB] and
CT IV urogram [CT IVU]).
• CT angiography and venography – for example, suspected limb or mesenteric vascular
occlusion, sagittal sinus thrombosis.
• Orthopaedic – complex fractures.
• CT-guided biopsy, surgery, and radiosurgery.

MAGNETIC RESONANCE IMAGING


MRI does not use any X-rays, thus does not expose the patient to ionising radiation. It is superior
to CT in obtaining detailed images of the soft tissues and also the brain. MRI uses strong mag-
netic fields, radio waves, and field gradients to generate the image.
In structural MRI, the images are obtained by proton alignment by an external magnet and
a subsequent radiofrequency pulse disrupts the equilibrium, which gives an MRI signal. Details
of MRI protocols and sequences are not needed for finals – T1- and T2-weighted are common
sequences (in the brain cerebrospinal fluid [CSF] appears bright/white on T2), and IV contrast can
also be used (gadolinium).

ADVANTAGES
• No ionising radiation exposure.
• Provides 2D and 3D cross-sectional images of the body.
• Superior to other imaging modalities in obtaining high-resolution images of the brain and
musculoskeletal system.
• Ideal for soft tissue structures, cartilage, and ligament imaging.
• Vascular and cardiac applications.

DISADVANTAGES
• Expensive equipment – the most expensive imaging modality.
• Time consuming, requiring patient cooperation, ability to lie still, often for 30–60 minutes.
• Contraindicated in patients with ferrous metal implants – pacemakers, cochlear implants,
metallic foreign bodies in the eyes.
• MRI is undertaken in a relatively enclosed space – unsuitable for patients with
claustrophobia and young children (may need general anaesthesia).
• Relatively contraindicated in pregnancy, particularly first trimester.

K30031_Book.indb 4 9/6/17 1:34 PM


Nuclear medicine

INDICATIONS
• Head and neck – neuroimaging – clear differentiation between the grey and white
matter, diagnosis of demyelinating disease, cerebrovascular disease, detailed imaging of
malignancies and infectious diseases, epilepsy imaging, functional MRI brain studies.
CT is more accurate in the detection of acute blood; new MRI techniques, e.g. diffusion
weighting, can detect cerebral ischaemia very early (minutes) when compared with CT.
• Spine imaging – nerve compression (cord and cauda equina), malignancies, disc disease.
• Hepatobiliary – liver, pancreas, and biliary lesions, MR cholangiopancreatography (MRCP)
for structural imaging of the biliary tree.
• Small bowel – Crohn’s disease diagnosis.
• Knee and other joints – used in cartilage and ligament imaging.
• Angiographic, vascular protocols, cardiac MRI.
• Prostate imaging, diagnosis, and staging of prostate cancer.
• Rectal, gynaecological cancer staging.

NUCLEAR MEDICINE
Nuclear medicine uses injected (or inhaled) radioactive isotopes to diagnose or treat many con-
ditions: endocrine, heart, and gastrointestinal (GI) diseases. It images the emission of isotope
radiation from within the body and can construct a 2D/3D image of the areas of the radioactive
substance uptake. It is used for functional imaging, rather than structural imaging, as contrast/
spatial resolution is poor. Some nuclear medicine is combined with CT/MRI to improve anatomi-
cal detail.

IMAGING MODALITIES
• Myocardial perfusion scan – assessment of the function of myocardium for diagnosis
of hypertrophic cardiomyopathy and coronary artery disease, in combination with
MRI +/– CT.
• Genitourinary scan – assessment of renal blood flow and function, evaluate renovascular
hypertension, and assess vesicoureteral reflux.
• Bone imaging – assessment of bone metastases, infection.
• PET – imaging of metastases, neuroimaging – imaging of brain activity in dementias,
combining injection of metabolically active substances, e.g. fluorodeoxyglucose (FDG) and
tomography/CT detection.

ADVANTAGES
• Provides functional information of organs and disease processes.
• Advancement of treatment options for cancer patients.
• Allows early or improved detection of metastases (PET).
• Provides detailed and accurate information in hard to reach areas.
• Radioisotopes are used to treat some cancers, e.g. radioiodine and papillary thyroid
cancer.

K30031_Book.indb 5 9/6/17 1:34 PM


1  Overview of imaging modalities

DISADVANTAGES
• High cost.
• Exposure to radiation doses, which may be significant, e.g. PET.
• Not all techniques are widely available, e.g. PET.

FLUOROSCOPY TECHNIQUES
Fluoroscopy combines ionising radiation from X-ray exposure with administration (ingested/
injected) of contrast medium, which is then imaged passing through the structures/organs of
interest to assess their function and structure in real time. Examples include:

• Contrast swallow – assessment of the structure and function of the pharynx and
oesophagus (largely replaced by oesophago-gastro-duodenoscopy [OGD]).
• Barium follow through – assessment of the structure and function of the small bowel
(MRI small bowel replacing).
• Contrast enema – assessment of structure and function of the large bowel and rectum
(colonoscopy replacing), used particularly to evaluate the integrity of postoperative bowel
anastomoses.
• Tubogram (hysterosalpingography) – assessment of the shape of the uterine cavity and the
shape and patency of the Fallopian tubes.
• Arteriogram, venogram (CT/MRI replacing).

ADVANTAGES
• Allow a ‘live’ assessment.
• Relatively inexpensive, readily available.
• Relatively noninvasive.

DISADVANTAGES
• Exposure to ionising radiation, which may be significant, e.g. barium enema.
• Poor soft tissue resolution.
• Endoscopy techniques are more accurate in bowel mucosal assessment and allow tissue
biopsies.

K30031_Book.indb 6 9/6/17 1:34 PM


Hints and tips for finals
Objective Structured Clinical

2 Examination
THOMAS KURKA

Logistics of preparation and the day itself 7 Examination stations 10


Your communication skills 8 Imaging, blood results, and other test
Communication stations 9 results in OSCE 12
History taking stations 10 Final words 13

The OSCE (Objective Structured Clinical Examination) is designed to test clinical and communi-
cation skills in a structured environment in real time. Many medical schools use the ‘integrated
station’ approach in their OSCE exams, which means that you may be asked to take a focused his-
tory, do a part of a clinical examination, and interpret a test result all in one station. This tests your
knowledge, skills, and your thinking process towards reaching a working diagnosis. Remember
that most people pass their OSCE and you are allowed to fail a small proportion of the stations –
your medical school will be able to advise on the specific rules of the exam.

LOGISTICS OF PREPARATION AND THE DAY ITSELF


• Practice ... practice ... practice! Then practice even more. It is important to have some regular
quality group study time before your OSCE. This exam is about your skills and practical
experience, and you cannot pass the OSCE if you only study from books.
• You should observe other students practicing OSCE-style scenarios, give each other
constructive feedback, and correct mistakes. It is important to be helpful and polite to your
colleagues and friends but it is very important to be constructive with your feedback and
verbalise what went wrong. Some people may not be aware of their mistakes and cannot
improve unless you tell them.
• Although it may seem intimidating, do ask doctors to assess you when on the wards. Most
are keen to teach and help you pass and it will give you more experience in presenting real
cases.
• The OSCE is a role play, not a real-life scenario. You need to learn to play the game. Speak
to previous students who passed finals OSCE at your medical school to understand the
structure of the stations and the day.
• Have a good night’s sleep before the OSCE day. Tiredness decreases concentration and
organisational skills and hinders your ability to communicate effectively. The OSCE is a
type of performance and you need to be fresh and alert to perform well.

K30031_Book.indb 7 9/6/17 1:34 PM


2  Hints and tips for finals Objective Structured Clinical Examination

• Read the OSCE station instructions properly and follow the script – this ensures you stay
on the topic of the OSCE station and will earn you points. If the station says take a history
from the patient you will not score any points on educating or advising the patient. Stay
focused on the tasks specified in your station brief.
• Begin every station with a polite introduction of yourself. Knock on the door before
entering and say hello with a smile on your face (even a nervous smile counts). Introduce
yourself with a full name and your role, and do not forget to articulate. Most feedback
from the patients from OSCE stations was that they could not understand the students’
names and introduction because they spoke too fast as they were nervous. Be the one to be
remembered for appearing calm, with a smile on your face and a clear introduction.
• Ask your patient’s permission to take their history and/or examine them – there is a mark
for gaining a verbal consent.
• Follow up with letting the patient tell you their story – this will allow you to have a minute
to catch your breath and to connect with the patient.
• Finally, the staff who are examining you want you to pass and you need to give them the
opportunity to give you the points!

YOUR COMMUNICATION SKILLS


• Smile and adopt an approachable body language.
• Make sure that each station is a dialogue between you and the patient. Avoid leading and
closed-end questions, especially in the history of the presenting complaint.
• There is a balance between letting the patient explain their symptoms or problems, and
them rumbling on for too long, which could be a distraction taking you off the path of
the station – keep the conversation focused to the topic of the station but ensure you do
not cut the patient off too soon, which could appear impolite and potentially damage the
doctor–patient relationship. If you need to interrupt their story, apologise for doing so,
acknowledge what they were saying, and offer to return to it if there is time at the end.
• Avoid all medical jargon! It is natural for medical students in the final year to be very
familiar and fluent in medical jargon but most patients do not understand these terms and
OSCEs will test that you can communicate using simple terms.
• Be clear and succinct when giving advice to the patients and always ensure their
understanding – the best way is to ask the patient to repeat it back to you in their own words.
• Do not ever sound patronising or forceful with any advice you give to the patient!
Remember, patients have a right to autonomy, which means that you should only advise
and they can choose to accept or decline your advice (assuming full mental capacity).
• Many students like to repeat the history back to the patients at the end to summarise and
buy some time to think about what next. This may not be recommended in finals especially
if your OSCE station is only 8–10 minutes long. During a finals OSCE, you will have more
than the history to get through (blood results, imaging or further questions) so do not waste
time on repetition as you could run out of time by the end and lose some valuable points.
• OSCE stations are often divided into two sections, an 8-minute station has 4 minutes for
history, for example, and then 4 minutes for further questions/looking at results/differential/
further management. The examiner will usually prompt you at 4 minutes if needed.
• Listen to your patients and respond directly to what they are saying. The patient (or actor)
is playing by the script and they will not mislead or give you any wrong information.

K30031_Book.indb 8 9/6/17 1:34 PM


Communication stations

It is important to acknowledge their worries and concerns directly, even if you need to
divert to continue gathering the essential information for your history. Sometimes patients
can talk for a long time and go off the topic, and it is your job to politely interrupt them,
acknowledge you will return to their point, and only then divert to what you want to talk
about. You need to appear to be in control but do it politely.
• The examiner will nudge you if you start slowing down or diverge from the main topic or
time is running short. Take the hint as they are trying to set you back on the right path, the
path of the marking sheet.
• Avoid talking too much. It can be tempting to try to talk a lot to show you know your
subject but remember this is a two-way discussion, not a monologue. This applies mainly to
communication stations when you are asked to explain a procedure, counsel the patient or
discuss a new treatment. It is tempting to quickly say everything you know about the subject
to impress your examiner but remember this is about giving information to the patient who
needs to understand it, be able to ask questions, and share their point of view with you.

COMMUNICATION STATIONS
• Communication stations are those where you are asked to discuss a certain treatment or
procedure with a patient, to break bad news or to deal with a complaint.
• Practice communication stations with your friends and colleagues.
• Many medical schools use communication stations in their finals OSCE. Commonly the
instructions prior to entering the communication stations will be very brief, allowing
consultation for the full time of the station. This can be both an advantage and a
disadvantage, as you need to be very organised to structure your discussion to fill the time
and cover the most important areas.
• It is crucial to have a general structure on how to approach any station. There are a number
of structures that ensure you are able to obtain and give all the necessary information about
any topic and allow for a two-way discussion. Prior to entering the examining room decide
which structure you are going to use. For example, when asked to explain a procedure,
discuss a new treatment or counsel a patient, always start by gaining permission to discuss
the topic with your patient: ‘I am here to talk to you about X, would it be OK?’ This is
usually followed by, ‘What do you know about X?’ By asking this question, you gain the
patient’s understanding, perceptions, and concerns about the topic. This often provides
the narrative you should use to elaborate on. Always ensure you pause regularly and check
the patient’s understanding and give time for questions. You have to address all of their
concerns and answer all their questions by the end of the station. It is good practice to start
winding down in the last minute of the station, recap all of the important points, and allow
for final questions.
• Remember you cannot know everything and it is important to admit it. It is appropriate
to say that you do not know but you would check with your senior and tell the patient later.
By doing this, you show that you understand your limitations and that you will be a safe
practitioner.
• It may happen that the station instructions ask you to discuss a topic you have absolutely no
knowledge about. Do not panic! In such situations, remember that following a script could
get you out of trouble. Allow the patient to tell you what they know about the topic, which
may trigger some of your knowledge. Be honest and acknowledge that this is a topic you do

K30031_Book.indb 9 9/6/17 1:34 PM


2  Hints and tips for finals Objective Structured Clinical Examination

not know a great deal about but state that you will find out. Also, if you find yourself totally
lost and have no more to talk about, remember to consider the patient’s ideas, concerns, and
expectations (ICE). One tip would be to discuss the patient’s social support – do they have a
partner, family or friends who they could talk to or get help from? Would they benefit from
counselling, group sessions or further information from the Internet or leaflets? Do they have
a general practitioner (GP) with whom they would feel comfortable discussing this further?
This not only keeps the conversation going but it shows that you understand that difficult life
situations and decisions require support from those who are closest to the patient.
• Sometimes you may encounter a difficult conversation station such as an angry patient or
relative, or having to break bad news. Many students feel that they have to show knowledge
of the topic to score all the points but often the main point is to be empathetic, respond to
the patients’ concerns, allow them to express their feelings and emotions, and remember
that the use of silence in difficult conversations can be exactly what the patient needs.
• Practice breaking bad news with your friends and colleagues before your OSCE. It is often
uncomfortable to be silent through a stressful or a sad discussion but it is important to use
silence at the right moment. The more you practice, the easier it becomes.

HISTORY TAKING STATIONS


• Practice history taking stations with your friends and colleagues.
• You may be asked to take a full history, focused history or medical history. Whatever it is
called, you should ensure that you always take a full history including past medical, drug,
family, and social history.
• Before you start your station, be clear on how long you have to obtain the medical history.
Sometimes you may only have 4 minutes out of an 8-minute station but this should be
clearly stated in the station instructions. Pace yourself and do not forget to ask about drugs,
allergies, smoking, alcohol intake, and social situation before you run out of time. You will
lose valuable marks on relatively simple questions, which can be rehearsed and used in
every history taking station.
• As a rule of thumb, in every adult history taking station, always ask the ‘B questions’ of
cancer screening: ‘Have you noticed any unexpected weight loss, if so how much and over
how long? Have you had any fevers or night sweats?’ You will never fail to score on these
points if you make these questions a habit.
• If you run out of steam when asking about history of the presenting complaint, skip to
the other sections of the history taking – drugs, allergies, family, and social history – and
then return back to history of the presenting complaint. By doing this, you score all the
important points for other sections and give yourself some time to think about other
aspects of the presenting complaint.

EXAMINATION STATIONS
• Practice examination stations with your friends and colleagues.
• Always gain an informed consent from your patient.
• Never hurt your patient during a physical examination. It is important to ask about pain
before your examination. Always check with the patient if you are causing them discomfort
during the examination and warn the patient if you have cold hands before you touch them!

10

K30031_Book.indb 10 9/6/17 1:34 PM


Examination stations

• Read your instructions clearly to understand which part of the body you are supposed to
examine. If it says to examine the cardiovascular system, than start at the bottom of the bed
with general inspection, moving onto the hands, face, neck, etc. If it says to examine the
precordium, then you are only being asked to concentrate on the chest. If in doubt, always
ask the examiner to clarify the instructions for you.
• Be systematic! You need to develop a sequence by which you can examine any body system.
The general rule is to OBSERVE, PERCUSS/PALPATE, AUSCULTATE/MOVE. You can
examine somebody’s shoulder by following the sequence of observe, palpate, and move
even if you cannot remember precisely how to do it.
• Students are never sure whether to narrate during the examination or not. Some medical
schools have specific rules about this and you should follow them. A rule of thumb is to
narrate only those parts of the examination that are not obvious to the examiner. For
example, when you are inspecting the hands during an abdominal examination, you should
comment on nicotine tar staining, clubbing, palmar erythema, etc. If you did not narrate
this part and the examiner had a separate point for each of these findings on their scoring
sheet, it would be difficult to award you all the points. On the other hand, if you were
auscultating the heart, the examiner can see the areas that you are auscultating and you
would not need to state this. However, you would need to state your findings with regard to
heart sounds.

• Avoid saying, ‘I am looking for...’ because this does not inform the examiner whether
you found it or not. You should always say, ‘There is no pitting oedema of the legs’ rather
than, ‘I am looking for leg oedema.’

• Practice summarising your examination findings in three succinct sentences. You do not
need to state everything. Unless you found any peripheral signs of a disease, it is perfectly
acceptable to say that there were no peripheral signs of disease. You have to mention all
your positive findings. For example: ‘I performed a full cardiovascular examination on
Mr X, a 35-year-old male who had no peripheral signs of cardiovascular disease. His blood
pressure was 135/70 mmHg with a regular pulse of 70 bpm, heart sounds one and two were
audible with no additional sounds, and his lung bases were clear. I would conclude this to
be a normal cardiovascular examination.’
• Practice using instruments. It is easy to spot a student who has never held a patellar
hammer or ophthalmoscope in their hand. When practicing for your OSCE, make sure you
have all the required equipment and you practice using it.
• Use alcohol gel when practicing examination stations with your colleagues. It is an easy
point on the mark scheme to gain and many students forget to use it because of stress.
Use it in your practice time to ensure it becomes second nature. You should gel your hands
before examining your patient and again after, just before leaving the room.
• If you find something abnormal during your OSCE examination and you cannot remember
what it is, what it is called or what sign of disease it represents, describe it to the examiner
in your own words and say that you recognise this as abnormal but you cannot remember
what it signifies. Also offer to seek advice from senior colleagues; this will demonstrate that
you are safe, and there are usually OSCE points for stating this.
• Do not forget to look around the bedside. Some patients may have a walking stick, inhaler,
glyceryl trinitrate (GTN) spray, glasses or hearing aids on the table. These are there to give
you a hint; take it!

11

K30031_Book.indb 11 9/6/17 1:34 PM


2  Hints and tips for finals Objective Structured Clinical Examination

• Often, if there is something on the patent’s table in the OSCE station, it is there to be used.
If you were asked to examine the thyroid gland, make sure you use the glass of water on
their table to assess swallowing. If you were to examine someone’s hands, make sure you
use the 50p coin on the table to observe grip and dexterity.
• Always thank the patient afterwards. In many stations the examiner will ask for the
opinion of the actor with regard to how you treated them and your general demeanour.
Patient opinion does not usually attract marks but will add to the overall impression of the
examiner. Often patients are volunteering their services, especially if they are real patients,
and although you are only doing the station once they will be repeating it with nervous
students multiple times. A kind word and a smile will go a long way.

IMAGING, BLOOD RESULTS, AND OTHER TEST


RESULTS IN OSCE
• You will encounter imaging and test results in many stations of your finals OSCE.
• They are often incorporated into the station but can be the main focus of the station as well.
You may be asked to take a history from the patient, be asked what investigation you would
want to do, and then be presented with the results of these to interpret.
• If you are asked to take a medical history or examine the patient, you should have a good
grasp of the presenting problems or signs and therefore be able to interpret the test results.
You should be almost able to predict what the blood results or CXR would show even before
you see it.
• Be systematic when approaching test interpretation and take into account any medical
history or examination findings.
• Remember that test results can be normal! Do not be scared to say that a CXR is normal if
that is what you think.
• At the end of a medical history or examination you may be asked what investigations you
would like to do. Always start with the easiest/noninvasive investigations first and build
it up. First, you should mention bedside tests – general observations, bloods (FBC, U&Es,
CRP, LFTs, TFTs, ESR, amylase, group and save, cross-match, Ca 2+, Mg2+, PO34− , and
glucose). Often not all of those are required; be guided by your differential diagnosis.
Also consider blood cultures, urine dip/MSU, capillary blood glucose, ECG, arterial/
venous blood gas, wound swab, etc. Then move onto imaging tests, starting with the
least invasive appropriate test first (e.g. US, CXR, AXR). Then, if appropriate, add more
complex diagnostic tests at the end if indicated (CT, MRI, diagnostic laparoscopy, etc.).
• It is important to emphasise that listing these key and baseline tests is essential to pick
up easy marks in the exam. Do not assume the examiner knows which tests you would
request; you need to specifically go through the lists of investigations and mention them to
the examiner.
• Many OSCEs will have imaging incorporated into the stations. This will most likely be
displayed on a computer screen, anonymised, and with an obvious pathology. You may only
have a minute or two to comment on the imaging results during the station so have a clear
system of reporting CXR, AXR, CT, and MRI. Do not forget to state that you would check
that the image is from the correct patient and is the most recent. Then continue describing
the abnormality on the film and correlate it with the history and examination.

12

K30031_Book.indb 12 9/6/17 1:34 PM


Final words

• Imaging in the exam is covered in detail throughout the book and may occur in the OSCE
or the written papers. Imaging pathology will be clearly apparent and the image is usually
nonadjustable on the computer screen. Having a clear method and approach to image
presentation is essential and will reassure the examiner that you have seen/presented
imaging many times in the past.

FINAL WORDS
• Practice ... practice ... practice: be systematic, practice more, and remember to be seen to
wash your hands as needed. Do not panic because by the time you undergo your OSCE
exams you should have a system to tackle any problem, practice again, smile, and be kind to
your patients.
• Do not rely on books only to prepare for OSCE. You must get involved in regular group
revision.
• Eat healthily, keep hydrated, exercise, and get some quality sleep during your revision
period.
• Allow yourself some downtime to relax. Watch some television, visit friends, play your
favourite sport, go for walks or anything else you used to do before you started this
revision. Do not allow it to completely take over your life but at the same time make it your
priority.
• Remember, there is light at the end of the tunnel and the skills you are learning now are
genuinely useful for the future.
• Most people pass!

13

K30031_Book.indb 13 9/6/17 1:34 PM


K30031_Book.indb 2 9/6/17 1:34 PM
The normal chest X-ray

3 THOMAS KURKA

Indications for requesting a CXR 15 Example OSCE stations with CXR


Medical ionising radiation exposure 15 interpretation 26
CXR reporting technique 16

The CXR is the most common radiological investigation performed. Interpretation can be dif-
ficult and often falls initially to those with relatively limited experience. It is important to have a
systematic approach to interpretation to ensure that the correct diagnosis is made and nothing is
missed. This will help both in the examination situation and in real life.
This chapter provides a step-by-step approach to reviewing the CXR. It provides a compre-
hensive problem-solving technique, which encourages a set format involving an introduction, a
detailed assessment of the key abnormality, and a systematic review of the rest of the film. It is
this systematic review that students have most difficulty with and we provide two different tech-
niques for dealing with it. We then illustrate these techniques with example cases.

INDICATIONS FOR REQUESTING A CXR


Even though a CXR may not be the diagnostic investigation of choice for pulmonary embolism
(PE), lung cancer or heart failure, for example, it can provide some very useful information and as
such is frequently used as the first-line investigation when cardiorespiratory patients present to
the hospital. Additionally, the radiation dose of a CXR is low (0.015 mSv) – around 440 times less
than a chest CT scan– making it the least invasive investigation of choice. A summary of the main
indications for CXR is shown in Table 3.1.

MEDICAL IONISING RADIATION EXPOSURE


X-rays and gamma rays damage DNA. Some of this damage is predictable and dose dependent.
There are dose-related and predictable effects, such as radiation sickness and alopecia, which
occur at set doses of radiation. Other effects, such as the development of cancer, are not dose
dependent and a safe level of radiation cannot be predicted. The chance of these events occurring
increases with dose but does not have a known safe threshold.

15

K30031_Book.indb 15 9/6/17 1:34 PM


3  The normal chest X-ray

Table 3.1  Common indications for requesting a CXR

Diagnoses Symptoms
Respiratory Malignancy (primary or secondary) Haemoptysis
Infection – pneumonia, TB Dyspnoea
Pulmonary embolism Chest pain
Pleural effusion Productive cough
Inhaled foreign body Severe abdominal pain
Chest trauma
Pneumothorax
Acute exacerbation of asthma
Acute exacerbation of COPD
Cardiac Heart failure
Heart murmurs
Surgical Pneumoperitoneum
Other NG tube position
Central venous catheter position

The lifetime risk of developing cancer is influenced by the dose and cumulative exposure
to radiation. According to the Royal College of Radiologists, exposures of less than 1 mSv
(equivalent to 70 CXR or 6 months of background radiation) confer a cancer development risk
of less than 1:20,000. This rises to about 1:4,000 for 5 mSv and 1:2,000 for 10 mSv exposures.
The risk ratio, however, is heavily influenced by age and sex, with infants and females at the
greatest risk. The risk of a medical exposure, however, should always be put into the context of
the population cancer risk (currently 1 in 3 in the UK) and be balanced against the investiga-
tion benefits.
Each of us is exposed daily to background radiation from the earth and space. The background
radiation in the UK is around 2.2 mSv per year with a regional variation of as much as 1.5–7.5 mSv
per year depending largely on rock type (notably granite in the Aberdeen area and the rocks of
Cornwall). In addition, we expose ourselves to further radiation during air flight. A return flight
from London, UK to New York, USA adds approximately 0.1 mSv radiation exposure, which is
equivalent to seven CXRs.
ALARA (as low as reasonably achievable) is an American safety principle and regulatory
requirement, which sets standards for a reasonable level of radiation exposure. The main princi-
ples are time (minimising the time of direct exposure), distance (double the distance, quarter the
dose), and shielding (using absorbent materials to reduce radiation exposure). Table 3.2 illustrates
the associated radiation dose of some common imaging tests with the equivalent dose in CXR or
background radiation.

CXR REPORTING TECHNIQUE


Much of this section refers to real-life CXR review and most is also applicable to the exam sce-
nario. Remember X-rays in the exam will appear on a computer screen or an examination ques-
tion sheet – they will be anonymised and pathology should be obvious. Image manipulation on
the screen is not usually allowed or needed.

16

K30031_Book.indb 16 9/6/17 1:34 PM


CXR reporting technique

Table 3.2  Radiation doses for the main imaging modalities

Equivalent period
Equivalent of background
Modality DOSE (mSv) in CXRs radiation
CXR 0.015 1 2.5 days
AXR 0.4 30 2 months
XR pelvis 0.3 20 1.5 months
XR skull 0.07 5 12 days
XR hip 0.8 60 4 months
XR hand/foot <0.001 <1 <2 days
XR cervical spine 0.05 3 7.5 days
XR thoracic spine 0.4 30 2 months
XR lumbar spine 0.6 40 3 months
CT head 1.4 90 7.5 months
CT chest 6.6 440 3 years
CT abdomen 5.6 370 2.5 years
CT abdomen/pelvis 6.7 450 3 years
CT chest/abdomen/pelvis 10 670 4.5 years
CT KUB 5.5 370 2.5 years
CT colonography 10 670 4.5 years
Barium swallow 1.5 100 8 months
Barium meal 2.0 130 11 months
Barium enema 2.2 150 1 year
Bone (Tc-99m) scan 3.0 200 1.4 years
DEXA scan 0.0004 <1 <2 days
Mammogram 0.5 35 3 months
PET scan 18 1200 8.1 years

LEARNING POINTS: REPORTING TECHNIQUE


▪▪ Introduction:
– ‘PPP’ (projection, personal demographics, previous CXR comparison).
– Technical factors: ‘RIP’ (rotation/inspiration/penetration).
▪▪ Describe the obvious abnormality: what and where.
▪▪ Systematic review: ABCDE (Table 3.4) or anatomical approach.

First, you need to introduce the CXR by checking the projection of the image and men-
tioning any available personal demographic information (i.e. ‘This is a PA CXR of an adult
female.’). It is really important to check you are reviewing the correct CXR for the correct
patient, and from the correct date and time. In the exam you will not be able to do this as
the XR should be anonymised but mention to the examiner that you would wish to do this
as part of your usual practice. Then consider its technical quality (i.e. ‘There is no rotation,
and inspiration and penetration are adequate.’). This ensures that any visible abnormality is

17

K30031_Book.indb 17 9/6/17 1:34 PM


3  The normal chest X-ray

likely to be related to pathology rather than an artefact of the film. Exam XRs should not have
any technical issues.
Second, describe any obvious abnormality in terms of what and where.
Third, a systematic review of the entire CXR is required to make sure that you have not missed
anything. Two different approaches to this (ABCDE or anatomical) are discussed below. Finally,
summarise your findings, give a diagnosis or differential diagnosis, and recommend further
management.
These three stages of reporting will now be discussed in greater detail.

INTRODUCTION
A number of factors should be considered for PPP.

PPP
Projection
• Pay attention to letters PA (posteroanterior) or AP (anteroposterior) on the CXR and also
the words erect or supine (Figure 3.1).
• A standard CXR is taken in a PA, erect position (i.e. the patient is standing up with
shoulders internally rotated, hands on hips, which moves the scapulae laterally so they are
less visible on the film). If it is not labelled, this is the default position.
• AP films are taken for patients that are difficult to mobilise and/or very unwell and may be
labelled portable where the patient is sat up in bed with the film cassette tucked behind them.
• AP films have more of the scapulae projected over the chest. They also have a more
prominent cardiac silhouette, which can be misinterpreted as cardiomegaly. Only assess
heart size on a PA projection (Figure 3.1).

PA AP

X-ray film X-ray film

Fig. 3.1  The size of the heart on the PA and AP CXR. Note on PA projection that the heart is closer to the
X-ray film and thus less magnified by the divergent beam than on the AP view.

18

K30031_Book.indb 18 9/6/17 1:34 PM


CXR reporting technique

PA erect is a standard for CXR


Projection
AP – cannot comment on the heart size

Personal Name, age (date of birth), hospital number, gender


demographics Date and time when film taken

Previous CXR Allows for differentiation between


comparison acute and chronic changes

Fig. 3.2  PPP (projection / personal demographics / previous CXR comparison).

• Other projections are available (i.e. lateral, lordotic, apical, rotated, and oblique) but they
have been almost entirely replaced by the use of CT. You will not see these projections in
medical school exams.

Personal demographics (Figure 3.2)


• Always ensure you present the full name, date of birth (age), and hospital number of the
patient (more applicable to ward rounds, rather than the exam).
• The CXR will be anonymised in the exam but you should offer to check the personal
demographics at this stage. You may be able to tell if they are adult or paediatric by the
presence of growth plates, also male or female by the breast shadows.
• If there is a date and time on the image remember to mention it.

Previous CXR comparison


• Offer to compare the current CXR with any previous films available. This helps to
differentiate between acute and chronic changes.
• It is also important to check you are reviewing the correct CXR for the correct patient from
the correct date and time.

RIP
Rotation
• To assess for rotation, find the medial heads of the clavicles and compare their distances
away from the spinous process of the adjacent vertebral body (Figures 3.3 and 3.4).
• If the spinous process of the vertebral body is equidistant between both clavicle heads then
there is no rotation.
• If the gap is less on the right then the patient is rotated to the right, and vice versa.

Inspiration
• Patients are asked to breathe in and hold their breath when a CXR is taken so that the lungs
are optimally visualised.
• Poor inspiratory effort may be caused by pain, confusion or respiratory distress.

19

K30031_Book.indb 19 9/6/17 1:34 PM


3  The normal chest X-ray

Fig. 3.3  Assessing the technical quality of CXR (M = medial clavicle).

Rotation Distance between the clavicular heads and


the spinous processes

9–11 posterior or 5–7 anterior ribs are visible


Inspiration
on a PA film

Vertebral bodies are just visible behind


Penetration
the heart

Fig. 3.4  Technical factors: RIP (rotation/inspiration/penetration).

20

K30031_Book.indb 20 9/6/17 1:34 PM


CXR reporting technique

• Hyperexpanded lungs may be seen in COPD patients with obstructive airway disease.
The diaphragms will appear flattened.
• Inspiratory effort is described as adequate when 9–11 posterior or 5–7 anterior ribs are seen.

Penetration
• The vertebral bodies should just be visible behind the heart for adequate penetration.
• If the CXR is either over- or underpenetrated, then you will not be able to fully assess all
the structures and compare their densities accurately.
• An underpenetrated XR appears overly opaque/dense/white.
• An overpenetrated XR appears too lucent/dark/black.

DESCRIBE THE OBVIOUS ABNORMALITY: WHAT AND WHERE


Sit back and look over the whole CXR to spot any obvious abnormality. If you see something
abnormal, describe this in terms of what and where before proceeding with the systematic review.
Practice using the correct terminology to describe the common pathologies as outlined below.

WHAT
• Shape: describe the shape of the abnormality (round, diffuse, well/poorly demarcated).
• Size: describe size of lesion.
• Density: say if it is hypo- (dark) or hyperdense (bright) compared with the surrounding
soft tissues, also if it is homogeneous (same density throughout) or heterogeneous (various
densities). Cavitating lesions have a soft tissue rim with a hypodense core and may contain
an air/fluid level. Is there calcium or fat density associated?
• Associated factors: presence of lung oedema, fluid level or air bronchogram. For pleural
effusions, describe which side is affected, comment on the presence of a meniscus and how
high the fluid level extends.

WHERE
• Site: say which lung is affected.
• Site: describe whether it is in the upper, middle or lower zone of the lung. This is much
easier than trying to assess which lobe is involved, this may be difficult to evaluate on the
frontal XR.

Common descriptions include:

• Pneumonia: mostly unilateral, patchy, soft tissue consolidation. Look for air bronchograms.
• Pulmonary oedema: mostly bilateral, patchy, soft tissue consolidation with associated
cardiomegaly and pleural effusions (Table 3.3).
• Pleural effusion: mostly unilateral, homogeneous, soft tissue opacification. Blunting of
costo- and cardiophrenic angles with a meniscus at the air–fluid level.
• Pneumothora: loss of lung markings in the lateral aspect of the thorax with a visible pleural line.
• Tension pneumothorax: as above with mediastinal and/or tracheal shift away from the
pneumothorax and flattening of the ipsilateral hemidiaphragm.
• Lobar collapse: mediastinal and/or tracheal shift towards the collapse, raised ipsilateral
diaphragm, displaced hilum, and rib space narrowing.

21

K30031_Book.indb 21 9/6/17 1:34 PM


3  The normal chest X-ray

Table 3.3  ABCDEF of pulmonary oedema


A Alveolar and interstitial shadowing
B Kerley B lines
C Cardiomegaly
D Upper lobe Diversion
E Effusion
F Fluid in the horizontal fissure

SYSTEMATIC REVIEW: ABCDE OR ANATOMICAL APPROACH


There are two different systematic approaches to reviewing the CXR. One follows the familiar
ABCDE approach to assessing the acutely unwell patient (Table 3.4 and Figure 3.5). The second
approach is used primarily by radiologists and follows the anatomical landmarks of the film.
These are just two examples of how to do it and in time you will establish your own approach.
Make sure before you start your systematic review that you have considered the nature of the film
and its technical qualities, as described above.

ABCDE APPROACH
Airway
• Trachea should be central. Deviation to the right may be related to ipsilateral lung
volume loss (lung or lobar collapse) or contralateral volume expansion (pneumothorax,
haemothorax, pleural effusion or large lung mass). It may also be deviated by a mediastinal
mass (thyroid goitre).
• Free gas in the soft tissues (surgical emphysema) secondary to penetrating trauma or severe
asthma.
• Neck masses, such as an enlarged thyroid goitre or calcified vascular calcification (subclavian
aneurysm), may be visible.

Breathing
• Lung apices should be compared. They should be symmetrical and have a similar density –
take care here as pathology in the apices can easily be missed!
• Upper, middle, and lower zones. Follow the lateral borders down to the bases and then up
towards the hila. Compare both sides (Figure 3.5).
• Pneumothorax. Close inspection of the lateral borders of each lung for a visible pleural line
and rim of absent lung markings. If you are shown a pneumothorax in the exam it will
usually be large and clearly demonstrated – small, subtle lesions will not be used.

Table 3.4  ABCDE of CXR


A Airway
B Breathing
C Circulation
D Diaphragm
E ‘Everything else’

22

K30031_Book.indb 22 9/6/17 1:34 PM


CXR reporting technique

Fig. 3.5  Airway and breathing structures on CXR. Trachea (1), carina (2), right hilum (3), left hilum (4), right
costophrenic angle (5), left costophrenic angle (6), right cardiophrenic angle (7), left cardiophrenic angle (8).
Note bilateral, normal, symmetrical breast outlines (9).

• Pleural angles. The costophrenic and cardiophrenic angles are checked for blunting and
increased density, as seen with consolidation (pneumonia), pleural effusion or chronic
pleural thickening.
• Hilar position, shape, and density. The left should sit at the same level or slightly higher
than the right. The hila are made up of pulmonary arteries, veins, bronchi, and lymph
nodes. They should be equal in size, shape, and density. A displaced hilum may suggest
lung volume loss. A dense or enlarged hilum may be caused by lymphadenopathy
(due to infection, malignancy or sarcoidosis) or pulmonary hypertension (due to COPD
or heart disease).
• Nodules and masses may be dense and well defined (calcified) or soft tissue density and
poorly defined: the latter are more concerning for malignancy. They may be single or
multiple. Remember to check behind the heart for a subtle mass in the left lower lobe and
also to assess the basal segments of the lower lobes through the upper abdomen/diaphragm.
• Fissures. Check the normal appearance and position of the fissures, as these will be
distorted in lobar collapse.

23

K30031_Book.indb 23 9/6/17 1:34 PM


3  The normal chest X-ray

Circulation
• Heart size (if it is a PA film). If you compare the width of the heart with that of the thorax,
the cardiothoracic ratio should be less than 50% in adults (Figure 3.6). Cardiomegaly may
be caused by heart failure.
• Mediastinal shift. If the heart no longer appears in the centre of the thorax the film may
be rotated, there may be volume loss pulling structures towards the pathology (lung or
lobar collapse) or volume increase pushing structures away from the pathology (tension
pneumothorax, haemothorax or large mass).
• Aortic arch (AA). This should be on the left. If small, there could be an atrial septal defect.
If enlarged, there may be hypertension, aortic stenosis or aortic dissection.
• Left heart border (LHB). The left atrium (LA) or left atrial appendage may be enlarged
in mitral valve disease (now rarely seen as rheumatic heart disease is less prevalent).

Fig. 3.6  Circulation structures on CXR. The left heart border (LHB) is made of the left atrium (LA)
superiorly and left ventricle (LV). The right heart border (RHB) is made up of the right atrium (dotted line)
only, as the right ventricle lies posteriorly. Aortic arch (AA), descending right pulmonary artery (PA). The
CTR (cardiothoracic ratio) is the greatest cardiac width ÷ the intrathoracic width at its widest point (inner rib
→ inner rib), <50% in adults.

24

K30031_Book.indb 24 9/6/17 1:34 PM


CXR reporting technique

The left ventricle (LV) may be enlarged in volume overload due to aortic or mitral
regurgitation, ischaemia or cardiomyopathy causing primary left ventricular disease, or
pericardial effusion. If the LA enlarges (e.g. mitral stenosis) this may widen the carina
(LA sits in the subcarinal region) and extend across to the right heart border (RHB) causing
an apparent ‘double’ heart border.
• RHB. The right atrium (RA) may be enlarged in tricuspid regurgitation.
• Pericardium. Gas shadows around the cardiac silhouette into the mediastinum may indicate
pneumomediastinum.

Diaphragm
• Position. The right hemidiaphragm should be slightly higher than the left due to the mass
effect of the liver. There should be curvature in both.
• Pneumoperitoneum is characterised by free subdiaphragmatic gas on an erect CXR. This is
usually caused by bowel perforation and would warrant urgent surgical review.

‘Everything else’
• Lines and tubes must all be commented upon. State whether they are adequately positioned
or need replacing [i.e. nasogastric (NG) tube, endotracheal (ET) tube, central venous line,
and pleural drain].
• Cardiac device. If present, describe its position, how many leads leave the device and where
they terminate, and is there lead fracture?
• Bone fractures. Check the clavicles and ribs. Make sure the acromioclavicular and
glenohumeral joints of the shoulders are intact. Look for rib metastases.
• Breast contours in female patients. Note any asymmetry or evidence of previous
mastectomy. There may be surgical clips.

THE RADIOLOGIST’S ANATOMICAL APPROACH


This alternative method is preferred by radiologists as it follows the usual format of a written
report. The detail of what to look for under each section is the same as above.
Like the ABCDE approach, it is always preceded by a note on the projection, patient demo-
graphics, and previous films for comparison (PPP). The technical quality is also checked in terms
of rotation, inspiration, and penetration (RIP). The radiologist will usually also check the lines
and tubes, and review the breast shadows at the outset.
Having described any immediately obvious abnormality, the following systematic review is
conducted.

Heart
• Heart size and contour.
• Mediastinal and hilar size and contour.

Lungs
• Lungs apices, upper, middle, and lower zones.

Bones
• All visible bones are checked for fractures and focal lesions.

25

K30031_Book.indb 25 9/6/17 1:34 PM


3  The normal chest X-ray

Review areas (where abnormalities are most likely to be missed)


• Peripheral soft tissues (soft tissue mass or injury).
• Behind the heart (left lower lobe collapse or a lung nodule).
• Costophrenic angles (lung nodule, consolidation, effusion).
• Below and behind the diaphragm (lower lobe lung nodules and subdiaphragmatic gas).

SUMMARY OF REPORTING TECHNIQUE – HOW TO PRESENT


IN THE EXAMS AND IN REAL LIFE
• Your summary should be succinct and straight to the point (2–3 sentences only). This is of
particular importance in the exam where time is tight.
• Summarise all positive findings and correlate these with the history and examination
findings.
• Offer a differential diagnosis and a sentence about further management. This might be a
recommendation for a further investigation or for urgent senior medical or surgical input.

EXAMPLE OSCE STATIONS WITH CXR INTERPRETATION

CASE 1
A 45-year-old female attends the emergency department (ED) with difficulty breathing and sharp
pleuritic chest pain. She has a past medical history of anxiety, depression, and hypertension. She
is a nonsmoker. Her respiratory rate is 26 bpm, oxygen saturation is 98% on air and she is tachy-
cardic at 100 bpm. A CXR is taken and you are asked to review it systematically (Figure 3.7).

Introduction (PPP RIP)


• Projection. PA erect chest radiograph (by default, as the image is not labelled).
• Personal demographics. Offer to check patient’s details as the film is anonymised.
• Previous films for comparison. Ask if there are previous CXRs available for comparison.
• Technical quality (RIP). The film is not rotated. There is adequate inspiratory effort and
penetration.

Describe the abnormality


• This is not immediately obvious so follow your systematic approach.

Systematic review
• Airway. The trachea is central. No evidence of free air in the soft tissue of the neck.
• Breathing:
• Right side. The apex is clear, the lateral thoracic border has no abnormality, the
costophrenic and cardiophrenic angles are visible, and the hilum is of normal size and is
positioned slightly lower than the left.
• Left side. The apex is clear, the lateral thoracic border has no abnormality, the
costophrenic and cardiophrenic angles are visible, and the hilum is of normal size and is
positioned slightly higher than the right.
• Circulation. The heart size is normal, the AA is visible, the left and right heart borders are
normal, and there is no mediastinal shift.

26

K30031_Book.indb 26 9/6/17 1:34 PM


Example OSCE stations with CXR interpretation

Fig. 3.7  Case 1: CXR, patient in the ED.

• Diaphragm. The hemidiaphragms are dome shaped, with the right slightly higher than the
left. There is no visible free gas under the diaphragm. The normal gastric bubble is seen on
the left.
• ‘Everything else’. There are no lines or tubes projected on the film. The breast contours are
present and symmetrical, and there is no evidence of previous surgery. There are no visible
fractures or bone lesions.

Summary
• This is a chest radiograph of a 45-year-old female who presents with dyspnoea and sharp
chest pain. The CXR is normal with no pathology identified. Additional investigations
would be advised to establish the nature of her presenting symptoms.

ALTERNATIVE ‘RADIOLOGIST-REVIEW’ APPROACH (WARD ROUND


EXAMPLE WITH AVAILABLE PATIENT DEMOGRAPHICS)
• Type of film and projection:
‘This is a PA chest radiograph of ...’
• Name:
‘Mrs Jane Smith …’

27

K30031_Book.indb 27 9/6/17 1:34 PM


3  The normal chest X-ray

• Hospital number:
‘Hospital number 123456789 ...’
• Date of birth:
‘Date of birth 22nd February 19XX …’
• Date:
‘Taken on 12th January 20XX …’
• Technical quality:
‘It is not rotated, and there is satisfactory inspiratory effort and penetration…’
• Lines, tubes, breast shadows:
‘There are no visible lines or tubes, and the breast shadows are symmetrical…’
• Check old films:
‘I would like to compare this image with any previous images to identify any changes and
to review previous history …’
• Heart, then lungs, then bones:
‘The heart is normal in size, both the left and right heart borders are normal, and both hila
are normal with the right hilum sitting slightly lower than the left. Both right and left lung
apices, upper, middle and lower zones are clear. There are no visible bone fractures or focal
bone lesions …’
• Review areas:
‘The peripheral soft tissues appear normal, both right and left costophrenic and
cardiophrenic angles are fully visible and clear, there is no evidence of left lower lobe
collapse or mass behind the heart. Both hemidiaphragms are dome shaped with the right
slightly higher than the left. No free subdiaphragmatic gas …’
• Summary:
‘In summary, this is a normal chest film with no gross pathology. If pulmonary embolus is
suspected urgent CT pulmonary angiogram would be recommended.’

CASE 2
A 64-year-old male was referred to the respiratory department with a 1-year history of progres-
sive dyspnoea and unintentional weight loss of 12 kg. He has no past medical history but is a
smoker with 35 pack-years. His respiratory rate is 16 bpm, oxygen saturation 98% on air, heart
rate 85 bpm, and BP 135/78 mmHg. A CXR is performed and you are asked to interpret it system-
atically (Figure 3.8).

Introduction (PPP RIP)


• Projection. PA erect chest radiograph (by default as the image is not labelled).
• Personal demographics. Offer to check patient’s details as the film is anonymised.
• Previous films for comparison. Ask if there are previous CXRs available for comparison.
• Technical quality (RIP). The patient is mildly rotated. There is adequate inspiratory effort and
penetration.

Describe the abnormality


• Where. There is a rounded lesion in the left lung located in the upper zone, projected
partially over the clavicle.

28

K30031_Book.indb 28 9/6/17 1:34 PM


Example OSCE stations with CXR interpretation

Fig. 3.8  Case 2: CXR, patient in the respiratory department.

• What. The lesion is round and well demarcated. It is soft tissue density, homogeneous and
noncavitating. There is no consolidation around the lesion and no other lesions are visible.
There is no calcification within the lesion.

Systematic review
• Airway. The distal trachea is not significantly deviated allowing for patient rotation.
• Breathing:
• Right side. The apex is clear, the lateral border has no abnormality, the costophrenic and
cardiophrenic angles are visible, the hilum is of normal size and is positioned slightly
lower than the left.
• Left side. The apex is clear, the lateral border has no abnormality, the costophrenic and
cardiophrenic angles are visible, the hilum is of normal size and is positioned slightly
higher than the right. The left upper zone mass is the only lung abnormality.
• Circulation. The heart size is normal, the AA is visible and normal, the left and right heart
borders are normal, and there is no mediastinal shift.
• Diaphragm. The hemidiaphragms are dome shaped, with the right slightly higher than the left.
There is no visible free gas under the diaphragm. The normal gastric bubble is seen on the left.
• ‘Everything-else’. There are no lines or tubes projected on the film. There is no evidence of
previous surgery. There are no visible fractures or bone lesions.

29

K30031_Book.indb 29 9/6/17 1:34 PM


3  The normal chest X-ray

Summary
• This is a chest radiograph of a 64-year-old male who presented with dyspnoea and
unintentional weight loss for over 1 year. There is a soft tissue, rounded, and well-
demarcated mass in the left upper zone with no associated consolidation. The appearance
is suspicious for lung malignancy. Further investigation with CT of the chest and
abdomen is recommended along with urgent thoracic multidisciplinary team (MDT)
review.

ALTERNATIVE ‘RADIOLOGIST-REVIEW’ APPROACH


• Type of film and projection:
‘This is a PA chest radiograph of ...’
• Name:
‘Mr John Smith...’
• Hospital number:
‘Hospital number 123456789 ...’
• Date of birth:
‘Date of birth 22nd February 19XX …’
• Date:
‘Taken on 12th January 20XX ...’
• Technical quality:
‘It is not rotated, and there is satisfactory inspiratory effort and penetration…’
• Lines, tubes, breast shadows:
‘There are no visible lines or tubes …’
• Check old films:
‘I would like to compare this image with any previous images to identify any changes and
to review previous history …’
• Heart, lungs, bones:
‘The heart is of normal size, both the left and right heart borders are normal, and both hila
are normal. There is a solitary round, well-demarcated, soft tissue, homogeneous lesion
in the left upper zone. It is partially projected over the left clavicle. There is no visible
cavitation. The right upper zone is clear, both right and left lung apices, middle and lower
zones are clear. There are no visible bone fractures or focal bone lesions.’
• Review areas:
‘The peripheral soft tissues appear normal, both right and left costophrenic and
cardiophrenic angles are fully visible and clear, there is no evidence of left lower lobe
collapse or mass behind the heart. Both hemidiaphragms are dome shaped with the right
slightly higher than the left. No free subdiaphragmatic gas …’
• Summary:
‘In summary, there is a round, well-demarcated, noncavitating, homogeneous lesion in the
left upper zone, lung malignancy to be excluded. Further investigation with CT of the chest
and abdomen is recommended along with urgent MDT review. ’

30

K30031_Book.indb 30 9/6/17 1:34 PM


Example OSCE stations with CXR interpretation

CASE 3
An 84-year-old female presented to the ED with sudden-onset right-sided chest pain and
­dyspnoea. She has a past medical history of asthma, for which she uses only a salbutamol inhaler,
and bilateral shoulder surgery joint replacement for rheumatoid arthritis. Her respiratory rate is
26 bpm, oxygen saturation 85% on air, heart rate 112 bpm, and blood pressure 135/78 mmHg. As
part of her work-up, a CXR is taken and you are asked to interpret it systematically (Figure 3.9).

Introduction (PPP RIP)


• Projection. PA erect chest radiograph (by default as the image is not labelled).
• Personal demographics. Offer to check patient’s details as the film is anonymised.
• Previous films for comparison. Ask if there are previous CXRs available for comparison.
• Technical quality (RIP). The patient is rotated. There is adequate inspiratory effort and
penetration.

Fig. 3.9  Case 3: Erect CXR, elderly female in the ED.

31

K30031_Book.indb 31 9/6/17 1:34 PM


3  The normal chest X-ray

Describe the abnormality


• What and where. The patient is markedly rotated with a pronounced thoracic scoliosis
concave to the right. There is tracheal and mediastinal shift to the right, associated with
complete opacification (homogeneous soft tissue shadowing /’whiteout’) of the right hemi-
thorax. Also on the right there is rib space narrowing in keeping with volume loss.

Systematic review
• Airway. The trachea is deviated to the right. Also there is apparent amputation seen at the
origin of the right main bronchus suspicious for bronchial occlusion.
• Breathing. Aerated right lung is not visible, there is complete opacification of the right hemi-
thorax, and the mediastinal structures are deviated to the right consistent with volume loss
in the right hemithorax. The left apex, upper, middle and lower lung zones, the costo-
phrenic and cardiophrenic angles, and the hilum are normal.
• Circulation. Heart size cannot be assessed as the heart borders are not fully visible. There is
mediastinal shift to the right. The AA and right heart border are not visible.
• Diaphragm. The right hemidiaphragm is not visible due to the underlying abnormality.
The left hemidiaphragm is normal. Normal air-filled bowel loops are seen beneath the left
hemidiaphragm. No free intraperitoneal air.
• ‘Everything else’. There are no lines or tubes projected on the film. The breast contours are
symmetrical. There are no visible fractures or bone lesions. There is evidence of bilateral
shoulder joint replacement.

Summary
• This is a chest radiograph of an 84-year-old female who attended the ED with a sudden
onset right-sided chest pain and dyspnoea. The CXR shows severe tracheal deviation and
mediastinal shift to the right, and complete opacification of the right hemithorax. There is
likely right bronchial occlusion. The appearances are consistent with right lung collapse. A
possible cause for this is bronchogenic malignancy. An urgent respiratory team review is
advised regarding further investigation and likely bronchoscopy.

ALTERNATIVE ‘RADIOLOGIST-REVIEW’ APPROACH


• Type of film and projection:
‘This is a PA chest radiograph of …’
• Name:
‘Mrs Jane Smith …’
• Hospital number:
‘Hospital number 123456789 ...’
• Date of birth:
‘Date of birth 22nd February 19XX …’
• Date:
‘Taken on 12th January 20XX.’
• Technical quality:
‘The patient is markedly rotated with a thoracic scoliosis concave to the right, with good
inspiratory effort and adequate penetration.’

32

K30031_Book.indb 32 9/6/17 1:34 PM


Example OSCE stations with CXR interpretation

• Lines, tubes, breast shadows:


‘There are no visible lines or tubes, and the breast shadows are present …’
• Check old films:
‘I would like to compare this image with any previous images to identify any changes and
to review previous history …’
• Heart, lungs, bones:
‘There is a significant tracheal deviation to the right with right mediastinal shift and a
complete loss of normal aerated right lung. There is likely occlusion at the origin of the right
main bronchus. There is right rib space narrowing. The heart size cannot be assessed as the
right heart border is obliterated by the abnormality. The left lung appears normal. There are
no bony lesions or fractures.’
• Review areas:
‘The peripheral soft tissues appear normal, the left costophrenic and cardiophrenic angles
are fully visible and clear, there is no evidence of left lobar collapse or mass behind the
heart. The left hemidiaphragm is dome shaped with some visible air in the bowel loops
below.’
• Summary:
‘In summary, there is a severe right tracheal deviation with mediastinal shift and complete
collapse of the right lung. A possible cause for this is a central obstructing lung malignancy
and the patient should be reviewed urgently by the respiratory team with a view to further
investigation and bronchoscopy.’

Note: This is an instructive case and should be compared with other causes of hemithoracic
complete opacification later in the book. With lung volume loss as a cause, the mediastinum will
move towards the affected lung. If a large pleural effusion is the aetiology this will have mass
effect and push the mediastinum away from the affected side. Consolidated lung may opacify the
hemithorax but will usually not displace the mediastinum, unless there is associated effusion or
collapse. US will readily differentiate effusion from collapse.

33

K30031_Book.indb 33 9/6/17 1:34 PM


K30031_Book.indb 2 9/6/17 1:34 PM
The normal abdominal X-ray

4 SEAN MITCHELL

Contraindications 35 AXR interpretation and presentation 37


Limitations 35 Exam technique 44
Radiation exposure 36 How to present your findings 45
Technique 36

The AXR is still commonly undertaken in the acute abdomen where perforation or acute bowel
obstruction is suspected, particularly in the elderly where acute abdominal signs and symptoms
are often nonspecific. You will also find that along with the CXR the AXR is one of the more com-
mon images that you will be expected to be able to interpret during your foundation years and as
part of your final year OSCE/written examinations.
Example indications for AXR include:
• Abdominal pain where there is a suspicion of perforation or obstruction.
• Renal calculi – however, CT KUB is becoming the imaging technique of choice.
• Inflammatory bowel disease, particularly where toxic dilatation is suspected
(CT increasingly being used).
• Foreign body ingestion.
• Postoperatively where there is a suspicion of ileus.

CONTRAINDICATIONS
An AXR is relatively contraindicated in the pregnant patient. If you are unsure, or if the woman
is of childbearing age, a pregnancy test should be undertaken. If the patient is certain she is not
pregnant then it is best practice to ask her to sign a disclaimer before proceeding. If in doubt dis-
cuss with the radiologist as alternative imaging methods may be more appropriate.
AXR is not usually indicated for the investigation of gallstones [right upper quadrant (RUQ)
pain] or in diagnosing constipation.

LIMITATIONS
The AXR is far less sensitive than CT for identifying intraperitoneal free air, ischaemic bowel,
the retroperitoneum, abscesses, and calcifications – particularly gallstones where only 10% at

35

K30031_Book.indb 35 9/6/17 1:34 PM


4  The normal abdominal X-ray

best will be seen on AXR. However, it is worth remembering that CT exposes the patient to a far
higher dose of radiation.

RADIATION EXPOSURE
• ALARA (as low as reasonably achievable)– keep dose down!!

You should always consider the amount of radiation that the patient will be exposed to and try
to minimise this where practicable. The exact amount of exposure will vary between individuals,
dependent on body habitus and type of imaging being undertaken (Table 4.1).
Before requesting an AXR it is worth considering if a nonionising imaging modality (that uses
no ionising radiation) would be more beneficial such as US or MRI. However, you need to con-
sider the benefits and risks to the patient. For example, if a patient presents with suspected renal
colic an AXR would not be of some benefit, whereas a CT KUB, despite exposing the patient to a
higher dose of radiation, would lead to a more accurate and rapid diagnosis. In cases such as these
you need to be able to justify a higher radiation exposure, i.e. ensuring that the clinical benefit to
the patient outweighs the risk.

• Remember – if in doubt, discuss with a senior colleague or radiologist.

Table 4.1  Modality dose comparisons

Equivalent
Imaging Dose mSv in CXR
CXR 0.015 1
AXR 0.4 30
CT abdomen 5.6 370
CT abdomen and pelvis 6.7 450
See Chapter 3 for a full table of imaging modalities and associ-
ated doses.

TECHNIQUE
The AXR is taken AP with the patient in the supine position. This is often combined with an erect
CXR where perforation is suspected, especially in the elderly as the erect CXR is far more sensitive
for free intraperitoneal air than an AXR and perforated viscus is in the differential for many acute
abdominal presentations, especially in older patients.

TOP TIPS
The patient should ideally be sat upright for 5–10 minutes prior to the erect CXR being taken, to allow any
free air to rise and collect under the diaphragm where it can be visualised. Check with the radiographer
that this has occurred if in doubt.

36

K30031_Book.indb 36 9/6/17 1:34 PM


AXR interpretation and presentation

Erect or decubitus AXRs, although still mentioned in many textbooks, are no longer under-
taken in clinical practice. If the patient is unable to lie supine it is always worth discussing with
the radiologist beforehand, as they will be able to advise you on how to best further investigate.

AXR INTERPRETATION AND PRESENTATION


When interpreting an AXR remember that because of the varying densities of the tissues within
the abdominal cavity, the amount of anatomy that can be identified may vary from individual to
individual dependent on factors such as the amount of fat surrounding the abdominal contents
and gas within the digestive tract. Therefore, prior to interpreting the AXR consider the underly-
ing anatomy; a sound understanding of this and those aspects that can usefully be visualised will
aid in identifying any abnormalities (Figure 4.1).
When presenting a radiographic image, particularly in an OSCE, it is worth remembering that
time is tight and you will not have the luxury of being able to go through everything in too much
detail, so be succinct and briefly explain what you would normally do. Examiners want to see that
you have presented and reviewed radiology images before and may allow/encourage you to move
on to the image abnormalities once they see you have a good technique. Some medical schools base
the whole OSCE case around one XR – which may be normal – just to examine technique, although
this is unusual. Start by identifying the type of film and projection, making note of the orientation
markers (L, R). Then check the patient demographics and image details to ensure you have the
right patient with the right image on the right date at the right time. In an OSCE this information
will be anonymised, but you should still ensure that you mention these aspects. Having done this
you should then ensure the film is adequate before systematically interpreting the image.
An adequately exposed film is one that includes the entire abdomen from diaphragm to pubic
symphysis and the lateral abdominal walls. This is vital to be able to accurately interpret the
image. In an adequately penetrated film you should also be able to identify the vertebrae but they
should not be so exposed that everything is too overexposed (excessively black) or underexposed
(excessively white). Not all cases in exams will be perfect and do not go into too much detail on
this – any abnormalities on the XR should be readily apparent.
Having ensured that you have the correct film and that it is adequate for diagnostic purposes
you then need to have a systematic approach to identifying any clinical abnormalities that may
aid you in your diagnosis, for example:

• Gases.
• Masses.
• Stones.
• Bones.
• Artefacts.
Some people find it useful to check artefacts first (tubes, lines, surgical clips) as this can give a
clue as to the patient’s presentation. However, the important thing is to have a system that suits
you and to stick to it.
Learning to interpret an XR in a systematic way is not just a skill for OSCEs but one that will
stand you in good stead throughout your career. Although it may seem pedantic to go through all
these steps, it ensures the appropriate image is being studied and that nothing is missed, which can
be easy to do when under pressure on a busy ward or in a busy medical/surgical assessment unit.

37

K30031_Book.indb 37 9/6/17 1:34 PM


4  The normal abdominal X-ray

Figs. 4.1A, B  Nonlabelled


AXR (4.1A). Labelled AXR (4.1B)
showing normal abdominal
anatomy. Depending on body
habitus and bowel gas pattern,
you may be able to identify the
liver, spleen, kidneys, stomach,
small bowel, large bowel, and
bladder (not the ureters). In this
patient the tips of the right lobe
of liver and spleen can be seen
and the collapsed stomach.
The renal outlines are largely
obscured by faecal residue/
A
bowel gas.

GASES
It is worth starting with a systematic review of the gas-filled GI tract, as this is often the most
prominent anatomy visible. Start with a general overview and ask yourself if it looks normal or
abnormal. As a general rule, if you are not sure or if it looks borderline it is often normal, as the
most common sign of abdominal pathology is dilated bowel, which is usually obvious and if used
in an OSCE will stand out clearly on a computer screen.
Then identify the small and large bowel (Figure 4.2).
The small bowel can usually be identified by its central location and the valvulae conniventes
that form complete bands across the bowel, often described as ‘stacks of coins’.
The large bowel is usually located at the peripheries. As it contains a mixture of gas and fae-
ces the pattern will vary between individuals and over time. You should be able to identify the
ascending, transverse, descending, and sigmoid colon as well as the rectum.
If the bowel is very dilated (>7 cm) it is most likely to be large bowel as small bowel does not
get this large before perforating. Note the ‘3-6-9 rule’ for size (Table 4.2). This topic is covered in
more detail in later chapters.
The stomach lies across the midline towards the left upper quadrant (LUQ) and may be identi-
fied by an air–fluid level formed by gastric body fluid and air in the fundus. The gas-filled pylorus
and duodenum may also be visible.
The rectum lies at the distal end of the GI tract and may contain gas or faeces.

38

K30031_Book.indb 38 9/6/17 1:34 PM


AXR interpretation and presentation

B
Fig. 4.1B

Fig. 4.2  AXR showing normal


loops of small and large bowel.
Note the differing appearances
of valvulae conniventes (B) and
haustra (A).

39

K30031_Book.indb 39 9/6/17 1:35 PM


4  The normal abdominal X-ray

Table 4.2  AXR features of small and large bowel

Small bowel Large bowel


Position Central Peripheral
Features Valvulae conniventes Haustra
Size <3 cm <6 cm or <9 cm in the caecum
Content Liquid and air Solid faeces, liquid, and air

MASSES (SOLID VISCERA)


The solid organs (liver, spleen, kidneys) are not always identifiable due to overlying bowel gas but
may become more conspicuous if there is an abnormality (Figure 4.3).
The liver fills the RUQ and may have a smooth well-defined inferior border visible just below
the right costal margin. Extension of the liver tip below this level raises suspicion of hepatomeg-
aly. An exception to this is the presence of a Riedel’s lobe, which is a common normal anatomical
variant with inferior projection of the right lobe of liver, and this can project below the inferior
costal cartilage and simulate pathology. The normal gallbladder and pancreas are not visible.
The spleen tip may be seen in the LUQ, just superior to the left kidney, inferior to the 9th –11th ribs.
The kidneys are located between T12 and L2 with the right kidney 2–3 cm lower than the left
owing to the position of the liver, and clear renal outlines are often difficult to see.

Fig. 4.3  AXR with annotated


structures. Tip of Riedel’s lobe
right liver (A), expected outline of
right kidney (B), iliopsoas muscle
outlines (C), left paracolic fat
stripe (D), spleen tip (E), collapsed
stomach (F). Left kidney obscured
by faecal matter.

40

K30031_Book.indb 40 9/6/17 1:35 PM


AXR interpretation and presentation

STONES (CALCIFICATIONS)
Identify structures that can become calcified or contain stones, particularly the kidneys and
­u rinary tract. Remember that 90% of kidney stones are radiopaque owing to their high calcium
content. Conversely, only 10% of gallstones are radiopaque and, therefore, are much less likely
to be visible. Calcium deposition can also be seen within the liver, spleen, pancreas, and arterial
walls and it may, for example, delineate the wall of an AAA.
To identify urinary tract calcification, start by identifying the course of the urinary tract
(Figure 4.4). The kidneys are not always seen but the path of the ureters can be traced from the
level of the renal pelvis, down the lateral borders of the lumbar transverse processes, over the iliac
vessels and sacroiliac joint and into the bladder at the vesicoureteric junction. The normal bladder
is located low in the pelvis and has the density of soft tissue on AXR. It may or may not be visible
depending on the degree of distension. Renal calculi may be visible anywhere along this tract and
if suspected may warrant further investigation with CT.
The aorta, iliac, and femoral arteries may be visualised if calcified, suggesting atherosclerosis.
An incidental AAA may be identified and warrant further investigation with CT.
Phleboliths are small calcifications located within veins, and are commonly found within the
pelvic veins of women. They can easily be mistaken for distal ureteric calculi and may require CT
to help tell them apart. Typically, however, phleboliths have a relatively lucent centre and look a
bit like a tiny polo mint when seen ‘en face’ (Figure 4.5).

Fig. 4.4  AXR with bilateral


ureteric stents and a right-sided
nephrostomy tube (arrow A). The
right stent demonstrates the normal
position of the renal pelvis, ureter,
and bladder. Note the mid left
stent is medially deviated (B) by a
retroperitoneal soft tissue mass (C).

41

K30031_Book.indb 41 9/6/17 1:35 PM


4  The normal abdominal X-ray

Fig. 4.5  Cropped pelvic XR showing multiple phleboliths (A) with central lucency, projected over the
distended urinary bladder (B).

Gallstones may also be visible in the gallbladder if they are radiopaque. Pancreatic calcification
(chronic pancreatitis) may be visible on AXR, although CT is far more sensitive.

BONES, ASSOCIATED STRUCTURES, AND OTHER ARTEFACTS


Having reviewed the abdominal organs, you should move on to the bones, associated structures
and artefacts.
At the superior border of the film the lower ribs should be visible along with both hemidia-
phragms. The normal AXR should not have any air under the hemidiaphragms; however, remem-
ber that the normal AXR is taken with the patient supine and therefore, even in the case of a
perforation, you are unlikely to see air under the diaphragm, hence the need for an erect CXR if
you have any suspicion of a visceral perforation.
The lower thoracic and lumbar vertebrae should also be clearly visible; if not the film may be
underpenetrated. The outline of the lumbar vertebrae should be traced to assess for any abnor-
malities such as a fracture. While assessing the lumbar spine you may also be able to identify the
outline of the iliopsoas muscles (these may enlarge in cases of retroperitoneal tumour, abscess or
bleed).
To complete the examination of the bones you should check the pelvis, sacroiliac joints, sym-
physis pubis, and hip joints for any deformity.

42

K30031_Book.indb 42 9/6/17 1:35 PM


AXR interpretation and presentation

ARTEFACTS
Artefacts, such as surgical clips, drains, tubes, body piercings, and foreign bodies, may be vis-
ible on the AXR and provide useful clues about the patient’s history (Figures 4.6 and 4.7). For
example, surgical clips in the RUQ may represent previous cholecystectomy or, in the female
pelvis, may suggest previous sterilisation. In female patients you may also see a ring pessary or
an intrauterine coil.
Findings such as these may warrant further investigation into the patient’s past surgical
history.

• Remember, postsurgical adhesions represent the major cause of small bowel obstruction
(in over 90% of cases). A past surgical history is essential, as is examining the abdomen
carefully for scars.

Medical devices, such as ECG leads and oxygen tubing, may also be visible. In an OSCE you
should endeavour to point these out as they give a clue to the patient’s clinical condition.
Finally, take the opportunity to review content on the AXR that is unrelated to the abdo-
men, such as the lung bases (pneumonia, mass), the peripheral abdominal wall soft tissues,

Fig. 4.6  AXR showing


metal artefacts from
umbilical jewellery (A) and an
intrauterine coil (B).

43

K30031_Book.indb 43 9/6/17 1:35 PM


4  The normal abdominal X-ray

Fig. 4.7  AXR showing surgical clips (A) in the RUQ, likely to relate to previous cholecystectomy. There is
a further clip visible in the pelvis (B), which could relate to a migrated cholecystectomy clip or, possibly a
sterilisation clip.

and the groins. This may reveal findings such as hernias or consolidation, which can add to
the overall clinical picture.
Review areas:

• Ribs.
• Lung bases.
• Hernial orifices.
• Abdominal wall.

EXAM TECHNIQUE
When preparing for your OSCE exam you should consider the context in which the AXR is likely
to be presented. The examiners will be looking for a systematic approach, a reasonable differential
diagnosis, and a management plan.
Cases in exams will be common and radiology findings will not be subtle/equivocal. Images
will have obvious pathology that can be clearly seen, usually on a computer screen. Imaging in

44

K30031_Book.indb 44 9/6/17 1:35 PM


How to present your findings

exams is usually presented on a personal computer screen (often a laptop) and will not be adjust-
able, while images are also increasingly being used in written papers. The imaging should be
anonymised but do not let this distract you into forgetting to mention the importance of noting
patient details as these are easy marks in an OSCE. You need to clearly state to the examiner right
from the outset that you would normally check the patient demographics. Radiology in OSCEs
usually features in the second half of the case where further discussion around the case occurs.
Occasionally a radiology image will form the basis of the whole case, and rarely a normal AXR
may be used for discussion.
Common OSCE AXR radiological presentations may include:

• Bowel perforation with free intraperitoneal air (more usually erect CXR utilised).
• Small or large bowel obstruction.
• Sigmoid volvulus.
• Foreign body (e.g. swallowed coin or migrated intrauterine coil).
• Inflammatory bowel disease with ‘thumbprinting’ or ‘toxic megacolon’.
• Pancreatic calcification (chronic pancreatitis).

Remember that although in real life we do not always know the answer, in the exam you need
to describe your findings with confidence. Likewise, in any exam you can still pass without get-
ting the correct diagnosis so long as you are confident and systematic.

TOP TIPS
▪▪ In an exam you will not have time to make a full presentation but will need to indicate how you
would normally approach an AXR. Therefore, demonstrate a systematic approach.
▪▪ Ask to review previous imaging – always compare with previous imaging if available.
▪▪ Normal/abnormal – clearly state whether the bowel gas pattern is normal or abnormal.
▪▪ Offer the examiner an initial management plan to include bedside tests and bloods.
▪▪ Escalation – say who you would discuss this with and why. Remember that if you are unsure it is
never wrong to seek advice.
▪▪ If you suspect bowel perforation you should review an erect CXR.
▪▪ If undertaking an erect CXR the patient must be allowed to sit upright beforehand to allow any
free air to rise up below the diaphragm.
▪▪ If you are considering requesting an AXR for a diagnosis other than bowel perforation or
obstruction, consider whether another imaging modality may be more appropriate.

HOW TO PRESENT YOUR FINDINGS


• Practice ... practice ... practice.

This will build confidence. You are not expected to know everything so, if you do not know the
answer, say you do not know but suggest how you might find out or who you would seek advice
from – be safe. Use the example cases below to get started (Figures 4.8–4.10). Consider the dif-
ferent ways you might structure your presentation along the lines described in Chapter 2.

45

K30031_Book.indb 45 9/6/17 1:35 PM


4  The normal abdominal X-ray

CASE 1

Fig. 4.8  Case 1: AXR, young


female with abdominal pain.

• Type of film and projection:


‘This is a supine abdominal radiograph of …’
• Name:
‘Miss Jo Smith …’
• Hospital number:
‘Hospital number 123456789 …’
• Date of birth:
‘Date of birth 22nd February 1992 …’
• Date:
‘Taken on 12th January 2016 …’
• Exposure + rotation:
‘It is adequately exposed …’
• Penetration:
‘There is adequate penetration…’

46

K30031_Book.indb 46 9/6/17 1:35 PM


How to present your findings

• Gases, masses, stones, bones:


‘The bowel gas pattern is within normal limits. No dilated loops of small or large bowel are
seen. No intraperitoneal free gas. No organomegaly. No abnormal calcifications. The bones
appear normal. The review areas are clear.’
• Check old films:
‘I would like to compare this image with any previous images to identify any changes and
to review the previous history.’
• Summary and plan:
‘In summary, this is a normal abdominal radiograph with no gross pathology. I would
like to review any blood tests, bedside observations, and urinalysis, and correlate with the
clinical presentation.’

CASE 2

Fig. 4.9  Case 2: AXR


of an elderly female with
increasing abdominal pain
and distension.

• Type of film and projection:


‘This is a supine abdominal radiograph of …’
• Name:
‘Mrs Edwina Bloggs …’

47

K30031_Book.indb 47 9/6/17 1:35 PM


4  The normal abdominal X-ray

• Hospital number:
‘Hospital number 123456789 …’
• Date of birth:
‘Date of birth 17th February 1949 …’
• Date:
‘Taken on 2nd August 2015 …’
• Exposure + rotation:
‘It is adequately exposed and not rotated. The upper abdomen and right lateral border of the
abdomen and iliac crest are not included in the field of view as the image is not central …’
• Penetration:
‘There is adequate penetration …’
• Gases, masses, stones, bones:
‘There is marked dilatation of several centrally located loops of bowel with valvulae conniventes
evident, in keeping with small bowel. The large bowel is not clearly visible. The liver, spleen,
and kidneys are not visible. There are no obvious calcifications within the renal tract, or
visible phleboliths. The lumbar spine is scoliotic, concave to the left, with degenerative change
between L4 and S1. There is the tip of a urinary catheter present, no surgical clips are seen.’
• Check old films:
‘I would like to compare this image with any previous images to identify any changes and
to review previous history and clinical presentation.’
• Summary and plan:
‘In summary, this is an abnormal abdominal radiograph with signs of small bowel
dilatation. The appearances would be consistent with small bowel obstruction, although
ileus could have a similar radiological appearance. Correlation with clinical findings
and previous surgical history is essential. She will also need an erect CXR to exclude
perforation. This patient will need to be admitted have an NG tube inserted and started on
IV fluids. I would like to review any blood tests, bedside observations and urinalysis, and
then refer her to the surgical team for further management.’

CASE 3
• Type of film and projection:
‘This is a supine abdominal radiograph of …’
• Name:
‘Mrs Alice Jones …’
• Hospital number:
‘Hospital number 123456789 …’
• Date of birth:
‘Date of birth 15th March 1948 …’
• Date:
‘Taken on 30th January 2016 …’
• Exposure + rotation:
‘The upper abdomen is not fully included in the field of view, otherwise the coverage and
exposure are adequate. The radiograph is not rotated.’

48

K30031_Book.indb 48 9/6/17 1:35 PM


How to present your findings

Fig. 4.10  Case 3: elderly


female with abdominal pain,
distension, and weight loss.

• Penetration:
‘There is adequate penetration …’
• Gases, masses, stones, bones:
‘The bowel gas pattern is within normal limits. No dilated loops of small or large bowel
are seen. There is abnormal punctate calcification within the pelvis, which likely relates
to uterine fibroid calcification. There is pronounced smooth enlargement of the liver and
spleen, no calcification is seen. The bones are normal. In the left upper quadrant I can see
two ports suggestive of a PIC line.’
• Check old films:
‘I would like to compare this image with any previous images to identify any changes and
to review previous history and the clinical presentation.’
• Summary and plan:
‘In summary, this is an abnormal abdominal radiograph with hepatosplenomegaly in a
patient with a likely underlying chronic disease as indicated by the PIC line. This suggests
a myeloproliferative disorder such as chronic myeloid leukaemia or myelofibrosis. There is
calcification within the pelvis that is likely to be longstanding. I would like to undertake a
full physical examination including a set of bedside observations and urinalysis. I would
also like to take a set of bloods to include FBC, U&Es, and LFTs. The patient is likely to
need a CT scan of chest, abdomen, and pelvis, and I would refer her to haematology as she
may already be known to them.’

49

K30031_Book.indb 49 9/6/17 1:35 PM


K30031_Book.indb 2 9/6/17 1:34 PM
Thoracic cases

5 HANNAH ADAMS, SARAH HANCOX,


CRISTINA RUSCANU, AND
DAVID C HOWLETT

Case 5.1 52 Case 5.14 115


Case 5.2 58 Case 5.15 119
Case 5.3 62 Case 5.16 124
Case 5.4 67 Case 5.17 130
Case 5.5 71
Case 5.18 135
Case 5.6 76
Case 5.19 141
Case 5.7 81
Case 5.20 145
Case 5.8 86
Case 5.21 150
Case 5.9 91
Case 5.10 95 Case 5.22 155
Case 5.11 100 Case 5.23 159
Case 5.12 105 Case 5.24 164
Case 5.13 109 Case 5.25 169

51

K30031_Book.indb 51 9/6/17 1:35 PM


Case 5.1

Fig. 5.1A  PA CXR.

52

K30031_Book.indb 52 9/6/17 1:35 PM


Case 5.1: Questions

A 69-year-old male presents to his GP with an ongoing cough for 8 weeks. He also reports feeling
tired all the time and has noticed some weight loss, which he associates with his poor appetite.
He gave up smoking 5 years ago after undergoing a coronary artery bypass graft (CABG). Prior to
this he smoked 20 cigarettes per day for 40 years.
On examination he has normal vital observations. A 1 cm hard supraclavicular lymph node is
palpated on the left and clubbing of his nails is noted. Cardiorespiratory examination is normal
with no added sounds on auscultation.
The GP arranges for an urgent CXR (Figure 5.1A). Routine blood tests are also requested,
which show:

FBC Normal Urea 2.4 mmol/L (1.7–8.3 mmol/L)


Sodium 120 mmol/L (135–146 mmol/L) Creatinine 74 micromol/L
(62–106 micromol/L)
Potassium 3.1 mmol/L (3.2–5.1 mmol/L) LFTs Normal

CASE 5.1: QUESTIONS


1 What does the CXR show?
2 What is the most likely diagnosis?
3 What is the likely explanation for this patient’s hyponatraemia?
4 What further investigations should be arranged?
5 What are the treatment options for this condition?

53

K30031_Book.indb 53 9/6/17 1:35 PM


5  Thoracic cases

CASE 5.1: ANSWERS


1 What does the CXR show?
The PA view CXR shows a spiculated opacity in the left hilum (M) (Figure 5.1B). The mass is
clearly separate from the aortic knuckle (A). This is an example of the silhouette sign; the left
pulmonary artery/hilar structures are obscured, confirming that the origin of the hilar density is
at the hilum rather than in front or behind the hilum when the hilar outline would then be clearly
seen. The mass is also separate from the aortic knuckle, which can be clearly seen. Sternotomy
wires confirm the history of previous cardiothoracic surgery.

Fig. 5.1B  CXR showing a spiculated mass in the left hilum (M), separate from the aortic knuckle (A). Note:
obscuration of normal left hilar structures and also sternotomy clips.

2 What is the most likely diagnosis?


The most likely diagnosis is primary lung carcinoma. The combined features of persistent cough,
weight loss, lymphadenopathy, and appearance on CXR are highly suggestive of malignancy.
Bronchogenic cancer accounts for 95% of primary lung tumours and is now the commonest can-
cer in the UK. It has an extremely poor prognosis with 25% of patients surviving 1 year and only
7% surviving 5 years.

54

K30031_Book.indb 54 9/6/17 1:35 PM


Case 5.1: Answers

Risk factors for developing lung cancer can be divided into modifiable and nonmodifiable:

• Modifiable.
• Smoking.
• Asbestos exposure.
• Air pollution.
• Ionising radiation (radon gas).
• Nonmodifiable.
• Family history/genetic factors.
• Male gender.
There are four main histological types of lung cancer (Table 5.1):

• Small cell carcinoma.


• Non-small cell carcinoma.
• Squamous cell carcinoma.
• Adenocarcinoma (previously bronchioalveolar carcinoma).
• Large cell carcinoma.

Table 5.1  Lung cancer histological types and features

Small cell Squamous cell Large cell


carcinoma carcinoma Adenocarcinoma carcinoma
Incidence (%) 15 40 30 15
Tissue type Endocrine Squamous Glandular Undifferentiated
(Kulchitsky cells) epithelium (goblet cells/clara cells) cells
Location Hilar Hilar Peripheral Peripheral/central
Treatment Chemotherapy Surgery Surgery Surgery
Metastatic spread Early Late Intermediate Early

3 What is the likely explanation for this patient’s hyponatraemia?


The most likely cause for hyponatraemia in a patient with lung cancer is paraneoplastic syndrome.
These syndromes arise at distant sites from the primary tumour and result from the production
of hormones, antibodies, peptides or prostaglandins by the carcinoma. Syndrome of inappropri-
ate antidiuretic hormone (SIADH) secretion is often seen in patients with small cell carcinoma,
resulting in hyponatraemia, hypokalaemia, and hypouricaemia, confirmed with low serum and
raised urine osmolarity. Treatment consists of fluid restriction (<1.5 L/day) and prescription of a
vasopressin receptor antagonist.
This patient underwent investigation and was found to have SIADH secondary to his small
cell lung cancer.

4 What further investigations should be arranged?


Investigations required in the diagnosis and staging of lung cancer include:

• CT chest (Figure 5.1C) and abdomen with contrast.


• PET.

55

K30031_Book.indb 55 9/6/17 1:35 PM


5  Thoracic cases

Fig. 5.1C  Axial thoracic CT with contrast demonstrating a large mediastinal/hilar mass (M). Normal
ascending aorta (A), carina (C), descending aorta (D), left pulmonary artery (P), and superior vena cava (S).

• Bronchoscopy +/− biopsy or washings.


• Transthoracic fine needle aspiration biopsy (dependent on tumour location).
• Sampling of pleural fluid in the presence of a pleural effusion.

Staging of non-small cell lung cancer uses the tumour, nodes, metastasis (TNM) classification.
Small cell lung cancers are, however, described simply as being limited or extensive.

5 What are the treatment options for this condition?


There is a variation in treatment regimens dependent on the histological classification of the lung
cancer:

• Surgical intervention.
• Considered in non-small cell lung cancers.
• Majority of cases are inoperable at the time of presentation.
• Must undergo full staging investigations to ensure the tumour has not metastasised and
lung function tests to demonstrate sufficient respiratory reserve.
• Radiotherapy.
• Used for inoperable cancers.
• Radiation pneumonitis and fibrosis are recognised complications.
• Used for relief of symptoms such as superior vena cava obstruction and chest wall pain.
• Chemotherapy.
• Generally reserved for small cell lung cancer.
• Not a curative intervention, improves prognosis by months only.

56

K30031_Book.indb 56 9/6/17 1:35 PM


Case 5.1: Answers

• Palliation.
• Symptom control – transbronchial stenting.
• Opioid analgesia and laxatives.
• Steroids can be used to improve appetite.

LEARNING POINTS: LUNG CANCER


▪▪ Lung cancer is the most common cause of cancer deaths per annum.
▪▪ Cigarette smoking is the major risk factor for developing lung cancer.
▪▪ Small cell lung cancers are rapidly growing, aggressive, and have usually metastasised at the time
of presentation.
▪▪ Chemotherapy and radiotherapy improve quality of life when surgical resection is not a valid
treatment option.
▪▪ Paraneoplastic syndromes are most common with small cell lung cancer.

57

K30031_Book.indb 57 9/6/17 1:35 PM


Case 5.2

Fig. 5.2A  PA CXR.

58

K30031_Book.indb 58 9/6/17 1:35 PM


Case 5.2: Questions

A 76-year-old female presents to Respiratory Outpatients with a persistent dry cough, poor appe-
tite, and weight loss. She reports a 4–5-month history of worsening right shoulder pain radiating
into her axilla, described as an aching sensation from which she has little relief. She is a current
smoker and has smoked 10 cigarettes a day for 40 years.
On examination she has normal vital observations but appears cachectic. No lymphadenopa-
thy is present and examination of the shoulder joint is unremarkable. Respiratory examination
reveals equal expansion and air entry, normal percussion sounds, and normal breath sounds
throughout. A CXR has been performed (Figure 5.2A).

CASE 5.2: QUESTIONS


1 What radiological findings are demonstrated on this CXR?
2 What is the most likely diagnosis?
3 What is the syndrome often associated with this condition?
4 What further investigations should be undertaken?
5 What are the treatment options available?
6 What is ‘pack-year history’ and how is it calculated?

59

K30031_Book.indb 59 9/6/17 1:35 PM


5  Thoracic cases

CASE 5.2: ANSWERS


1 What radiological findings are demonstrated on this CXR?
The CXR demonstrates (Figure 5.2B):
• A right apical lung opacity within the superior sulcus (A).
• Destruction of the posterior aspect of the right second rib (expected course of rib marked
with dashed line).
• Presence of a permanent dual chamber pacemaker.

Fig. 5.2B  CXR demonstrating a large right apical opacity (A) with destruction of the posterior aspect of
the right second rib (dashed line shows expected position of the rib). The normal posterior aspect of the
left second rib is shown by the solid lines, first ribs arrowed (B).

2 What is the most likely diagnosis?


Right apical opacity with rib destruction is indicative of a Pancoast (superior sulcus) tumour. The
opacity alone could be a pleural tumour (fibroma, lipoma, metastasis), a neural tumour such as
neurofibroma, possibly TB or fungus ball (mycetoma), or an aneurysm.
3 What is the syndrome often associated with this condition?
The history, examination, and CXR are suggestive of a Pancoast tumour. This is the description
given to a malignant neoplasm at the extreme lung apex; involvement of the brachial plexus and
sympathetic chain by the tumour results in Pancoast syndrome. This syndrome is represented by:
• Shoulder pain (chest wall invasion).
• Ipsilateral C8–T2 radicular pain.
• Horner’s syndrome resulting from infiltration of the tumour into the thorax involving the
sympathetic chain causing ptosis, miosis, enophthalmos, and anhydrosis.
60

K30031_Book.indb 60 9/6/17 1:35 PM


Case 5.2: Answers

• Only 25% of patients with Pancoast tumour are diagnosed with Pancoast syndrome as the
majority do not exhibit complete signs of Horner’s syndrome.
Patients report pain in the shoulder and axilla due to erosion of the ribs. Anhydrosis occurs
secondary to involvement of the stellate ganglion, a collection of sympathetic nerves anterior to
the sixth and seventh cervical vertebrae. Invasion of the tumour into the brachial plexus results
in wasting of the small muscles of the hand and neuropathic pain, while invasion of the vertebral
foramen can lead to spinal cord compression, a medical emergency.
There is often a delayed diagnosis of Pancoast syndrome as the apical lung cancer can be slow
growing and insidious, and may not be visualised on initial CXRs. Once the patient presents with
worsening symptoms the cancer has often already invaded nearby structures. Delayed diagnosis
can also occur due to prominence of musculoskeletal symptoms, which are incorrectly inter-
preted, such as shoulder symptoms and radicular pain.
4 What further investigations should be undertaken?
CT provides vital information on the extent and spread of the suspected lesion, and can be used to
plan the most appropriate approach to obtain a sample for histological diagnosis.
The majority of Pancoast tumours are squamous cell carcinomas but samples should be
obtained for diagnostic purposes. Adenocarcinomas may also occur in old areas of scarring
­( history of TB). Sputum samples may be beneficial in patients who cannot tolerate invasive inves-
tigations. Bronchoscopy or CT-guided biopsy is used to obtain histology, as these are peripheral
lesions usually and more amenable to CT access.
MR scanning is particularly useful in providing multiplanar visualisation of the chest wall and
the brachial plexus.
PET scans can provide further detail regarding the extent of the disease and location of metas-
tases. This information can be used in a multidisciplinary meeting to determine the best course
of treatment for each patient.
5 What are the treatment options available?
Surgery is indicated in patients with early disease, which is only seen in a minority of patients.
Contraindications to surgery include metastatic spread, nodal mediastinal extension, and inva-
sion into the neck, brachial plexus, and vertebrae.
Medical management is suitable for most patients, providing a degree of palliation, such as
radiotherapy for relief of associated symptoms. Chemotherapy can be considered but is often
more appropriate in preoperative patients.
6 What is ‘pack-year history’ and how is it calculated?
Pack-year history is a measurement of a smoking history. It is calculated by multiplying the num-
ber of packs of 20 cigarettes smoked per day by the number of years the person has smoked. For
example, 20 cigarettes/day for 1 year = 1 pack-year history, or in the case of this patient 10 ciga-
rettes/day for 40 years = 20 pack-year history.

LEARNING POINTS: PANCOAST TUMOUR


▪▪ Pancoast tumour is a malignant neoplasm at the extreme lung apex.
▪▪ Pancoast syndrome is a triad of shoulder pain, C8–T2 radicular pain, and Horner’s syndrome.
▪▪ Investigations include CT chest and liver, sputum samples, bronchoscopy +/− biopsy, MRI, and
PET scan.
▪▪ Treatment is often limited to radiotherapy due to the late presentation.
▪▪ Pack-year history = number of cigarettes (day) × number of years.

61

K30031_Book.indb 61 9/6/17 1:35 PM


Case 5.3

Fig. 5.3A  PA CXR.

62

K30031_Book.indb 62 9/6/17 1:35 PM


Case 5.3: Questions

A 52-year-old male presents to the ED with a 3-week history of feeling generally unwell,
fatigue and coughing up purulent green sputum. He has lost his appetite but does not report
any weight loss. He has no fixed abode, drinks over 60 units of alcohol per week, and is a
nonsmoker.
On examination he has a temperature of 38.4°C, blood pressure 129/78 mmHg, pulse rate of
90 bpm, respiratory rate 20 bpm, and oxygen saturations 95% breathing room air. He looks gener-
ally unwell, pale, and clammy. Respiratory examination reveals some dullness to percussion and
decreased breath sounds in the right middle zone.
An urgent CXR is requested (Figure 5.3A) and bloods are performed:

Hb 132 g/L (130–180 g/L) U&Es Normal


WCC 17.8 × 10 /L (4.0–11.0 × 10 /L)
9 9 LFTs Normal
Neutrophils 15.2 × 109/L (2.0–7.5 × 109/L) CRP 246 mg/L (<5 mg/L)
Platelets 288 × 109/L (150–450 × 109/L)

CASE 5.3: QUESTIONS


1 What key radiological findings are demonstrated?
2 What is the most likely diagnosis?
3 What are the possible causes for this appearance?
4 What investigations and management are required?
5 What are the complications that may occur with this condition?

63

K30031_Book.indb 63 9/6/17 1:35 PM


5  Thoracic cases

CASE 5.3: ANSWERS


1 What key radiological findings are demonstrated?
There is a large ill-defined opacity in the right lung containing two lucent areas, with an air–fluid
level (Figure 5.3B).

Fig. 5.3B  Frontal CXR


with arrows delineating
air–fluid levels within a
large ill-defined area of
opacification in the right
middle zone.

2 What is the most likely diagnosis?


The most likely diagnosis is a cavitating abscess of the right lung (causative organisms are given
in Table 5.3). Pulmonary cavities are gas-filled areas within a mass or area of consolidation. They
are referred to as a lung abscess when they contain areas of infected parenchyma with necrosis
and suppuration, and are typically thick walled.

Table 5.3  Causative organisms

Anaerobes Aerobes Other


Bacteroides spp. Staphylococcus aureus Mycobacterial infection
Peptostreptococcus spp. Streptococcus pyogenes Fungus – Aspergillus spp.
Fusobacterium spp. Pseudomonas auriginosa Parasites – Entamoeba spp.
Klebsiella pneumoniae

64

K30031_Book.indb 64 9/6/17 1:35 PM


Case 5.3: Answers

Risk factors for lung abscesses include:

• Immunosuppression (human immunodeficiency virus, chemotherapy).


• Alcoholism and drug use.
• Lung disease (cystic fibrosis).
• Aspiration (cerebral palsy, cerebrovascular accident).

3 What are the possible causes for this appearance?


Possible causes for lung cavitation include (mnemonic CAVITY):

• Cancer (primary – squamous cell; secondary – head and neck, oesophagus).


• Autoimmune (Wegener’s granulomatosis, rheumatoid nodules).
• Vascular (septic pulmonary emboli).
• Infection (TB, abscess).
• Trauma (pneumatocoeles).
• Youth (bronchogenic cyst).

4 What investigations and management are required?


Management follows an initial ABCDE assessment and stabilisation of the patient. Blood cul-
tures and sputum samples provide crucial information about correct antibiotic therapy. Further
imaging with CT will help to determine the exact cause of the cavity prior to starting treatment
(Figure 5.3C). CT-guided aspiration and drainage may also be required to obtain a sample of the
organism and relieve the collection.

Fig. 5.3C  Axial postcontrast CT section through right posterior lung lesion demonstrating a large thick-
walled mass in the right lower lobe (arrows), with necrotic fluid centre (A) and areas of internal cavitation (B).

65

K30031_Book.indb 65 9/6/17 1:35 PM


5  Thoracic cases

Prompt broad-spectrum IV antibiotic therapy to include anaerobe cover is recommended in


the first instance.
Resolution is normally rapid following the commencement of suitable antibiotics. Patients
must be followed up with CT scans to ensure complete resolution and that there is no underlying
malignancy.

5 What are the complications that may occur with this condition?
With the correct treatment, lung abscesses should heal completely to leave a small fibrous scar.
However, complications can occur such as:

• Empyema (needing CT/surgical drainage).


• Haemorrhage (caused by erosion into nearby vessels).
• Septicaemia.
• Pyopneumothorax.

LEARNING POINTS: CAVITATING LUNG ABSCESS


▪▪ Pulmonary cavities are gas-filled areas within a mass or area of consolidation.
▪▪ Mnemonic ‘CAVITY’ can be used to remember the causes of cavitating lesions.
▪▪ Immediate broad-spectrum IV antibiotic therapy to include anaerobes is essential.
▪▪ Follow-up using CT is an indicator of treatment progress.
▪▪ Complications include empyema and haemorrhage.

66

K30031_Book.indb 66 9/6/17 1:35 PM


Case 5.4

Fig. 5.4A  PA CXR.

A 59-year-old male presents to his GP with a 3-week history of persistent painless macroscopic
haematuria. Prior to this he thinks he lost approximately 10 kg in weight over 5– 6 months. His
wife reports she has noticed he is recently short of breath when he climbs the stairs.
On examination he is apyrexial with a blood pressure of 134/85 mmHg, heart rate 78 bpm,
respiratory rate of 16 bpm, and oxygen saturations 96% breathing room air. Examination reveals
a soft abdomen with some tenderness in the left loin but no palpable mass. Auscultation of his
chest is normal.
Urine dipstick confirms 3+ blood but nil else. A CXR is requested for his shortness of breath
(Figure 5.4A) and he is referred urgently to the haematuria clinic at the local hospital.

CASE 5.4: QUESTIONS


1 What radiological findings are demonstrated?
2 What is the most likely diagnosis?
3 Is there a differential for this appearance?
4 How should this patient be investigated?

67

K30031_Book.indb 67 9/6/17 1:35 PM


5  Thoracic cases

CASE 5.4: ANSWERS


1 What radiological findings are demonstrated?
Radiological features include (Figure 5.4B):
• Multiple large, bilateral, well-circumscribed round pulmonary lesions.
• No calcification in lesions, early cavitation in some lesions.
• No obvious pleural effusion.
• No bone lesions.
• No mediastinal lymphadenopathy.
• Heart size normal.

Fig. 5.4B  CXR showing multiple large, bilateral, well-circumscribed round pulmonary lesions (arrows).
Note small areas of lucency in some lesions consistent with early cavitation (A).

2 What is the most likely diagnosis?


The most likely diagnosis based on the patient’s history is renal (or urinary tract) carcinoma with
pulmonary ‘cannonball’ metastases. Metastases spread to the lung via the following routes:
• Haematogenous – tumours that have direct venous drainage to the lungs, such as breast,
kidney, melanoma, testes, and head and neck squamous carcinoma.
• Lymphatic – tumour cells deposited in mediastinal lymph nodes, which spread along the
lymphatics into the lung, such as stomach, colon, and breast.
• Direct invasion – e.g. malignant melanoma directly through the chest wall.
68

K30031_Book.indb 68 9/6/17 1:35 PM


Case 5.4: Answers

Pulmonary metastases are seen in up to 50% of extrathoracic malignancies, with the lungs
being the second most common site of metastatic spread. Pulmonary metastasis indicates a poor
prognosis as a result of the late stage of disease.
Appearances of lung metastases on CXR include (see Table 5.4):

• Cannonball appearance.
• Ill-defined ‘snowstorm’ appearance (Figure 5.4C).
• Solitary lung nodule (+/− cavity).
• Multiple lung nodules (+/− cavities, Figure 5.4D, page 70).
• Miliary nodularity (military = seed = fine and small, 1–3 mm).
• Lymphangitis carcinomatosa.

Table 5.4  Lung metastases appearance and tumour origin

Cannonball appearance Snowstorm appearance Solitary nodules


Kidney Kidney Kidney
Bladder Bladder Bowel
Uterus Breast Breast
Testes Prostate
Bowel Thyroid
Melanoma

Fig. 5.4C  A CXR in a patient with colon cancer demonstrating multiple bilateral ill-defined pulmonary nodules.
69

K30031_Book.indb 69 9/6/17 1:35 PM


5  Thoracic cases

Fig. 5.4D  CT thorax


(lung window settings)
image in a patient being
staged for testicular
cancer demonstrating
circumscribed lung
metastases (arrows).

Complications associated with pulmonary metastases include:


• Pleural effusion.
• Pericardial effusion.
• Lobar collapse.

3 Is there a differential for this appearance?


Differential diagnoses for multiple lung nodules:
• Metastases.
• Sarcoidosis.
• TB.
• Pulmonary lymphoma.
• Kaposi sarcoma.
• Rheumatoid nodules.
• Pulmonary/septic infarcts (tend to be ill-defined +/− cavitate).

4 How should this patient be investigated?


This patient should be investigated urgently in the haematuria clinic with the following
investigations:
• Bloods, including renal function, FBC, bone profile, LFTs.
• CT chest, abdomen, pelvis (Figure 5.4D).
• Cystoscopy.

LEARNING POINTS: PULMONARY METASTASES


▪▪ Metastases can spread via haematogenous and lymphatic routes or by direct invasion.
▪▪ Lung metastases can be solitary or multiple with a cannonball or snowstorm appearance.
▪▪ Common primary sites include breast, melanoma, GI tract, and kidney.
▪▪ Complications of lung metastases include pleural effusion and lobar collapse.

70

K30031_Book.indb 70 9/6/17 1:35 PM


Case 5.5

Fig. 5.5A  PA CXR.

A 64-year-old male presents to his GP with a 4-month history of shortness of breath and a dull
ache in the right side of his chest. He has not noticed any weight loss and reports having a good
appetite. He denies a smoking history. He has recently retired but prior to this worked as a self-
employed plumber for 45 years.
On examination his respiratory rate is 18 bpm and oxygen saturations are 93% breathing room
air. He has decreased lung expansion on the right side with dullness to percussion but no pain on
palpation of his chest wall. He does, however, have bibasal crackles on auscultation. An urgent
CXR is performed (Figure 5.5A).

CASE 5.5: QUESTIONS


1 What are the radiological findings?
2 What is the most likely diagnosis?
3 What occupational exposure might be to blame?

71

K30031_Book.indb 71 9/6/17 1:35 PM


5  Thoracic cases

CASE 5.5: ANSWERS


1 What are the radiological findings?
Radiological features include (Figure 5.5B):

• Decreased lung volume in the right hemithorax.


• Mediastinal shift to the right.
• Lobulated pleural thickening encasing the right hemithorax (arrows).
• Pleural opacity at the right costophrenic angle, indicating a pleural effusion or further
pleural thickening.
• Some increased reticular density in the right mid/lower zone, possibly relating to an
inflammatory process.

Fig. 5.5B  CXR showing increased reticular density in the right mid/lower zone with lobulated pleural
thickening (arrows) and right hemithoracic volume loss.

2 What is the most likely diagnosis?


Mesothelioma, in the context of his symptoms, occupational history, pleural plaques, and irregu-
lar pleural thickening on CXR. CT is the technique of choice for further assessment of the lungs
and pleura and in cases of suspected malignancy (Figure 5.5C).

72

K30031_Book.indb 72 9/6/17 1:35 PM


Case 5.5: Answers

Fig. 5.5C  An axial postcontrast CT section at the level of the left ventricle in another patient with
mesothelioma, demonstrating pleural thickening (A) around the right hemithorax, with pleural masses (B)
and enlarged subcarinal nodes (C). There is also extension of a large right pleural mass into the right chest
wall (D).

Asbestos-related pleural disease can be benign or malignant:

• Benign pleural disease.


• Pleural effusion: early sign of asbestos exposure (need to exclude underlying malignancy).
• Diffuse pleural thickening: owing to irritation of the pleura.
• Pleural plaques (calcified, noncalcified or hairy): following longstanding asbestos
exposure (>20 years).
• Rounded atelectasis: atypical, peripheral lung collapse adjacent to pleural plaque.
• Asbestosis-related pulmonary fibrosis (asbestosis, predilection for lower zones).
• Malignant pleural disease.
• Pleural mesothelioma: irregular pleural thickening owing to a primary tumour,
following asbestos exposure.
• Bronchogenic carcinoma: increased risk in smokers with history of asbestos exposure.

Pleural plaques (Figure 5.5D) are often reported as an incidental finding on CXR, and may
calcify. They are an indication of possible asbestos exposure and in themselves do not cause any
lung function impairment. Extensive pleural thickening, however, can lead to a restrictive picture
causing shortness of breath. A diagnosis of pleural plaques is important as it allows patients to
apply for compensation following exposure to asbestos.

73

K30031_Book.indb 73 9/6/17 1:35 PM


5  Thoracic cases

Fig. 5.5D  CXR in a different patient showing widespread bilateral and symmetric calcified pleural plaques
with a ‘holly leaf’ effect. Diaphragmatic (A) and mediastinal (B) calcified plaques are also shown.

Mesothelioma is a malignancy arising from the pleura and has a strong association with asbes-
tos exposure (90% of cases). The lag interval from exposure to diagnosis is approximately 30–40
years and, as precautionary measures were not introduced until the 1970s, it is predicted that the
prevalence of mesothelioma is likely to continue to rise for another 5–10 years.
Mesothelioma exhibits an aggressive disease progression (as indicated in Figure 5.5C), encas-
ing the lung, pericardium or peritoneum depending on the tumour location. The mass infiltrates
the chest wall, resulting in chest pain, despite there often being little or no bony destruction.
Mesothelioma regularly metastasises through haematogenous spread. Although treatment
options such as radical pneumonectomy, radiotherapy, and chemotherapy are available, the
response rate is generally poor and most patients die within 2 years of diagnosis.

3 What occupational exposure might be to blame?


The most hazardous occupational exposure is contact with asbestos. Asbestos is the collective
term given to a number of naturally occurring mineral silicates, which were previously used in
construction for their insulating and fire resistant properties. The two main categories are:

• Serpentine asbestos fibres (white asbestos). Long flexible fibres, which are more difficult
to inhale.

74

K30031_Book.indb 74 9/6/17 1:35 PM


Case 5.5: Answers

• Amphibole asbestos fibres (blue or brown asbestos). Short straight, brittle fibres, which
penetrate deeply into lung tissues on inhalation, these are considered more hazardous to
the body, and result in pleural disease.

Occupations associated with asbestos exposure include:

• Miners.
• Plumbers.
• Builders/demolition workers.
• Fire fighters.
• Dock workers.

In the 1970s, precautionary measures were introduced to limit the exposure to asbestos
through the use of personal protective equipment such as respiration masks. However, owing to
the long interval between asbestos exposure and development of disease, asbestos-related lung
conditions are still commonly diagnosed.

LEARNING POINTS: MESOTHELIOMA AND PLEURAL PLAQUES


▪▪ Mesothelioma is a malignancy of the pleura caused by inhalation of asbestos.
▪▪ Blue and brown asbestos have been identified as the most hazardous forms.
▪▪ A diagnosis of mesothelioma should be considered in patients presenting with pleural thickening
and chest discomfort.
▪▪ Mesothelioma has a poor prognosis.

75

K30031_Book.indb 75 9/6/17 1:35 PM


Case 5.6

Fig. 5.6A  PA CXR.

76

K30031_Book.indb 76 9/6/17 1:35 PM


Case 5.6: Questions

A 67-year-old male presents to his GP with ongoing shortness of breath on exertion for several
months. He does not report having a cough, ankle swelling or chest pain. His past medical history
is unremarkable and he does not take any regular medications. He is a retired farmer and denies
having a smoking history.
On examination he is apyrexial, BP 129/72 mmHg, pulse rate 67 bpm, respiratory rate 17 bpm,
and oxygen saturations 92% breathing room air. He has noticeable clubbing of his nails, jugular
venous pressure is within normal limits, and heart sounds are normal. Respiratory examination
reveals equal expansion of the chest wall, normal percussion, and bibasal end-inspiratory crepita-
tions. A CXR is performed (Figure 5.6A).

CASE 5.6: QUESTIONS


1 What are the key radiological findings?
2 What are the possible causes for this condition?
3 What further investigations would be helpful?
4 How should this patient be further managed?

77

K30031_Book.indb 77 9/6/17 1:35 PM


5  Thoracic cases

CASE 5.6: ANSWERS


1 What are the key radiological findings?
These include (Figures 5.6B and 5.6C):

• Irregular reticular density throughout both lung fields.


• ‘Shaggy’ ill-defined cardiac borders.
• Loss of costophrenic angles bilaterally.
• Decreased lung volumes.

Reticular = ‘net-like’, also called interstitial, and this increase in density is caused by pathology
that involves and thickens the lung interstitium (borders the air space). These appearances with
history and examination findings would be consistent with interstitial pulmonary fibrosis.

Fig. 5.6B  CXR demonstrates reduced volume lungs with diffuse reticular, interstitial increase in lung
density. Note the ‘shaggy’ irregular left heart border (A). There is additional pleural thickening in the right
hemithorax (B) with right basal pleural opacity (C), which may relate to thickening or effusion.

78

K30031_Book.indb 78 9/6/17 1:35 PM


Case 5.6: Answers

Fig. 5.6C 
A magnified
view of a section
of lung better
demonstrating the
interstitial, reticular
‘net-like’ pattern
of interstitial
fibrosis (see areas
highlighted).

2 What are the possible causes for this condition?


There are a large number of possible aetiologies for lung fibrosis, although often patients will
present with advanced disease and the cause is unknown (idiopathic). Some diseases may involve
certain parts of the lung.

Upper zones Lower zones


Bronchopulmonary aspergillosis Connective tissue disease
Radiation Asbestosis
Extrinsic allergic alveolitis Rheumatoid arthritis
Ankylosing spondylitis Drugs (methotrexate, amiodarone)
Sarcoidosis ASpiration
TB
Silicosis
Note: Mnemonic BREASTS. Note: Mnemonic CARDS.

Neurofibromatosis, histocytosis, and tuberose sclerosis can present with advanced disease,
the so-called ‘honeycomb’ lung.

3 What further investigations would be helpful?


When considering a diagnosis of pulmonary fibrosis, these should include:

• Blood profile: FBC, rheumatoid factor, antinuclear antibodies, and avian precipitins.
• LFTs: a restrictive pattern will be demonstrated.
• High-resolution CT: may indicate the presence of acute inflammation (‘ground-glass
density’), which may indicate a potential for response to steroid treatment. CT can also be
used to guide diagnostic lung biopsy in difficult cases (Figure 5.6D).
• Bronchoscopy with washings: lymphocytic picture is indicative of sarcoidosis.

79

K30031_Book.indb 79 9/6/17 1:35 PM


5  Thoracic cases

Fig. 5.6D  A high-resolution CT, lung window setting, in another patient, demonstrating subpleural
interstitial fibrotic changes in the left upper lobe (A) and posterior emphysematous/bullous changes (B).

4 How should this patient be further managed?


Pulmonary fibrosis is the overarching term given to a number of conditions that result in inflam-
mation and scarring of the alveoli, distal airways, and interstitium of the lung, thus causing
impaired gas diffusion and shortness of breath. It most commonly affects older men in their sev-
enth decade of life who have been exposed to environmental dusts such as metal and wood dust.
Idiopathic pulmonary fibrosis is the commonest subtype.
Treatment is dependent on the cause but steroids and other immunosuppressants, such as pir-
fenidone, can often be used to reduce inflammation and relieve symptoms. If the cause is a drug
reaction it is important to stop the offending drug. Pulmonary fibrosis patients have an increased
risk of developing lung cancer if they have a smoking background, and cessation advice should
be provided.

LEARNING POINTS: PULMONARY FIBROSIS


▪▪ Pulmonary fibrosis is the term given to inflammation and scarring of the alveoli, distal airways and
interstitium of the lung.
▪▪ Causes of upper zone pulmonary fibrosis can be remembered using the mnemonic ‘BREASTS’;
lower zone ‘CARDS’.
▪▪ Investigations include lung function tests and high-resolution CT.
▪▪ Idiopathic pulmonary fibrosis is the most common subtype.
▪▪ Steroids and immunosuppressive agents can be used to relieve symptoms.

80

K30031_Book.indb 80 9/6/17 1:35 PM


Case 5.7

Fig. 5.7A  PA CXR.

A 26-year-old male presents to the ED complaining of sudden onset of left-sided chest pain,
which is worse on deep inspiration and is associated with shortness of breath. He is usually fit and
well, and does not report a cough.
On examination he is not distressed but is mildly short of breath. He is apyrexial, has a BP of
108/76 mmHg, heart rate of 89 bpm, respiratory rate of 19 bpm, and oxygen saturations of 94%
breathing room air. He has a central trachea, reduced expansion, and increased resonance on
percussion of the left side. There is also reduced air entry on auscultation and reduced vocal reso-
nance on the left. A CXR has been performed portably in the department (Figure 5.7A).

CASE 5.7: QUESTIONS


1 What key radiological findings are demonstrated?
2 What is the likely diagnosis?
3 How should this patient be managed?
4 What information should this patient be provided with on discharge?

81

K30031_Book.indb 81 9/6/17 1:35 PM


5  Thoracic cases

CASE 5.7: ANSWERS


1 What key radiological findings are demonstrated?
Key findings are (Figure 5.7B):

• Large left-sided pneumothorax with a large air-filled pleural space and loss of lung
markings in the left hemithorax.
• Collapse of the left lung with a visible ‘pleural line’ (arrows).
• Mild mediastinal shift to the right suggesting the pneumothorax is under tension.

A pneumothorax is the presence of air in the pleural space. Air enters the pleural space through
a hole in the soft tissues and causes pressure to build in the intrapleural space, resulting in pro-
gressive collapse of the lung.
Young patients are often able to compensate for a large pneumothorax for a long period of time
before becoming unwell quickly, requiring urgent intervention.

Fig. 5.7B  CXR showing a left-sided pneumothorax; the margin of the collapsed left lung is shown
(arrows).

82

K30031_Book.indb 82 9/6/17 1:35 PM


Case 5.7: Answers

2 What is the likely diagnosis?


A spontaneous primary pneumothorax with an element of tension. Pneumothoraces are classi-
fied as:

• Spontaneous.
• Primary: in the absence of lung disease.
• Secondary: in the presence of lung disease, such as COPD or cystic fibrosis.
• Tension.
• Medical emergency.
• Air enters the pleural space during each inspiration but is unable to leave as the pleural
tear acts as a one-way valve.
• Intrapleural pressure increases causing the lung to collapse and venous return to be
impaired.
• Commonest causes include thoracic trauma (rib fractures, stab wounds) and positive
pressure ventilation.
• Iatrogenic: following central line insertion or biopsy.

3 How should this patient be managed?


The initial management of this patient should involve an ABCDE assessment to ensure resuscita-
tion and stabilisation of the patient with administration of oxygen.
Tension pneumothorax is a medical emergency, and patients should be assessed immediately.
If a tension pneumothorax is apparent clinically and the patient is showing signs of distress, they
should undergo aspiration of the pneumothorax under senior guidance, before proceeding to
chest drain insertion. Do not wait for a CXR in these cases: if unsure ask for an urgent portable
radiograph in A/E so the patient can be observed.
Following these initial steps the management should be according to local/national guidelines:
• Primary pneumothorax.
• >2 cm* and/or respiratory distress → aspirate with large bore cannula and review.
• <2 cm and/or no respiratory distress → no intervention, consider discharge and review
in 2–4 weeks.
• Secondary pneumothorax.
• >2 cm and/or respiratory distress → admit for insertion of a chest drain.
• 1–2 cm → aspirate with large bore cannula and review.
• <1 cm → admit for high-flow oxygen (unless contraindicated) and observation for 24 hours.
• Tension pneumothorax.
• If suspected do not wait for a CXR, it is a clinical diagnosis.
• Immediate aspiration with a large bore cannula.
• Definitive management by inserting a chest drain.
Note: 2 cm* refers to the measurement of the intrapleural distance at the level of the hilum.

If there is no improvement in the pneumothorax following aspiration then a chest drain should
be inserted.
A chest drain is inserted into the pleural cavity to allow drainage of air in the context of a
pneumothorax. Effective drainage requires adequate positioning of the drain with an air-tight,
one-way seal to maintain subatmospheric pressure, allowing re-expansion of the lung. An aseptic
technique should be used within a sterile field. Local anaesthetic must be infiltrated with suf-
ficient time for good effect. Chest drains can be inserted using the Seldinger technique, which

83

K30031_Book.indb 83 9/6/17 1:35 PM


5  Thoracic cases

incorporates the use of a guide wire and dilator system over which the chest drain is passed, or via
an open surgical incision (thoracostomy). Chest drains should be inserted into the ‘safe triangle’.
Borders of the ‘safe triangle’:

• Medially – lateral border of pectoralis major.


• Laterally – anterior axillary line.
• Inferiorly – 4th to 5th intercostal space.

Once the drain is positioned adequately it should be connected to a closed drainage system
using an underwater seal acting as a one-way valve to prevent re-entry of air into the pleural
space during inspiration. Application of an airtight dressing follows this.
Post drain insertion:

• Monitor for oscillation/swinging of the underwater seal during inspiration to ensure the
drain is patent.
• Repeat CXR to confirm the position of the drain and determine re-expansion of the lung
(Figure 5.7C).

Fig. 5.7C  CXR following insertion of a small-calibre drain (D) into the left pleural space. The left-sided
pneumothorax remains (lung edge, arrows), the mediastinal shift has resolved.

84

K30031_Book.indb 84 9/6/17 1:35 PM


Case 5.7: Answers

4 What information should this patient be provided with on discharge?


The patient should be advised of the following:

• To return to hospital immediately if their symptoms deteriorate (‘safety netting’).


• The chance of a recurrent pneumothorax on the same side is 50%.
• Smoking increases the risk of further pneumothoraces.
• Air travel should not be undertaken until 1 week following resolution of the pneumothorax
on CXR.
• Underwater diving cannot be undertaken unless preventative surgery has been performed.

LEARNING POINTS: PNEUMOTHORAX


▪▪ A pneumothorax is defined as the presence of air in the pleural space.
▪▪ Primary pneumothorax (absence of lung disease), secondary pneumothorax (presence of
underlying lung disease).
▪▪ Tension pneumothorax is a medical emergency, diagnosed clinically, CXR should not delay
aspiration.
▪▪ Treatment is based on the 2 cm rule, using aspiration +/− chest drain insertion.
▪▪ Chest drains should be inserted into the ‘safe triangle’.

85

K30031_Book.indb 85 9/6/17 1:35 PM


Case 5.8

Fig. 5.8A  PA CXR.

86

K30031_Book.indb 86 9/6/17 1:35 PM


Case 5.8: Questions

A 28-year-old female migrant from Somalia who has been resident in the UK for 2 months
is referred to the respiratory clinic by her GP. She has a 4-month history of a cough, which is
associated with purulent sputum and is occasionally streaked with blood. She reports some
shortness of breath when she climbs the stairs but no chest pain, fevers or night sweats. Over
the last 2 weeks she also reports feeling more tired than usual and has been experiencing a loss
of appetite.
On examination she appears well and is apyrexial, with a normal BP and pulse rate. Respiratory
rate is 16 bpm and oxygen saturations are 97% breathing room air; there is no palpable lymph-
adenopathy. Respiratory examination reveals equal expansion and air entry, normal percus-
sion sounds, and some coarse crepitations anteriorly on the right. A CXR has been performed
(Figure 5.8A).

CASE 5.8: QUESTIONS


1 What radiological features are demonstrated?
2 What is the most likely diagnosis?
3 What investigations should be performed?
4 What is the management of this patient?
5 What are the extrathoracic manifestations of this condition?
6 Is there a facility in place to prevent this condition?

87

K30031_Book.indb 87 9/6/17 1:35 PM


5  Thoracic cases

CASE 5.8: ANSWERS


1 What radiological features are demonstrated?
Radiological features (Figure 5.8B):

• Nodular infiltrates in the right upper zone.


• Right hilar mass.
• Right paratracheal nodes.
• Apical consolidation.

Fig. 5.8B  Magnified CXR demonstrating nodular infiltrates in the right upper zone (A) with apical
consolidation (B), right paratracheal nodes (C), and right hilar nodes (D).

2 What is the most likely diagnosis?


Primary tuberculosis (TB) is most likely. TB is a notifiable disease and its prevalence in the UK
is increasing. The causative agent is Mycobacterium tuberculosis; however, atypical mycobacteria
should be considered in immunosuppressed patients or those with pre-existing lung conditions.
Atypical mycobacteria include Mycobacterium avium complex (MAC) and Mycobacterium kansaii.
Stages of TB are defined as:
• Primary TB.
• Initial lesion, which is normally solitary, located in the upper or middle zones of the lung.
• The focus of the primary infection is called the Ghon focus.

88

K30031_Book.indb 88 9/6/17 1:35 PM


Case 5.8: Answers

• Secondary TB (postprimary TB).


• Reactivation of primary TB or reinfection.
• Usually bilateral cavitating lung lesions.
• Miliary tuberculosis (Figure 5.8C).
• Acute, diffuse dissemination of tubercle bacilli (‘millet seed’ size) via haematogenous
spread.
• Formation of small granulomas in other organs.
• A result of either primary or secondary TB, usually fatal without treatment.

Fig. 5.8C  Axial CT image at the level of the carina using lung window setting, demonstrating multiple tiny
lung nodules, consistent with miliary TB. Differential diagnoses include sarcoid and metastases.

3 What investigations should be performed?


Patients suspected of having TB who require admission to hospital must be treated in an isolated
environment. Those who can be treated at home must be advised to wear a mask to ensure pro-
tection for others coming into contact with the patient.
A CXR must be performed in those suspected of having TB. Cavitating apical lung lesions
are characteristic of TB but are not diagnostic as other conditions may demonstrate a similar
appearance, such as lung malignancy, Wegener’s granulomatosis, and rheumatoid nodules. In
this patient the combination of upper lobe infiltrates with mediastinal lymphadenopathy is highly
suggestive of TB.
Sequential sputum samples should be obtained over at least 3 consecutive days and sent to the
microbiology laboratory for Ziehl–Neelsen (ZN) staining, a method used to identify acid-and-
alcohol fast bacilli (AAFB). Cultures have a higher sensitivity than staining sputum; however, it
may take up to 8 weeks for a positive culture and a further 2 weeks for antibiotic sensitivity test-
ing. If sputum samples cannot be obtained, alternatives such as bronchial washings or biopsies,
can be used. Pleural fluid aspiration is also appropriate if a pleural effusion is present.

89

K30031_Book.indb 89 9/6/17 1:35 PM


5  Thoracic cases

DNA techniques using polymerase chain reaction (PCR) are available in some specialist cen-
tres. These techniques have a similar sensitivity rate to culture and results are available within
days; however, this technique is much more expensive.

4 What is the management of this patient?


The majority of patients are treated in an outpatient setting using a combination therapy (mne-
monic RIPE) outlined below. All four antibacterial drugs are given for the first 2 months of initial
therapy and two are given for a further 4 months in the continuation phase. All drugs are given
once daily:

• Rifampicin (6 months).
• Isoniazid (6 months).
• Pyrazinamide (2 months).
• Ethambutol (2 months).

Supervision of treatment is essential and patients should be reviewed on a monthly basis to


ensure they are compliant. Poor compliance is a major cause of treatment failure and directly
observed therapy should be considered in patients who are likely to have difficulty adhering to the
treatment regime. This can also help in the prevention of drug-resistant TB.

5 What are the extrathoracic manifestations of this condition?


TB can occur in most organs, including the skin, kidney, adrenal glands, and bone. In the brain,
TB can lead to meningitis or tuberculoma formation (a space-occupying lesion). In the adrenal
glands, TB can cause primary adrenocortical insufficiency and must be considered as an underly-
ing cause in a new diagnosis of Addison’s disease.

6 Is there a facility in place to prevent this condition?


Prevention of TB is facilitated by the Bacille Calmette–Guérin (BCG) vaccine, prepared using
attenuated live Mycobacterium bovis. The vaccine was previously offered to schoolchildren aged
12–13 years in the UK; however, this vaccination programme has now ceased. The vaccine is now
given to individuals and neonates from high-risk groups, such as migrants and those with a fam-
ily history of TB. Immunisation has been shown to decrease the risk of developing TB by 60–80%.
Effectiveness of the vaccine varies and is dependent on genetic differences in the population,
environmental factors, and exposure to other infections.

LEARNING POINTS: TUBERCULOSIS


▪▪ Causative agent is Mycobacterium tuberculosis.
▪▪ Stages of TB: primary, secondary, and miliary.
▪▪ Investigations include CXR, ZN staining, and sputum cultures.
▪▪ Treatment is with ‘RIPE’.
▪▪ Extrapleural manifestations of TB can cause other disease processes, such as meningitis and
Addison’s disease.

90

K30031_Book.indb 90 9/6/17 1:35 PM


Case 5.9

Fig. 5.9A  PA CXR.

A 35-year-old female presents to the ED with a 2-week history of a productive cough with green
purulent sputum, shortness of breath, and fever. She is a nonsmoker with a past medical history
of asthma, which is well controlled with both short- and long-acting bronchodilators and a steroid
inhaler.
On examination she appears clammy, has cold peripheries, and a capillary refill time of 4 sec-
onds. She has a temperature of 38.3°C, BP 102/65 mmHg, pulse rate of 105 bpm, respiratory rate
24 bpm, and oxygen saturations 91% breathing room air. Respiratory examination reveals a cen-
tral trachea with reduced expansion on the left, dullness to percussion in the left upper zone with
reduced air entry and crepitations on auscultation. An urgent CXR is performed (Figure 5.9A).

CASE 5.9: QUESTIONS


1 What are the key radiological findings?
2 What is the most likely diagnosis?
3 What are the possible causes for this condition?
4 Name a scoring system used to determine the severity of this condition.

91

K30031_Book.indb 91 9/6/17 1:35 PM


5  Thoracic cases

CASE 5.9: ANSWERS


1 What are the key radiological findings?
Key radiological findings include (Figure 5.9B):

• Extensive consolidation in the upper lobe of the left lung.


• Infiltrates in the middle zone of the right lung.
• Air bronchograms on the left.

Mnemonic for remembering the features of consolidation (A2BC2):

• Air bronchograms – air-filled bronchi made visible by the opacification of surrounding


alveoli.
• Acinar rosettes.
• Bat wing appearance (bilateral, symmetrical changes, more common in pulmonary
oedema, Figure 5.9C).
• Consolidation (diffuse, segmental/lobar).
• Confluent ill-defined (fluffy) appearance.

Fig. 5.9B  CXR showing a large area of consolidation in the left upper lobe (dashed circle), containing air
bronchograms (B) and right-sided infiltrates (A).

92

K30031_Book.indb 92 9/6/17 1:35 PM


Case 5.9: Answers

Fig. 5.9C  CXR of a different patient demonstrating symmetrical, bilateral, and perihilar consolidation
(A) suggestive of pulmonary oedema. Note the sternotomy wires (B) and ill-defined left heart border (C) and
small bilateral basal pleural effusions (D).

2 What is the most likely diagnosis?


Consolidation is a nonspecific radiological sign, therefore the history, examination, and review of
previous CXRs are very important. The most likely diagnosis given the history of purulent spu-
tum, pyrexia, and large area of consolidation on CXR is a lobar pneumonia of the left upper lobe,
with focal changes seen in the right lung also.
Causes for a consolidated appearance include:

• Pus (infection), as in this case.


• Blood (trauma, bleeding diathesis).
• Tumour (lung cancer).
• Water (pulmonary oedema) (Figure 5.9C).

3 What are the possible causes for this condition?


The most common organism seen in community-acquired pneumonia is Streptococcus pneumoniae;
however, pneumonia can be caused by other typical and atypical organisms shown in Table 5.9A.

93

K30031_Book.indb 93 9/6/17 1:35 PM


5  Thoracic cases

Table 5.9A  Organisms associated with pneumonia

Typical organisms Atypical organisms


Streptococcus pneumoniae Mycoplasma pneumoniae
Haemophilus infuenzae Legionella pneumophila
Moraxella catarrhalis Chlamydia spp. (C. pneumoniae)
Influenza A Coxiella burnetii

4 Name a scoring system used to determine the severity of this condition.


CURB-65 score (Table 5.9B) can be used to determine the severity of pneumonia and the most
appropriate setting for treatment, scoring 1 point for each positive category:

• Confusion: new onset mini mental test score <8.


• Urea >7 mmol/L.
• Respiratory rate ≥30 bpm.
• Blood pressure ≤90 mmHg systolic and/or ≤60 mmHg diastolic.
• Age ≥65 years.

Table 5.9B  CURB-65 score used to determine the severity of pneumonia

CURB-65 Score Severity Appropriate treatment location


0 or 1 Mild Community
2 Moderate Hospital – short stay
3 or more Severe Hospital – consider ITU

Other features that should be taken into consideration when determining the most appropri-
ate location for treatment include:

• Co-morbidities: particularly lung conditions (COPD, asthma, cystic fibrosis).


• Involvement of more than one lobe or a bilateral picture.
• Hypoxia requiring oxygen therapy (PaO2 <8 kPa or Sats <92%).

Treatment for moderate and severe pneumonia is with IV antibiotics, usually a penicillin and
macrolide combination to cover typical and atypical organisms. Less severe pneumonia is treated
with oral antibiotics according to local hospital protocol.
Patients require a follow-up CXR after 6 weeks to ensure there is substantial resolution of the
consolidation. If signs of consolidation remain, further investigations may be warranted to ensure
there is not an underlying malignancy.

LEARNING POINTS: PNEUMONIC CONSOLIDATION


▪▪ Features of consolidation can be remembered with mnemonic A2BC2.
▪▪ Differential diagnoses include infection, pleural effusion, and congestive cardiac failure.
▪▪ Most likely organism causing community-acquired pneumonia is Streptococcus pneumoniae.
▪▪ Consider atypical organisms or alternative diagnosis if not responding to treatment.
▪▪ Repeat CXR after 6 weeks to ensure resolution.

94

K30031_Book.indb 94 9/6/17 1:35 PM


Case 5.10

Fig. 5.10A  Erect CXR.

A 54-year-old male presents to the ED with abdominal pain, nausea, and having vomited once.
He has had recent epigastric discomfort for which he has been taking over-the-counter medica-
tion but is otherwise fit and well. He has a history of abdominal surgery following a road traffic
accident 30 years ago.
On examination, he is lying still on his back, in pain but alert and orientated. He is tachycardic
at 110 bpm and hypotensive at 110/76 mmHg but apyrexial at 36.4°C. His abdomen is mildly dis-
tended and rigid with generalised tenderness with rebound tenderness and guarding throughout.
Bowel sounds are quiet.
A supine AXR and erect CXR (Figure 5.10A) are arranged. The AXR (not shown) demon-
strates no acute abnormality.

CASE 5.10: QUESTIONS


1 What is the key radiological finding 3 Is further imaging required?
demonstrated on the CXR? 4 How would you manage this patient?
2 What is the likely diagnosis?

95

K30031_Book.indb 95 9/6/17 1:35 PM


5  Thoracic cases

CASE 5.10: ANSWERS


1 What is the key radiological finding demonstrated on the CXR?
The key abnormality on the erect CXR is presence of free air beneath the diaphragm (Figure 5.10B).
This is indicative of free intraperitoneal gas, otherwise known as pneumoperitoneum. An erect
CXR is the most sensitive XR method of detecting free intraperitoneal air, as air rises to lie beneath
the diaphragm. It is important that patients sit up for several minutes prior to the erect CXR to
allow time for the air to migrate. A good quality erect CXR can detect as little as 1 cc of free air,
with AXR being relatively insensitive. Occasionally ‘Rigler’s sign’ can be seen on AXR; this is also
known as the double wall sign and indicates air on both sides of the intestinal wall. Common
causes of free intraperitoneal air are shown in Table 5.10A.

Fig. 5.10B  Erect CXR showing a large amount of free subdiaphragmatic air (∗). This is seen overlying the
liver on the right (L) beneath the right hemidiaphragm (A) and beneath the left hemidiaphragm (B) in the
LUQ where it is distinguishable from the gastric bubble (C). Note absent splenic density in the LUQ likely to
relate to previous splenectomy.

Table 5.10A  Common causes of free intraperitoneal air


• Perforated peptic ulcer
• Perforation following bowel obstruction
• Perforated diverticulitis/appendicitis
• Recent abdominal surgery
• Peritoneal dialysis

96

K30031_Book.indb 96 9/6/17 1:35 PM


Case 5.10: Answers

TOP TIPS
▪▪ When asked in a practical exam a question such as ‘What are the causes of free intraperitoneal air?’
use a surgical sieve (with a mnemonic such as VITAMIN D) to ensure that you do not forget anything.
▪▪ VITAMIN D: Vascular, Infection/Inflammatory, Trauma, Autoimmune, Metabolic, Iatrogenic/
Idiopathic, Neoplastic, Degenerative.
▪▪ Alternatively, remember the most common causes and list them in order with the ability to talk a
little about each.
▪▪ You do not have to have a cause in each of the categories, just use it as an aide memoire for
causes when under pressure.

2 What is the likely diagnosis?


The clinical and XR findings indicate a perforated viscus, most likely a gastric or duodenal ulcer
in this patient (note: history of epigastric discomfort). Other causes of perforated viscus include
diverticulitis, bowel obstruction, and ischaemic bowel, although the clinical findings in this case
are not suggestive of these entities. A nonpathological finding of free subdiaphragmatic gas is
also sometimes seen following recent laparoscopic surgery or peritoneal dialysis. A more subtle
case of subdiaphragmatic free air is shown in Figure 5.10C. A mimic of subdiaphragmatic free air
is shown in Figure 5.10D.

Fig. 5.10C  A more subtle finding of pneumoperitoneum with free air visible below the diaphragm (A).
Normal gastric bubble (B).

97

K30031_Book.indb 97 9/6/17 1:35 PM


5  Thoracic cases

Fig. 5.10D  Magnified X-ray of the chest right lower zone shows small bowel loops interposed between
the right hemidiaphragm and the liver. Bowel loops are distinguishable from free intraperitoneal air by the
presence of valvulae conniventes (A). This is ‘Chilaiditi’s sign’ and an important mimic of free air.

TOP TIPS
Know a little about each major cause of free intraperitoneal air and be able to talk for 2 minutes about
each.

For example, peptic ulcers are associated with the use of nonsteroidal anti-inflammatory drugs (NSAIDs),
steroids, aspirin, and gastro-oesophageal reflux disease. Perforation occurs when the ulcer erodes
through the full thickness of the gut wall and the gastric contents spill into the peritoneum causing
peritonitis and free intraperitoneal air. The most frequent site of perforation is the anterior wall of the first
part of the duodenum. Posterior duodenal perforation is more likely to result in haematemesis owing to
gastroduodenal artery involvement. Longstanding peptic ulcers can result in obstruction due to fibrosis
or oedema.

3 Is further imaging required?


In cases where the diagnosis is uncertain and where patient condition allows, a CT scan may be
helpful. This will/can confirm the diagnosis and help guide surgical management. CT is highly
accurate and extremely sensitive in the detection of free intraperitoneal air and also in demon-
strating the likely aetiology. It is particularly useful in elderly patients who often have relatively
nonspecific symptoms and signs.

98

K30031_Book.indb 98 9/6/17 1:35 PM


Case 5.10: Answers

In patients with clear clinical findings and free intraperitoneal air on erect CXR, urgent senior
surgical opinion should be requested.

4 How would you manage this patient?


Initial management follows an ABCDE approach (Table 5.10B) as these patients are often par-
ticularly unwell and in severe pain. Early senior support should be sought in each case (this is
very important to mention in the exam situation). Resuscitation with IV fluids to maintain blood
pressure and cardiac monitoring is necessary. Pain control is usually by IV opiates. The definitive
management is surgical once the diagnosis is confirmed by a senior surgeon.

Table 5.10B  Initial management of patient with free intraperitoneal air


Be able to discuss the ABCDE approach to initial management
Airway (is the patient talking? Use of adjuncts if necessary)
Breathing (central trachea, RR, O2 sats, listen, percuss, equal expansion?)
Circulation (HR, BP, IV access, bloods, ABG, fluids, ECG)
Disability (AVPU: alert, voice, pain, unresponsive, GCS, pupils)
Exposure (abdomen, genitalia, legs, sacral/peripheral oedema, glucose)

In an OSCE scenario, a CXR will be given that shows obvious free intraperitoneal air. Once
you have recognised this, the examiner will want to know how you will manage the patient.
Depending on how unwell the patient is, always start with ABCDE then say you would take a full
history and examine the patient. Following on from this, bloods are necessary (say which bloods
you need and why – ensure to include clotting and group, and save as this is a patient likely to
need surgery). An ABG or VBG is often very helpful in these situations as it is an indicator of how
unwell a patient is and whether senior ITU support is necessary. You will ALWAYS need to inform
a senior colleague and ask for their advice/support.

TOP TIPS
ALWAYS ask for senior support and mention this in the OSCE.

It may be helpful to know a little about the type of surgery to be performed (open or laparo-
scopic gastrectomy or vagotomy) but the examiners will not expect you to go into any detail. It
is most important to recognise from the history, examination, and test results that this patient is
unwell and to discuss the immediate management.

LEARNING POINTS: FREE INTRAPERITONEAL AIR


▪▪ When examining a peritonitic abdomen it is important to obtain an erect CXR to look for signs of
bowel perforation, usually in combination with a supine AXR.
▪▪ Erect CXR is the most sensitive type of radiograph in the detection of free intra-abdominal air.
▪▪ Pneumoperitoneum can be subtle on CXR and therefore a CT scan may be indicated for definitive
diagnosis but always ask for senior advice at an early stage.
▪▪ The most common causes of pathological pneumoperitoneum are perforation of bowel
(i.e. secondary to a peptic ulcer, diverticulitis or appendicitis) or it can be a normal finding up to
10 days following laparoscopic surgery.

99

K30031_Book.indb 99 9/6/17 1:35 PM


Case 5.11

Fig. 5.11A  Erect CXR.

100

K30031_Book.indb 100 9/6/17 1:35 PM


Case 5.11: Questions

A 25-year-old male is readmitted through the ED 2 weeks following a laparoscopic appendicec-


tomy for a perforated appendix. He has severe RUQ and shoulder tip pain, has vomited, and is
feeling feverish.
On examination he looks flushed but is alert and orientated. He has RUQ tenderness but his
abdomen is soft and not distended. Per rectum (PR) exam reveals soft brown stool. He is hypo-
tensive at 90/63 mmHg, tachycardic at 110bpm, tachypnoeic at 25 bpm, and febrile at 38.8°C. An
erect CXR is arranged initially (Figure 5.11A). Bloods are perfomed:

Hb 106 g/L (130–180 g/L) K+ 3.7 mmol/L (3.2–5.1 mmol/L)


WBC 22 × 109/L (4.0–11.0 × 109/L) Urea 6.2 mmol/L (1.7–8.3 mmol/L)
CRP 266 mg/L (<5 mg/L) Creatinine 90 mmol/L (62–106 mmol/L)
Na+ 138 mmol/L (135–146 mmol/L) Lactate 4.2 mmol/L (0.5–2.2 mmol/L)

CASE 5.11: QUESTIONS


1 What is the main radiological finding?
2 What are your differential diagnoses?
3 How would you confirm the diagnosis?
4 How would you manage this patient?

101

K30031_Book.indb 101 9/6/17 1:35 PM


5  Thoracic cases

CASE 5.11: ANSWERS


1 What is the main radiological finding?
The CXR shows a large loculation of gas under the right hemidiaphragm (Figure 5.11B). The
loculation outlines a thickened hemidiaphragm and overlies a hazy looking superior liver edge
indicating free fluid. There appears to be an air–fluid level within the collection. There are no
bowel wall markings or signs elsewhere of pneumoperitoneum. The appearances are therefore
consistent with a large subphrenic abscess.

• The difficulty here lies with differentiating this from free intraperitoneal air.
The hemidiaphragm thickening and air–fluid level are the most useful signs.
• Other signs that may be visible on CXR in a patient with a subphrenic abscess include
a raised hemidiaphragm, fluid within the ipsilateral costophrenic angle, and collapse or
consolidation at the lung base.
• Additionally, the raised inflammatory markers and high lactate indicate that the patient is
septic and has likely developed a postoperative abdominal infection.

Fig. 5.11B  Erect CXR showing a large gas loculation (A) under the thickened right hemidiaphragm
(B) overlying the liver (L). There is an air–fluid level within the collection (C). Note the lack of valvulae
conniventes (small bowel) or haustral (large bowel) markings.

102

K30031_Book.indb 102 9/6/17 1:35 PM


Case 5.11: Answers

2 What are your differential diagnoses?


The history suggests either postoperative perforation or infection causing an abscess and dia-
phragmatic irritation with referred pain to the shoulder. Normal postlaparoscopic intraperitoneal
air would be unlikely at 2 weeks post surgery.
Knowing a few key points about abscess formation within the abdomen will get you marks in
an exam situation:

• A subphrenic abscess develops due to accumulation of fluid between the diaphragm and
the liver or spleen, which becomes infected.
• The abscess can be simple or complex (multiloculated).
• Abdominal abscesses are most common in the subphrenic, subhepatic, pelvic, and paracolic
gutter locations.
• The symptoms and signs are location dependent but usually include fever, pain, and
diarrhoea or ileus.
• Subphrenic abscesses can cause diaphragmatic irritation and referred shoulder tip pain.
• They may complicate abdominal surgery, often originally associated with peritonitis, and
usually present 2–3 weeks following surgery.

3 How would you confirm the diagnosis?


Further imaging would be necessary to confirm the diagnosis of a subphrenic abscess. In an
emergency setting, US can identify large collections or the presence of fluid or air within the peri-
toneal cavity; however, a CT scan is more sensitive (Figure 5.11C). CT is also useful in guiding
percutaneous drainage.

Fig. 5.11C  Axial postcontrast CT of the upper abdomen showing free fluid (A) around the anterior border
of the liver (B). A pocket of free gas (C) has risen to the superior antidependent aspect of the peritoneal
cavity (the patient is lying on his back) and an air–fluid level is seen (D). This is a subphrenic fluid and gas
collection. Stomach (E).

103

K30031_Book.indb 103 9/6/17 1:35 PM


5  Thoracic cases

4 How would you manage this patient?


Initial management would be via an ABCDE approach as the patient is clinically septic. It would
be necessary to start analgesia, IV fluid resuscitation, and IV antibiotics following blood cultures.
Antibiotic choice should follow the local trust guidelines but should include broad-spectrum
enteric and anaerobic cover.
Calling for senior help at an early point is important and clearly mention this in the exam.
The on-call surgical team should be informed and it is also good practice to inform the surgeon
who performed the original operation.
Definitive management may be conservative, interventional or surgical:

• If the patient is not septic or particularly unwell, antibiotic therapy may be sufficient to clear
the abscess.
• If the abscess needs draining then an interventional radiologist may be able to do this
percutaneously.
• If the patient is too unwell or the abscess unsuitable for percutaneous drainage, then the
patient may need a full abdominal washout and surgical drainage.

The important point in cases like this is to recognise that the patient is unwell. Remember to
mention the following points in management:

• ABCDE, bloods, blood cultures, monitoring, analgesia, antibiotics, and resuscitation.


If bloods are not included in the exam case history then inform the examiner what blood
tests you would perform and why.
• Full history and examination.
• Inform a senior colleague.
• Further imaging to confirm diagnosis.
• Definitive management is surgery or radiological drainage in some cases.

LEARNING POINTS: SUBPHRENIC ABSCESS


▪▪ Subphrenic abscess on CXR: look for a well-defined collection with an air–fluid level beneath a
thickened hemidiaphragm.
▪▪ Be able to differentiate this from pneumoperitoneum and bowel loops.
▪▪ A subphrenic abscess can develop following peritonitis and complicated abdominal surgery. It is
an important consideration in a patient who improves postoperatively but deteriorates 2–3 weeks
later.
▪▪ A CT scan is useful for confirming the diagnosis and for treatment planning.

104

K30031_Book.indb 104 9/6/17 1:35 PM


Case 5.12

Fig. 5.12A  Magnified CXR view of the upper lung zones.

An 82-year-old male presents to the ED with a 6-week history of productive cough with yellow
sputum, occasionally bloodstained, associated with wheeze and shortness of breath. He is an
ex-smoker, with a past medical history of asthma for which he takes a short-acting bronchodilator.
He is a keen gardener in his spare time.
On examination he is apyrexial, BP 132/76 mmHg, pulse rate 87 bpm, respiratory rate 19 bpm,
and oxygen saturation 94% breathing room air. Respiratory examination reveals equal expan-
sion of the chest wall, dullness to percussion in the right apex with associated coarse crepitations
anteriorly on the right.
A CXR is requested (Figure 5.12A) and bloods are performed:

Hb 139 g/L (130–180 g/L) Platelets 328 × 109/L (150–450 × 109/L)


WCC 12.8 × 109/L (4.0–11.0 × 109/L) U&Es Normal
Neutrophils 8.4 × 10 /L (2.0–7.5 × 10 /L)
9 9 LFTs Normal
Eosinophils 2.8 × 109/L (0–0.4 × 109/L) CRP 106 mg/L (<5 mg/L)

CASE 5.12: QUESTIONS


1 What are the key radiological findings?
2 What are the differential diagnoses in this case?
3 What treatment should be initiated for this gentleman?
4 Discuss the spectrum of lung involvement that may be associated with this condition.

105

K30031_Book.indb 105 9/6/17 1:35 PM


5  Thoracic cases

CASE 5.12: ANSWERS


1 What are the key radiological findings?
Key findings (Figure 5.12B) include:

• Scarring of the left apex.


• Opacity in the right apex and volume loss, with elevated right hilum.
• There is a curvilinear lucent halo present within the right apical opacity.
• Note also calcified lung granulomata indicative of likely previous tuberculous infection.

This is the pathognomonic appearance of cavitation around a fungus ball that has formed in
an old area of right upper lobe lung scarring, usually due to previous TB. The main differential
diagnosis is malignancy. Another example is given in Figure 5.12C.

Fig. 5.12B  Magnified CXR view of the upper lung zones with arrows delineating a lucent halo within the
right apical lung mass. Left apical scarring (A), right hilum (B), which is elevated indicative of right upper
lobe volume loss and scarring, calcified granulomata (C).

2 What are the differential diagnoses in this case?


The following differential diagnoses should be considered in this case:

• Mycetoma or pulmonary aspergilloma.


• Lung malignancy.
• Possibly cavitating infection (reactivation TB).

106

K30031_Book.indb 106 9/6/17 1:35 PM


Case 5.12: Answers

Fig. 5.12C  Coronal postcontrast CT through the lung apices of another patient with mycetoma.
A lucency (A) surrounds the central fungus ball (B) in an old area of TB scarring. Normal anatomical
features include aortic arch (AA), left main pulmonary artery (L), left atrium (LA), right main pulmonary
artery (R), and trachea (T).

Mycetoma is a mass caused by a fungal infection, predominantly Aspergillus spp. The fun-
gus grows in a previously formed lung cavity or invades healthy lung tissue in immunocompro-
mised individuals. Pulmonary aspergilloma and fungus ball are alternative names used to replace
mycetoma.
People who inhale aspergillus particles when they come into contact with them do not nor-
mally develop mycetoma as the immune system destroys the fungus rapidly. However, patients
who are immunocompromised or have an underlying lung disease, such as TB, COPD or cystic
fibrosis, are more likely to develop the condition.

3 What treatment should be initiated for this gentleman?


The diagnosis of mycetoma uses a combination of CXR or CT, sputum samples, which are posi-
tive for Aspergillus spp. in 50% of patients, and a serum Aspergillus precipitin antibody test for the
presence of IgG, IgM, and IgE.
Treatment is usually considered when the patient is symptomatic: haemoptysis is the most
common symptom. Antifungal medication is first-line treatment, using various routes (IV, inha-
lation, and CT-guided percutaneous administration).
Embolisation of the pulmonary artery is beneficial in patients with life-threatening haemop-
tysis but is often a temporary measure as haemoptysis recurs due to collateral vessel formation.

107

K30031_Book.indb 107 9/6/17 1:35 PM


5  Thoracic cases

Surgical resection of the cavity containing the mycetoma is beneficial in patients with recur-
rent haemoptysis providing their lung function is sufficient and often provides positive outcomes.
However, it is not always without complications such as haemorrhage, haematogenous spread of
fungal infection, and worsening shortness of breath.
Patients at risk of mycetoma must be educated to avoid environments that are likely to contain
Aspergillus fungus, e.g. compost heaps, dead leaves, marshland, forests, and grain stores.

4 Discuss the spectrum of lung involvement that may be associated with


this condition.
Aspergillus spp. causes a spectrum of clinical syndromes depending on the patient’s immune sys-
tem and the presence of pulmonary disease:

• Noninvasive.
• Allergic bronchopulmonary aspergillosis (hypersensitivity reaction): background of
asthma, atopy or cystic fibrosis.
• Aspergilloma/mycetoma: background of cavitating lung disease (TB).
• Locally invasive.
• Chronic necrotising aspergillosis: mildly immunocompromised or background of
COPD.
• Severe disease.
• Invasive pulmonary aspergillosis: immunocompromised.

LEARNING POINTS: MYCETOMA


▪▪ Mycetoma is a mass caused by a fungal infection, predominantly Aspergillus spp.
▪▪ More common in people who are immunocompromised or have an underlying lung disease such
as asthma, COPD or cystic fibrosis.
▪▪ Treatment includes antifungal medications and surgical resection.
▪▪ Patients should be educated to avoid environments containing Aspergillus.
▪▪ Aspergillus causes a spectrum of syndromes, largely dependent on the immune status of the
host.

108

K30031_Book.indb 108 9/6/17 1:35 PM


Case 5.13

Fig. 5.13A  PA CXR.

A 65-year-old female presents to the ED with a 2-month history of gradually worsening short-
ness of breath associated with a dry cough and pleuritic chest pain. She had been diagnosed and
treated for breast cancer 5 years previously.
On examination there is reduced expansion of the right hemithorax with stony dull percus-
sion and absence of breath sounds on auscultation. She is mildly tachypnoeic at 24 bpm and her
oxygen saturation on room air is low at 90%.
As part of her initial management, a CXR is arranged (Figure 5.13A).

CASE 5.13: QUESTIONS


1 What are CXR findings?
2 What is the diagnosis?
3 What further initial imaging investigation would be helpful?
4 How would you further manage this patient?

109

K30031_Book.indb 109 9/6/17 1:35 PM


5  Thoracic cases

CASE 5.13: ANSWERS


1 What are the CXR findings?
The CXR shows near complete opacification of the right hemithorax with the presence of a meniscus
at the superior margin (Figure 5.13B). The right hemidiaphragm and heart border are obscured.
There is no significant mass effect or mediastinal shift. There are no bone lesions. Note the irregu-
larity and reduction in size of the right breast shadow (consistent with previous breast surgery).

Fig. 5.13B  CXR showing near complete opacification of the right hemithorax (A) with a meniscus at
the superior margin (B). There is no mediastinal shift. Note blunting of the left costophrenic angle (C) and
asymmetry of right breast outline (D). The trachea is central and undisplaced (E).

2 What is the diagnosis?


There are large right and small left pleural effusions – in the context of previous breast cancer,
metastatic disease is to be excluded.
Types of pleural effusion:

• Hydrothorax (serous fluid).


• Haemothorax (blood).

110

K30031_Book.indb 110 9/6/17 1:35 PM


Case 5.13: Answers

• Chylothorax (chyle).
• Pyothorax (empyema).

3 What further initial imaging investigation would be helpful?


Further investigation of the pleural effusion is warranted and an US-guided diagnostic and/or
therapeutic thoracocentesis is the next step. US guidance is recommended for all pleural proce-
dures as it allows improved site identification and is safer. US can also identify pleural/lung solid
lesions and whether an effusion is multiloculated, which will render drainage ineffectual. US is
used to select a site for aspiration with sufficient volume, no intervening lung on maximal inspira-
tion, and no adjacent structures (heart, liver, spleen) (Figure 5.13C).

Fig. 5.13C  Sagittal US image of the right lung base and liver showing a large pleural effusion (A),
collapsed lung segment (B), diaphragm (C), liver with simple liver cysts (D), and ascites (E).

4 How would you further manage this patient?


Check carefully for other signs of breast metastatic disease – palpable nodes supraclavicular fossa
or axilla, liver edge, and scar recurrence.

• ABCDE systematic approach with initial resuscitation of the patient and close monitoring.
• Early senior advice to be sought.
• Blood profile: FBC, U&Es, LFTs, bone profile, coagulation screen.

111

K30031_Book.indb 111 9/6/17 1:35 PM


5  Thoracic cases

• ABG (identify type 1 respiratory failure, detect possible CO2 retention).


• ECG and possibly echocardiogram (to assess LV function).
• Pleural US (to confirm the effusion, check for fibrous septations, and guide diagnostic/
therapeutic thoracentesis or pleural drain insertion) (Table 5.13A).
• CT thorax (to identify lung pathology). If metastatic breast malignancy is suspected, a
staging CT of the chest, abdomen, and pelvis, and a bone scan can be performed.

Pleural effusions can be transudates (protein <30 g/L) or exudates (protein >30 g/L). Light’s
criteria for exudates (causes Table 5.13B):

• Protein: effusion albumin/plasma albumin >0.5.


• LDH: effusion LDH/plasma LDH >0.6.
• LDH: effusion LDH >2/3 upper limit of the reference range for the serum LDH.

Table 5.13A  Pleural fluid analysis


Macroscopic • Normal – light yellow and clear fluid (clear ultrafiltrate of plasma that originates
appearance from the parietal pleura)
• Milky fluid – seen with chylothorax and is due to high triglyceride levels
• Bloody fluid – seen after a traumatic thoracentesis and in traumatic
haemopneumothorax, malignancy, pulmonary embolism, and TB
• Purulent fluid – seen in empyema
Laboratory tests • pH level
• Gram stain and culture
• Cell count and differential diagnosis
• Glucose, protein, and LDH levels
• Cytology
• Amylase (if oesophageal perforation or pancreatitis is suspected)
• Triglycerides (if chylothorax is suspected)

Table 5.13B  Causes of pleural effusions

Transudative pleural effusion Exudative pleural effusion


Liver cirrhosis Malignant conditions
Cardiac Metastatic pleural disease
Congestive cardiac failure Carcinomatosis
Hypoalbuminaemia Primary mesothelioma
Nephrotic syndrome Meigs’ syndrome
Protein-losing enteropathy Infection
Miscellaneous Empyema
Myxoedema TB
Superior vena cava obstruction Parapneumonic effusion
Miscellaneous
Connective tissue disease
Vasculitis
Sarcoidosis
Pancreatitis
Asbestos pleural effusion

112

K30031_Book.indb 112 9/6/17 1:35 PM


Case 5.13: Answers

Fig. 5.13D  Erect CXR in another patient with a large left pleural effusion with opacified left hemithorax. In
this case the effusion has mass effect with mediastinal shift to the right – tracheal deviation (A) is present
and the right heart border is also significantly displaced (B).

Another patient with a pleural effusion and mediastinal shift is demonstrated in Figure 5.13D.
The causes of opacified hemithorax and its investigation are discussed elsewhere in this book,
and it is important to be able to differentiate between lung collapse and pleural effusion on CXR.
Assessment of the positioning of the mediastinum should allow this, with US being used in
equivocal cases.

113

K30031_Book.indb 113 9/6/17 1:35 PM


5  Thoracic cases

LEARNING POINTS: PLEURAL EFFUSION/OPACIFIED HEMITHORAX


Causes of an opacified hemithorax:

▪▪ Total lung collapse or pneumonectomy (trachea and heart displaced towards the opacified
hemithorax, look for surgical clips).
▪▪ Consolidation (trachea remains central, look for air bronchograms).
▪▪ Pleural effusion (if large trachea plus heart pushed away from the opacified hemithorax).

Look for clues in the clinical assessment for either malignancy or infection.

US is useful to confirm the diagnosis and guide percutaneous drainage/aspiration.

114

K30031_Book.indb 114 9/6/17 1:35 PM


Case 5.14

Fig. 5.14A  CXR.

A 35-year-old female presents to the ED with acute breathlessness following a 1-week history of
mild productive cough with thick green sputum. She has a history of asthma and currently uses a
beclometasone dipropionate inhaler twice daily and salbutamol inhaler as needed.
On examination there is reduced expansion of the left hemithorax with reduced breath sounds
on auscultation. She is tachypnoeic at 28 bpm, hypoxic with oxygen saturations on air of 85%,
tachycardic at 110 bpm, and mildly hypertensive at 145/85 mmHg. As part of her initial investiga-
tions, a CXR is arranged (Figure 5.14A).

CASE 5.14: QUESTIONS


1 What are the CXR findings?
2 What is the diagnosis?
3 How would you manage this patient?

115

K30031_Book.indb 115 9/6/17 1:35 PM


5  Thoracic cases

CASE 5.14: ANSWERS


1 What are the CXR findings?
There is reduced volume of the left hemithorax with a double contour of the LHB (sail sign,
Figure 5.14B). There is loss of clarity of the medial left hemidiaphragm and the descending tho-
racic aorta (silhouette sign). The inferior mediastinum is also shifted towards the left. The right
lung is clear.

Fig. 5.14B  CXR showing collapse of the left lower lobe causing a double LHB appearance (sail sign)
(A), obscuration of the medial left hemidiaphragm (B), and descending thoracic aorta (C). The inferior
mediastinum is deviated to the left, secondary to the collapse. Note reduced volume of left hemithorax and
also increased translucency compared with the right owing to left upper lobe expansion.

2 What is the diagnosis?


Left lower lobe collapse. Given the history of recent infective exacerbation of asthma, the likely
cause is a mucus plug obstructing the left lower lobe bronchus.
• Lobar collapse is recognised by an increase in density of an area of lung (in this case behind
the heart) associated with loss of lung volume. General signs include:
• Ipsilateral raised hemidiaphragm.
• Displaced hilum (towards collapse).

116

K30031_Book.indb 116 9/6/17 1:35 PM


Case 5.14: Answers

• Tracheal (and mediastinal) shift towards the side of collapse.


• Ipsilateral narrowing of the intercostal spaces.
• Left lower lobe collapse can be subtle as it is projected behind the heart. The key signs to
look out for are the increased density with double heart border (sail sign) and obscuration
of the medial aspect of the left hemidiaphragm (silhouette sign). Silhouette sign – on
XR, borders of structures are often demarcated at the interface of tissues of different
radiographic density. In this case, for example, the descending thoracic aorta and left
hemidiaphragm are usually clearly seen as aerated/dark lung abuts soft tissue/white.
When the lobe collapses this also becomes of soft tissue density and where it abuts the
diaphragm/descending aorta the air/soft tissue interface is lost and is now soft tissue/soft
tissue.
• Lobar collapse is usually related to endobronchial obstruction, which may be intrinsic
or extrinsic. In adults the most common causes of intrinsic obstruction are tumours and
mucus plugs. In the clinical context of a middle-aged or elderly smoker, lobar collapse
should always be considered to be cause by bronchogenic carcinoma until proven
otherwise (bronchoscopy needed). In children, inhaled foreign bodies or mucus plugs
are most common. Extrinsic compression may be caused in any age by mediastinal
lymphadenopathy or other mediastinal masses. Large pleural effusions can also cause
external compression.

3 How would you manage this patient?


Initial management would follow an ABCDE approach with early senior advice. Investigations
would include routine blood tests, ABG, CXR, continuous bedside monitoring, and peak expira-
tory flow rate (PEFR) every 15–30 minutes. Medical therapy for her asthma exacerbation will need
to be commenced including:

• Supplementary oxygen, maintaining saturation pressure of oxygen (SPO2) at 94–98%.


• Nebulised therapy: beta2-agonist bronchodilators (salbutamol), consider repeat doses every
15–30 minutes. Ipratropium bromide 0.5 mg every 4–6 hours.
• Steroid therapy (oral or IV).
• Antibiotics. Consider them only when evidence of bacterial infection is present: most
asthma exacerbations are caused by viral infections.

The patient should be referred to the chest team and for urgent physiotherapy to dislodge the
mucus plug. Early repeat CXR to confirm complete lung re-expansion – if collapse persists bron-
choscopy will be needed.
Another left lung lobar collapse that can be difficult to recognise is left upper lobe collapse,
where the upper lobe collapses anteriorly and medially. The result is that the main adjacent struc-
tures (AA and hilum) lose their clarity on CXR. This is demonstrated in Figure 5.14C and on CT
in Figure 5.14D (both on page 118).

LEARNING POINTS: LEFT LOWER LOBE COLLAPSE


▪▪ In children, usually caused by inhaled foreign body.
▪▪ In young adults/asthmatics, mucus plug most common.
▪▪ In older patients/smokers, malignancy to be excluded.

Patients need respiratory team referral and early bronchoscopy if there is not prompt lung reinflation.

117

K30031_Book.indb 117 9/6/17 1:35 PM


5  Thoracic cases

Fig. 5.14C  CXR in a


different patient showing
left upper lobe collapse
with a typical veil sign
(the left lung field appears
as though covered by
a veil). The left upper
lobe collapses anteriorly
and medially, and as it
encroaches upon the
left hilum and aortic
knuckle these structures
will become obscured
(silhouette sign again).
The left hilum is also
elevated towards collapse
and is not well seen. Note
obscuration of the aortic
arch as collapsed lung
lies against it (A) – as well
as tenting/elevation of the
left hemidiaphragm (B)
with cardiac and minor
tracheal shift to the left,
and reduced volume of
left lung. Left upper lobe
collapse is usually caused
by a central obstructing
malignancy.

Fig. 5.14D  Coronal


CT image of the thorax
showing left upper lobe
collapse (A). Trachea (B),
right main bronchus (C),
and left main bronchus
(D). Note collapsed lobe
abutting upper pole
left hilum (E) and aortic
knuckle (F) with loss of
aerated margin (silhouette
sign).

118

K30031_Book.indb 118 9/6/17 1:35 PM


Case 5.15

Fig. 5.15A  CXR.

A 65-year-old male presents to the respiratory clinic referred by his GP with a 6-month history
of gradually progressive shortness of breath, cough with haemoptysis, and weight loss. He is a
smoker with a 45 pack-year history. He has no past medical history of note.
He is tachypnoeic at 35 bpm, tachycardic at 120 bpm, normotensive at 125/90 mmHg, hypoxic
with SPO2 of 88% on room air, and apyrexial.
As part of his initial investigations, a CXR is arranged (Figure 5.15A).

CASE 5.15: QUESTIONS


1 What are the CXR findings?
2 What is the diagnosis?
3 How would you investigate and manage this patient?

119

K30031_Book.indb 119 9/6/17 1:35 PM


5  Thoracic cases

CASE 5.15: ANSWERS


1 What are the CXR findings?
The findings are (Figure 5.15B):

• Opacity of the right upper zone.


• Elevation and tenting of the right hemidiaphragm.
• Elevation of the right hilum. Note: the right hilum cannot be seen clearly but the curved S
configuration of the hilum is suggestive of a mass (Golden’s ‘S’ sign).
• Tracheal shift towards the collapsed side (right).
• Crowding of the ribs on the right side, reduced volume of the right hemithorax.

Fig. 5.15B
PA CXR. There is
volume loss in the
right hemithorax
with elevation
of the right
hemidiaphragm (A).
There is right upper
zone opacity and
the right hilum is
elevated and blends
with right upper
zone opacity: an
underlying mass
is likely (Golden’s
“S” sign, B). The
trachea is shifted to
the right (C).

2 What is the diagnosis?


The CXR shows features of a collapsed right upper lobe likely caused by a central obstructing hilar
mass: a diagnosis of lung malignancy is highly likely in this case.
Figures 5.15C and 5.15D are radiographs in another patient with middle lobe collapse (right
lower lobe collapse is not dissimilar to left lower lobe collapse in terms of appearances in the
­collapsed lobe, Figure 5.15E).

120

K30031_Book.indb 120 9/6/17 1:35 PM


Case 5.15: Answers

Figs. 5.15C, D  PA and lateral


CXRs, respectively. (Right) middle
lobe collapse. Note the increased
density adjacent to and obscuring
the right heart border (A) with
depression of the right hilum
on the PA CXR. The collapsed
dense middle lobe (M) is seen on
the lateral view, outlined by the
depressed horizontal fissure (B)
D
and deviated oblique fissure (C).

121

K30031_Book.indb 121 9/6/17 1:35 PM


5  Thoracic cases

Fig. 5.15E  CXR of right lower lobe collapse. Note the triangular shaped collapsed lower lobe (A),
depressed hilum, and reduced volume in the right hemithorax.

Right middle lobe collapse radiological features:

• Difficult to identify on PA CXR owing to subtle changes.


• Ill-defined increased density of the right lower zone.
• Depressed right hilum.
• Silhouette sign, with loss of aerated middle lobe abutting the RHB; increased density in
the right lower zone with loss of clarity of the RHB points to pathology in the middle lobe –
consolidation or collapse.
• On the lateral CXR, a collapsed opacified middle lobe can be identified by outlining the
depressed horizontal fissure and an elevated distorted right oblique fissure.

Right lower lobe collapse is also shown (Figure 5.15E).

122

K30031_Book.indb 122 9/6/17 1:35 PM


Case 5.15: Answers

3 How would you investigate and manage this patient?


Use ABCDE as always initially, oxygen by mask, and monitoring. Routine bloods and discuss
with seniors/respiratory team:

• Blood profile: FBC (anaemia), U&Es (SIADH), LFTs (metastatic liver disease, raised alkaline
phosphatase (ALP) in metastatic bone disease), bone profile (high calcium levels secondary
to metastatic bone disease or part of a paraneoplastic syndrome).
• Sputum cytology.
• Bronchoscopy.
• CT chest/abdomen.

LEARNING POINTS: RIGHT UPPER LOBE COLLAPSE


▪▪ The age of the patient and distribution of the lobar collapse is important in aetiology.
▪▪ Upper lobe collapse is more usually due to malignancy.
▪▪ Older patients/smokers need early bronchoscopy to exclude malignancy.

123

K30031_Book.indb 123 9/6/17 1:35 PM


Case 5.16

Fig. 5.16A  CXR.

124

K30031_Book.indb 124 9/6/17 1:35 PM


Case 5.16: Questions

A 65-year-old male smoker presents to the medical assessment unit with a 6-month history
of gradually worsening shortness of breath, dry cough, and decreased exercise tolerance. He
describes no change in appetite and no recent weight loss. In the last 4 days he has noticed that
his breathing has deteriorated acutely and his cough has become productive with small amounts
of brown sputum.
He has been started on a bronchodilator inhaler by his GP with minimal improvement in
symptoms. He has had recurrent chest infections over the past 6 months and has taken several
courses of antibiotics.
On examination he is dyspnoeic, has a hyperexpanded chest, and on auscultation there is
expiratory polyphonic wheeze and reduced breath sounds at the apices. He is afebrile, tachy-
pnoeic at 35 bpm, tachycardic at 120 bpm, and normotensive at 130/70 mmHg. SPO2 is 85% on
room air.
As part of the initial assessment a CXR is arranged (Figure 5.16A).

CASE 5.16: QUESTIONS


1 What are the CXR findings?
2 What is the diagnosis?
3 How would you investigate and manage this patient?

125

K30031_Book.indb 125 9/6/17 1:35 PM


5  Thoracic cases

CASE 5.16: ANSWERS


1 What are the CXR findings?
The CXR shows hyperexpansion of the lungs with flattening of diaphragms and reduced cardiac
silhouette (Figure 5.16B). Lung vascular markings are significantly attenuated in the lower zones,
which appear hyperlucent.

Fig. 5.16B  CXR showing hyperexpanded lungs with flattened hemidiaphragms and small appearance
of the heart (A). The lower zones are translucent. Note the increased visible anterior ribs (10 are visible,
labelled on right, normal 5–7) and posterior ribs (12 are visible, labelled on left, normal 7–9).

126

K30031_Book.indb 126 9/6/17 1:35 PM


Case 5.16: Answers

2 What is the diagnosis?


These findings, associated with the clinical presentation, confirm a diagnosis of COPD with
emphysematous changes. The current presentation can be attributed to a likely infective exacer-
bation of COPD.
• COPD is a progressive, poorly reversible condition of airflow limitation, caused by a
persistent inflammatory response in the lungs. It is associated in most cases with smoking
but can also be caused by atmospheric pollution or alpha-1-antitrypsin deficiency.
• In clinical practice, the radiographic assessment of hyperinflation is usually subjective but
specific quantitative indices are sometimes applied. Thus, the diaphragm is considered
low if the level of the right dome is at or below the anterior aspect of the 7th rib, and flat if
the maximum curvature of the dome is less than 1.5 cm in height. Causes of hyperinflated
lungs are shown in Table 5.16.
Table 5.16  Causes of hyperexpanded lungs
• Emphysema/chronic bronchitis/COPD
• Acute asthma attack
• Alpha-1-antitrypsin deficiency
• Cystic fibrosis

3 How would you investigate and manage this patient?


Investigations:
• Blood tests: FBC (raised WCC in associated infection, high Hb), consider serum albumin
(severity), alpha-1-antitrypsin deficiency (younger patient with positive family history).
• ABG: type II respiratory failure (low PaO2 and high PaCO2).
• ECG and echocardiography: assess for features of cor pulmonale.
• CXR: hyperexpanded lungs, bullae, pneumothorax.
• Pulmonary function tests: obstructive ventilator defect with a low gas transfer coefficient.
Treatment of acute exacerbation:
• ABCDE resuscitation with early senior input.
• Controlled O2 supplementation – aim for an O2 saturation of 88–92%, to be administered
through a Venturi mask, with monitoring for possible CO2 retention and type II respiratory
failure.
• Nebulised bronchodilators then oral steroids for 7 days.
• Antibiotics if evidence of an associated bacterial infection.
• Chest physiotherapy.
• Consideration of noninvasive ventilation in the presence of type II respiratory failure.
• High-resolution CT to look for features of lung disease, bullae, and fibrosis (Figures 5.16C
and 5.16D).
• Respiratory specialist review.
• Exclude an associated malignancy owing to the increased risk in smokers (cigarette
smoking is linked to about 90% of lung cancers).
Long-term medical treatment:
• Smoking cessation.
• Long-term oxygen therapy if required.

127

K30031_Book.indb 127 9/6/17 1:35 PM


5  Thoracic cases

Fig. 5.16C  Axial CT thorax using lung window setting, showing numerous large bullae (A) with thin
septations in a different patient with severe COPD.

Fig. 5.16D  CXR in a patient with more overt emphysematous changes, notably in the right lung.

128

K30031_Book.indb 128 9/6/17 1:35 PM


Case 5.16: Answers

• Pulmonary rehabilitation programmes should include multicomponent, multidisciplinary


interventions, which are tailored to the individual patient’s needs. The rehabilitation
process should incorporate a programme of physical training, disease education, and
nutritional, psychological, and behavioural intervention.
• Vaccinations – pneumococcal vaccination and an annual influenza vaccination should be
offered to all patients with COPD.

Lung surgery (in selected, suitable patients):

• Bullectomy – patients who are breathless, have a single bulla on a CT scan, and an FEV1
less than 50% predicted should be referred for consideration of bullectomy.
• Lung reduction surgery – in patients with severe COPD who remain breathless with
marked restrictions of their activities of daily living, despite maximal medical therapy and
who are meeting specific criteria.
• Single lung transplant – patients with severe COPD who remain breathless with marked
restrictions of their daily activities despite maximal medical therapy, bearing in mind
comorbidities and local surgical protocols.

LEARNING POINTS: CHRONIC OBSTRUCTIVE PULMONARY DISEASE


▪▪ Emphysema/chronic bronchitis/COPD is the most common cause of hyperexpanded lungs.
▪▪ CXR findings include a flattened diaphragm, small appearance of the heart, and lucent bullae.
Bullae and fibrosis, however, are better demonstrated on HRCT.
▪▪ Smoking is the most common cause, although alpha-1-antitrypsin deficiency should be
considered in the young.
▪▪ Any discussion about long-term medical management should include mention of smoking
cessation and long-term oxygen therapy.

129

K30031_Book.indb 129 9/6/17 1:35 PM


Case 5.17

Fig. 5.17A  CXR.

A 40-year-old female with a 15-year history of recurrent chest infections with a productive cough
presents to the ED with an exacerbation of her cough, productive of purulent sputum, unwell, and
febrile. She is a nonsmoker, keeps no pets, and with no history of recent foreign travel.
She is clubbed, pyrexial 37.5°C but otherwise well. On auscultation there are scattered inspira-
tory crackles and widespread inspiratory wheeze.
As part of her initial management a CXR is arranged (Figures 5.17A and 5.17B).

130

K30031_Book.indb 130 9/6/17 1:35 PM


Case 5.17: Questions

Fig. 5.17B  Magnified view of the right upper zone.

CASE 5.17: QUESTIONS


1 What are the CXR findings?
2 What is the diagnosis?
3 How would you investigate and manage this patient?

131

K30031_Book.indb 131 9/6/17 1:35 PM


5  Thoracic cases

CASE 5.17: ANSWERS


1 What are the CXR findings?
The CXR shows abnormal reticular ‘tram-track’ opacification in the right upper zone in keeping
with grossly thickened and dilated bronchi. There is no associated volume loss or mediastinal
lymphadenopathy (Figure 5.17C).

Fig. 5.17C  CXR showing bronchiectasis in the right upper zone. ‘Tram-track’ lines represent a dilated,
thick-walled bronchus (A). A tubular dilated opacified bronchus is present, likely containing mucus/pus (B).

2 What is the diagnosis?


The radiographic findings together with a history of recurrent chest infections and productive
cough are consistent with a diagnosis of bronchiectasis:

• Bronchiectasis is a longstanding disease in which the bronchi become irreversibly


thickened and dilated. It is caused by a combination of excess production of secretions with
impaired clearance, often with impaired host defence mechanisms. It can be localised to a
lobe or generalised. Bronchiectasis is a cause of clubbing.

132

K30031_Book.indb 132 9/6/17 1:35 PM


Case 5.17: Answers

• Types: cylindrical, varicose, cystic or traction. Recurrent infections cause recurrent


bronchial wall damage, scarring, fibrosis, and dilatation.
• Causes:
• Congenital: cystic fibrosis, Kartagener’s syndrome, immunodeficiency, congenital
kyphoscoliosis.
• Acquired: infection (viral, bacterial, fungi), mechanical bronchial obstruction
(mass, foreign body), recurrent aspiration or associated with autoimmune disorders
(rheumatoid arthritis, inflammatory bowel disease).
• Pathogens associated with bronchiectasis:
– Pseudomonas aeruginosa.
– Haemophilus influenza.
– Streptococcus.
– Aspergillus spp. (particularly in upper lobe bronchiectasis).
• Complications:
• Pulmonary: recurrent infections, haemoptysis (can be life threatening), empyema,
abscess, cor pulmonale, scarring and fibrosis of the lung.
• Extrapulmonary: anaemia, cerebral abscess, secondary amyloidosis.
• Cerebral abscess.

3 How would you investigate and manage this patient?


Investigations include CXR and HRCT (Figure 5.17D) to define the distribution of bronchiectasis
and to look for a possible cause, as well as respiratory team referral. HRCT involves taking very
thin CT slices through the chest but with small gaps between the slices.

Fig. 5.17D  Axial HRCT thorax using lung window setting showing cylindrical bronchiectasis and the signet
ring sign. Note the dilated thick-walled bronchi in the middle lobe (A). In cross-section the bronchus and
pulmonary artery branch should be the same size, whereas in bronchiectasis the bronchus is markedly
dilated and thick walled appearing like a signet ring seen end-on. The dilated bronchus (B) is thick walled
and can be seen to be larger than the adjacent pulmonary artery branch (C), the signet ring sign.

133

K30031_Book.indb 133 9/6/17 1:35 PM


5  Thoracic cases

Additionally, blood tests (Hb and inflammatory markers), sputum cultures (including acid-fast
bacilli and cytology), and pulmonary function tests (obstructive pattern with limited reversibility)
should be performed.
To identify an underlying cause, cystic fibrosis sweat and genetic testing, TB elispot, serum
immunoglobulins, serum electrophoresis and Aspergillus precipitins may be useful.
Bronchoscopy may be helpful in some cases.
Treatment (acutely):

• ABCDE initial approach with fluid/oxygen, resuscitation as needed, and early senior input.
• Routine bloods and blood cultures if pyrexial.
• Chest physiotherapy and postural drainage.
• Keep well hydrated – IV fluids if required.
• Bronchodilators (nebulised/inhaled).
• Mucolytics (carbocisteine).
• Antibiotics if evidence of bacterial infection.
• Treatment of underlying cause; might include surgery in carefully selected cases.

LEARNING POINTS: BRONCHIECTASIS


Radiographic features to look for on CXR:

▪▪ Dilated and thickened airways (tramlines, ring shadows).


▪▪ Mucus plugging with bronchoceles (‘gloved finger sign’).
▪▪ Volume loss secondary to consolidation.

HRCT is the diagnostic test of choice.

134

K30031_Book.indb 134 9/6/17 1:35 PM


Case 5.18

Fig. 5.18A  AP erect CXR.

A 76-year-old male patient on the ward who has suffered a cerebrovascular accident coughs when
drinking and recently choked on soft food. The nurses are worried about his ability to swallow
and you have arranged for the speech and language therapists to assess him. The team decide that
he should be fed for the time being via an NG tube. You have inserted this and want to check its
position before it is used for feeding. Attempted aspiration did not yield enough fluid for the pH
verification test and you therefore request a CXR (Figure 5.18A).

CASE 5.18: QUESTIONS


1 What does the CXR show and can feeding be commenced through the NG tube?
2 What are the possible complications of an incorrectly placed NG tube?
3 What other ways can correct NG tube placement be checked?
4 What other indications are there for NG tube insertion?

135

K30031_Book.indb 135 9/6/17 1:35 PM


5  Thoracic cases

CASE 5.18: ANSWERS


1 What does the CXR show and can feeding be commenced through the NG tube?
The CXR shows incorrect placement of the NG tube. The tube has passed via the right main
bronchus into the right lower lobe of the lung (Figure 5.18B). Feeding must not be commenced.
A correctly placed NG tube on CXR (Figure 5.18C) should:

• Follow the path of the oesophagus/avoid the contours of the bronchi.


• Clearly bisect the carina.
• Cross the diaphragm in the midline.
• Have a tip clearly visualised below the left hemidiaphragm.
• See Figures 5.18D and 5.18E for another example.
• Owing to the close proximity of the oesophagus to the larynx, NG tube placement can be
difficult as the tip can pass via the larynx into the trachea and into either the right or left
main bronchus of the lung.

Fig. 5.18B  AP erect CXR showing the NG tube in the right lower lobe of the lung (A).

136

K30031_Book.indb 136 9/6/17 1:35 PM


Case 5.18: Answers

Fig. 5.18C  CXR showing satisfactory NG tube placement. Note the arrows following the path of the tube,
which bisects the carina and crosses the diaphragm in the midline. The tube tip lies well beneath the left
hemidiaphragm.

KEY POINTS
If you are ever unsure of NG tube placement, always ask a senior doctor or radiologist to review the CXR
before feeding. Document your findings and recommendations in the notes. Such decisions are best
made in working hours when senior colleagues are freely available for advice.

Before confirming NG tube placement on CXR it is important to check the patient name, hospital number
and date of birth. Additionally, check the time and date of the film you are reviewing. ITU patients may
have several CXRs every day, so you must ensure that you are reviewing the correct one. Always docu-
ment your findings in the patient notes.

137

K30031_Book.indb 137 9/6/17 1:35 PM


5  Thoracic cases

Figs. 5.18D, E  (5.18D) Mobile AP CXR in a different patient showing consolidation in the right lung,
likely caused by aspiration pneumonia. Although the radiopaque NG tube tip is projected below the left
hemidiaphragm, the tube clearly does not bisect the carina and has passed into the left main bronchus.
The dashed line (A) shows the expected path of a normally positioned NG tube. The solid black line
(B) shows the actual passage of the NG tube down the left main bronchus. (5.18E) The outline of the
trachea and the carina (C) are shown in a solid black line. Note that although the tip of the NG tube
is below the left hemidiaphragm, there is failure of the tube to bisect the carina (C) and the actual
path of the tube overlaps the left main bronchus due to incorrect positioning. This patient also has
an incorrectly placed right subclavian central venous line, which passes cranially up the right internal
jugular vein (D).

2 What are the possible complications of an incorrectly placed NG tube?


Complications of NG tube placement include:

• Immediate.
• Epistaxis.
• Oesophageal perforation.
• Intracranial placement of the tube.
• Early.
• Incorrect placement of tube in the lung.
• Pneumonitis.
• Lung collapse.

138

K30031_Book.indb 138 9/6/17 1:35 PM


Case 5.18: Answers

Fig. 5.18E

• Late.
• Aspiration pneumonia.
• Tube obstruction.
• Feed related complications: diarrhoea, abdominal cramps, nausea.
Note: In an exam situation when asked for complications of a procedure, split this into imme-
diate, early, and late complications. This is a good way to classify the information you give and
ensures that you do not forget anything.

3 What other ways can correct NG tube placement be checked?


The National Patient Safety Agency (NPSA) issued guidance in 2005 for safe placement and posi-
tion checking of nasogastric tubes. There are two methods that can be used to check NG tube
placement:

• First-line check is by aspiration of gastric fluid and measurement of pH of the aspirate using
pH indicator paper. If the aspirate pH is below 5.5 then feeding can commence.
• CXR is indicated as second line test (not routine) and can only be checked by those trained
to do so. If there is any uncertainty, the CXR should be checked by a radiologist.
• Other methods have been highlighted as insufficient, such as the ‘whoosh’ test (listening
for bubbling sounds after air entry) or use of litmus paper in testing acidity.

139

K30031_Book.indb 139 9/6/17 1:35 PM


5  Thoracic cases

TOP TIPS
▪▪ Know what a correctly placed NG tube should look like on CXR (Figures 5.18C and 5.18D, E).
▪▪ Follow the NG tube down from the most superior aspect where it is first seen on the CXR.
▪▪ The NG tube should bisect the carina centrally.
▪▪ Ensure it follows the central line of the oesophagus and not down a main bronchus.
▪▪ The tip should be within the stomach/duodenum, which is on the left side below the diaphragm.
▪▪ Take senior advice if you are unsure and document your findings. Arrange NG tube changes/CXR
if possible in working hours, when senior advice is readily available. Delay any feeding until correct
position confirmed.

4 What other indications are there for NG tube insertion?


The indications are:

• Evaluation of an upper GI bleed.


• Aspiration of gastric contents.
• Administration of radiographic contrast to the GI tract.
• Gastric or small bowel decompression.
• Feeding* or administration of medication.

LEARNING POINTS: NASOGASTRIC TUBE PLACEMENT AND ASSESSMENT


▪▪ NG tube position can be checked using pH assessment of gastric aspirate. If pH is below 5.5, it
confirms correct NG tube position.
▪▪ CXR is a second line investigation. If correctly placed, the NG tube should follow the line of the
oesophagus, bisect the carina and cross the diaphragm in the midline, tip lying beneath the left
hemidiaphragm.
▪▪ If there are any concerns over the positioning, review the CXR with a senior colleague, make a
record in the medical notes, and do not commence feeding.

* NG tubes are used for short-term feeding for up to 6 weeks in patients with dysphagia or for those on ven-
tilators. Longer-term feeding is better delivered via a gastrostomy or jejunostomy tube.

140

K30031_Book.indb 140 9/6/17 1:35 PM


Case 5.19

Fig. 5.19A  Spot


image from an upper
GI contrast study.

A 53-year-old male presents to his GP with difficulty in swallowing solids and liquids over the
past 6 months, often with regurgitating of his food. This is associated with retrosternal chest pain.
Otherwise he feels well and has not lost any weight recently. He has no relevant past medical or
family history.
His observations are normal. Abdominal examination reveals a soft and nontender abdomen
with no organomegaly or palpable masses. His GP performs a set of routine bloods that reveals a
Hb of 129 g/L (130–180 g/L) and refers to the gastroenterology team. The patient could not toler-
ate an upper GI endoscopy and therefore the following imaging test is arranged (Figure 5.19A).

CASE 5.19: QUESTIONS


1 What is the imaging modality shown?
2 What is the abnormality demonstrated?
3 How would you further investigate this condition?
4 How would you manage this patient?

141

K30031_Book.indb 141 9/6/17 1:35 PM


5  Thoracic cases

CASE 5.19: ANSWERS


1 What is the imaging modality shown?
The imaging (Figure 5.19A) shows an upper GI contrast study called a barium swallow. Barium
solution is swallowed and coats the lining of the oesophagus, stomach, and duodenum.
• Barium studies of the upper GI tract have largely been replaced by upper GI endoscopy,
which allows more accurate mucosal assessment, avoids irradiation, and also provides a
biopsy facility. Barium studies still have a role, however, in the investigation of motility
disorders and in patients who cannot tolerate endoscopy.

2 What is the abnormality demonstrated?


There is a tight, smooth, narrowing of the lower oesophagus and gastro-oesophageal junction,
characteristic of achalasia. Prestenotic dilatation of the lower oesophagus is also noted. Contrast
has passed through the constriction into the stomach (Figure 5.19B).
• The constriction has a smooth tapered margin (‘bird’s beak’ sign). This is important in
differentiating it from a malignant stricture. The imaging shows no features of malignancy,
with no shouldering or visible mucosal erosion.

Fig. 5.19B  Spot image


from a barium swallow
showing dilation of the
lower oesophagus with
pooling of contrast (A)
above a tight smooth
narrowing (‘bird’s
beak’ sign) of the lower
oesophageal sphincter
(B). Some contrast has
passed through into the
fundus of the stomach,
with visible rugae (C).

142

K30031_Book.indb 142 9/6/17 1:35 PM


Case 5.19: Answers

Features of achalasia:

• Primary achalasia is failure of organised oesophageal peristalsis.


• There is impaired relaxation of the lower oesophageal sphincter.
• This is due to denervation of the muscles in the lower oesophagus, with unknown cause.
• This results in dilatation of the oesophagus and food stasis (Figure 5.19C).
• Both solids and liquids are equally affected, dysphagia is nonprogressive.
• Little or no weight loss.
• Involves a short segment (<3.5 cm) of the distal oesophagus.

Complications of achalasia:

• Oesophageal carcinoma, 5% of cases caused by chronic irritation of the mucosa by stasis of


food and secretions.
• Aspiration pneumonia.
• Candida oesophagitis.
• Acute airway obstruction (emergency) requiring
NG tube decompression.

Fig. 5.19C  Barium swallow


image from a different patient
with achalasia. This also shows
prestenotic dilatation of the lower
oesophagus with pooling of contrast
(A). There is smooth tapering of
the oesophageal stricture with a
‘bird’s beak’ abnormality (B). Air
and contrast (C) are seen within the
fundus of the stomach. Note the
food residue in the dilated upper
oesophagus (D).

143

K30031_Book.indb 143 9/6/17 1:35 PM


5  Thoracic cases

DIFFERENTIAL DIAGNOSIS OF ACHALASIA OF THE OESOPHAGUS


When presented with an image you will usually be asked for a differential diagnosis. In this case the
following need to be considered:

▪▪ Benign stricture.
– Achalasia (bird’s beak).
– Chagas disease (caused by protozoan Trypanosoma cruzi, endemic to Latin America with
multiple-organ involvement including myocarditis).
▪▪ Malignant stricture.
– Oesophageal malignancy or gastric carcinoma (irregular margin or shouldering of the stricture.
Clinical signs are also important).

3 How would you further investigate this condition?


Upper GI endoscopy, to exclude malignancy, is usually the first investigation for dysphagia.
Early signs of achalasia, however, can be subtle endoscopically and findings may appear normal.
Following endoscopy, patients with suspected achalasia will usually undergo a barium swallow.
Visualisation of barium passage in real time is most accurate in demonstrating dysmotility. In the
early stages, barium swallow may show loss of normal peristalsis and delayed emptying before
formation of a typical beak-like narrowing.
CT is only used if malignancy is suspected but may show a dilated thin-walled oesophagus
filled with fluid or food debris. It may also show evidence of aspiration pneumonia.

4 How would you manage this patient?


Treatment is aimed at allowing adequate drainage of the oesophagus into the stomach. A gastro-
enterologist usually manages this condition. Management options include:

• Calcium channel blockers (used as a bridge to definitive management).


• Endoscopic dilatation (85% effective).
• Local botulinum toxin injection at the level of the gastro-oesophageal sphincter (risk of
scarring to submucosa).
• Surgical myotomy (usually alongside Nissen fundoplication owing to risk of later reflux).

LEARNING POINTS: ACHALASIA OF THE OESOPHAGUS


▪▪ Achalasia classically produces a smooth, beak-like narrowing of the distal oesophagus, best
demonstrated on barium swallow.
▪▪ It is important to differentiate achalasia from a malignant stricture that presents with progressive
dysphagia and weight loss and produces an irregular stricture on barium swallow.
▪▪ 5% of patients with achalasia progress to develop oesophageal carcinoma.
▪▪ Endoscopic dilatation of achalasia is the mainstay of management; this also allows biopsies to be
taken in cases where malignancy is suspected.

144

K30031_Book.indb 144 9/6/17 1:35 PM


Case 5.20

Fig. 5.20A  Lateral oblique


image from a barium swallow
series.

A 67-year-old female presents to her GP with progressive dysphagia to solids and liquids with
absolute dysphagia over the past 24 hours. She regurgitates anything swallowed. Six months ago
she weighed 75 kg but today weighs 64 kg. She has been feeling more lethargic recently. She is a
heavy smoker with a 40 pack-year history and drinks 2–3 glasses of wine per night.
An upper GI endoscopy was attempted but could not pass the midoesophagus. A barium swal-
low is performed to further delineate the oesophageal anatomy (Figure 5.20A).

CASE 5.20: QUESTIONS


1 What does the barium study show? 3 What therapeutic options are available for
2 How would you manage this patient symptomatic treatment of this condition
initially? and what are the complications?

145

K30031_Book.indb 145 9/6/17 1:35 PM


5  Thoracic cases

CASE 5.20: ANSWERS


1 What does the barium study show?
The image shows an abrupt irregular stricture of the midthoracic oesophagus with mucosal irreg-
ularity and shouldering (Figure 5.20B). There is prestenotic oesophageal dilatation and minimal
passage of contrast through the stricture into the stomach.

• Features that favour a malignant cause for the stricture include a history of a progressive
then absolute dysphagia, recent weight loss, and smoking. Long-term reflux or Barrett’s
oesophagus are also significant risk factors (not present in this case).
• Examination findings might include cachexia and palpable cervical lymph nodes,
particularly Virchow’s node in the left supraclavicular fossa (Troisier’s sign), which is
associated with upper GI malignancy. Examine also for metastatic liver enlargement and
features of lung aspiration pneumonia.
• The tightness of the stricture and the asymmetric shouldering of the stricture margin also
suggest malignancy.

Fig. 5.20B  Lateral oblique image


from a barium swallow series
showing a tight midoesophageal
stricture. There is prestenotic
dilatation and pooling of contrast
(A) and irregular shouldering of
the oesophagus at the level of
the stricture (B). Minimal contrast
bypasses the stricture (C) to enter
the stomach (D). The normal
cardiac outline is also labelled (E).

146

K30031_Book.indb 146 9/6/17 1:35 PM


Case 5.20: Answers

2 How would you manage this patient initially?


Management of the patient depends on the likelihood of malignancy. She requires urgent assess-
ment and admission to hospital under gastroenterology because of the absolute dysphagia. This
patient has a number of the red flag symptoms for malignancy:

• Dysphagia.
• Vomiting.
• Anorexia.
• Weight loss.
• Upper GI bleeding (haematemesis or melena).
• Rapidly progressing symptoms.
• Iron deficiency anaemia.
• Abdominal mass.

These symptoms should point you towards the diagnosis and the need for further investiga-
tion. Be aware, however, that symptoms can overlap with benign conditions and formal investi-
gation and diagnosis is necessary in all cases.
A sensitive discussion with the patient should be had with support from a specialist nurse or
relatives to explain that malignancy is a diagnosis that is being considered.
Following review from a senior gastroenterologist the following investigations may be performed:

• Repeat endoscopy with biopsy for histological diagnosis.


• Endoscopic US to look for enlarged paraoesophageal nodes.
• A CT chest and abdomen for tumour staging (Figure 5.20C).
• PET-CT is often used as a supplementary staging technique.

Fig. 5.20C  Axial CT of the same patient with an obstructing oesophageal malignancy. This large tumour
(A) compresses the oesophageal lumen, which is no longer visible. The normal descending aorta (B) and
left atrium (C) are also labelled. CT will demonstrate mediastinal lymph node involvement (endoscopic US
more sensitive and allows biopsy), also lung/liver/bone/abdominal lymph node metastases.

147

K30031_Book.indb 147 9/6/17 1:35 PM


5  Thoracic cases

The differential diagnoses of an oesophageal stricture are:


• Benign oesophageal stricture – results from scarring from long-term reflux disease and
Barrett’s oesophagus (lower oesophagus).
• Postoperative stricture.
• Malignant oesophageal stricture.
• Oesophageal webs.
• Eosinophilic oesophagitis.
• Extrinsic compression of the oesophagus (from a lung mass, nodal mass or vascular anomaly).

3 What therapeutic options are available for symptomatic treatment of this condition
and what are the complications?
Oesophageal malignancy carries a poor prognosis and management is often palliative. In oper-
able patients with early stage tumours, oesophagectomy is the treatment of choice. Chemo-
radiation may be used in some patients preoperatively, and is the main form of treatment for more
advanced tumours.
Stents may be inserted using radiological or endoscopic techniques to relieve the stricture
for palliative symptomatic control. The stent is measured so that it crosses the full length of the
stricture and is self-expanding; a degree of ‘waisting’ in the middle is expected initially where the
stent traverses the tumour (Figure 5.20D).

Fig. 5.20D  An oesophageal malignancy


after stenting. The proximal oesophagus
is visualised above the upper part of the
stent (A). There is compression/waisting
from the tumour on either side of the
stent (B) between the upper (C) and lower
part of the stent (D). Contrast is seen
passing through the stent into the distal
oesophagus (E).

148

K30031_Book.indb 148 9/6/17 1:35 PM


Case 5.20: Answers

The complications of stent insertion are:

• Immediate.
• Oesophageal perforation.
• Early.
• Failure of the stent to relieve the obstruction owing to tumour overgrowth or
incomplete coverage of the stricture by the stent (sometimes additional stents are
needed).
• Late.
• Proximal or distal stent migration.
• Aspiration pneumonia.

The types of oesophageal malignancy are:

• Adenocarcinoma (arises from glandular cells, lower one-third of oesophagus).


• Associated with reflux and obesity.
• Follows Barrett’s oesophagus owing to longstanding inflammation causing metaplasia
and then dysplasia of the squamous cells to columnar cells and adenocarcinoma.
• Squamous cell carcinoma (arises from epithelial cells lining oesophagus).
• Associated with alcohol, smoking, and achalasia.

LEARNING POINTS: OESOPHAGEAL CARCINOMA


▪▪ If there are any red flag symptoms for malignancy refer to gastroenterology urgently using the
2-week-wait rule.
▪▪ A malignant oesophageal stricture may have an irregular contour with asymmetric shouldering on
barium swallow.
▪▪ Management is often palliative and based on symptom control.
▪▪ Endoscopic or radiological stent placement can palliatively relieve the obstruction.

149

K30031_Book.indb 149 9/6/17 1:35 PM


Case 5.21

Fig. 5.21A  AP erect CXR.

150

K30031_Book.indb 150 9/6/17 1:35 PM


Case 5.21: Questions

A 65-year-old male presents to his GP with heartburn, a chronic dry cough, and excess burp-
ing that has been present for the past year. He denies any weight loss, anorexia or shortness of
breath and is otherwise fit and well. He has tried a 4-week course of omeprazole, which has
not improved his symptoms, and he has had an Helicobacter pylori stool antigen test, which is
negative.
On examination he has a body mass index (BMI) of 33. His heart sounds are normal and chest
is clear. His abdomen is soft and he has minimal tenderness on palpation of the epigastrium.
A CXR is performed (Figure 5.21A).

CASE 5.21: QUESTIONS


1 What does the CXR show?
2 What are the long-term complications of this condition?
3 How would you manage this condition?

151

K30031_Book.indb 151 9/6/17 1:35 PM


5  Thoracic cases

CASE 5.21: ANSWERS


1 What does the CXR show?
The CXR shows a large hiatus hernia, with the gastric fundus projected behind the heart, and a
clear air–fluid level (Figure 5.21B).

• The gastric fundus has protruded through the oesophageal hiatus of the diaphragm into
the thoracic cavity.
• These are usually sliding hiatus hernias (90%) where the gastro-oesophageal junction has
displaced through the oesophageal hiatus. Rolling paraoesophageal hernias (10%) occur
when the gastro-oesophageal junction remains in its normal position while a portion of the
stomach herniates above the diaphragm.
• Most hiatus herniae are asymptomatic, some may present with reflux symptoms,
postprandial fullness, chest pain or nausea and vomiting.
• Risk factors for hiatus hernia are those that raise intra-abdominal pressure, such as obesity,
heavy lifting, chronic cough, and pregnancy.
• The main differential for this appearance on a CXR is a cavitating lung mass. It is often
possible, however, to differentiate between the two on CXR but ask for senior advice if
unsure. CT is helpful in equivocal cases or to aid with hernia repair surgical planning
(Figure 5.21C).

Fig. 5.21B  AP erect CXR showing a large hiatus hernia with protrusion of the gastric fundus (A) into the
thorax with an air–fluid level (B) above the left hemidiaphragm (C).

152

K30031_Book.indb 152 9/6/17 1:35 PM


Case 5.21: Answers

Fig. 5.21C  Axial CT of the thorax confirming a large sliding hiatus hernia. The gastric fundus with air–
fluid level (A) and NG tube (B) is seen above the diaphragm, behind the heart (C). The normal descending
thoracic aorta (D) and liver (E) are also labelled.

2 What are the long-term complications of this condition?


The long-term complications include the risks of developing incarceration of the herniated bowel
or development of a gastric volvulus. Associated gastro-oesophageal reflux disease (GORD)
over many years can lead to the development of strictures, ulcers, and subsequent bleeding.
Additionally, with long-term reflux the risk of oesophageal adenocarcinoma secondary to Barrett’s
oesophagus is increased and this condition will require endoscopic surveillance.

EXAM TIP
When asked about complications of a condition in an exam, make sure to structure your answer. The
easiest way to do this is to break it down into immediate, early, and late complications. This is especially
helpful in surgical conditions where, for example, bleeding may be an immediate complication, sepsis an
early complication, and failure of the procedure a late complication.

3 How would you manage this condition?


Management options depend on whether the hernia is symptomatic or not. Most are incidental
findings on CXR or endoscopy that require no specific management. Some, as in this case, are
found through investigation for chronic acid reflux. Another CXR case is shown in Figure 5.21D.
Upper GI endoscopy may be used to determine any complications associated with GORD such
as strictures, bleeding or ulceration, which can happen over time in severe cases.

153

K30031_Book.indb 153 9/6/17 1:35 PM


5  Thoracic cases

Fig. 5.21D  A large hiatus hernia (white arrows) projected overlying the cardiac silhouette. Note the air–
fluid level (A).

Management of hiatus hernias can be divided into conservative, medical, and surgical options:

• Conservative: if asymptomatic then there is no indication for treatment.


• Medical: if reflux symptoms are troubling then a trial of proton pump inhibitors (such as
omeprazole or lansoprazole) may be helpful alongside lifestyle modification (e.g. smoking
cessation and weight loss).
• Surgical: if medical management of GORD has failed and there are complications of the
hiatus hernia (e.g. strictures, ulcers, bleeding). Those patients with large, symptomatic
paraoesophageal hernias are at risk of incarceration and surgical repair is usually advised
prior to this happening. Nissen fundoplication is the procedure most often performed for
both types of hernia.

LEARNING POINTS: HIATUS HERNIA


▪▪ Hiatus herniae are usually asymptomatic and picked up incidentally on CXR.
▪▪ There are two main types of hiatus hernia: sliding (90%) and paraoesophageal (10%).
▪▪ Complications can be related to the hernia itself, such as incarceration in paraoesophageal
hernias, or to long-term reflux symptoms such as ulceration.
▪▪ Surgical repair is indicated in most paraoesophageal hernias or for sliding hernias that are
particularly large or associated with resistant symptoms.

154

K30031_Book.indb 154 9/6/17 1:35 PM


Case 5.22

Fig. 5.22A  CXR.

A 87-year-old male patient presents to his GP with recent discomfort over his left chest. On exam-
ination there is obvious deformity over his left thorax and there is minor tenderness of the left
upper ribs, which he noticed after gardening. He appears otherwise well. A CXR is arranged for
further assessment (Figure 5.22A).

CASE 5.22: QUESTIONS


1 What are the CXR findings?
2 What is the diagnosis?
3 What surgical treatments were used for this condition in the past?

155

K30031_Book.indb 155 9/6/17 1:35 PM


5  Thoracic cases

CASE 5.22: ANSWERS


1 What are the CXR findings?
The CXR shows a severe thoracic deformity with crowding of abnormally modelled ribs and
absent ribs in the left upper zone (Figure 5.22B). There is left hemithoracic volume loss with tra-
cheal deviation to the left and a raised left hemidiaphragm. The left costophrenic angle is blunted,
likely related to pleural thickening. Calcified granulomas are noted in the right upper zone and
left lung.

Fig. 5.22B  A CXR showing loss of volume in the left hemithorax. The left upper ribs are deformed and
crowded (A) and calcified granulomata can be seen in both lungs (B). The costophrenic angle on the left is
blunted, likely longstanding (C).

2 What is the diagnosis?


The patient has had a previous left thoracoplasty with rib resection for TB, as evidenced by the
CXR findings. Old calcified lung granulomata are indicative of previous TB infection.

156

K30031_Book.indb 156 9/6/17 1:36 PM


Case 5.22: Answers

3 What surgical treatments were used for this condition in the past?
Thoracoplasty was used from the early 1900s through to the late 1950s as one form of collapse
therapy for TB prior to anti-TB medical therapy (Figure 5.22C). These historical techniques are re-
emerging as a treatment option owing to the increasing incidence of multidrug resistant TB strains.
Collapse therapy was based on the observation that healed TB cavities were closed and that
closing the cavities helped to inactivate the disease. In thoracoplasty this was achieved through
resection of multiple ribs.

Fig. 5.22C  Another patient (female) with evidence of previous left thoracoplasty. Note the extensive left
pleural calcifications (arrows) – this is likely to be the result of a healed TB empyema, although oleothorax
may also cause a pleural calcified lesion (oleothorax tends to be more circumscribed).

Procedure types are:

• Intrapleural thoracoplasty: involves multiple rib excisions as well as resection of the parietal
pleura, periosteum, intercostal muscles, and intercostal neurovascular bundles.
• Extrapleural thoracoplasty: the rib periosteum, intercostal muscle, and parietal pleura are
preserved.
• Plombage thoracoplasty: Plombe or filler (well-circumscribed radiopaque or radiolucent
densities) are inserted in the space created between the rib cage, endothoracic fascia, and
periosteum (Figure 5.22D).
• Phrenic nerve crush: diaphragm paralysis (look for scar in the supraclavicular fossa), often
used in combination with an artificial pneumothorax.

157

K30031_Book.indb 157 9/6/17 1:36 PM


5  Thoracic cases

Fig. 5.22D  CXR showing previous left plombage treatment. Note the multiple lucent spheres in the left
upper zone (arrows) and also calcified lung granulomata.

• Apical lobectomy.
• Recurrent medical pneumothoraces.
• Oleothorax: involved insertion of an oil filled capsule between ribs and pleura to collapse
adjacent lung. These lesions may be seen as calcified pleural masses.

4 How would you manage this patient?


This patient has no clinical evidence to suggest TB recurrence and has reassuring chest radio-
graph findings. His rib discomfort is likely to be musculoskeletal in origin after gardening. He can
be discharged with an invitation to return if the thoracic discomfort persists/worsens.

LEARNING POINTS: THORACOPLASTY


▪▪ Thoracoplasty was used to treat TB prior to the introduction of medical therapy and is
recognisable on CXR as upper zone volume loss with chest wall deformity.
▪▪ Other forms of surgical collapse treatment included plombage and oleothorax.
▪▪ The radiological findings are characteristic and once seen are readily recognisable.

158

K30031_Book.indb 158 9/6/17 1:36 PM


Case 5.23

Fig. 5.23A  CXR.

A 55-year-old Caucasian male presents to his GP with a persistent dry cough that has been pres-
ent for 2 months. He has been feeling more lethargic than usual and has noticed about 6 kg of
recent weight loss.
On examination, there is palpable, firm, nontender cervical lymphadenopathy. The chest is
clear, heart sounds are normal, and the abdomen is soft with no palpable masses. A set of bloods
is performed and a CXR is arranged (Figure 5.23A). His blood results are:
Hb 116 g/L (130–180 g/L) Potassium 4.2 mmol/L (3.2–5.1 mmol/L)
MCV 92 fL (80–100 fL) Urea 4.2 mmol/L (1.7–8.3 mmol/L)
WBC 4.6 × 109/L (4.0–11.0 × 109/L) Creatinine 86 micromol/L (62–106 micromol/L)
CRP 9 mg/L (<5 mg/L) Corrected calcium 2.65 mmol/L (2.15–2.55 mmol/L)
Sodium 140 mmol/L (135–146 mmol/L)

CASE 5.23: QUESTIONS


1 What is the key radiological finding? 4 How would you further investigate this
2 What are the differential diagnoses? patient?
3 What further imaging is necessary?

159

K30031_Book.indb 159 9/6/17 1:36 PM


5  Thoracic cases

CASE 5.23: ANSWERS


1 What is the key radiological finding?
The CXR shows bilateral symmetrical lobulated enlargement of the hila secondary to lymph-
adenopathy (Figure 5.23B).

Fig. 5.23B  CXR showing bilateral hilar lymphadenopathy (A). The pulmonary arteries cannot be
separated from the hilar masses, confirming the origin of the masses to the hila. Also note the normal aortic
arch (B) and left heart border (C) seen clearly separate to the mass, confirming the mass is not anterior and
abutting the heart (border would be lost).

2 What are the differential diagnoses?


The differential diagnoses for bilateral hilar lymphadenopathy are:

• Symmetrical.
• Sarcoidosis.
• Lymphoma.
• Chronic lymphoid leukaemia (CLL).
• Asymmetrical (or unilateral) – note symmetrical causes may also cause asymmetric
changes.
• Tuberculosis.
• Lung malignancy.

160

K30031_Book.indb 160 9/6/17 1:36 PM


Case 5.23: Answers

In this patient, the clinical information and CXR suggest sarcoidosis; however, it is usually
not possible to distinguish the cause of bihilar lymphadenopathy without further investigations.

EXAM POINT
Know a little about each of the causes of bilateral hilar lymphadenopathy as they are common and
should not be missed:

▪▪ Bilateral.
– Malignancy (CLL, lymphoma).
– Sarcoidosis.
▪▪ Asymmetrical.
– Tuberculosis.
– Malignancy (lung, metastases).

3 What further imaging is necessary?


CT of the thorax and abdomen is necessary to confirm the findings and allow differentiation of
the masses within the hilum. It also allows visualisation of the lungs and a review for abnormali-
ties and lymph nodes elsewhere.
A CT thorax of the same patient is shown (Figure 5.23C). Try to identify the normal and
abnormal structures yourself.

EXAM POINT
When asked in an exam what investigations you would perform, remember to structure your answer.
Always remember to mention you would take senior advice prior to arranging more advanced investiga-
tions, after bedside baseline tests.

▪▪ Bedside tests – bloods, urine dip/MSU, ECG, cardiac monitoring, blood cultures, ABG/VBG
(as applicable, be able to say why you would perform each test).
▪▪ Imaging – be specific about which part of the body you wish to image, which investigation, and
why.
▪▪ More invasive investigations – e.g. endoscopy, biopsy, interventional radiology.

• Sarcoidosis is a disease of unknown aetiology involving abnormal collections of


inflammatory noncaseating chronic granulomas. Initially it tends to affect the lungs, skin or
lymph nodes; however, it can affect multiple other organs such as the heart, liver, brain, and
eyes. Often there are few or no symptoms but it may cause shortness of breath and cough.
An early sign is a high blood serum calcium level (with normal parathyroid hormone). Later
clinical findings in progressive respiratory disease are due to lung fibrosis, which can cause
severe right-sided heart failure.
• Thoracic sarcoidosis is staged according to the imaging findings:
• Stage 0 - normal CXR findings.
• Stage I - bilateral hilar lympadenopathy.
• Stage II - bilateral hilar lymphadenopathy and pulmonary infiltrates.
• Stage III - pulmonary infiltrates alone.
• Stage IV - end-stage lung disease with pulmonary fibrosis and honeycombing.

161

K30031_Book.indb 161 9/6/17 1:36 PM


5  Thoracic cases

Fig. 5.23C  CT thorax of the same patient showing bilateral hilar lymphadenopathy (arrows). Normal
mediastinal structures including the ascending aorta (A), pulmonary trunk (B), right pulmonary artery (C),
descending aorta (D), and air-filled oesophagus (G) are demonstrated. Note the normal black and air-filled
right (E) and left main bronchi (F).

4 How would you further investigate this patient?


Further investigations are necessary to confirm the diagnosis and to assess the extent of lung
disease. These may be started by the GP but would also require a referral to a specialist respira-
tory consultant.

• Bloods: FBC and CRP to look for infection and a blood film to look for leukaemic cells.
Check serum calcium as this can be high in sarcoidosis and malignancy. Serum ACE is also
increased; however, this is a nonspecific marker and rarely used.
• CT scan, for the reasons discussed above. HRCT reformats are used to look at the lungs in
more detail.
• Image-guided core biopsy of an accessible lymph node, (this patient had an US-guided core
biopsy of a cervical lymph node that confirmed sarcoid granuloma infiltration).
• Bronchoscopy with washout if sarcoidosis or TB suspected, with biopsy of any lesions for
histology and confirmation of diagnosis.
• Pulmonary function tests and a diffusion capacity of the lung for carbon monoxide (DLCO)
test are used routinely in evaluation and follow-up of patients with sarcoidosis and chronic
lung disease.

Treatment with corticosteroids is only required if the patient is symptomatic. NSAIDs are
used for any associated arthralgia (which is common). In patients who do not respond to steroid

162

K30031_Book.indb 162 9/6/17 1:36 PM


Case 5.23: Answers

therapy or where it is not tolerated, immune modulating therapy is used (e.g. methotrexate, aza-
thioprine, infliximab).
Sarcoid eye disease with neuro-ophthalmic involvement can present with diplopia owing to
cranial nerve palsies or decreased vision caused by optic nerve infiltration/oedema. Ocular sar-
coidosis presents with symptoms of uveitis (blurred vision, photophobia, floaters, redness, and
pain). Mass lesions can also develop. These need urgent review by an ophthalmologist owing to
the risk of blindness. Systemic disease is more likely to be progressive in these patients.

TOP TIP
You will be expected to recognise bihilar lymphadenopathy on CXR and suggest a differential diagnosis
as well as further investigations such as bloods, imaging (CT), and referral to a respiratory consultant. Be
able to discuss differential diagnoses such as sarcoidosis, tuberculosis, and lung malignancy.

LEARNING POINTS: SARCOIDOSIS, BILATERAL HILAR LYMPHADENOPATHY


▪▪ Bilateral hilar lymphadenopathy is a common exam scenario and you should be able to recognise
the CXR findings.
▪▪ Common causes of bilateral hilar lymphadenopathy include sarcoidosis, lymphoma, and
tuberculosis.
▪▪ Thoracic sarcoidosis is the most common form and is staged according to the imaging findings,
which range from normal to significant fibrosis.
▪▪ Diagnosis is best confirmed with lymph node or lung nodule biopsy or via bronchoscopy.

163

K30031_Book.indb 163 9/6/17 1:36 PM


Case 5.24

Fig. 5.24A  CXR.

164

K30031_Book.indb 164 9/6/17 1:36 PM


Case 5.24: Questions

A 75-year-old female presents to her GP with a sensation of fullness in her neck, worse when she
lies flat and also mildly increasing shortness of breath on exercise. On clinical examination there
is a firm, nontender swelling in her neck, palpable above the manubrium. A CXR is arranged
initially (Figure 5.24A).

CASE 5.24: QUESTIONS


1 What are the CXR findings?
2 What is the diagnosis? Is there a differential diagnosis?
3 How would you investigate and manage this patient?

165

K30031_Book.indb 165 9/6/17 1:36 PM


5  Thoracic cases

CASE 5.24: ANSWERS


1 What are the CXR findings?
The CXR shows a large mediastinal mass with a lobulated contour that involves the superior,
anterior, and middle mediastinum. The trachea is deviated to the right (Figure 5.24B).

Fig. 5.24B  CXR showing a large mediastinal mass (white dotted line) involving the superior, anterior, and
middle mediastinum. Note the silhouette of the pulmonary vessels is obliterated by the mass, indicating
contact with the hila in the middle mediastinum (hilum overlay sign). The trachea is deviated to the right
(arrow). Note also the mediastinal mass extends up to and above the clavicles but the supraclavicular
portion is not well defined.

2 What is the diagnosis? Is there a differential diagnosis?


The likely diagnosis is that of a multinodular goitre of the thyroid with retrosternal, intratho-
racic extension. As can be seen from the CXR, the intrathoracic component is significant and
the patient describes symptoms of mass effect with pressure effects in the neck and shortness of
breath, which may relate to tracheal compression/deviation. Dysphagia may also occur in large
thyroid lesions.

166

K30031_Book.indb 166 9/6/17 1:36 PM


Case 5.24: Answers

Clinically patients with goitre are often hypothyroid, although a toxic nodule within the goitre
may cause hyperthyroidism.
As described this is a large mediastinal lesion involving several compartments. Typically an
intrathoracic goitre tends to involve the anterior mediastinum and is one of the causes of an ante-
rior mediastinal mass.
Causes of an anterior mediastinal mass – think of the 4 ‘Ts’:

• Thyroid.
• Teratoma.
• Thymus (thymic tumour, e.g. thymoma).
• ‘Terrible’ lymph nodes, usually lymphoma.
Radiologically on CXR, the anterior mediastinum does not extend above the clavicles, there-
fore any mass that clearly extends above this level towards the root of the neck is likely to be
extending from the neck itself (cervicothoracic sign).

3 How would you investigate and manage this patient?


This patient requires full history and examination. Look particularly for evidence of hypo/hyper-
thyroidism in the history/examination. A thyroid lump is a common OSCE scenario; familiarise
yourself with thyroid examination.
Baseline bloods, i.e. FBC/TFTs, clearly are important.

Fig. 5.24C  Coronal postcontrast thoracic CT confirming a large lobulated multinodular goitre with
retrosternal extension into the mediastinum (white dotted line). Note the right (R) and left (L) lobes of the
thyroid and communication of the mediastinal thyroid mass with the left lobe (arrow A).

167

K30031_Book.indb 167 9/6/17 1:36 PM


5  Thoracic cases

In view of the significant intrathoracic extension of the goitre and evidence of mass effect
plus possible airway compromise, this patient requires urgent referral to an ear, nose, and throat
(ENT) department.
Once in hospital, imaging investigations will include:

• US neck/thyroid to confirm multinodular nature of the goitre and demonstrate any


suspicious intrathyroid lesions requiring fine-needle aspiration cytology. US will also assess
the neck for suspicious nodes, but will not be able to access the intrathoracic component.
• CT neck/chest – to accurately delineate the size and extent of the thyroid enlargement
(Figure 5.24C). This will precisely gauge the degree of tracheal deviation/compression and
also the relationship of the goitre to intrathoracic vessels. The need for surgical intervention
will depend on the operative fitness of the patient and whether airway compression is of
concern (surgery will require ENT/cardiothoracic approach).

LEARNING POINTS: MULTINODULAR THYROID/SUPERIOR MEDIASTINAL MASS


▪▪ Multinodular goitres are seen more commonly in women aged 35–50 years. The gland may be
hyperfunctioning or hypofunctioning.
▪▪ Nodules may harbour malignancy and US is performed to look for any suspicious features or
dominant nodules, (large, hypervascular, with microcalcifications) that may require fine-needle
aspiration and cytology.
▪▪ Thyroid goitre is on the differential list for an anterior/superior mediastinal mass on CXR, and is
associated with the cervicothoracic sign and also deviation of the trachea.
▪▪ CT can be useful to fully characterise the extent of a retrosternal goitre.
▪▪ Treatment may be indicated if the goitre is symptomatic (hyperfunctioning or hypofunctioning
thyroid) or if it is causing significant mass effect.

168

K30031_Book.indb 168 9/6/17 1:36 PM


Case 5.25

Fig. 5.25A  Lateral oblique mammographic views of both breasts (R/L).

A 57-year-old female presents to her GP having felt a lump in her left breast. On examination
there is a fixed, nonmobile nodule in the upper outer quadrant of her left breast. The GP refers the
patient urgently to the local breast unit. A mammogram is arranged following clinical assessment
(Figure 5.25A).

CASE 5.25: QUESTIONS


1 What are the imaging findings?
2 What are the indications for mammography?
3 What do you know about mammography?
4 How would you further investigate and manage this patient?

169

K30031_Book.indb 169 9/6/17 1:36 PM


5  Thoracic cases

CASE 5.25: ANSWERS


1 What are the imaging findings?
The mammogram shows a lobulated dense, ill-defined mass in the superior aspect of the left
breast with overlying skin retraction, in keeping with malignancy (Figure 5.25B). Benign lesions
tend to be circumscribed and smooth with no associated parenchymal or skin changes – all cases
will need US for further characterisation.

Fig. 5.25B  Magnified lateral oblique mammographic view of the left breast showing a mass in the
upper breast (A). This is lobulated and poorly defined in part, and there is distortion of the adjacent breast
parenchyma (B). Note the overlying skin retraction (C – clinically this will appear as a dimple) and normal
pectoralis major muscle shadow (D).

2 What are the indications for mammography?


Indications for mammography:

• Screening programme (in the UK ages vary, currently screening is being offered to women
aged 47–73 years).
• Assessment of symptomatic breast patients (usually >35 years old).
• Follow-up of previously treated breast cancer patients.

3 What do you know about mammography?


Mammography is a dedicated radiographic technique for imaging the breast. It is used for both
screening and diagnostic symptomatic imaging. Two standard views are obtained, craniocaudal
(CC) and mediolateral oblique (MLO).

170

K30031_Book.indb 170 9/6/17 1:36 PM


Case 5.25: Answers

Digital mammography (full field digital mammography: FFDM) is a newer technique that
provides higher resolution imaging and is more sensitive in younger women with denser breast
tissue. Mammography does involve a radiation dose to the breast, although this is not large and
is reduced in newer digital units.

4 How would you further investigate and manage this patient?


The assessment of the symptomatic breast involves the concept of triple assessment: this is the key
phrase to mention in the exam. Triple assessment of the symptomatic breast includes:

• Clinical assessment (examination of both breasts and axillae and supraclavicular fossae/
neck if there is a lump). In the exam you will usually encounter a mannequin or actor with
a prosthetic breast. Remember the axillae and always check you are not hurting the patient.
Warn the patient if you have cold hands before you examine her!
• Imaging with either mammography and/or US.
• US only for women <35 years old.
• Mammography for women >35 years old (due to the less dense breasts) and US of breast
and ipsilateral axilla if palpable or mammographic abnormality (Figures 5.25C and 5.25D).
• US-guided core biopsy and histology if needed.

Fig. 5.25C  US of a left axillary lymph node (callipers) showing features of malignant infiltration including
enlarged size, eccentric thickened cortex (A), and a displaced node hilum (B).

171

K30031_Book.indb 171 9/6/17 1:36 PM


5  Thoracic cases

Fig. 5.25D  US of the left breast mass showing an ill-defined, hypoechoic mass (arrows) with posterior
acoustic shadowing (arrows S) consistent with a carcinoma.

MRI of the breasts is not used in initial assessment but has a valuable role as a problem-solving
tool – MRI is also excellent in the evaluation of the treated breast and breast implants.
Further investigation and management includes completion of the triple assessment with
US-guided core biopsy of the breast mass and core biopsy or fine-needle aspiration of any suspi-
cious axillary lymph nodes.
A score is given for each aspect of the triple assessment (Examination, Imaging, and Cytology)
between 1 and 5. The higher the score, the greater the suspicion of malignancy. The above patient
received a score of E4 (suspicious lump on examination), M5/U5 (malignant on mammogram
and US), and B5 (malignant on histology).
These results are discussed at a multidisciplinary team meeting involving surgeons, oncolo-
gists, radiologists, histopathology, and nurse specialists.
The results are then carefully explained to the patient with counselling offered and discus-
sion of management options. Management is likely to involve surgery to the breast (wide local
­excision  or mastectomy) and also axillary surgery. Postoperatively, the patient may require
­chemotherapy or radiotherapy depending on tumour type and node status.

172

K30031_Book.indb 172 9/6/17 1:36 PM


Case 5.25: Answers

PATHOLOGY OF BREAST CANCER


Virtually all breast cancers are adenocarcinomas (tumour derived from epithelial cells of glands or ducts):

▪▪ Preinvasive carcinomas are tumours confined to the ducts or the acini of the lobules without
infiltration of the basement membrane. These are called either ductal or lobular carcinomas in situ
(DCIS/LCIS).
▪▪ Invasive carcinomas are malignant tumours that have penetrated the basement membrane of
the tissue of origin and spread to other tissues. Of these, 80% are invasive ductal carcinoma and
10–15% are invasive lobular carcinomas.

Tumours may spread either locally (directly into the surrounding tissue), via lymph nodes (to the axillary
and periclavicular nodes) or via the blood (to the lungs, bones, liver, brain, and adrenal glands).

TREATMENT OPTIONS FOR BREAST CANCER


▪▪ Surgery gives the best outcomes with either wide local excision or mastectomy. Surgery is
combined with sentinel axillary node sampling or axillary clearance depending on tumour type
and patient.
▪▪ Adjuvant radiotherapy is given to the chest wall after mastectomy for tumours with a high risk of
local recurrence.
▪▪ Adjuvant chemotherapy improves survival, particularly in younger patients with node-positive
disease. Anthracyclines are usually combined with other agents. Hormone therapy (tamoxifen
blocks oestrogen receptors, aromatase inhibitor blocking oestrogen production) is used longer
term postsurgery in oestrogen-positive tumours. Some breast cancers are stimulated by human
epidermal growth factor receptor 2 (HER-2), while biological therapy, e.g. trastuzamab, blocks the
effects of HER-2.

LEARNING POINTS: BREAST LUMPS/BREAST CARCINOMA


▪▪ Examination of a breast lump is a common OSCE scenario.
▪▪ Triple assessment, involving clinical examination, imaging, and biopsy with discussion at a MDT
meeting, is key to initial management.
▪▪ Mammography is less effective at detecting the early changes of breast cancer in women <35
years, as their breast tissue is often dense. US usually initially preferred in patients below this age
with a palpable abnormality.
▪▪ Always review both CC and MLO images for any mammogram and be able to orientate yourself.
Comment on any asymmetry, masses, calcification, skin changes, and enlarged axillary nodes.

173

K30031_Book.indb 173 9/6/17 1:36 PM


K30031_Book.indb 2 9/6/17 1:34 PM
Cardiovascular cases

6 HANNAH ADAMS, SARAH HANCOX,


CRISTINA RUSCANU, AND
DAVID C HOWLETT

Case 6.1 176 Case 6.4 190


Case 6.2 181 Case 6.5 195
Case 6.3 185 Case 6.6 199

175

K30031_Book.indb 175 9/6/17 1:36 PM


Case 6.1

Fig. 6.1A  Axial contrast-enhanced CT thorax at the level of the pulmonary trunk.

176

K30031_Book.indb 176 9/6/17 1:36 PM


Case 6.1: Questions

A 63-year-old male presents to the ED with a severe, constant, sharp, central chest pain radiating
to his back, which started 1 hour ago. He is on medication for hypertension and has a 40 pack-year
smoking history.
On examination he looks pale. Heart sounds are normal and his chest is clear. He is tachy-
cardic at 110 bpm and has a BP in the right arm of 140/96 mmHg and in the left arm of 104/74
mmHg. Radial pulses are present but weaker in the left than the right and femoral pulses also
are weak.
The ECG shows a sinus tachycardia and the CXR reports a small left pleural effusion only.
After discussion with a senior colleague, you organise a CT thorax (Figure 6.1A).

CASE 6.1: QUESTIONS


1 What does the CT thorax show?
2 What is the likely diagnosis?
3 How would you manage this patient?
4 What are the complications of this condition?

177

K30031_Book.indb 177 9/6/17 1:36 PM


6  Cardiovascular cases

CASE 6.1: ANSWERS


1 What does the CT thorax show?
This CT demonstrates an aortic dissection with an intimal dissection flap involving both the
descending and ascending aorta. The following structures are identifiable (Figure 6.1B):

• Ascending aorta true lumen with visible intimal flap and false lumen.
• Descending aorta with intimal flap and false lumen.

Fig. 6.1B  Postcontrast CT thorax at the level of the pulmonary trunk. The ascending aorta true lumen is
seen (A) with visible intimal flap (B) and false lumen (C). Descending aorta true lumen (F), intimal flap (G)
and false lumen (H). Note: left basal pleural effusion (I), normal main pulmonary artery (D), and right main
pulmonary artery (E).

2 What is the likely diagnosis?


The history and imaging findings are consistent with a diagnosis of aortic dissection. This is
where there is a tear in the tunica intima of the aorta causing blood to flow between the layers
of the wall of the aorta and forcing the layers apart. This classically presents with severe, tearing
chest pain, radiating to between the scapulae. This is often in a patient with cardiac risk factors
such as smoking, high blood pressure or high cholesterol.
If suspected, then an urgent CT aortogram with contrast is necessary alongside an echocar-
diogram to look at the aortic root (as this is more accurate for diagnosis of this type of dissection
where the dissection extends to involve the aortic root and there may be acute aortic regurgita-
tion, pericardial effusion, and also coronary artery involvement). You will need support from
senior colleagues and both ITU and cardiothoracic involvement.

178

K30031_Book.indb 178 9/6/17 1:36 PM


Case 6.1: Answers

There is a system of classification dependent on the extent of the dissection diagnosed on


imaging and this informs the management (Table 6.1 and Figure 6.1C).
Table 6.1  Stanford system of classification of aortic dissection
Type A Involves the ascending aorta and the aortic arch (AA) (proximal to the origin of left subclavian
artery) +/− descending aorta. The tear can originate in the ascending aorta, the AA or in the
descending aorta
Type B Involves the descending aorta or AA (distal to the origin of left subclavian artery), without
involvement of the ascending aorta/arch proximal to the left subclavian artery

Fig. 6.1C  A sagittal reconstruction of a CT aortogram postcontrast in another patient. It shows a type
B aortic dissection with origin (O) distal to the left subclavian artery (A) involving the descending thoracic
aorta (B) with a false lumen on the outside (C) and visible intimal flap (D). It does not involve the ascending
aorta or arch thereby classifying it as a type B dissection.

3 How would you manage this patient?


Management is via an ABCDE approach. These patients can become unstable very quickly; there-
fore, it is necessary for continuous cardiac monitoring with initial management in a specialist
cardiology/cardiothoracic unit:
• Bloods – FBC to check Hb levels, U&Es as patient can be in shock, clotting factors as
any derangement will need to be corrected, serum troponin to ensure that there is no
concurrent myocardial infarction (MI), and cross match as patient could need a transfusion.

179

K30031_Book.indb 179 9/6/17 1:36 PM


6  Cardiovascular cases

• It is important to perform an ECG and check cardiac enzymes as cardiac blood supply can
be affected in type A dissections.
• Early imaging is vital for prompt diagnosis and decision on management.

Acute aortic dissection can be treated surgically or medically. This is dependent on the patient’s
current clinical condition, comorbidities, and type of dissection (extent):

• Type A: emergency surgical repair.


• Type B: if a complicated type B dissection, then surgery is indicated. Otherwise, medical
management is the mainstay of treatment.

4 What are the complications of this condition?


Complicated type B dissections involve the following:

• Propagation of aorta (increasing diameter).


• Increasing size of haematoma.
• Compromise of major branches of the aorta.
• Impending rupture.
• Persistent pain despite adequate pain management.
• Bleeding into the pleural cavity.
• Development of saccular aneurysm.

Medical management remains the treatment of choice for type B dissections unless they are
leaking, ruptured or complicated (see above). The principles also relate to patients awaiting sur-
gery in order to decrease the intimal tear and propagation of the dissection, and also postopera-
tively. Antihypertensive treatment in the form of beta-blockers (IV labetolol) is usually used with
continuous cardiac monitoring.
Surgical management aims to alleviate the symptoms and decrease the frequency of complica-
tions. Ultimately the surgeon aims to prevent aortic rupture and death. The affected layers of the
aorta are sutured together and the aorta is reinforced with a Dacron graft. Advances in stenting
technology have also allowed this technique to be used by interventional radiologists in more
stable type B dissections.

TOP TIPS
A history of severe, sudden-onset, tearing chest pain with unequal four limb blood pressures in a sus-
ceptible patient should raise your suspicion of aortic dissection. ABCDE management and early involve-
ment of a senior doctor is important. Prompt imaging will help with your diagnosis.

LEARNING POINTS: AORTIC DISSECTION


▪▪ Aortic dissection presents with severe, sudden-onset, tearing chest pain radiating between the
scapulae.
▪▪ CT imaging will show a false lumen within the aorta but an echocardiogram is more accurate for
evaluation of aortic root involvement.
▪▪ Dependent on the level of the dissection, it can be classified as type A (involving ascending aorta)
or type B (descending aorta only below the level of the left subclavian artery).
▪▪ Emergency management of type A dissection is surgical while type B is often managed medically.

180

K30031_Book.indb 180 9/6/17 1:36 PM


Case 6.2

Fig. 6.2A  Axial arterial enhanced CT (‘aortogram’) at the level of the umbilicus.

A 68-year-old male presents to the ED with generalised abdominal pain that has been worsen-
ing over the last 24 hours and now radiates to his back. He does not report any change in bowel
habit or recent weight loss. He is on antihypertensive medication and reports that he has a high
cholesterol level. He smoked a pipe for 40 years but has now given this up.
On examination, he appears pale and is in some discomfort. He has a BMI of 32. He is hypo-
tensive at 110/72 mmHg and tachycardic at 90 bpm. He has an SPO2 of 98% on air. Heart sounds
are normal and his chest is clear. His abdomen is generally tender, most marked at the umbilicus,
but not distended and there is no palpable tenderness in the loins or down the spine. His blood
results are as follows:
Hb 109 g/L (130–180 g/L) LFTs normal
WBC 9 × 109/L (4.0–11.0 × 109/L) Serum amylase normal
CRP 8 mg/L (<5 mg/L)

The initial FAST scan (focused assessment with sonography for trauma) performed in the ED
is nondiagnostic owing to bowel gas, and a CT scan is arranged (Figure 6.2A).

CASE 6.2: QUESTIONS


1 What does the CT scan show? 3 How would you manage this patient?
2 What other imaging might show this 4 How might this condition be prevented in
abnormality in an emergency situation? the older population?

181

K30031_Book.indb 181 9/6/17 1:36 PM


6  Cardiovascular cases

CASE 6.2: ANSWERS


1 What does the CT scan show?
The CT ‘aortogram’ shows a ruptured AAA, which is a surgical emergency (Figures 6.2B and
6.2C). There is a contained leak from the AAA, which will lead to further bleeding and ultimately
death. Urgent surgical (or radiological) intervention is necessary.
Leaking AAA can present with very nonspecific symptoms ranging from abdominal pain
to back pain, and can often be misdiagnosed as renal colic or pyelonephritis. A retroperitoneal
­haematoma may compress the ureter and cause dipstick-positive haematuria causing further
potential confusion with renal colic.
AAA may be associated with:
• Atherosclerosis (most common).
• Chronic aortic dissection.
• Vasculitis, e.g. Takayasu arteritis.
• Connective tissue disorders, e.g. Marfan’s or Ehlers–Danlos syndrome.

Complications of AAA are:


• Bleeding due to leak or rupture.
• Fistula (aortoenteric would cause life-threatening bleeding into the bowel).

Fig. 6.2B  CT ‘aortogram’ showing a ruptured AAA. Note the large 9 cm aortic aneurysm (A), aortic lumen
with contrast (B), mural calcification (C), mural thrombus (D), and retroperitoneal haemorrhage (E). Also
demonstrated are the chronically scarred and atrophic right (F) and left (G) kidneys. Note the anterior and
right margins of the aneurysm are not well seen owing to adjacent retroperitoneal haematoma.

182

K30031_Book.indb 182 9/6/17 1:36 PM


Case 6.2: Answers

2 What other imaging might show this abnormality in an emergency situation?


CT angiography is ideal for confirming the diagnosis and for preoperative planning as it accu-
rately determines the size and shape of the AAA and its relationship to branch arteries and the
aortic bifurcation. It is also superior to US in detecting and sizing common iliac artery aneurysms.
Patients must be haemodynamically stable and monitored closely when they are referred to CT.
There are, however, a number of other imaging modalities that may detect a AAA:
• AXR: this is an insensitive test but may outline the curvilinear arterial calcification of an
aneurysm.
• US: this is simple, quick, inexpensive, and commonly used technique in the ED in
patients with nonspecific symptoms or where a AAA is suspected. It is not sensitive for
detecting leaks or rupture and may be limited by overlying bowel gas (as in this case)
or abdominal tenderness. US is, however, used for routine AAA screening of men over
65 years old in the UK.

3 How would you manage this patient?


Management of this patient is via an ABCDE approach. This is a surgical emergency and, there-
fore, urgent discussion with the surgical team and anaesthetist is required.
Do not forget:
• IV access with two large bore cannulae in the antecubital fossae.
• Bloods. Check Hb, U&Es, and clotting, and cross match 8–10 units.
• ECG and portable CXR, VBG, and cardiac monitoring.
• Imaging. US in the ED if sufficient expertise is available may show the aneurysm. CT aortogram
is the technique of choice following urgent senior surgical and radiological discussion.
If there is a suspicion of a ruptured AAA, careful BP control with hypotensive resuscitation is
advised, under the guidance of an acute medicine physician. Raising the BP too high may cause
clot migration and rebleeding. The ideal replacement fluid is blood but local resuscitation protocols
will apply. If the AAA is confirmed to be leaking then either open or endovascular aneurysm repair
(EVAR) will be required, depending on available expertise and the clinical condition of the patient.
AAA repair options:
• EVAR (radiology and surgical collaboration). A stent-graft is inserted via the femoral artery
in the groin up to the aorta, and deployed over the aneurysmal section. This supports the
aneurysm and restores normal blood flow.
• Open aneurysm repair (surgical). The abdomen is opened and the aorta exposed and
repaired with a graft. This is still the standard procedure in many institutions.
EVAR is less invasive than open surgery, has a lower morbidity and mortality rate, and reduces
postoperative recovery time. It is performed in elective aneurysm repairs as well as in patients
undergoing emergency repair for rupture. Not all aneurysms, however, are suitable for EVAR and
a CT aortogram is always required for preprocedural planning.

TOP TIPS
Elective management of AAA can be classified as conservative, medical, or surgical:
▪▪ Conservative: screening programme, monitoring of aneurysms >3 cm.
▪▪ Medical: risk factor management (BP, cholesterol, weight, smoking).
▪▪ Surgical: aneurysms >5.5 cm or growing >1 cm/year or after leak/rupture.

183

K30031_Book.indb 183 9/6/17 1:36 PM


6  Cardiovascular cases

Fig. 6.2C  CT aortogram of a different patient with aneurysmal dilation of the infrarenal aorta, measuring
6 cm diameter, with contrast in the aortic lumen (A), mural thrombus (B), and mural calcification (C). There
is also a large para-aortic haematoma (D), which has tracked into the retroperitoneum along the right
iliopsoas (E), which is enlarged (this finding may be visible on AXR). This contrasts with the normal looking
left iliopsoas (F).

4 How might this condition be prevented in the older population?


Owing to the mortality associated with a large AAA, the UK has a screening programme for at-
risk patients (males over 65 years old). Men are offered an US examination at the age of 65 years
and small aneurysms (>3 cm diameter) are followed up. If >5.5 cm or growing >1 cm in 1 year
then intervention is recommended.

LEARNING POINTS: LEAKING ABDOMINAL AORTIC ANEURYSM


▪▪ Rupture of a AAA is a surgical emergency with a high mortality rate and can present with collapse
in a patient who is cardiovascularly unstable.
▪▪ AAAs >5.5 cm are at risk of rupture and should be considered for either elective open or
endovascular repair.
▪▪ A leaking AAA can present with nonspecific symptoms, such as abdominal or back pain, and may
be misdiagnosed as renal colic.
▪▪ US is accurate for AAA screening; however, CT angiography (in stable patients) is the reference
standard test for diagnosis and preoperative planning.

184

K30031_Book.indb 184 9/6/17 1:36 PM


Case 6.3

Fig. 6.3A  CTPA postcontrast at the level of the right main pulmonary artery.

A 63-year-old female presents to the ED at night having woken with sudden onset shortness of
breath and sharp, right-sided, chest pain, which is worse on inspiration. She is undergoing che-
motherapy for breast cancer and had a left mastectomy 2 months ago. She is a previous smoker
with a 20 pack-year history.
On examination, she is normotensive at 128/74 mmHg, tachycardic at 110 bpm, tachypnoeic
at 30 bpm, and has an SPO2 of 88% on air. Chest and cardiovascular examinations are normal.
Her ECG shows a sinus tachycardia.
A CTPA is arranged (Figure 6.3A). ABG results (on air) are as follows:

PaO2 7.2 kPa (11.1–14.4 kPa) Bicarbonate 24.2 (22–28)


PaCO2 4 kPa (4.7–6.4 kPa) Base excess +1 (−2–+2)
pH 7.48 (7.35–7.45) Lactate 1.2 mmol/L (0.5–2.2 mmol/L)

CASE 6.3: QUESTIONS


1 From the history alone, what are your differential diagnoses?
2 What does the blood gas show?
3 What does the CTPA scan show?
4 What are the important factors in deciding when to perform a CTPA?
5 How would you manage this patient?

185

K30031_Book.indb 185 9/6/17 1:36 PM


6  Cardiovascular cases

CASE 6.3: ANSWERS


1 From the history alone, what are your differential diagnoses?
Sudden onset shortness of breath with a background of malignancy suggests:

• Pulmonary embolus.
• Myocardial infarction with acute heart failure.
• Malignant pleural effusion.
• Pneumonia secondary to chemotherapy (immunosuppression).
• Pneumothorax.
• Rib fracture.
• Metastases (lung or bone).

The sudden onset with severe hypoxia and pleuritic chest pain, however, makes pulmonary
embolism (PE) most likely.

2 What does the blood gas show?


The ABG results show a mild respiratory alkalosis with significant hypoxia. This is indicative
of type 1 respiratory failure. There does not seem to be any metabolic compensation (as shown
by the normal bicarbonate and base excess), which suggests an acute insult.

3 What does the CTPA scan show?


The CTPA shows large filling defects within the main pulmonary arteries bilaterally
(Figures 6.3B–6.3D). These are pulmonary emboli, outlined by the contrast within the vessels.
The lungs are clear without evidence of infarction.

TOP TIPS
Risk factors for pulmonary embolism:
▪▪ Non-modifiable.
– Malignancy.
– Pregnancy.
– Inherited clotting disorder.
– Immobility.
– Previous PE/deep vein thrombosis (DVT).
▪▪ Modifiable.
– Smoking, obesity, age, combined oral contraceptive pill.

4 What are the important factors in deciding when to perform a CTPA?


Having taken a history and assessed modifiable and nonmodifiable risk factors for thromboem-
bolism, a Wells score should be calculated. This scoring system is used to decide whether or not a
PE is likely and, therefore, requires further investigation with a CTPA (Table 6.3).

186

K30031_Book.indb 186 9/6/17 1:36 PM


Case 6.3: Answers

Fig. 6.3B  CTPA with large emboli within the right (A) and descending left (B) pulmonary arteries. Note:
normal pulmonary trunk (C), aortic root (D), and descending aorta (E). The normal air-filled right (F) and left
(G) main bronchi are also demonstrated. Superior vena cava (H).

Fig. 6.3C  CTPA of the same patient (more inferior slice) showing further emboli (arrows) outlined by
contrast within the lower lobe pulmonary arteries.

187

K30031_Book.indb 187 9/6/17 1:36 PM


6  Cardiovascular cases

Fig. 6.3D  Coronal CTPA with thrombus in the right main pulmonary artery (A) and left upper and lower
lobe pulmonary arteries (B).

Table 6.3  PE Wells score

Clinical feature Points


Clinically suspected DVT 3
Alternative diagnosis is less likely than PE 3
Tachycardia (HR >100) 1.5
Immobilisation (≥3 days)/surgery in previous 4 weeks 1.5
History of previous DVT or PE 1.5
Haemoptysis 1
Malignancy (with treatment within 6 months) or palliative 1

Clinical probability scores


PE likely (consider diagnostic testing) More than 4 points
PE unlikely (consider D-dimer to rule out PE) 4 points or less

188

K30031_Book.indb 188 9/6/17 1:36 PM


Case 6.3: Answers

USE OF D-DIMER AS A DIAGNOSTIC TOOL


▪▪ D-dimer is a useful test but only when used correctly. It is a fibrin degradation product, a small
protein fragment present in the blood after a blood clot is degraded by fibrinolysis. The absence
of a raised concentration implies no recent thromboembolic event. A positive concentration,
however, is fairly nonspecific and is found in malignancy, infection, and pregnancy as well as
thrombosis.
▪▪ It is, therefore, useful to rule out thromboembolic disease when the Wells score probability is low.

5 How would you manage this patient?


The patient should be resuscitated with an ABCDE approach. A large PE is a potential cause
for cardiac arrest and close monitoring is therefore required. An ABG should be performed and
CTPA will confirm the diagnosis. Remember: O2 by mask, analgesia, IV access, and bloods.
Other important investigations include:

• ECG. The most common finding is sinus tachycardia. Less commonly the ECG may show a
S1Q3T3 pattern (a large S wave in lead I, a Q wave in lead III, and an inverted T wave in lead
III indicates acute right heart strain).
• CXR to rule out pneumothorax as a cause of the pain. The CXR is usually normal in PE but
may show atelectasis.
• Echocardiogram. This is usually performed in patients presenting with suspected PE to
look for right-sided heart strain.
Once PE is confirmed, IV thrombolysis may be considered if the patient is haemodynamically
unstable. Direct pulmonary angiography with embolectomy may also be considered (in patients
with a massive embolus and where interventional radiology facilities are available).
Initially in the majority of cases, patients will be treated with a treatment dose of low molecular
weight heparin (LMWH). This can then be converted to oral medication such as warfarin, or the
more recently introduced rivaroxaban, on discharge.

LEARNING POINTS: PULMONARY EMBOLISM


▪▪ CTPA is the diagnostic test of choice for PE and has largely replaced ventilation/perfusion (V/Q)
scanning. (Nuclear medicine perfusion scanning alone is sometimes used in pregnant patients to
reduce maternal radiation dose).
▪▪ The Wells score allows assessment of patients on an individual basis to determine whether further
tests are necessary to rule out thromboembolism.
▪▪ PE can be fatal and, therefore, it is important to get early senior help in managing patients to
ensure prompt diagnosis and treatment.
▪▪ Long-term management is with anticoagulation and the length of treatment is dependent on
precipitating factors.

189

K30031_Book.indb 189 9/6/17 1:36 PM


Case 6.4

Fig. 6.4A  Erect


AP CXR.

A 78-year-old male presents to the ED having woken up in the night severely short of breath. He
does not report any chest pain. He had an MI 1 year ago and since then has been increasingly
short of breath on exertion (SOBOE), only being able to walk about 50 metres at a time. His regu-
lar medications include amlodipine, ramipril, furosemide, simvastatin, and aspirin. He is a past
smoker with a 30 pack-year history.
On examination he is visibly breathless while sitting upright. There are fine crepitations with
reduced air entry in both lung bases. Heart sounds are normal and his abdomen is soft and non-
tender. There is no peripheral or sacral oedema. His SPO2 is 88% on air, BP 135/84 mmHg, HR 96
bpm, respiration rate 26 bpm, and temperature 36.4°C. You arrange an urgent CXR (Figure 6.4A).

CASE 6.4: QUESTIONS


1 What does the CXR show?
2 What other investigations would you perform?
3 How would you manage this patient?

190

K30031_Book.indb 190 9/6/17 1:36 PM


Case 6.4: Answers

CASE 6.4: ANSWERS


1 What does the CXR show?
The CXR shows widespread interstitial oedema with septal lines in the lower zones. There
are bilateral pleural effusions with fluid tracking along the horizontal fissure. The heart is not
enlarged. These changes are consistent with acute pulmonary oedema (Figure 6.4B). There is
upper lobe blood diversion (indicting pulmonary venous hypertension with upper lobe venous
blood redistribution).

Fig. 6.4B
Magnified view
of the right lower
zone, better
demonstrating the
small, subpleural,
peripheral horizontal
septal lines of
interstitial oedema
(A). There is also a
small pleural effusion
(B) with fluid tracking
into the horizontal
fissure (C).

TOP TIPS
A way to remember the CXR features of heart failure (ABCDE):
Alveolar oedema (bat wing appearance).
Kerley B lines (septal lines of interstitial oedema).
Cardiomegaly.
Dilated prominent upper lobe vessels.
Effusion (pleural).

191

K30031_Book.indb 191 9/6/17 1:36 PM


6  Cardiovascular cases

Heart failure is when the heart is unable to pump blood sufficiently to maintain the body’s
needs. The cause may be acute or chronic, or acute on chronic, and is usually multifactorial:

• Acute: MI, fluid overload or arrhythmia.


• Chronic: coronary heart disease, hypertension, cardiomyopathy, anaemia, hyperthyroidism.

Kerley B lines (also known as septal lines) represent interstitial oedema in the interlobular
lymphatics. Causes include pulmonary oedema, lymphangitis carcinomatosa, and sarcoidosis.
A good history and examination will help differentiate these. It is also helpful if you can compare
with any previous CXRs. Remember that in acute, ‘flash’ pulmonary oedema the heart size may
be normal, with enlargement occurring later in the disease process. Initially, interstitial oedema
may be seen and fluid may extend into the airspace/alveolus also, and then has the appearance of
consolidation, radiopaque and often perihilar (Figure 6.4C).

Fig. 6.4C  CXR showing the typical perihilar (bat wing) appearance of pulmonary oedema (in a patient
on dialysis). There is a central venous catheter placed within the superior vena cava. The heart is slightly
enlarged and there are no pleural effusions. This appearance is nonspecific, and infection (particularly
atypical, e.g. Pneumocystis jiroveci [formerly known as P. carinii] pneumonia) could also explain these XR
appearances.

192

K30031_Book.indb 192 9/6/17 1:36 PM


Case 6.4: Answers

It is important to distinguish pulmonary oedema from other causes of consolidation on CXR,


such as pneumonia. Always ask for help if unsure. As well as the clinical differences (SOBOE,
crackles, hypertension versus cough and fever), there are a few helpful radiological differences
(Table 6.4A).

Table 6.4A  Differentiation of causes of consolidation on CXR

Pulmonary oedema Pneumonia


Usually bilateral Usually unilateral
Typically perihilar Typically lobar, segmental or focal
Associated interstitial densities (septal lines) Usually a single and denser area of opacity. No interstitial
and cardiomegaly markings
Usually bilateral pleural effusions Possible associated unilateral pleural effusion

2 What other investigations would you perform?


Other investigations are:

• ECG to exclude an acute MI.


• Blood tests.
• FBC and CRP for concurrent infection and to rule out anaemia.
• Troponin/cardiac enzymes to rule out acute MI.
• TFTs to check for hyperthyroidism as a cause.
• U&Es as diuretics are a mainstay of management.
• Brain natriuretic peptide (BNP) for diagnosis and management of chronic heart failure.
• Echocardiogram, to evaluate cardiac function.

3 How would you manage this patient?


Initial management is via an ABCDE approach. This includes sitting the patient up, administer-
ing high-flow oxygen, taking a more detailed history, getting IV access, taking bloods (see above),
and arranging an ECG and CXR with early senior review.
Medical management then aims to reduce preload and afterload pressures on the heart:

• Preload: sublingual GTN and diuretics (oral or IV furosemide).


• Afterload: long-term BP management with ACE inhibitors and angiotensin receptor blocker
(ARB) agents.
• In the acute and unstable setting, inotropic agents may be necessary with ICU support.

The New York Heart Association (NYHA) classification of chronic heart failure is shown in
Table 6.4B.

TOP TIPS
When asked how to manage a patient with acute cardiogenic pulmonary oedema start with the ABCDE
approach, a full history, and investigations including bloods, ECG, and CXR. Mention senior clinical
review before talking about the use of GTN and diuretics to offload fluid from the lungs. The examiners
want to know that you can comfortably manage a patient like this in an acute setting.

193

K30031_Book.indb 193 9/6/17 1:36 PM


6  Cardiovascular cases

Table 6.4B  NYHA classification of heart failure


This is a simple way of classifying the functional extent of heart failure and is widely used in the
management of patients. It often comes up in exams, both written and practical, so is handy to
remember:
I Cardiac disease but no symptoms or limitation in ordinary physical activity
II Mild symptoms (mild shortness of breath and/or angina) and slight limitation during ordinary
activity
III Marked limitation in activity due to symptoms, even during less-than-ordinary activity, e.g.
walking short distances (20–100 m), comfortable only at rest
IV Severe limitations with symptoms at rest (mostly bed-bound patients)

LEARNING POINTS: CARDIAC FAILURE


▪▪ CXR changes in a patient with heart failure do vary but include alveolar oedema, ‘Kerley B’ septal
lines of interstitial oedema, cardiomegaly, dilated prominent upper lobe vessels, and pleural
effusions.
▪▪ Try to differentiate the consolidation seen in pulmonary oedema from that seen in pneumonia,
although this can be difficult (bilateral symmetrical versus unilateral asymmetrical).
▪▪ Management is via an ABCDE approach with high-flow oxygen, GTN, and IV diuretics to offload
the heart. Always exclude an acute MI.
▪▪ NYHA classification is used to determine functional status in chronic heart failure.

194

K30031_Book.indb 194 9/6/17 1:36 PM


Case 6.5

Fig. 6.5A  CXR.

A 78-year-old female is admitted to hospital with sudden-onset, left-sided weakness and dysar-
thria. She has a CT of her brain, which shows early signs of an acute right middle cerebral artery
stroke. She had an anterior MI 4 years previously and has been under the care of cardiology,
and in the last few months she has been complaining of lethargy, nonspecific chest pains, and
palpitations.
As part of her initial investigations she has a CXR performed (Figure 6.5A).

CASE 6.5: QUESTIONS


1 What are the CXR findings?
2 What is the CXR diagnosis?
3 How would you investigate and manage this patient?

195

K30031_Book.indb 195 9/6/17 1:36 PM


6  Cardiovascular cases

CASE 6.5: ANSWERS


1 What are the CXR findings?
The CXR shows cardiomegaly (Figure 6.5B). The left atrium and ventricle are particularly
enlarged with splaying of the carina secondary to left atrial enlargement. There is calcification of
the left ventricle (LV) wall at the apex. A dual lead pacemaker with implantable cardiac defibrilla-
tor (ICD) is in situ. The lungs are clear.
Cardiomegaly in adults is defined as a cardiothoracic ratio (CTR) that exceeds 50% on a PA
projection CXR. In this case the left side in particular of the heart is enlarged:

• Left atrial enlargement: double RHB (caused by the enlarged left atrium), enlarged left
atrial appendage, and splaying of the carina (>70°).
• Left ventricular enlargement, with globular chamber enlargement.

Fig. 6.5B  CXR showing cardiomegaly with enlarged left atrium, causing splaying of the carina (C): the
normal carina angle is <60–70°, splaying occurs with left atrial or subcarinal nodal enlargement. The
cardiothoracic ratio (A/B) is >50% – this widest point of transverse cardiac diameter/widest intrathoracic
diameter from inner rib margins in adults should be <50%. There is curvilinear left ventricular myocardial
calcification (D). Note: implantable defibrillator/pacing device.

196

K30031_Book.indb 196 9/6/17 1:36 PM


Case 6.5: Answers

2 What is the diagnosis?


The diagnosis is a calcified left ventricular aneurysm (LVA), likely secondary to her previous MI:

• Myocardial calcification is a sign of prior infarction, while pericardial calcification is


strongly associated with constrictive pericarditis. Therefore, detecting and recognising
calcification related to the heart on chest radiography and other imaging modalities, such as
fluoroscopy, CT, and echocardiography, may have important clinical implications.
• Aneurysmal myocardial calcification is identified as thin and curvilinear shaped and
usually appears toward the apex of the LV. There is often an associated bulge in the
ventricular contour and myocardial calcification is deeper than pericardial, which tends
to outline the periphery of the cardiac silhouette. Myocardial calcification follows the
ventricular contour and will not cross the midline following the pericardium.
• LVA is a rare complication of a MI that can cause serious morbidity or death (4% of cases).
Acute MI damages the muscle wall and this can then bulge and develop into a true
aneurysm, where there is a full-thickness breach in the myocardial lining with a broad
neck. Acutely this may rupture, or there may be fibrosis, scarring, and healing with a
chronic aneurysm and this may calcify (Table 6.5). Rarely the rupture is contained by the
pericardium, creating a false aneurysm.

Table 6.5  Complications of LVA


Acute • Sudden death/cardiac rupture
Chronic • Thromboembolism (embolic events from thrombus within the aneurysmal sac)
• Ventricular arrhythmia (note the presence of pacemaker) often refractory to drug treatment
• Congestive heart failure
• Refractory angina

3 How would you investigate and manage this patient?


Investigations:

• ECG: it is important for identifying evidence of acute or prior myocardial ischaemia, as well
as associated arrhythmias. A LVA acutely can cause persistent ST elevation.
• Blood profile: troponin (to exclude acute MI), FBC (anaemia or infection can exacerbate
pre-existing heart failure), U&Es (renal impairment, hyponatraemia, establish baseline
electrolytes prior to starting diuretics or ACE inhibitors), LFTs, blood glucose (to detect
underlying diabetes mellitus).
• Echocardiography: assess atrial and ventricular sizes, global left and right ventricular
systolic function, diastolic function of the LV, regional wall abnormalities, and mural
thrombus and valve abnormalities.

Refer for cardiology specialist review (further management depends on whether the presenta-
tion is acute or chronic).
Treatment (in chronic cases):

• Lifestyle advice: stopping smoking, control BP/diabetes, weight loss.


• Anticoagulation.
• Heart failure medication for symptom control (ACE inhibitors also help to prevent
ventricular remodelling and the formation of aneurysms).

197

K30031_Book.indb 197 9/6/17 1:36 PM


6  Cardiovascular cases

• Arrhythmia management.
• Definitive surgery with left ventricular reconstruction is often not needed in chronic cases,
particularly with smaller aneurysms.

LEARNING POINTS: LEFT VENTRICULAR ANEURYSM


▪▪ A chronic left ventricular (true) aneurysm is an area of well-delineated, thinned, and fibrotic
myocardial wall, devoid of muscle, that is a result of a healed transmural MI. Lesions may calcify
when healed and are associated with arrhythmias and thromboembolic events.
▪▪ A ventricular pseudoaneurysm (false aneurysm) develops after an acute MI that has been
complicated by cardiac wall rupture that is contained by localised pericardial adhesions. These
lesions are at high risk of cardiac rupture with sudden death, and urgent surgical intervention is
needed.

198

K30031_Book.indb 198 9/6/17 1:36 PM


Case 6.6

Fig. 6.6A  Digital subtraction


angiogram of the right upper leg/thigh.

A 58-year-old male presents to his GP complaining of pain at rest in his right leg for the previ-
ous week and numbness in the foot and calf. This was preceded by several months of pain in the
calf and foot on minimal exertion. He is taking medication for hypertension and is a smoker of
10 cigarettes a day.
On examination the right foot is pale and cold to touch. There is reduced sensation, pedal
pulses are absent, and the popliteal pulse is weak. There are no abdominal masses. Femoral
pulses are present. He is referred to the hospital for an urgent vascular assessment.
As part of his management he undergoes femoral angiography (Figure 6.6A).

CASE 6.6: QUESTIONS


1 What are the angiographic findings? 3 How would you investigate and manage
2 What is the diagnosis? this patient?

199

K30031_Book.indb 199 9/6/17 1:36 PM


6  Cardiovascular cases

CASE 6.6: ANSWERS


1 What are the angiographic findings?
The angiogram of the right upper leg shows an occlusion of the superficial femoral artery just
above the adductor canal with collateral circulation and reconstitution of the artery more distally
(Figure 6.6B). The adductor canal is a common site of occlusion/stenosis as it is an area of poten-
tial narrowing/vascular compression as the artery passes through the fascia at this level.

Fig. 6.6B  Digital subtraction


angiogram of the right upper leg
showing occlusion (white arrow)
of the right superficial femoral
artery (A) with reconstitution of
the distal femoral artery (B) via
collateral circulation. Femur (F).

2 What is the diagnosis?


This patient has occlusion of the right superficial femoral artery that will need urgent treatment.
The likely aetiology is acute thrombosis on a background of pre-existing chronic stenosis of the
distal superficial femoral artery.

200

K30031_Book.indb 200 9/6/17 1:36 PM


Case 6.6: Answers

Possible aetiologies of acute vascular occlusion include:


• Thrombosis: acute on chronic (embolus from rupture of proximal atherosclerotic plaque) or
chronic (gradual extension of thrombus with development of collateral circulation). This is
peripheral artery occlusive disease (PAOD), which has presented with claudication/pain.
• Embolism: secondary to atrial fibrillation, post-MI, prosthetic heart valves, aortic
aneurysm, proximal atheromatous stenosis or malignancy.
• Trauma: penetrating injury with laceration of the vessel.
• Raynaud’s syndrome: peripheral arterial spasm when exposed to cold temperatures,
smoking, or emotional stress.
• Compartment syndrome: occurs when perfusion pressure falls below tissue pressure in a
closed anatomical space. This may follow orthopaedic (fracture), vascular (haemorrhage) or
soft tissue (crush, burn) injury.
There are several limb ischaemia classification systems. Fontaine’s four stages are the easiest to
remember:
• Stage I: asymptomatic.
• Stage IIa: intermittent claudication >200 m walk.
• Stage IIb: intermittent claudication <200 m walk.
• Stage III: rest pain.
• Stage IV: ischaemic ulcers or gangrene.
The clinical features of an acutely ischaemic limb (‘the 6 Ps’) are:
• Pale.
• Pulseless.
• Painful.
• Paralysed.
• Paraesthetic.
• ‘Perishing with cold’.

3 How would you investigate and manage this patient?


Investigations would include:
• Doppler US of both feet to identify any distal arterial flow.
• Blood tests including FBC (anaemia can worsen the ischaemia), ESR (giant cell arteritis or
other connective tissue disorders), glucose (diabetes mellitus), lipids, thrombophilia screen,
and coagulation screen.
• Investigations to identify an embolic source including ECG and echocardiogram.
• CT or MR angiography to assess the abdominal aorta and limb vasculature, and the
stenosis and collateral circulation.
• Digital subtraction angiography (DSA), as performed above. This is an invasive technique
that involves femoral catheter insertion. It is the reference standard technique for assessing
stenoses, occlusions, and collateral circulations, and is performed in interventional
radiology and usually as a precursor to radiological treatment of the stenosis.
Management (remember ABCDE) includes:
• Urgent admission (this is a surgical emergency).
• IV heparin (may double the limb salvage rate).
• Analgesia.

201

K30031_Book.indb 201 9/6/17 1:36 PM


6  Cardiovascular cases

Short-term management has a number of options depending on patient condition and find-
ings. Radiological intervention is most common in the first instance (remember in the exam to
mention that you would seek urgent senior advice).

• If thrombotic: intra-arterial thrombolysis, angioplasty or bypass surgery.


• If embolism: radiological or surgical embolectomy or local intra-arterial thrombolysis;
bypass graft if the embolectomy fails.
• If the limb is irreversibly ischaemic: amputation will be required.
• If significant arterial stenosis: radiological angioplasty with or without a stent
(Figure 6.6C).

Fig. 6.6C  Digital subtraction angiogram before (left) and after (right) catheter thrombectomy and
angioplasty in the same patient. Note: contrast opacified superficial femoral artery with central catheter
tip (A) just proximal to the stenosis on the left image. The postprocedural image on the right shows
contrast flowing normally through the previously stenotic portion of femoral artery after balloon dilatation
angioplasty (B). In this procedure the catheter is deployed near the stenosis and a guidewire navigated over
the stenosis. The catheter is then advanced and a balloon inflated to reduce the stenosis. This process can
be repeated and if unsuccessful a metallic stent can be deployed.

202

K30031_Book.indb 202 9/6/17 1:36 PM


Case 6.6: Answers

Long-term management involves:

• General management for underlying cardiovascular disease: exercise, smoking cessation,


treatment of hypertension and hyperlipidaemia, and achieving optimal diabetes control.
• Low-dose aspirin or clopidogrel.
• ACE inhibitor therapy: reduces the morbidity and mortality of cardiovascular disease in
patients with peripheral arterial disease by up to 25%.

LEARNING POINTS: ISCHAEMIC LEG


▪▪ Thrombolysis usually takes between 6 and 72 hours to achieve clot lysis, hence patients with
limb-threatening ischaemia are not usually candidates for local thrombolysis and require urgent
radiological/surgical intervention.
▪▪ Without revascularisation, complete acute ischaemia can lead to extensive tissue necrosis within
6 hours.
▪▪ Ischaemia is considered acute if the symptoms and signs have developed over less than 2 weeks.
▪▪ Management involves close collaboration between vascular surgery and interventional radiology.

203

K30031_Book.indb 203 9/6/17 1:36 PM


K30031_Book.indb 2 9/6/17 1:34 PM
Abdomen and pelvis cases

7 FAYE CUTHBERT, AMANDA JEWISON,


AND OLWEN WESTERLAND

Case 7.1 206 Case 7.15 265


Case 7.2 211 Case 7.16 269
Case 7.3 215 Case 7.17 273
Case 7.4 219 Case 7.18 276
Case 7.5 223
Case 7.19 281
Case 7.6 227
Case 7.20 286
Case 7.7 230
Case 7.21 291
Case 7.8 236
Case 7.22 294
Case 7.9 240
Case 7.23 299
Case 7.10 245
Case 7.11 249 Case 7.24 303
Case 7.12 253 Case 7.25 307
Case 7.13 257 Case 7.26 311
Case 7.14 261 Case 7.27 315

205

K30031_Book.indb 205 9/6/17 1:36 PM


Case 7.1

Fig. 7.1A  Longitudinal US image of the left kidney.

206

K30031_Book.indb 206 9/6/17 1:36 PM


Case 7.1: Questions

A 74-year-old male presents to urology with a 4-week history of painless haematuria. He describes
‘rosé’ coloured urine but no clots. He denies pain, weight loss or dysuria.
On examination he is well. He has tar stains on his fingers and is a heavy smoker of 20 cig-
arettes per day for over 40 years. There is no abdominal tenderness or any palpable masses.
Observations are stable. Urinalysis shows blood +++ but no leucocytes, nitrites or glucose. Blood
tests reveal mild normochromic, normocytic anaemia, and slightly elevated urea and creatinine.
A renal US is requested (Figure 7.1A).

CASE 7.1: QUESTIONS


1 What does the US show?
2 What are the possible causes?
3 What investigation should be carried out next?
4 How should this patient be managed?

207

K30031_Book.indb 207 9/6/17 1:36 PM


7  Abdomen and pelvis cases

CASE 7.1: ANSWERS


1 What does the US show?
The US shows mild hydronephrosis of the left kidney (Figure 7.1B). Hydronephrosis is dilatation
of the renal pelvis and calyceal system, usually caused by obstruction of urine outflow. It can be
physiological, for example in pregnant women where the gravid uterus compresses the ureters, or
pathological. It can be unilateral or bilateral, acute or chronic.

Fig. 7.1B  Longitudinal ultrasound image of a left kidney showing mild hydronephrosis. Renal cortex (A).
Renal sinus fat, which is bright (B). Dilated renal pelvis (C). Dilated proximal ureter (D) and dilated upper and
lower pole calyces (E). The upper pole of the kidney lies to the left of the image. Note the preservation of
thickness of renal cortex, which may suggest an acute/subacute cause (longer standing obstruction will
usually cause cortical loss/damage and thinning).

2 What are the possible causes?


Common causes of urinary tract obstruction are shown in Table 7.1. Obstruction can occur at any
level from the pelvicalyceal system to the urethra. In this case, the obstruction is unilateral and
painless, therefore malignancy is the cause to be excluded.

208

K30031_Book.indb 208 9/6/17 1:36 PM


Case 7.1: Answers

Table 7.1  Common causes of urinary tract obstruction

Intrinsic (within the lumen or wall) Extrinsic (external compression or infiltration)


Calculi Malignancy (retroperitoneal mass, lymph nodes, pelvic malignancy)
Blood clot Prostatic enlargement
Malignancy (transitional cell carcinoma) Congenital (pelviureteric junction obstruction)
Ureteric or urethral stricture Retroperitoneal fibrosis

3 What investigation should be carried out next?


The patient should be referred urgently to urology for a cystoscopy to look for a bladder tumour.
The patient will also require a CT IVU or CT urogram: this involves a precontrast study through
the renal tract to look for stones and then a postcontrast examination to look for other lesions, and
in particular transitional cell carcinoma of the urinary tract, which may occur from renal pelvis to
bladder and may be multifocal.

4 How should this patient be managed?


Management of the patient depends on the underlying cause and whether the renal function is
compromised. If the renal function is compromised or there is suspicion of infection, the patient
may require ureteric stenting. This involves passing a thin tube up the ureter to relieve the
obstruction (usually cystoscopically) or inserting a percutaneous nephrostomy tube into the renal
pelvicalyceal system to decompress the kidney. If malignancy is found, it will need to be staged
and discussed at the urology MDT meeting in order to plan appropriate treatment. Transitional
cell carcinoma can involve the urothelium anywhere in the renal tract. Smoking is an important
risk factor.
This patient had a cystoscopy and CT, which revealed a large mass in the bladder causing
obstruction of the left ureter at the vesicoureteric junction (Figure 7.1C). Biopsy confirmed transi-
tional cell carcinoma. The ureters and upper tracts were, however, normal on CT.

Fig. 7.1C  Axial CT image at the level of the bladder. Contrast in the bladder (A) helps to demonstrate that
the posterior wall of the bladder is irregular and thickened, in keeping with a bladder tumour (B). Femoral
vessels: vein medial (C), artery lateral (D). Coccyx (E), rectum (F), seminal vesicles (G).

209

K30031_Book.indb 209 9/6/17 1:36 PM


7  Abdomen and pelvis cases

LEARNING POINTS: HAEMATURIA/HYDRONEPHROSIS


▪▪ Painless haematuria is caused by malignancy until proven otherwise and warrants an urgent
referral to urology.
▪▪ CT urogram and cystoscopy are first-line investigations.
▪▪ It is important to exclude infection clinically because an infected obstructed system can lead to
rapid loss of function unless urgently decompressed.
▪▪ Hydronephrosis can be caused by intrinsic blockage or external compression at any level of the
urinary tract.
▪▪ Common causes of hydronephrosis include urinary tract calculi, ureteric or bladder tumours,
pelvic masses, and congenital pelviureteric junction obstruction.

210

K30031_Book.indb 210 9/6/17 1:36 PM


Case 7.2

Fig. 7.2A  AXR.

A 65-year-old male presents to the ED with sudden-onset severe abdominal pain. On exam-
ination he is pale and sweaty. He has generalised abdominal tenderness with reduced bowel
sounds and guarding in the upper abdomen. Observations reveal tachycardia 120 bpm, BP 80/60
mmHg, and temperature 37.5°C. A supine AXR is performed amongst other initial investigations
(Figure 7.2A).

CASE 7.2: QUESTIONS


1 What radiological sign is shown and what does it indicate?
2 What is the most likely cause?
3 How should the patient be managed?

211

K30031_Book.indb 211 9/6/17 1:36 PM


7  Abdomen and pelvis cases

CASE 7.2: ANSWERS


1 What radiological sign is shown and what does it indicate?
The AXR shows Rigler’s sign. This is when air is seen on both sides of the bowel wall (Figure 7.2B).
It indicates pneumoperitoneum, so air outlines the bowel wall clearly, with air within the bowel
lumen and also outside of the bowel. Rigler’s sign was first described by L.G. Rigler in 1941; a posi-
tive Rigler’s sign does require relatively large amounts of free intraperitoneal air.
Other abdominal radiographic features of free intraperitoneal air include:

• ‘Football’ sign, oval-shaped free air in the upper abdomen, which may outline the falciform
ligament.
• Triangular free air collection in Morrison’s pouch beneath right lobe of liver.
• Perihepatic free air.
• ‘Cupola’ sign, free air trapped beneath the central tendon of the diaphragm.

Fig. 7.2B  Cropped AXR showing Rigler’s sign. Air is clearly seen on both sides of the bowel wall (within
lumen A, outside bowel B).

2 What is the most likely cause?


Pneumoperitoneum is free intra-abdominal gas, either as a result of recent surgery or visceral
perforation. The most common sites of a perforated viscus are the duodenum and sigmoid colon,
although a perforation can occur at any part of the GI tract. Common causes include a peptic
ulcer, bowel obstruction, and inflammatory processes such as diverticulitis or appendicitis.
Another example of Rigler’s sign is shown in Figure 7.2C.

212

K30031_Book.indb 212 9/6/17 1:36 PM


Case 7.2: Answers

Fig. 7.2C  AXR in another patient showing Rigler’s sign (arrows A). Note the widespread free
intraperitoneal air throughout abdomen (arrows B).

3 How should the patient be managed?


The patient is unwell and needs a structured ABCDE approach initially, including IV access and
fluid resuscitation. It is important to state in an OSCE situation that you would seek senior input
early. Bowel perforation carries high mortality if untreated because the leaked bowel contents
cause inflammation and infection in the peritoneum (peritonitis) and can rapidly lead to severe
sepsis and death. Therefore, urgent surgery is usually the most appropriate next management
step in order to repair or resect the area of bowel that has perforated and to thoroughly wash out
the abdomen.

213

K30031_Book.indb 213 9/6/17 1:36 PM


7  Abdomen and pelvis cases

Often, a CT scan of the abdomen and pelvis (Figure 7.2D) is requested, if patient condition
allows, prior to surgery in order to confirm the presence of pneumoperitoneum and help guide the
surgeon to the site and cause of perforation.
Do also remember that an erect CXR is far more sensitive than an AXR in the detection of free
intraperitoneal air. CT is the technique of choice for difficult or equivocal cases and can detect
tiny amounts of free gas. A good quality erect CXR can detect amounts of free intraperitoneal air
down to 1 mL.
In this patient the majority of free gas was located in the upper abdomen, and there was no evi-
dence of diverticular disease or colonic tumour on CT, therefore it was thought that the likely source
of perforation was the upper GI tract. A perforated duodenal ulcer was confirmed at laparotomy.

Fig. 7.2D  Axial CT on lung window setting to help further evaluate the patient from this case. Free gas is
present in the peritoneal cavity (A). Note the falciform ligament (B) surrounded by air. One can appreciate
how Rigler’s sign appears on AXR; air within small bowel lumen (C) and free air outside the bowel (D)
outline the small bowel wall.

LEARNING POINTS: PNEUMOPERITONEUM


▪▪ Rigler’s sign is where air is seen on both sides of the bowel wall. It indicates pneumoperitoneum.
▪▪ An erect CXR is more sensitive and more useful than an AXR at detecting pneumoperitoneum.
▪▪ Always ask about recent surgery.
▪▪ Free gas on AXR indicates a large volume of free gas. Smaller volumes can be present without
being visible on CXR or AXR, therefore high clinical suspicion of perforation should prompt further
investigation with CT.

214

K30031_Book.indb 214 9/6/17 1:36 PM


Case 7.3

Fig. 7.3A  Supine AXR.

A 75-year-old male presents to the ED with a 3-day history of abdominal pain and distension. He
reports no bowel movements for 4 days and is not passing flatus. He complains of nausea and has
vomited once. He has no past medical history of note.
On examination, observations are temperature 36.5°C, BP 100/70 mmHg, HR 100 bpm, and
respiratory rate 18 bpm. He has a distended abdomen and is tender in the epigastrium. Tinkling
bowel sounds are present.
An AXR is requested (Figure 7.3A).

CASE 7.3: QUESTIONS


1 What three key radiological signs are present?
2 What is the likely diagnosis?
3 How should the patient be managed?

215

K30031_Book.indb 215 9/6/17 1:36 PM


7  Abdomen and pelvis cases

CASE 7.3: ANSWERS


1 What three key radiological signs are present?
The AXR (Figure 7.3B) shows small bowel dilatation, biliary air (aerobilia), and a calcified gall-
stone lying outside of the expected position of the gallbladder. Note the dilated small bowel is
subtle, not the classic dilated air-filled loop. The dilated small bowel in this patient in the left iliac
fossa is largely fluid filled with only bubbles of air seen lying on top of the fluid.

Fig. 7.3B  AXR


showing gas in
the biliary tree (A),
dilated small bowel
(B), and a calcified
gallstone (C).

2 What is the likely diagnosis?


The likely diagnosis is gallstone ileus. This term is actually a misnomer as it describes small bowel
obstruction due to a gallstone that has passed through the gallbladder wall into the small bowel.
Gallstone ileus comprises about 1% of cases of bowel obstruction.

• The gallstone often becomes lodged at the ileocaecal valve, a site of physiological bowel
narrowing, and causes mechanical small bowel obstruction. It is a rare complication
of chronic cholecystitis where the long-standing inflammation allows a fistula to form
between the gallbladder and adjacent bowel wall and air passes from bowel into the
biliary tree. Small stones will pass into the colon and then exit the bowel in stool.

216

K30031_Book.indb 216 9/6/17 1:36 PM


Case 7.3: Answers

Rarely a stone may pass in a fistula to the duodenum and obstruct at the duodeno-jejunal
flexure: Bouveret’s syndrome.
• Classic gallstone ileus on an AXR consists of three radiological features: small bowel
dilatation, biliary air, and a gallstone. However, beware that not all gallstones are calcified
therefore all three features may not be present. It is also usually very hard to detect
gallstones when they lie in fluid-filled bowel. Air within the bile ducts is often not seen as
the cystic duct may be inflamed and occluded so air cannot enter the bile ducts, although
air may still be seen in the gallbladder itself.

3 How should the patient be managed?


The patient has small bowel obstruction and should be managed as such. He will need IV fluids,
insertion of a NG tube to decompress the stomach and close monitoring of input and output. Urgent
senior and surgical review are required. A CT may be helpful if the diagnosis is in doubt, or to
look for complications such as bowel ischaemia or pneumoperitoneum (Figures 7.3C and 7.3D).
Treatment of gallstone ileus is surgery in most cases. This may involve just a minilaparotomy in the
first instance with removal of the obstructing stone from the bowel lumen. CT can help significantly
in planning minimally invasive surgery. The gallbladder region is often avoided at initial surgery
owing to chronic inflammation and adhesions, although it can be targeted at a later date.

Fig. 7.3C  Coronal


contrast-enhanced CT of
the abdomen and pelvis
showing dilated small
bowel (A) and gas in the
biliary tree (B). Distended
stomach (C), liver (D), and
aorta (E).

217

K30031_Book.indb 217 9/6/17 1:36 PM


7  Abdomen and pelvis cases

Fig. 7.3D  Axial contrast-enhanced CT at the level of the right iliac fossa showing an obstructing gallstone
in the small bowel lumen in the right iliac fossa (A) and dilated loops of small bowel (B).

LEARNING POINTS: GALLSTONE ILEUS


▪▪ A rare complication of chronic cholecystitis.
▪▪ A gallstone passes through a fistula from the gallbladder into the small bowel and causes small
bowel obstruction.
▪▪ The impacted gallstone often becomes lodged at the ileocaecal valve.
▪▪ Key radiological features are:
– Small bowel dilatation.
– Air in the biliary tree.
– Gallstone in the right lower quadrant.

218

K30031_Book.indb 218 9/6/17 1:36 PM


Case 7.4

Fig. 7.4A  Supine AXR.

A 46-year-old female presents to the ED with abdominal pain and vomiting, she cannot keep any
fluids down, and is only passing small amounts of flatus. Her past medical history includes two
caesarean sections and a laparascopic sterilisation.
Examination reveals she is afebrile, with HR 100 bpm, and BP 120/80 mmHg. There is
abdominal distension and general tenderness, and bowel sounds are tinkling and loud. She has
evidence of abdominal surgical scars consistent with a previous caesarean section and laparo-
scopic surgery.
An AXR is requested (Figure 7.4A).

CASE 7.4: QUESTIONS


1 What does the AXR show?
2 What are the most common causes?
3 How should the patient be managed?

219

K30031_Book.indb 219 9/6/17 1:36 PM


7  Abdomen and pelvis cases

CASE 7.4: ANSWERS


1 What does the AXR show?
The AXR shows dilated loops of small bowel in the central abdomen (Figure 7.4B). There is mini-
mal bowel gas distally in the large bowel or rectum. No free gas. There are metal sterilisation clips
in the pelvis. The dilated loops of bowel, together with the clinical history and findings, indicate
small bowel obstruction (SBO). Remember that both obstruction and ileus will cause bowel dilata-
tion and the two are not distinguishable on XR alone: the clinical findings should help differentiate.

Fig. 7.4B  AXR showing central dilated small bowel loops (A), faeces in the nondistended right colon (B),
absence of bowel gas in the expected position of the rectum (C), and sterilisation clips in the pelvis (D).
These dilated small bowel loops are arranged in what is sometimes called a ‘stepladder’ configuration.

Why is this small bowel?

• It is centrally located.
• It does not contain faeces.
• It contains valvulae conniventes (Figures 7.4C and 7.4D).
• It is dilated >3 cm but not markedly so (e.g. 7–10 cm), which would be more consistent with
large bowel.

220

K30031_Book.indb 220 9/6/17 1:36 PM


Case 7.4: Answers

C D

Figs. 7.4C, D  Another example of SBO on AXR (magnified view of upper right quadrant shown in
Figure 7.4D). There are dilated small bowel loops (A) and absence of gas within the rectum, normal calibre
large bowel (B). Note the presence of valvulae conniventes, and mucosal folds that traverse the small
bowel lumen (C), unlike mucosal mural indentations – haustra – seen in the large bowel.

TOP TIPS
When faced with small bowel dilatation on an AXR in an obstructed patient, look for clues with regards
to the cause of obstruction, for example:

▪▪ Surgical clips (previous surgery increases the likelihood of adhesions).


▪▪ Bowel gas below the inguinal ligament (this indicates the presence of a hernia, which can become
strangulated).
▪▪ Biliary gas and a gallstone in the right iliac fossa suggestive of gallstone ileus.

2 What are the most common causes?


The most common causes of SBO in adults are shown in Table 7.4. Taking a careful history from
the patient will give important information to help determine the cause.

Table 7.4  Common causes of SBO


• Adhesions (>90% of cases) from previous surgery
• Malignancy (carcinoma, lymphoma)
• Inflammatory bowel disease (Crohn’s disease with strictures)
• Hernia (obstructed inguinal or femoral)
• Small bowel volvulus
• Gallstone ileus

221

K30031_Book.indb 221 9/6/17 1:36 PM


7  Abdomen and pelvis cases

3 How should the patient be managed?


The patient needs close monitoring and a senior review. Initial management should consist of
insertion of a NG tube and administration of IV fluids, known as ‘drip and suck’. She will also
need an erect CXR to exclude a perforation. The patient must be kept nil by mouth, will need
routine blood tests to look for evidence of infection or dehydration, and may require a CT scan in
order to determine the location and cause of obstruction. Subsequent management depends on
the cause. If the patient has had no previous abdominal surgery (a ‘virgin abdomen’), the obstruc-
tion is less likely to resolve with conservative management and to require surgical exploration. CT
will often help make this decision.

• In this case, a CT was requested (Figure 7.4E), which showed dilated small bowel with a
point of calibre change in the pelvis. No obstructing mass was demonstrated: therefore,
the cause was thought likely to be adhesions. After a day of conservative management, the
symptoms did not settle therefore a laparotomy was performed to relieve the obstruction
and treat the adhesions.

Fig. 7.4E  Axial contrast-enhanced CT of the abdomen showing loops of dilated small bowel containing
air/fluid levels (arrows).

LEARNING POINTS: SMALL BOWEL OBSTRUCTION


▪▪ >3 cm can be used as an approximate cut-off for small bowel dilatation.
▪▪ Always take a careful history including past surgical history.
▪▪ When small bowel dilatation is seen on an abdominal radiograph, check for clues as to the cause
(e.g. surgical clips/anastomoses, gas below the inguinal ligament, aerobilia, and calcified gallstone
outside the gallbladder).

222

K30031_Book.indb 222 9/6/17 1:36 PM


Case 7.5

Fig. 7.5A  Supine AXR.

A 60-year-old male presents to the ED with 2 days of abdominal pain and absolute constipation.
He has not vomited. He describes alternating diarrhoea and constipation for several months and
feeling generally tired and lethargic.
On examination, he is pale but afebrile with normal BP and mild tachycardia. His abdomen is
distended and tympanic. Digital rectal examination reveals an empty rectum. Initial blood tests
show microcytic anaemia but normal renal function, electrolytes, and inflammatory markers.
An AXR is requested (Figure 7.5A).

CASE 7.5: QUESTIONS


1 What does the AXR show?
2 What is the likely diagnosis?
3 What is the next most appropriate imaging investigation?
4 How should the patient be managed?

223

K30031_Book.indb 223 9/6/17 1:36 PM


7  Abdomen and pelvis cases

CASE 7.5: ANSWERS


1 What does the AXR show?
The AXR shows gas-filled dilated large bowel down to the level of the sigmoid colon. The small
bowel is not dilated. Table 7.5A and Figures 7.5B and 7.5C show a key difference between small
and large bowel on AXR.

Figs. 7.5B, C
Valvulae
conniventes in
small bowel (A) and
haustra in large
bowel (B). Valvulae
conniventes
traverse the lumen
and haustra
indent, although
sometimes
C haustral folds can
also traverse.

224

K30031_Book.indb 224 9/6/17 1:36 PM


Case 7.5: Answers

Table 7.5A  Radiological differentiation of dilated small


and large bowel on AXR

Small bowel Large bowel


Smaller calibre (>3 cm) Larger calibre (>6 cm)
Central location Peripheral location
Valvulae conniventes Haustrations
No faecal matter May contain faeces

2 What is the likely diagnosis?


The most likely diagnosis at this stage is large bowel obstruction (LBO). Common causes of LBO
in adults are listed in Table 7.5B.

Table 7.5B  Common causes of LBO in adults


• Malignancy (colorectal carcinoma)
• Inflammatory strictures, most commonly complicating diverticulitis
• Volvulus (sigmoid)
• Intussusception, less common

3 What is the next most appropriate imaging investigation?


Assuming the patient’s condition does not require urgent laparotomy, the patient will require an
erect CXR (as in most cases of acute abdomen) to exclude a perforation; it will also demonstrate
lung metastases. A CT scan of the abdomen and pelvis is often requested to determine the cause
of the obstruction and to look for evidence of complications such as perforation or bowel isch-
aemia. CT will also allow staging of tumours.
In this case, the CT scan showed a tumour in the sigmoid colon causing luminal obstruction,
without perforation.

4 How should the patient be managed?


Initial management will depend on the clinical status of the patient. Dehydration and electrolyte
disturbances should be corrected and a nasogastric tube should be inserted if the patient is vom-
iting. Subsequent management is guided by the cause of the obstruction and whether there is
evidence of perforation. If there is a perforation or bowel ischaemia, the patient will likely require
emergency surgery without delay for further imaging. Sometimes conservative management may
be appropriate.
In cases of an obstructing tumour, if time allows the patient will need to be discussed
at the colorectal MDT meeting in order to stage the cancer and decide what treatment should
be offered. A colonic stent might be appropriate to relieve the obstruction, which can be pal-
liative or a temporary measure prior to surgical resection. Colonic stents are inserted radio-
logically (Figure 7.5D). Colonic stents are being increasingly used to relieve acute LBO; they
are used for more distal lesions and allow patient resuscitation to facilitate safer surgery if
indicated.

225

K30031_Book.indb 225 9/6/17 1:36 PM


7  Abdomen and pelvis cases

Fig. 7.5D  Fluoroscopy images during colonic stent insertion. First, contrast (A) introduced by a rectal
catheter is used to demonstrate the location of the tumour (circled) and the degree of stenosis. A wire (B) is
guided per rectum through the narrowing caused by the tumour then a self-expanding metallic stent (C) is
fed per rectum over the guidewire and the wire removed. The stent expands and relieves the obstruction.
This is a classical ‘applecore’ stricture of carcinoma, in this case in the distal transverse colon. The wire (B)
lies in the splenic flexure. Note the air-filled descending colon (D) and also ‘waisting’ of the stent (C) where
the stent traverses the stricture.

LEARNING POINTS: LARGE BOWEL OBSTRUCTION


▪▪ It is important to distinguish between small and large bowel on AXR.
▪▪ Small bowel dilatation often coexists secondary to LBO.
▪▪ Common causes are malignancy and inflammatory strictures.
▪▪ A CT scan can help determine the cause of the obstruction, which will in turn guide management.
▪▪ When discussing management of a patient, always use an ABCDE approach initially and be able
to discuss several different options of further treatment.

226

K30031_Book.indb 226 9/6/17 1:36 PM


Case 7.6

Fig. 7.6A
Supine AXR.

A 48-year-old female is seen in the gastroenterology clinic with recurrent RUQ pain. She has a
history of diet-controlled type 2 diabetes, gastro-oesophageal reflux disease, and fibroids. Clinical
examination reveals mild tenderness in the RUQ but is otherwise unremarkable. Observations
are normal. Blood tests including a full blood count, urea and electrolytes, inflammatory markers,
and LFTs are within the normal range. She has had an US, arranged in the community by her GP,
of the RUQ, which showed ‘shadowing’ in the gallbladder fossa of uncertain significance.
An AXR is requested (Figure 7.6A).

CASE 7.6: QUESTIONS


1 What does the AXR show? 3 What should the next management step
2 What could the abnormality represent? involve?

227

K30031_Book.indb 227 9/6/17 1:36 PM


7  Abdomen and pelvis cases

CASE 7.6: ANSWERS


1 What does the AXR show?
It shows a peripherally calcified lesion in the RUQ (Figure 7.6B). There is no biliary gas. The bowel
gas pattern is normal.

Fig. 7.6B  Porcelain gallbladder (dense calcification in the RUQ [A]). Note the curvilinear splenic artery (B)
and left iliac artery (C) calcification.

2 What could the abnormality represent?


The RUQ density is peripherally calcified, a pattern known as ‘eggshell calcification’. This lesion
could be within liver, gallbladder fossa or at the upper pole of the right kidney. The rim-like nature
of the calcification suggests it is likely to be within the gallbladder wall. This is called a ‘porcelain
gallbladder’. This condition is associated with gallstones and chronic cholecystitis, and also an
increased risk of gallbladder carcinoma.

228

K30031_Book.indb 228 9/6/17 1:36 PM


Case 7.6: Answers

3 What should the next management step involve?


The diagnosis and recognition of porcelain gallbladder is important as this condition carries
an increased risk of malignancy (up to 7–10% incidence); therefore, once porcelain gallblad-
der is diagnosed the treatment is normally cholecystectomy. An appropriate next investigation
would be CT to confirm that the calcification is within the gallbladder (Figure 7.6C) and also
to exclude an invasive gallbladder mass, nodes or liver metastases, following which there can
be a discussion with the patient regarding an elective cholecystectomy. US is less useful in this
condition as calcific shadowing obscures the gallbladder (as in this case).

Fig. 7.6C  Coronal


postcontrast CT RUQ
showing porcelain
gallbladder. Note the
curvilinear calcification
in a thickened
gallbladder wall (A) and
stone in the gallbladder
lumen (B). Stones
and sludge can form
a dense mixture in
the gallbladder, and
tumour can have a
similar appearance.

LEARNING POINTS: PORCELAIN GALLBLADDER


▪▪ Calcification of the gallbladder wall, known as porcelain gallbladder owing to the brittle nature of
the calcified gallbladder.
▪▪ Often detected on AXR as rim-like calcification in the RUQ.
▪▪ A CT or US scan should be performed to confirm that the calcification is in the wall of the gallbladder.
▪▪ Normally asymptomatic.
▪▪ Treated with cholecystectomy owing to the increased risk of gallbladder carcinoma.
▪▪ The differential diagnosis of RUQ calcification is gallstones, milk of calcium bile (bile that contains high
levels of calcium salts), and calcified lesions in the kidney, pancreas, adrenal gland or liver.

229

K30031_Book.indb 229 9/6/17 1:36 PM


Case 7.7

Fig. 7.7A  Coronal unenhanced CT image at the level of the kidneys.

A 45-year-old male presents to the ED with sudden-onset severe left-sided loin pain radiating
inferiorly and anteriorly to his groin. He describes the pain as colicky in nature.
On examination between attacks his abdomen is soft with no palpable masses but during the
attacks it is not possible to examine him because he is rolling around and in pain. He is tachy-
cardic at 100 bpm, BP 140/80 mmHg, and apyrexial. Urinalysis shows blood ++ but is negative for
leucocytes, nitrites, and glucose. Routine blood tests are normal.
An unenhanced CT of the abdomen and pelvis is requested (CT KUB study) (Figures 7.7A
and 7.7B).

230

K30031_Book.indb 230 9/6/17 1:36 PM


Case 7.7: Questions

Fig. 7.7B  Axial unenhanced CT image at the level of the bladder.

CASE 7.7: QUESTIONS


1 What is the diagnosis?
2 What key radiological findings do these CT images show?
3 What are the possible causes?
4 What are the potential complications?
5 How should this patient be managed?

231

K30031_Book.indb 231 9/6/17 1:36 PM


7  Abdomen and pelvis cases

CASE 7.7: ANSWERS


1 What is the diagnosis?
This patient has left renal colic as a result of an obstructing left ureteric calculus.

2 What key radiological findings do these CT images show?


The CT shows a 4 mm calculus at the left vesicoureteric junction (VUJ) with resultant left
­hydronephrosis and associated perinephric inflammatory change (‘fat stranding’). (Figures 7.7C
and 7.7D). This stone is small and lies at the VUJ, therefore it is likely to pass spontaneously.

Fig. 7.7C  Coronal CT image demonstrating moderate left hydronephrosis (A) with some inflammatory
change in the perinephric fat (B). Note the dilated ureter in the pelvis (C). Iliopsoas muscles (P).

232

K30031_Book.indb 232 9/6/17 1:36 PM


Case 7.7: Answers

Fig. 7.7D  Axial CT image in the pelvis demonstrates a 4 mm calculus at the left VUJ (A). Note: these
scans are acquired with the patient prone; the calculus is, therefore, nondependent (it has not fallen
anteriorly in the bladder with patient prone) indicating that it is lodged in the VUJ.

3 What are the possible causes?


The majority of renal calculi are idiopathic. Most stones are composed of calcium and are visible
on unenhanced CT (if large enough, >4 mm, they may be visible on AXR). Causes of calcium
stones are listed in Table 7.7.

Table 7.7  Common causes of calcium renal calculi


• Idiopathic hypercalciuria
• Hypercalcaemia (e.g. primary hyperparathyroidism)
• Low fluid intake
• Urinary tract malformations (e.g. horseshoe kidney)
• Recurrent urinary tract infections
• Prolonged bed rest

Other common types of renal calculi include uric acid (usually radiolucent), struvite, and
­c ysteine-based stones.

233

K30031_Book.indb 233 9/6/17 1:36 PM


7  Abdomen and pelvis cases

4 What are the potential complications?


The complications associated with renal calculi are:

• Infection of the obstructed kidney and resultant septicaemia.


• Rupture of the renal pelvis.
• Renal impairment.
• Stone build up resulting in staghorn calculus and nonfunctioning kidney (Figure 7.7E).
• Bladder stones and associated complications including recurrent cystitis and urinary
frequency.
• Chronic irritation of urothelium, squamous metaplasia, and malignancy.

Fig. 7.7E  Supine AXR in an elderly female patient demonstrating a large staghorn calculus of the right
kidney (A). This patient also has an inferior vena caval filter in situ (B). Note the previous left dynamic hip
screw.

234

K30031_Book.indb 234 9/6/17 1:36 PM


Case 7.7: Answers

5 How should this patient be managed?


Initial management of renal colic requires strong analgesia (e.g. NSAID or opiate). Most ureteric
stones that are < 5 mm will pass spontaneously. Indications for intervention include:

• Ongoing pain.
• Sepsis.
• Failure for the stone to spontaneously pass.
• Larger impacted stones.

If intervention is required then the options for management include:

• Extracorporeal shock wave lithotripsy (ESWL).


• Ureteroscopy and removal or ‘in situ’ stone fragmentation.
• Percutaneous nephrostomy may be required to drain/relieve an obstructed or infected
pelvicalyceal system.
• Open surgery.

LEARNING POINTS: RENAL COLIC


▪▪ Renal colic presents classically as intermittent severe loin to groin pain with haematuria.
▪▪ Unenhanced low-dose abdominal CT is the imaging technique of choice and may show extrarenal
causes of pain in addition (e.g. appendicitis, diverticulitis).
▪▪ Look for opaque calculi within the kidneys, ureters, and bladder.
▪▪ Look for a dilated renal pelvicalyceal system or ureter to indicate urinary obstruction.
▪▪ Always check renal function (plasma urea, creatinine, and GFR) and for signs of infection.

235

K30031_Book.indb 235 9/6/17 1:36 PM


Case 7.8

Fig. 7.8A  Axial noncontrast CT image at the level of the left kidney.

Fig. 7.8B  A second axial noncontrast CT image, more inferior than Fig. 7.8A, but also at the level of the
left kidney.

236

K30031_Book.indb 236 9/6/17 1:36 PM


Case 7.8: Questions

A 68-year-old female presents acutely unwell to the ED with fever and confusion. She is unable to
give a coherent history. She is known to have type 2 diabetes. On examination she is haemody-
namically unstable with the following observations: temperature 39.5°C, HR 130 bpm, respiratory
rate 28 bpm, and BP 105/80 mmHg. She is hyperglycaemic on fingerprick testing. Urinalysis is
positive for glucose, white cells, and nitrites. Cardiorespiratory examination is unremarkable but
she is very tender in the left loin region with guarding. Bowel sounds are present. A CT KUB study
is performed (Figures 7.8A and 7.8B).

CASE 7.8: QUESTIONS


1 What do the CT images show?
2 What is the likely diagnosis?
3 How should the patient be managed?

237

K30031_Book.indb 237 9/6/17 1:36 PM


7  Abdomen and pelvis cases

CASE 7.8: ANSWERS


1 What do the CT images show?
The left kidney is enlarged and oedematous. There is an intraparenchymal cortical gas-contain-
ing crescenteric collection and associated inflammatory perinephric fat stranding (Figures 7.8C
and 7.8D). There is no evidence of hydronephrosis, stones or lymph node enlargement.

Fig. 7.8C  CT abdomen at the level of the left kidney mid-pole. This shows gas within the left renal cortical
parenchyma (A). Note the normal ureter (B), aorta (C), and right lobe liver (D).

2 What is the likely diagnosis?


The diagnosis is emphysematous pyelonephritis. This is a serious condition characterised by gas
within the collecting system and renal parenchyma secondary to a urinary tract infection.

• The causative organism is usually Escherichia coli and patients are typically diabetic or
immunocompromised.
• In contrast, gas confined to the collecting system, called emphysematous pyelitis, confers a
more favourable prognosis.

3 How should the patient be managed?


ABCDE approach, IV access, routine bloods (including blood sugar, inflammatory markers, and
renal function), blood cultures, and midstream urine for culture are also needed. The patient
should be treated as per the local hospital sepsis pathway, including aggressive IV fluid and anti-
biotic therapy. The patient should be commenced on an insulin sliding scale with regular moni-
toring of blood glucose levels. This condition has a high mortality rate and the patient should be

238

K30031_Book.indb 238 9/6/17 1:36 PM


Case 7.8: Answers

Fig. 7.8D  CT abdomen at the level of the kidneys showing an enlarged left kidney (LK) with an
intrasubstance crescenteric parenchymal gas collection (A) and perinephric fat stranding (B). Normal right
kidney (C).

urgently referred to the urology team and diabetic physicians. Urgent percutaneous drainage of
perinephric collections should be considered. Ultimately, nephrectomy may be required.
Emphysematous pyelonephritis is a condition with a high mortality and diabetics are predis-
posed. Infection may spread to/from the ureter and bladder (emphysematous cystitis). Urgent
recognition and treatment is needed.

LEARNING POINTS: EMPHYSEMATOUS PYELONEPHRITIS


▪▪ Gas within the ureter or renal collecting system on AXR should raise the suspicion of
emphysematous pyelonephritis.
▪▪ The diagnosis is best demonstrated with CT, which may also show complications such as renal or
extrarenal fluid collections.
▪▪ Emphysematous pyelonephritis is a urological emergency and urgent specialist referral should be
undertaken.

239

K30031_Book.indb 239 9/6/17 1:36 PM


Case 7.9

Fig. 7.9A  Longitudinal US image of the uterus. The endometrial thickness has been measured (calipers).

Fig. 7.9B  Transverse and longitudinal US images of the left adnexa. The radiologist has labelled the left
ovary and a suspected ectopic pregnancy.

240

K30031_Book.indb 240 9/6/17 1:36 PM


Case 7.9: Questions

A 24-year-old female presents to the ED with sudden-onset severe lower abdominal pain. She is
sexually active and cannot remember the date of her last menstrual period (LMP).
On examination there is lower abdominal tenderness. Observations reveal pulse 120 bpm,
BP 90/60 mmHg, and normal temperature. She is apyrexial. A urine HCG test is positive and her
other blood results are normal.
An urgent transvaginal US is performed (Figures 7.9A and 7.9B).

CASE 7.9: QUESTIONS


1 What is an ectopic pregnancy?
2 How do patients with ectopic pregnancy normally present?
3 What do the US images show?
4 How would you manage this patient?

241

K30031_Book.indb 241 9/6/17 1:36 PM


7  Abdomen and pelvis cases

CASE 7.9: ANSWERS


1 What is an ectopic pregnancy?
An ectopic pregnancy is when the embryo implants outside the uterus, most commonly in the
Fallopian tube:

• It is often idiopathic; however, it is more common in patients who have damaged Fallopian
tubes from previous pelvic inflammatory disease or previous tubal surgery.

2 How do patients with ectopic pregnancy normally present?


If a tubal ectopic causes rupture, then bleeding can be catastrophic and the patient presents with
collapse and abdominal pain. This is a surgical emergency and laparoscopy is required:

• With increasing use of US, increasingly patients are diagnosed early and while
asymptomatic. If a patient with a positive pregnancy test undergoes US that does not detect
an intrauterine pregnancy it may be because:
• The gestation is too early (<5 weeks).
• There has been a complete miscarriage.
• The pregnancy is ectopic.

3 What do the US images show?


The US images show an empty uterus with normal endometrial thickness. The left ovary contains
a normal corpus luteum, which is sustaining the pregnancy. In the left adnexal region (in the left
Fallopian tube) is a mass with a hyperechoic ring and internal material suggestive of a fetal pole.

Fig. 7.9C  Longitudinal US image of the uterus. Normal hyperechoic endometrium (A) and normal
myometrium (B) are noted. There is no evidence of an intrauterine pregnancy.

242

K30031_Book.indb 242 9/6/17 1:36 PM


Case 7.9: Answers

Fig. 7.9D  Transverse US image of the left adnexal region. The normal left ovary containing a corpus
luteum is noted (A). Medial to this (in the left Fallopian tube) is a round structure with a hyperechoic rim (B)
containing echogenic material (C) in keeping with a fetal pole. Appearances are those of a left tubal ectopic
pregnancy.

A heartbeat was present and according to fetal pole length the expected gestation is 6 weeks
(Figures 7.9C and 7.9D). Although not imaged, there was also a moderate volume of echogenic
free fluid in the pelvis suggestive of blood.

4 How would you manage this patient?


Management of symptomatic patients with suspected ectopic pregnancy includes:

• Nil by mouth, IV access.


• FBC and cross match.
• Urine pregnancy test.
• Gynaecology referral on an urgent basis.

243

K30031_Book.indb 243 9/6/17 1:36 PM


7  Abdomen and pelvis cases

• Urgent US should be undertaken if appropriate/available. Do not let this delay surgical


intervention if that is required (patient unstable, shocked).
• Laparoscopy or medical management if appropriate, i.e. asymptomatic patient and early
gestation.
• Particular support must be given to these patients who not only have ‘lost their baby’ and
had to undergo emergency surgery but also are likely to have reduced fertility in the future.

LEARNING POINTS: ECTOPIC PREGNANCY


▪▪ A pregnancy test (urine HCG) must be performed in all women of childbearing age who present
with abdominal pain.
▪▪ A urine HCG is almost always positive in patients with an ectopic pregnancy.
▪▪ A transvaginal US will not always visualise an ectopic pregnancy (depending on the stage of
gestation) but it should detect an intrauterine pregnancy.
▪▪ Imaging should not delay intervention if that is required on clinical grounds.

244

K30031_Book.indb 244 9/6/17 1:36 PM


Case 7.10

Fig. 7.10A  Erect CXR.

A 27-year-old Afro-Caribbean male presents to the ED by ambulance, with generalised severe


and nonspecific chest and abdominal pain. On examination, he is febrile 37.5°C, with HR 130 per
min, BP 90/65 mmHg, respiratory rate 29 bpm, and oxygen saturations of 94% on 2 litres of
­oxygen by mask. A CXR is performed as part of his investigations (Figure 7.10A).

CASE 7.10: QUESTIONS


1 What does the CXR show?
2 What underlying medical condition is this patient likely to have?
3 How should the patient be managed?

245

K30031_Book.indb 245 9/6/17 1:36 PM


7  Abdomen and pelvis cases

CASE 7.10: ANSWERS


1 What does the CXR show?
There are a number of key findings in the CXR (Figure 7.10B):

• Diffusely increased bone density in keeping with bony sclerosis.


• Sclerotic (dense) and partially collapsed humeral heads bilaterally indicating avascular
necrosis (‘snowcap’ appearance).
• Calcified spleen in the LUQ.

Fig. 7.10B  CXR showing diffuse bony sclerosis, avascular necrosis of the humeral heads (A), and a
calcified atrophic spleen (B) in keeping with sickle cell disease. Note the oxygen tubing (C).

2 What underlying medical condition is this patient likely to have?


The combination of bony sclerosis, avascular necrosis, and calcified spleen in a young patient
suggests an underlying diagnosis of sickle cell disease. He is likely to have a painful sickle
crisis; infection commonly precipitates a painful crisis. The spleen is particularly prone to
vaso-­occlusion and damage. Vaso-occlusion in blood vessels causes bone infarcts, pain, and
progressive bony sclerosis. As the spleen is progressively damaged it atrophies and over time

246

K30031_Book.indb 246 9/6/17 1:36 PM


Case 7.10: Answers

will calcify. Hyposplenism predisposes individuals to infection with certain bacteria, including
Streptococcus pneumoniae, non-typhi Salmonella spp. and Haemophilius influenzae type b.

• Sickle cell disease is a genetic autosomal recessive condition particularly common


in people of African descent, in which a mutated form of haemoglobin causes RBCs
to become sickle-shaped in conditions of stress or hypoxia. The abnormal RBCs are
viscous and fragile, have a reduced lifespan, and adhere very easily to endothelium
leading to chronic haemolytic anaemia plus vaso-occlusion with pain and distal organ
damage.

3 How should the patient be managed?


The abnormal observations mean that this patient is unstable, therefore a structured ABCDE
approach is essential. In an OSCE situation you will gain marks if you say you would seek
senior help. A blood gas may provide key information while waiting for formal blood results.
An ECG should also be performed, especially in view of the patient’s tachycardia. He will
need opiate analgesia, oxygen, fluid resuscitation, monitoring, and antibiotics (after blood,
urine and sputum cultures) if there is any suggestion of infection. Urgent haematology review
is needed.
In the long term, management of sickle cell disease is focussed on treating anaemia, pain,
and vaso-occlusive crises. Prevention of further complications, such as infection, is also impor-
tant, therefore people with sickle cell disease who are hyposplenic are offered additional
vaccinations.

LEARNING POINTS: SICKLE CELL DISEASE


▪▪ Inherited condition common in people of African descent.
▪▪ Diagnosis by haemoglobin electrophoresis.
▪▪ Causes anaemia and vaso-occlusive crises, which may be triggered by dehydration, hypoxaemia,
and temperature changes.
▪▪ Initially causes splenic enlargement followed by repeated infarction leading to atrophy and
calcification.
▪▪ Increased risk of infections due to autosplenectomy, particularly encapsulated bacteria.
▪▪ CXR appearances include bony sclerosis, avascular necrosis, H-shaped vertebrae (due to
endplate infarction and collapse (Figure 7.10C), pulmonary infarcts and pneumonia, calcified
spleen, and osteomyelitis.

247

K30031_Book.indb 247 9/6/17 1:36 PM


7  Abdomen and pelvis cases

Fig. 7.10C  AP view of the lumbar spine in a sickle-cell patient. He has classical ‘H’ vertebrae with
endplate depressions (A). Note the calcified and hypoplastic spleen (B). The endplates are particularly
prone to ischaemic damage as they are a ‘watershed’ area of blood supply, and with progressive vaso-
occlusive events they soften and collapse causing the ‘H’ appearance. The vertebral pedicles (C) and
spinous processes (D) are also shown.

248

K30031_Book.indb 248 9/6/17 1:36 PM


Case 7.11

Fig. 7.11A  Supine AXR, with a magnified view of the epigastric region.

A 45-year-old male presents to the ED with a 48-hour history of severe upper abdominal pain
radiating to his back. He has a long history of alcohol misuse and has seen his GP several times
over the past 4 years with recurrent abdominal pains.
On examination, he is pale, looks malnourished, and is tender in the epigastrium. No abdomi-
nal masses are palpable and he is not jaundiced. Observations reveal he is apyrexial, has oxygen
saturations 99% on air, HR 95 bpm, and BP 115/85 mmHg.
An AXR is performed (Figure 7.11A). Blood test results are:

Hb 108 g/L (130–180 g/L) ALT 143 IU/L (0–41 IU/L)


MCV 90 fL (80–100 fL) AST 214 IU/L (0–40 IU/L)
Platelets 400 × 10 /L (150–450 × 10 /L)
9 9 ALP 110 IU/L (40–129 IU/L)
INR 1.2 (0.8–1.2) Bilirubin 20 micromol/L (0–21 micromol/L)
Sodium 136 mmol/L (135–146 mmol/L) Amylase 300 IU/L (28–100 IU/L)
Potassium 3.3 mmol/L (3.2–5.1 mmol/L) GGT 469 IU/L (10–71 IU/L)
Urea 2.0 mmol/L (1.7–8.3 mmol/L) Albumin 31 g/L (34–48 g/L)
Creatinine 120 micromol/L (62–106 micromol/L) CRP 73 mg/L (0–5 mg/L)

CASE 7.11: QUESTIONS


1 What does the AXR show? 3 What findings should be specifically
2 What does this abnormality represent and looked for on clinical examination?
what are the causes of this condition? 4 How should the patient be managed?
249

K30031_Book.indb 249 9/6/17 1:36 PM


7  Abdomen and pelvis cases

CASE 7.11: ANSWERS


1 What does the AXR show?
Diffuse punctate epigastric calcification is present in the region of the pancreas on AXR
(Figure 7.11B).

Fig. 7.11B  AXR showing diffuse punctate pancreatic calcification (A). Note the air-filled stomach (B) and
duodenum (C).

2 What does this abnormality represent and what are the causes of this condition?
Pancreatic calcification is caused by chronic pancreatitis in the vast majority of cases. Repeated
episodes of inflammation cause progressive fibrotic destruction of pancreatic glandular tissue and
over time it loses function and calcifies. Most cases of chronic pancreatitis are related to long-term
alcohol misuse but some are idiopathic or secondary to autoimmune pancreatitis.

3 What findings should be specifically looked for on clinical examination?


Given the alcohol history, it is important to look for signs of chronic liver disease such as:

• Clubbing.
• Jaundice.
• Spider naevi.
• Ascites.
• Muscular atrophy.

250

K30031_Book.indb 250 9/6/17 1:36 PM


Case 7.11: Answers

• Gynaecomastia.
• Palmar erythema.
• Encephalopathy.
• Hepatomegaly.

4 How should the patient be managed?


The patient will require further investigation to confirm position and distribution of pancreatic
calcification. CT (Figure 7.11C) will also help exclude pancreatic carcinoma, which may compli-
cate this condition (more commonly in smokers). MRCP can be helpful in some cases to confirm
changes of pancreatic atrophy and duct dilatation with calculus formation. The organ damage is
irreversible so there is no specific treatment for chronic pancreatitis, other than pain control and
lifestyle modification, particularly reducing alcohol intake. If the organ damage is sufficiently
severe the patient may develop diabetes, which will need to be tested for and treated. Patients may
also have diarrhoea and malabsorption resulting from lack of pancreatic enzymes, and enzyme
supplements can help. Patients with chronic pancreatitis have an increased risk of developing
pancreatic cancer: there is up to a 5% increase of malignancy over a 20-year period.

Fig. 7.11C  Contrast-enhanced axial CT of the upper abdomen showing punctate calcifications within the
pancreatic head (A). Normal left kidney (B), right lobe of liver (C), aorta containing contrast (D), and lumbar
vertebra (E).

251

K30031_Book.indb 251 9/6/17 1:36 PM


7  Abdomen and pelvis cases

LEARNING POINTS: CHRONIC PANCREATITIS


▪▪ Associated with recurrent epigastric abdominal pain radiating to the back.
▪▪ Mainly affects men.
▪▪ Causes include chronic alcohol misuse and, less commonly, autoimmune pancreatitis. Some
cases are idiopathic.
▪▪ No specific treatment other than pain control and lifestyle modification.
▪▪ Chronic inflammation leads to pancreatic calcification and dysfunction over time.
▪▪ Increased risk of diabetes and pancreatic cancer.

252

K30031_Book.indb 252 9/6/17 1:36 PM


Case 7.12

Fig. 7.12A  Liver US, section through the left lobe of liver.

A 65-year-old male presents to his GP with malaise and RUQ abdominal discomfort. He has a 20
pack-year smoking history but no other relevant medical history. On examination he has a pal-
pable, tender, and irregular liver edge. He is noted to have deranged liver function tests and mild
anaemia. A liver US is performed (Figure 7.12A).

CASE 7.12: QUESTIONS


1 What does the US show?
2 What are the possible causes?
3 What investigation should be carried out next?
4 How should the patient be managed?

253

K30031_Book.indb 253 9/6/17 1:36 PM


7  Abdomen and pelvis cases

CASE 7.12: ANSWERS


1 What does the US show?
The grey-scale image (Figure 7.12B) shows multiple well-defined hyperechoic liver lesions. The
liver contour is smooth and there are no imaging features of hepatic cirrhosis. The liver capsule
and smooth liver surface are seen anteriorly.

Fig. 7.12B  Liver US showing hyperechoic (brighter than liver) liver lesions (arrows A) and liver capsule/
anterior margin (arrows B).

2 What are the possible causes?


There are several possible causes for multiple solid liver lesions (including benign lesions such as
haemangiomata, focal nodular hyperplasia, and adenomata). The latter two are more common in
young females. However, in an older patient the most likely cause is hepatic metastases. Patients
may be asymptomatic at presentation. Alternatively, if there is a high tumour burden, patients
may present with abdominal, liver capsular pain or features of hepatic decompensation such as
jaundice and ascites.

3 What investigation should be carried out next?


A CT chest, abdomen (Figure 7.12C), and pelvis with contrast should be performed in order to
identify and stage the primary malignancy.

254

K30031_Book.indb 254 9/6/17 1:36 PM


Case 7.12: Answers

Fig. 7.12C  Contrast-enhanced CT abdomen through the liver showing multiple hypodense liver
metastases throughout both hepatic lobes.

4 How should the patient be managed?


The patient should be discussed at the appropriate MDT meeting where further treatment options
can be discussed. Histology can be obtained via US-guided biopsy of one of the liver lesions in
order to confirm the diagnosis if the primary site is not apparent or suitable for biopsy. The diag-
nosis in this patient was caecal carcinoma; this lesion was demonstrated on CT (Figure 7.12D)
and biopsied colonoscopically.
Tumours that metastasise to the liver include:

• GI tract (colon, stomach) and pancreas.


• Breast.
• Lung.
• Melanoma.
• Kidney.
• Ovaries.

255

K30031_Book.indb 255 9/6/17 1:36 PM


7  Abdomen and pelvis cases

Fig. 7.12D  Axial CT right iliac fossa level in the same patient confirms a caecal mass consistent with likely
caecal carcinoma (arrows). This was confirmed on colonoscopic biopsy.

LEARNING POINTS: LIVER METASTASES


▪▪ The most common cause of multiple solid liver lesions in an older patient is metastases.
▪▪ Patients are often asymptomatic or have few symptoms and the diagnosis is often made while
investigating abnormal liver function tests.
▪▪ The site of the primary tumour should be sought. Many different tumours metastasise to the liver;
however, the commonest types are colorectal, pancreatic, oesophageal, breast, and lung cancer.

256

K30031_Book.indb 256 9/6/17 1:36 PM


Case 7.13

Fig. 7.13A  Axial contrast-enhanced CT of the abdomen at the level of the patient’s kidneys.

A 45-year-old male is referred by his GP for an US for a suspected gall stone. Incidental note is
made of a solid lesion arising from his right kidney. He is asymptomatic. His bloods, in particular
his renal function, are normal.
A CT is arranged for further characterisation (Figure 7.13A).

CASE 7.13: QUESTIONS


1 What does the CT show?
2 How may this condition present?
3 How should this patient be treated?

257

K30031_Book.indb 257 9/6/17 1:36 PM


7  Abdomen and pelvis cases

CASE 7.13: ANSWERS


1 What does the CT show?
A heterogeneously enhancing mass arising from the interpolar region of the right kidney. There
is a solitary enlarged aortocaval lymph node (Figure 7.13B). The appearances are consistent with
likely right renal cell carcinoma (RCC, adenocarcinoma) and aortocaval nodal metastasis.

Fig. 7.13B  Axial CT scan demonstrating the right kidney (A) containing a solid and heterogeneously
enhancing mass (B) measuring 3.5 cm. In addition there is an enlarged aortocaval lymph node (C),
suggestive of nodal metastasis. Note the normal left kidney (D), abdominal aorta (E), and inferior vena cava
(F) compressed by the node.

2 How may this condition present?


Although this lesion was picked up incidentally, patients are often symptomatic. They may pres-
ent with:

• Haematuria.
• Loin pain.
• Mass in the flank.
• Malaise.
• Weight loss.

258

K30031_Book.indb 258 9/6/17 1:37 PM


Case 7.13: Answers

• Fever (RCC is a cause of pyrexia of unknown origin).


• Polycythaemia, hypercalcaemia, and thrombocytosis are associated paraneoplastic
syndromes.

As a general rule, the larger the tumour the more likely the patient is to be symptomatic
(Figure 7.13C – the patient presented with constitutional symptoms, polycythaemia, and a left
loin mass). Smoking, obesity, and hypertension are significant risk factors for RCC, and this
tumour type is more common in men >65 years of age.

Fig. 7.13C  Axial postcontrast renal CT scan. Only a small amount of the left renal cortex is visible (A), it is
largely replaced by a huge left RCC (B), and this had metastasised to the lungs at the time of diagnosis.

3 How should this patient be treated?


Treatment depends on the stage at presentation. All patients require full staging of the chest and
urinary tract with CT and MDT discussion. PET scanning may be helpful in complex cases. RCC
has a tendency to involve the renal vein and then to metastasise haematogenously; lung metasta-
ses are most common. There may be involvement of regional lymph nodes. Surgery is the treat-
ment of choice for localised disease; regional involved lymph nodes can also be removed if there
are no distant metastases.

• The presenting patient in this case should be considered for a nephrectomy, although
the left kidney must be scrutinised to make sure there is not a contralateral tumour.
A PET/CT scan will help to establish if there is disease in the aortocaval lymph node, which

259

K30031_Book.indb 259 9/6/17 1:37 PM


7  Abdomen and pelvis cases

may affect treatment. Many tumours are resistant to chemotherapy and radiotherapy, and
immunotherapy may be helpful (interferon, interleukin-2).
• Smaller tumours at the upper or lower pole can be considered for ‘nephron-sparing’ partial
nephrectomy surgery.
• If the patient is not fit for surgery then image-guided tumour ablation could be considered.
• The second patient in the case may need to be considered for surgery for symptom control.

LEARNING POINTS: RENAL CELL CARCINOMA


▪▪ RCC is the most common type of malignant renal tumour.
▪▪ It classically presents with loin pain and haematuria but may be diagnosed in the asymptomatic
patient on incidental imaging.
▪▪ Contrast-enhanced CT of the chest, abdomen, and pelvis is needed for staging.
▪▪ The patient should be discussed urgently at the local urology MDT meeting and treatment options
discussed.

260

K30031_Book.indb 260 9/6/17 1:37 PM


Case 7.14

Fig. 7.14
Supine AXR.

A 22-year-old male presents to the ED with a 1-week history of worsening abdominal pain and
­d istension. He also describes increasing loose bowel motions containing dark and altered blood
over the past month and he feels unwell with episodes of shivering and sweats.
On examination he looks pale, tachycardic 100 bpm, BP 105/65 mmHg, and pyrexial at 37.8°C.
His abdomen is distended and generally tender with guarding in the upper abdomen. Bowel
sounds are barely audible. An AXR has been undertaken (Figure 7.14A).

CASE 7.14: QUESTIONS


1 What three key radiological findings are demonstrated?
2 What is the likely diagnosis?
3 Would you arrange any other imaging?
4 How should this patient be managed?

261

K30031_Book.indb 261 9/6/17 1:37 PM


7  Abdomen and pelvis cases

CASE 7.14: ANSWERS


1 What three key radiological findings are demonstrated?
The AXR (Figure 7.14B) demonstrates:

• Dilatation of the transverse colon (>6 cm maximum transverse dimension).


• Mucosal oedema causing mural and haustral thickening of the transverse colon.
• Air within the thickened wall of the transverse colon (pneumatosis coli).

Fig. 7.14B  Cropped


and enlarged section
of the same AXR
showing the abnormal
distal transverse
colon, which is
pathologically dilated.
Note the pronounced
thickening of the bowel
wall with oedema of
the haustra (A). The
haustral thickening
gives the classical
radiological
appearance of
‘thumb printing’, i.e.
one could imagine a
thumb imprint causing
the impression. The
other important
finding is air within the
thickened wall, termed
pneumatosis coli (B).

262

K30031_Book.indb 262 9/6/17 1:37 PM


Case 7.14: Answers

2 What is the likely diagnosis?


The likely diagnosis in this patient is acute toxic colitis with dilatation of the transverse colon,
also known as toxic megacolon. Toxic megacolon may involve all or parts of the colon and is a
complication of several colitides:

• Most commonly associated with inflammatory bowel disease, ulcerative colitis > Crohn’s
disease.
• Infective, and a number of organisms are associated, including salmonella, shigella,
campylobacter and cytomegalovirus in HIV. Toxic megacolon as a complication of
Clostridium difficile pseudo membranous colitis is also increasingly recognised.
• Other causes include irradiation and ischaemia.
• There are recognised and diagnostic criteria for toxic megacolon, which include:
• Radiological evidence of colonic dilatation (usually >6 cm).
• Fever, tachycardia, leucocytosis, and anaemia.
• Evidence of dehydration or shock.
The pathogenesis of toxic megacolon, however, remains unclear.

3 Would you arrange any other imaging?


These patients are at high risk of bowel perforation. Sepsis and shock are the other key compli-
cations. At presentation an erect CXR should be undertaken to exclude free intraperitoneal air.
The use of other imaging will depend on the condition of the patient. If not seriously unwell, CT
can be valuable to provide further information on the distribution of disease, potential ischaemic
bowel, and to look for occult free intraperitoneal air. Seriously unwell and peritonitic patients may
need to go straight to theatre.

4 How should this patient be managed?


The initial management involves an ABCDE assessment. Patients require IV access, rehydration,
routine bloods including cross-match, and urgent discussion with senior surgical, gastroen-
terological, and anaesthetic colleagues. It is essential in OSCE scenarios to emphasise the ini-
tial resuscitation of the patient, monitoring, and baseline investigation, and indicate that senior
advice would be sought. All these statements will gain marks and need to be stated even if they
seem obvious.
• In this patient the presentation is highly suggestive of underlying inflammatory bowel
disease. The patient failed to respond to initial IV steroids and antibiotics, and following
senior review underwent a subtotal colectomy with end ileostomy formation. The final
diagnosis histologically was Crohn’s disease. An AXR of toxic megacolon in a patient with
ulcerative colitis is shown in Figure 7.14C.

263

K30031_Book.indb 263 9/6/17 1:37 PM


7  Abdomen and pelvis cases

Fig. 7.14C  AXR showing toxic megacolon in a patient with ulcerative colitis. Note the pelvic intrauterine
contraceptive device (IUCD).

LEARNING POINTS: PNEUMATOSIS COLI


▪▪ This term refers to gas in the bowel wall.
▪▪ The pathogenesis is poorly understood.
▪▪ In adults most cases are asymptomatic, and there is an association with COPD.
▪▪ When associated with diseases leading to bowel necrosis (such as toxic megacolon) the presence
of pneumatosis coli is an ominous finding and associated with a high mortality.
▪▪ Pneumatosis coli does also occur in neonates, usually secondary to necrotising enterocolitis.

264

K30031_Book.indb 264 9/6/17 1:37 PM


Case 7.15

Fig. 7.15A  Upper abdominal US showing the liver and gallbladder.

A 42-year-old female presents to the ED acutely unwell with a 1-day history of fever, vomiting,
and severe RUQ abdominal pain. She has no relevant past medical history but has a raised BMI of
28 kg/m2. Baseline observations: temperature 38.5ºC, HR 120 bpm, respiratory rate 20 bpm, and
BP 130/80 mmHg. On examination she appears dehydrated and has a positive Murphy’s sign on
palpation of the right upper abdominal quadrant. An abdominal US is performed (Figure 7.15A).
She has raised WCC and inflammatory markers.

CASE 7.15: QUESTIONS


1 What does the US show?
2 What additional findings should be looked for on US?
3 How should the patient be managed?

265

K30031_Book.indb 265 9/6/17 1:37 PM


7  Abdomen and pelvis cases

CASE 7.15: ANSWERS


1 What does the US show?
The abdominal US image shows a large echobright (hyperechoic) calculus at the gallbladder neck
(Figure 7.15B). There is posterior acoustic shadowing as the US waves cannot penetrate the dense
calculi and are reflected back towards the US probe (a characteristic imaging finding in both
gallbladder and renal calculi). The gallbladder wall is thickened and inflamed, confirming the
clinical diagnosis of acute cholecystitis. Note the positive Murphy’s sign, gallbladder tenderness
on palpation; this sign may also be elicited with the US probe.

Fig. 7.15B  Upper abdominal US showing the liver capsule anteriorly (A), gallbladder lumen (B), gallstone (C),
and posterior acoustic shadow (D). The gallbladder wall is thickened (E, callipers).

2 What additional findings should be looked for on US?


It is important sonographically to look for a dilated common bile duct (CBD) and assess for the
presence of an obstructing stone (choledocholithiasis). In the context of sepsis this would imply
a diagnosis of acute cholangitis and would alter the patient’s management, as the patient may
then need to undergo ERCP (endoscopic retrograde cholangiopancreatography), which involves

266

K30031_Book.indb 266 9/6/17 1:37 PM


Case 7.15: Answers

CBD cannulation via endoscopy and stone retrieval. MRCP is often used to visualise the bile
ducts noninvasively. US is very accurate at demonstrating the gallbladder and upper CBD but
often, owing to pain or bowel gas, the distal bile duct is usually not well seen. MRCP is needed to
exclude an intraductal calculus, particularly if US demonstrates CBD dilatation or if derangement
in liver function tests persists. CT is the technique of choice in acutely ill patients with suspected
gallbladder empyema or gallbladder perforation (Figure 7.15C).
Gallstones are extremely common, with a prevalence of approximately 10–15% in the adult
population. However, only 1–4% of individuals are symptomatic. Gallstones are more common in
middle-aged female Caucasians. Risk factors include obesity, pregnancy, hypercholesterolaemia,
family history, and diabetes. Patients may present with biliary colic (pain following eating) owing
to temporary obstruction of the cystic duct. Prolonged obstruction may result in inflammation of
the gallbladder wall (acute cholecystitis). Complications of acute cholecystitis include gallbladder
necrosis, gangrene, perforation, and abscess formation/empyema (Figure 7.15C).
Other complications of gallstones include:

• Chronic cholecystitis (gallbladder wall may calcify, ‘porcelain’ gallbladder).


• Increased risk of gallbladder cancer, particularly in ‘porcelain’ gallbladder.
• Gallstone ileus.
• Stone in CBD leading to cholangitis, obstructive jaundice, pancreatitis.

Fig. 7.15C  Axial postcontrast CT at the level of the gallbladder demonstrating gallbladder empyema.
The gallbladder (G) is distended with a thickened wall (A) and contains material of mixed density. The right
lobe of liver (L), incidental chronic left pelviureteric junction obstruction (P), and indwelling common bile duct
stent (B) are seen. Note the inflammatory changes in the mesenteric fat adjacent to the gallbladder (C).

267

K30031_Book.indb 267 9/6/17 1:37 PM


7  Abdomen and pelvis cases

3 How should the patient be managed?


The patient should be referred to the upper GI surgery team. The patient’s acute management
should include being placed nil by mouth, IV fluid resuscitation, IV antibiotic treatment, and
analgesia (remember opiates can worsen pain with spasm of the sphincter of Oddi).
After resolution of the acute episode, an elective cholecystectomy is performed some weeks
later. The reason for delay in surgery is to allow for inflammation of the gallbladder to resolve,
which is thought to result in reduced operative morbidity and reduced need for conversion from
a laparoscopic to an open approach. MRCP can also then be performed to exclude any further
­C BD stones. However, not all surgeons agree with this approach, some preferring ‘hot’ surgical
intervention during the acute episode.

LEARNING POINTS: ACUTE CHOLECYSTITIS


▪▪ Acute cholecystitis is a surgical emergency and the diagnosis should be considered in patients
with fever, vomiting, and RUQ pain. Remember the ABCDE concept and to call for senior advice.
▪▪ The initial diagnostic imaging test of choice is US, which may show gallstones and gallbladder
wall thickening. MRCP is used to demonstrate common duct stones when indicated.
▪▪ Complications of acute cholecystitis include gallbladder necrosis, perforation, and empyema.

268

K30031_Book.indb 268 9/6/17 1:37 PM


Case 7.16

Fig. 7.16A  Coronal T2-weighted image from MRI small bowel through the right iliac fossa region.

A 23-year-old male presents to gastroenterology outpatients with a 3-month history of intermit-


tent abdominal pain and diarrhoea. He also reports unintentional weight loss of 15 kg over the
preceding months but has no relevant past medical history. On direct questioning he has also
experienced joint pains and itching of both eyes. On examination he has a low-grade fever. Blood
tests reveal the following abnormalities:

WCC 13 × 109/L (4.0–11.0 × 109/L) CRP 120 mg/L (<5 mg/L)

A small bowel MRI is performed (Figure 7.16A).

CASE 7.16: QUESTIONS


1 What does the MRI small bowel show? 3 How should the patient be managed?
2 What is the likely diagnosis? 4 What are the complications of this disease?

269

K30031_Book.indb 269 9/6/17 1:37 PM


7  Abdomen and pelvis cases

CASE 7.16: ANSWERS


1 What does the MRI small bowel show?
There is a long segment of terminal ileal thickening and stricturing (Figure 7.16B) when com-
pared with the remaining visualised small bowel (where wall thickness does not exceed the
upper normal limit of 3 mm). There is also mild increased bowel wall signal within the thickened
segment, in keeping with oedema, indicating active inflammation. There is luminal narrowing,
which was shown to persist on all sequences, in keeping with a stricture. Post-IV contrast images
show increased enhancement of the thickened terminal ileum (Figure 7.16C), confirming the
diagnosis of terminal ileitis.

Fig. 7.16B  Coronal T2-weighted image from MRI small bowel. The MRI confirms diffuse stricturing of a
thick-walled terminal ileum (A). Note: caecum (B), ascending colon (C), right lobe liver (D), and normal ileal
loops (E).

270

K30031_Book.indb 270 9/6/17 1:37 PM


Case 7.16: Answers

Fig. 7.16C
T1 fat-saturated post-
gadolinium coronal image
showing thickening and
enhancement of the
terminal ileum (arrows A).

2 What is the likely diagnosis?


The most likely diagnosis is inflammatory bowel disease, most likely Crohn’s disease with this
distribution. The large bowel appeared unremarkable (images not shown although normal cae-
cum and ascending colon can be seen). However, it should be remembered that backwash ileitis
may occur in patients with ulcerative colitis. The differential diagnosis of terminal ileitis also
includes TB; however, the caecum is usually affected in this condition and there are usually intra-
abdominal necrotic lymph nodes. Atypical infection, for example amoebiasis and typhilitis owing
to chemotherapy, should not be included in the differential diagnosis in this particular patient as
there is no relevant history of travel/immunocompromise or treatment for cancer.

3 How should the patient be managed?


The patient should be managed by the gastroenterology team with discussion at the inflamma-
tory bowel disease MDT meeting. In addition:
• The diagnosis of Crohn’s disease can be confirmed with colonoscopy, terminal ileoscopy,
and terminal ileal biopsy.

271

K30031_Book.indb 271 9/6/17 1:37 PM


7  Abdomen and pelvis cases

• Treatment is with steroids and appropriate immunosuppressive agents (e.g. azathioprine)


or immunomodulatory therapies (e.g. infliximab), as directed by the gastroenterology team,
with appropriate monitoring.
• Surgery may be considered in the event of future complications, for example SBO caused by
fibrotic strictures or following abscess or fistula formation.

4 What are the complications of this disease?


Crohn’s disease is an inflammatory disorder of the GI tract that most often affects the terminal
ileum but may affect any part of the tract from mouth to anus. It has a multifactorial aetiology
where autoimmune and genetic factors are thought to play important roles. Patients typically
present in the second to third decades of life although there is a second peak of presentation in the
seventh decade. Typical presenting symptoms include abdominal pain and weight loss but non-
GI manifestations, such as arthralgia and episcleritis, may also be observed. Imaging may reveal
small bowel inflammation (typically affecting the terminal ileum) with characteristic features
including transmural inflammation, skip lesions, and strictures.
Complications of Crohn’s disease include SBO owing to strictures, fistulae (between small
bowel loops, small bowel and large bowel, and between the bowel and the skin), and abscess
formation. Perianal disease is also typical of Crohn’s disease. Patients are also at increased risk of
small bowel cancer and lymphoma.
MRI small bowel is the most sensitive imaging test for the diagnosis of small bowel Crohn’s
disease and is preferable in young patients as it does not involve ionising radiation. However, CT
is an excellent alternative in patients presenting with an acute abdomen (possible megacolon,
perforation, abscess). This is because CT is more readily available in most centres and is quick to
perform (therefore better tolerated by acutely unwell patients).
Early diagnosis and aggressive medical therapy has been shown to improve prognosis in
patients with Crohn’s disease.
Extraintestinal complications of Crohn’s disease include:

• Skin – erythema nodosum, pyoderma gangrenosum.


• Joints – sacroiliitis, ankylosing spondylitis.
• Eyes – uveitis, episcleritis.
• Liver – hepatitis, sclerosing cholangitis, cirrhosis, gallbladder carcinoma.

Note: this patient also had eye and joint symptoms.

LEARNING POINTS: CROHN’S DISEASE


▪▪ Crohn’s disease is the most common cause of terminal ileitis.
▪▪ Small bowel MRI is the most sensitive imaging test and may show bowel wall thickening and
increased enhancement, strictures, and complications (e.g. fistulae and abscesses). However,
terminal ileal biopsy is the reference standard diagnostic test.
▪▪ CT abdomen and pelvis should be considered in patients presenting with a surgical acute
abdomen to look for complications of megacolon, perforation, and abscess.
▪▪ Local complications of Crohn’s disease include strictures, abscesses, and fistulae.

272

K30031_Book.indb 272 9/6/17 1:37 PM


Case 7.17

Fig. 7.17A  Axial CT image upper abdomen post IV contrast.

A 65-year-old man presents to his GP with upper abdominal pain radiating to his back and
­malaise. On direct questioning he says he has lost one stone in weight over 6 weeks. He has no
other medical complaints but gives a 20 pack-year history of smoking and consumes approxi-
mately four bottles of wine per week. On examination he is haemodynamically stable but appears
cachectic and has pale sclerae. He is not jaundiced. His abdomen is nontender; however, he has a
palpable liver edge and is tender in the epigastrium.
CXR and AXR are unremarkable. CT of the abdomen and pelvis is undertaken (Figure 7.17A).
Blood investigations reveal the following abnormalities:

Hb 90 g/L (130–180 g/L) ALT 90 IU/L (<41 IU/L)


Bilirubin 50 micromol/L (<21 micromol/L) ALP 200 IU/L (40–129 IU/L)

CASE 7.17: QUESTIONS


1 What does the image show?
2 What is the diagnosis?
3 How should the patient be managed?

273

K30031_Book.indb 273 9/6/17 1:37 PM


7  Abdomen and pelvis cases

CASE 7.17: ANSWERS


1 What does the CXR show?
The CT image shows a large low-density mass in the pancreatic tail, multiple hypovascular liver
metastases, and a necrotic peripancreatic lymph node (Figure 7.17B). There is also LUQ ascites
and peritoneal stranding as well as nodularity anterior to the spleen.

Fig. 7.17B  Axial CT slice through the upper abdomen at the level of the pancreas. Pancreatic tail
mass (A), liver metastases (B), necrotic peripancreatic lymph node (C) and ascites (D). Infiltration of the
mesenteric fat is present (E). Note the stomach (F), right adrenal (G), spleen (H), and also normal pancreatic
body (I). The pancreatic tumour encases adjacent vessels.

2 What is the diagnosis?


The diagnosis is pancreatic cancer with hepatic, nodal, and peritoneal metastases. Pancreatic car-
cinoma is an adenocarcinoma, with high mortality owing to frequently inoperable disease at
presentation. Increasing age, smoking, obesity, and diabetes are risk factors as well as alcohol
excess and chronic pancreatitis.

• Note that the mass is in the pancreatic tail and, therefore, has not caused biliary
obstruction, hence the patient did not present with jaundice. More commonly the tumour
will occur in the pancreatic head and block the common bile duct causing biliary duct
dilatation and painless jaundice.

274

K30031_Book.indb 274 9/6/17 1:37 PM


Case 7.17: Answers

3 How should the patient be managed?


The patient should be admitted urgently to hospital for management of his pain, for diagnosis,
and for discussion of treatment options:

• ABCDE approach.
• CT chest to complete staging.

The patient should also be referred urgently to the hepatobiliary MDT for surgical and onco-
logical opinion. The diagnosis of metastatic pancreatic carcinoma was diagnosed on percutane-
ous US-guided liver biopsy. In this case surgery was not an option owing to the multiple hepatic
metastases and peritoneal disease. The patient was referred for palliative chemotherapy. His
prognosis is extremely poor.

LEARNING POINTS: PANCREATIC CANCER


▪▪ Pancreatic cancer is a common cause of painless jaundice caused by biliary obstruction.
▪▪ Risk factors include smoking and alcohol excess.
▪▪ CT may demonstrate the primary tumour and metastatic disease, including liver, nodal, lung, and
peritoneal metastases, as well as relationship of the primary tumour to the mesenteric vessels.
▪▪ Patients often present with irresectable disease.
▪▪ Factors affecting suitability for surgery include relationship of the tumour with the mesenteric
vessels, presence of metastatic disease, and patient fitness for surgery.

275

K30031_Book.indb 275 9/6/17 1:37 PM


Case 7.18

Fig. 7.18A  Transabdominal transverse US of the uterus (delineated with calipers).

A 44-year-old female presents to her GP with a 6-month history of menorrhagia and dysmenor-
rhea. On examination a firm suprapubic mass is palpable. Blood tests reveal a slightly low Hb
at 118 (125–165 g/L) but blood parameters are otherwise normal.
The GP suspects a diagnosis of uterine fibroids and requests a pelvic US (Figures 7.18A
and 7.18B).

276

K30031_Book.indb 276 9/6/17 1:37 PM


Case 7.18: Questions

Fig. 7.18B  Transvaginal US of the uterus in longitudinal section (delineated with calipers).

CASE 7.18: QUESTIONS


1 What are fibroids?
2 What does the US show?
3 What other imaging is available for suspected fibroids?
4 What are the potential complications of fibroids?
5 How are fibroids managed?

277

K30031_Book.indb 277 9/6/17 1:37 PM


7  Abdomen and pelvis cases

CASE 7.18: ANSWERS


1 What are fibroids?
Fibroids (also known as leiomyomata) are benign tumours of the myometrium. They are more
common in Afro-Caribbean women and in women approaching the menopause. They may com-
press the endometrial cavity (submucosal fibroids) and cause symptoms of menorrhagia and dys-
menorrhea, or arise from the surface of the uterus (serosal fibroids) and be asymptomatic. Fibroids
may also be ‘pedunculated’, arising from the uterine surface as a pedunculated mass.
Fibroids can vary in size from a few millimeters to a massive tumour occupying the abdomen.
Their growth is oestrogen-dependent and they therefore tend to increase in size during preg-
nancy and regress after the menopause.
Patients with fibroids may be asymptomatic or present with:

• Abnormal vaginal bleeding.


• Pain.
• Infertility.
• Palpable pelvic mass.
• Pressure effects.

Fig. 7.18C
Transabdominal
transverse US
image of the uterus.
Fundal low-density
(hypoechoic) fibroid
noted (A). The
endometrial cavity
is not visualised.
Note: the cervix (B),
vagina (C), and empty
collapsed bladder (D).

278

K30031_Book.indb 278 9/6/17 1:37 PM


Case 7.18: Answers

2 What does the US show?


The US demonstrates a fibroid at the fundus of the uterus compressing the superior aspect of the
endometrial cavity (Figures 7.18C and 7.18D).

Fig. 7.18D  Transvaginal US image of the uterus in longitudinal section. Note: the fundal fibroid (A)
compressing and distorting the endometrial cavity. Uterine cervix (B), vagina (C).

3 What other imaging is available for suspected fibroids?


US is the workhorse for the diagnosis of fibroids. It is essential to acquire transabdominal and
transvaginal views to ensure full visualisation and assessment of the extent of the fibroids (the
transvaginal US probe only has a sound penetration depth of around 4 cm, which will underes-
timate large fibroids).
Where there is doubt about the diagnosis or if further treatment planning is required, an MRI
scan will provide further information (Figure 7.18E).
CT will demonstrate fibroids as a soft tissue density mass in the pelvis but does not provide
good anatomical detail and it can be difficult to differentiate uterine fibroids from adnexal lesions.
There is also a significant associated radiation dose.

4 What are the potential complications of fibroids?


Complications of fibroids include:

• Menorrhagia and subsequent anaemia.


• Torsion of a pedunculated fibroid.
• Degeneration (can be associated with pain).
• Malignancy (leiomyosarcoma). This is rare, but if the fibroid is growing rapidly, in
particular after menopause, then malignancy should be considered.

279

K30031_Book.indb 279 9/6/17 1:37 PM


7  Abdomen and pelvis cases

Fig. 7.18E  Sagittal T2-weighted MRI image of the female pelvis. A large fundal fibroid is noted (A), which
is displacing the endometrium posteriorly (B). Note the cervix (C), vagina (D), bladder (E), and rectum (F).

5 How are fibroids managed?


Treatment options for fibroids depend on their size and location, the extent of symptoms, the age
of the patient, and patient wishes. They include:

• Myomectomy.
• Hormone administration.
• Hysterectomy (in a patient not wishing to preserve her fertility).
• Uterine artery embolisation (radiological procedure involving uterine artery catheterisation
via the femoral artery and embolisation of feeding vessels).

LEARNING POINTS: UTERINE FIBROIDS


▪▪ Uterine fibroids are commonly asymptomatic but can cause menorrhagia and dysmenorrhea if
submucosal.
▪▪ Transabdominal and transvaginal US are required for complete initial assessment.
▪▪ MRI may be used as an adjunct in complex cases and for pretreatment planning.
▪▪ Treatment options include myomectomy, hysterectomy, hormone replacement, and uterine artery
embolisation.

280

K30031_Book.indb 280 9/6/17 1:37 PM


Case 7.19

Fig. 7.19A  Axial CT abdomen at the epigastric level post IV contrast.

A 35-year-old male presents to the ED with 2 days of severe central abdominal pain and vomiting.
The pain radiates to the back and is mildly relieved by sitting forwards. He has no relevant past
medical history, takes no regular medications and is a nonsmoker. He drinks 50 units of alcohol
per week. On examination he appears unwell and is haemodynamically unstable with the follow-
ing observations: HR 130 bpm, respiratory rate 24 bpm, BP 100/80 mmHg, and temperature 38°C.
He has central abdominal guarding to palpation. Blood tests reveal the following abnormalities:

WCC 20 × 109/L (4–11 × 109/L) Urea 14 mmol/L (1.7–8.3 mmol/L)


CRP 200 mg/L (<5 mg/L) Creatinine 150 micromol/L (62–106 micromol/L)

ABG reveals a metabolic acidosis. CT of the abdomen is arranged (Figure 7.19A).

CASE 7.19: QUESTIONS


1 What does the imaging show and what 3 What are the underlying causes of this
is the diagnosis? What other blood tests condition?
might help? 4 What are the complications of this
2 How should the patient be managed? condition?

281

K30031_Book.indb 281 9/6/17 1:37 PM


7  Abdomen and pelvis cases

CASE 7.19: ANSWERS


1 What does the imaging show and what is the diagnosis? What other blood tests
might help?
There is fluid and soft tissue density inflammatory fat stranding surrounding the pancreas, with
loss of the normal dark fat density (Figures 7.19B and 7.19C), in keeping with acute pancreatitis.
Additional findings include:
• The pancreas enhances normally with no low-density gas areas to suggest necrosis, no
focal abscess.
• The splenic vein enhances normally and is patent.
• The gallbladder is thin walled and contains no gallstones (on this image).
• The CBD is not dilated.
Serum amylase and serum lipase would be helpful. Both are raised in acute pancreatitis, and
a mild to moderate increase in amylase can also be seen in other abdominal inflammation dis-
orders, and when pancreatitis is subacute. Lipase is more specific to alcohol-related pancreatitis.

Fig. 7.19B  Axial CT abdomen post IV contrast at the level of the pancreas. The pancreas (A) appears
swollen and there is diffuse soft tissue attenuation around the pancreas (B) with fluid also present (C). The
splenic vein appears patent (D) and the gallbladder appears distended but thin walled (E). Note: spleen (F),
distal nondilated common bile duct (G), and patent superior mesenteric artery (H).

282

K30031_Book.indb 282 9/6/17 1:37 PM


Case 7.19: Answers

Fig. 7.19C  More caudal axial CT abdomen in the same patient. There is extensive fluid surrounding
the pancreas (A). There is also extensive inflammatory change in the peripancreatic fat (B). Note: swollen
pancreatic tissue, head and tail (C), duodenum second part (D), superior mesenteric artery (E), and vein (F).

2 How should the patient be managed?


The patient should be kept nil-by-mouth and managed supportively on the high dependency
unit with IV fluid resuscitation, prophylactic antibiotics to be considered, and adequate analgesia.
Additionally:

• A NG tube should be sited in order to prevent vomitus and abdominal distension, and
aspiration pneumonia (Figure 7.19D).
• Fluid balance and blood gases should be monitored (the patient may need a urinary
catheter and central venous access).
• The underlying cause of pancreatitis should be investigated and complications managed as
appropriate.
• Abdominal US has a higher sensitivity than CT for detection of gallstones in the
gallbladder and should be considered if the underlying cause is unknown, Consider MRCP
for suspected CBD stones when patient condition allows.

283

K30031_Book.indb 283 9/6/17 1:37 PM


7  Abdomen and pelvis cases

3 What are the underlying causes of this condition?


The commonest causes of acute pancreatitis are CBD gallstones and alcohol (alcohol in this case).
Less common causes include:

• Iatrogenic (e.g. post ERCP).


• Medications (e.g. steroids, azathioprine).
• Autoimmune.
• Hyperlipidaemia.
• Viral (e.g. mumps, Coxsackie B).
• Miscellaneous (e.g. trauma).
• Idiopathic.

Fig. 7.19D  Axial CT abdomen in the same patient 1 week later. A NG tube has been inserted (A). There
is now a peripancreatic fluid collection (B). These are common in the early stages of acute pancreatitis but
may progress into a pseudocyst. Pseudocysts have a rim of granulation tissue and form approximately
6 weeks after the acute episode. These can be drained via a percutaneous, endoscopic or surgical
approach.

284

K30031_Book.indb 284 9/6/17 1:37 PM


Case 7.19: Answers

4 What are the complications of this condition?


Complications of acute pancreatitis include:

• Pancreatic infection, abscess, and necrosis.


• Peripancreatic fluid collection and late stage pseudocyst formation (Figure 7.19D).
• Splenic vein thrombosis.
• Splenic artery pseudoaneurysm.
• Acute respiratory distress syndrome.
• Cullen’s sign (periumbilical bruising) and Grey Turner’s sign (flank bruising) are both
clinical signs indicative of severe pancreatitis and suggest a poor prognosis.

LEARNING POINTS: ACUTE PANCREATITIS


▪▪ Epigastric pain radiating to the back and relieved on sitting forwards is the typical presentation of
acute pancreatitis.
▪▪ The commonest causes are gallstones and alcohol excess.
▪▪ CT may show pancreatic oedema, peripancreatic inflammation, and complications, e.g.
pancreatic necrosis, collection, and pseudocyst.
▪▪ Patients should be treated supportively with fluid resuscitation, analgesia, and prophylactic
antibiotics and may benefit from nasojejunal/NG tube.
▪▪ Patients may be septic and should be closely monitored in a high dependency environment, and
close liaison with specialist regional unit is recommended.

285

K30031_Book.indb 285 9/6/17 1:37 PM


Case 7.20

Fig. 7.20A  Axial postcontrast CT image of the upper abdomen.

A 63-year-old female presents to her GP with a 6-month history of abdominal distension, nausea,
and bloating. Clinical examination and routine blood tests are normal but her serum CA125 is
raised, at 1,988 units/mL (0–35 units/mL).
The GP refers the patient to the gynaecology oncology department who arrange a CT scan of
the chest, abdomen, and pelvis (Figures 7.20A, 7.20B, and 7.20C). She is subsequently discussed
at the MDT meeting.

CASE 7.20: QUESTIONS


1 What abnormalities do these CT images show?
2 What is the differential diagnosis?
3 How is this disease diagnosed and staged?
4 What complications is this patient at risk of?

286

K30031_Book.indb 286 9/6/17 1:37 PM


Case 7.20: Questions

Fig. 7.20B  Axial postcontrast CT image of the upper pelvis.

Fig. 7.20C  Axial postcontrast CT image of the mid pelvis.

287

K30031_Book.indb 287 9/6/17 1:37 PM


7  Abdomen and pelvis cases

CASE 7.20 ANSWERS


1 What abnormalities do these CT images show?
The CT demonstrates ascites and peritoneal metastases. There is stranding and thickening of the
anterior omentum (‘omental cake’), with pelvic peritoneal nodular thickening and enhancement,
and a pelvic mass (Figures 7.20D, 7.20E, and 7.20F).

Fig. 7.20D  Axial CT image of the upper abdomen. Note: the perihepatic and perisplenic free fluid (ascites)
(A), and the ill-defined soft tissue and nodular infiltration in the LUQ (B). This is peritoneal metastatic
disease.

2 What is the differential diagnosis?


Metastatic primary ovarian cancer is the most likely diagnosis. The differential diagnoses are:

• Metastases from breast, stomach or colon carcinoma; lobular breast cancer in particular can
cause this pattern of disease. Ovarian metastases are also known as Krukenberg tumours
and comprise 5–10% of all ovarian tumours; they are signet-ring adenocarcinomas.
• Peritoneal mesothelioma.
• Peritoneal tuberculosis.

288

K30031_Book.indb 288 9/6/17 1:37 PM


Case 7.20 Answers

Fig. 7.20E  Axial CT image of the upper pelvis shows thick omental disease (‘omental cake’ – A).
In addition there is a trace of free fluid in the right paracolic gutter (B) and a subcutaneous tumour deposit
can be seen near the umbilicus (C).

Fig. 7.20F  Axial CT image of the mid pelvis shows nodular thickening of the peritoneum (A), a left adnexal
mass, and likely ovarian primary malignancy (B) and ascites (C). Normal uterus (D).

289

K30031_Book.indb 289 9/6/17 1:37 PM


7  Abdomen and pelvis cases

Malignant spread in ovarian cancer can occur via:

• Intraperitoneal seeding and direct invasion (most commonly).


• Haematogenous dissemination.
• Lymphatic dissemination (rare).

As a result, if a patient presents with an indeterminate cystic ovarian lesion, biopsy is not rec-
ommended because if malignant it can rupture and leak, causing peritoneal seeding and upstag-
ing the tumour.

3 How is this disease diagnosed and staged?


The diagnosis may be acquired as a result of surgical excision of a complex ovarian mass. If dis-
ease is more advanced, ascitic tap and cytology or percutaneous biopsy of omental tumour may be
used. Currently, depending on the extent of disease at presentation, patients will either undergo
early surgical debulking followed by chemotherapy, or primary chemotherapy (for more advanced
disease).
If the patient undergoes surgical debulking then biopsy for histological diagnosis and staging
is often performed at the time of surgery.
If the patient undergoes primary chemotherapy then histology is acquired via a radiological-
guided biopsy procedure (such as US-guided omental biopsy) and the CT scan is relied on for
radiological staging.

4 What complications is this patient at risk of?


Complications of peritoneal disease may require treatment for palliation and include:

• Bowel obstruction as a result of serosal surface deposits. Serosal infiltration of the liver and
splenic capsules may also be observed.
• Malignant ascites (requiring repeated drainage). This is best done under US guidance as
thick omental disease should be avoided during the drain insertion.

LEARNING POINTS: OVARIAN CARCINOMA


▪▪ Ovarian carcinoma is often clinically silent and asymptomatic, and patients, therefore, typically
present late with more advanced disease.
▪▪ In the UK there is currently no screening programme for ovarian cancer. High-risk patients may
undergo annual CA125 estimation and pelvic US.
▪▪ Imaging features of primary ovarian malignancy include: rapid growth of an observed ovarian
lesion, abdominal ascites, and an ovarian mass with complex (part solid, part cystic) appearance
with increased vascularity on US/CT of the solid component.

290

K30031_Book.indb 290 9/6/17 1:37 PM


Case 7.21

Fig. 7.21A  Maximum intensity projection (MIP) coronal T2-weighted image from MRCP of the upper
abdomen.

A 42-year-old Caucasian female presents acutely to the ED with a 2-day history of fevers and RUQ
pain. Her partner notes that she has developed a yellowish discolouration of the skin. She has a
past surgical history of laparoscopic cholecystectomy. Clinical examination reveals a temperature
of 39°C, HR 120 bpm, BP 100/80 mmHg, respiratory rate 24 bpm, and oxygen saturation 98% on
room air. There is a yellowish discolouration of her sclera and skin. She has severe RUQ pain on
superficial and deep palpation that radiates to the right shoulder tip. Her salient abnormal blood
results are:
WCC 15 × 109/L (4.0–11.0 × 109/L) Bilirubin 150 micromol/L (<21 micromol/L)
CRP 180 mg/L (<5 mg/L) ALP 300 IU/L (35–104 IU/L)

Initial US obtains poor quality views owing to bowel gas but suggests intrahepatic biliary tree
dilatation. MRCP is performed (Figure 7.21A).

CASE 7.21: QUESTIONS


1 What does the MRCP show? 3 How should the patient be managed?
2 What is the diagnosis?

291

K30031_Book.indb 291 9/6/17 1:37 PM


7  Abdomen and pelvis cases

CASE 7.21: ANSWERS


1 What does the MRCP show?
The intra- and extrahepatic biliary tree is dilated owing to two large obstructing calculi within the
distal common bile duct (CBD) (Figure 7.21B).

Fig. 7.21B  MIP coronal image from an MRCP. These images are highly fluid sensitive so that fluid
appears bright/high signal. Obstructing calculi within the dilated CBD are seen as low signal filling
defects (A). The cystic duct (B) and CBD (C) are also dilated. Note cholecystectomy. The intrahepatic
ducts are dilated (D), normal duodenum (E), stomach (F), cerebrospinal fluid in the spinal canal (G) and
normal pancreatic duct (H).

2 What is the diagnosis?


Choledocholithiasis with secondary ascending cholangitis, i.e. stones within the biliary tree,
resulting in biliary obstruction and subsequent infection.

292

K30031_Book.indb 292 9/6/17 1:37 PM


Case 7.21: Answers

3 How should the patient be managed?


Ascending cholangitis is a surgical emergency:

• The patient is septic and should be treated with IV fluids and antibiotics, analgesia, and
careful monitoring of fluid balance. ABCDE approach, monitoring, routine investigations to
include blood cultures, and senior discussion.
• Urgent ERCP should be performed, the CBD stones retrieved, and the obstruction relieved.
A CBD stent may need to be sited if there is a stricture.

LEARNING POINTS: CHOLEDOCHOLITHIASIS


▪▪ Choledocholithiasis is the commonest cause of painful obstructive jaundice.
▪▪ US abdomen is usually performed in the first instance to look for biliary dilatation; however,
MRCP is the most sensitive imaging test and can add detail about the number of stones, level of
obstruction, and alternative pathologies (e.g. biliary strictures).
▪▪ ERCP is the definitive treatment.
▪▪ If the patient is septic this should be managed as an emergency.

293

K30031_Book.indb 293 9/6/17 1:37 PM


Case 7.22

Fig. 7.22A  Supine AXR.

294

K30031_Book.indb 294 9/6/17 1:37 PM


Case 7.22: Questions

A 60-year-old male presents to the ED with a 5-day history of colicky abdominal pain and vom-
iting. He has been unable to eat or drink for the past 24 hours. He has no relevant past medical
history but had a previous laparotomy many years ago for appendicitis with localised perfora-
tion. Clinical examination reveals cool peripheries, a prolonged capillary refill time of 4 seconds,
tachycardia (HR 120 bpm), and mild hypotension (BP 110/80 mmHg). He is apyrexial. Abdominal
examination reveals marked distension, generalised discomfort to palpation, and tinkling bowel
sounds on auscultation. Routine bloods reveal a mild leukocytosis and elevated CRP. Supine AXR
is performed (Figure 7.22A). Erect CXR is unremarkable.

CASE 7.22: QUESTIONS


1 What does the AXR show and what is the diagnosis?
2 What further imaging could be performed?
3 How should the patient be managed?

295

K30031_Book.indb 295 9/6/17 1:37 PM


7  Abdomen and pelvis cases

CASE 7.22: ANSWERS


1 What does the AXR show and what is the diagnosis?
There is a very dilated colonic loop, which arises from the right-sided abdomen (Figure 7.22B).
The dilated colonic loop displays haustral markings. The large bowel is otherwise nondi-
lated. There is no free intra-abdominal gas. This gas-filled bowel loop is massively dilated,
12–15 cm at least, and also contains haustra, which partially traverse the bowel lumen. This
lesion is large bowel at this size and in this position, and the configuration is consistent with
a dilated caecum, secondary to caecal volvulus – the ‘kidney bean’ sign.

Fig. 7.22B  Annotated AXR. Note: massive distension of the caecum (loop outlined: A) and presence of
haustrations (B), which do not fully traverse the bowel lumen. A loop this large cannot be small bowel, and
is most likely caecum in this position and with this configuration.

296

K30031_Book.indb 296 9/6/17 1:37 PM


Case 7.22: Answers

2 What further imaging could be performed?


CT abdomen and pelvis could be performed if there is diagnostic doubt or concern regarding
complications of caecal volvulus, e.g. bowel ischaemia or perforation (Figure 7.22C).

• CT further demonstrates a dilated bowel loop within the right-sided abdomen in the
approximate position of the caecum.
• CT may show a change in large bowel calibre and twist of the mesentery at the caecal level.

Fig. 7.22C  Coronal section CT abdomen and pelvis in the same patient showing a grossly distended
bowel loop within the right abdomen in the approximate position of the caecum. Note the massive
distension and haustration (arrows).

3 How should the patient be managed?


The patient should be managed supportively with nil by mouth, IV fluids, and NG tube (‘drip and
suck’). Also:

• Erect CXR to look for perforation (CT is more sensitive).


• General/colorectal surgery referral urgently.

297

K30031_Book.indb 297 9/6/17 1:37 PM


7  Abdomen and pelvis cases

• Surgical options include laparotomy with (usually) right hemicolectomy; in some patients
caecopexy (the caecum is fixed to the abdominal wall) or caecostomy (stoma between
caecum and abdominal wall) may be considered.
• If the patient is unfit for surgery, colonoscopic decompression may be attempted.

The term volvulus means twisting of the bowel/mesentery on its vascular pedicle.

• Caecal volvulus tends to occur in younger patients than sigmoid volvulus and accounts for
10–20% of cases of large bowel volvulus.
• Patients either have a congenital defect in peritoneal fixation or have an acquired
predisposition, e.g. as a result of previous abdominal surgery, or a pelvic mass.
• The caecum may rotate in the transverse plane (dilated loop appears in RLQ) or may twist
and invert (dilated loop in LUQ).
• Complications include bowel infarction and perforation.

LEARNING POINTS: THE ILEOCAECAL VALVE


▪▪ If competent, this will prevent caecal decompression and these patients will present acutely with
closed-loop caecal obstruction and secondary small bowel dilatation: high risk of perforation.
▪▪ If valve is incompetent (in elderly), this will allow intermittent caecal decompression and a more
subacute presentation.

LEARNING POINTS: CAECAL VOLVULUS


▪▪ Caecal volvulus is an uncommon cause of LBO.
▪▪ AXR shows a dilated large bowel loop within the right lower abdominal quadrant or sometimes in
the LUQ.
▪▪ Erect CXR should be performed to look for perforation.
▪▪ Patients are usually managed surgically.
▪▪ Complications include bowel ischaemia and perforation.

298

K30031_Book.indb 298 9/6/17 1:37 PM


Case 7.23

Fig. 7.23A  Supine AXR.

A 72-year-old female presents to the ED with a 10-day history of worsening generalised colicky
abdominal pain and vomiting. She has not opened her bowels for 4 days and is now unable to
pass flatus. She is afebrile, mildly tachycardic (HR 110 bpm), and mildly tachypnoeic (respiratory
rate 24 bpm). Her BP is 160/90 mmHg. Clinical examination reveals a grossly distended abdomen,
generalised tenderness to abdominal palpation, and tinkling bowel sounds. Blood tests reveal a
mild leukocytosis and mildly elevated CRP. Supine AXR is performed (Figure 7.23A).

CASE 7.23: QUESTIONS


1 What does the AXR show and what is the diagnosis?
2 What further imaging would you recommend?
3 How should the patient be managed?

299

K30031_Book.indb 299 9/6/17 1:37 PM


7  Abdomen and pelvis cases

CASE 7.23: ANSWERS


1 What does the AXR show and what is the diagnosis?
There is a grossly dilated loop of large bowel arising from the pelvis with an inverted U shape
(Figure 7.23B).

Fig. 7.23B  Supine AXR with a grossly dilated loop of sigmoid colon. The afferent (dotted white line) and
efferent (solid black line) loops are outlined, converging into the pelvis, with summation overlap line (solid
white line, A) evident. The dilated left proximal colon is shown (B). Haustrations can be seen in one of the
sigmoid loops. Note: in places, these appear to almost traverse the bowel lumen (C) and are not to be
confused with valvulae conniventes. (The two round densities in the lower right part of the XR are buttons
on the patient’s gown.)

300

K30031_Book.indb 300 9/6/17 1:37 PM


Case 7.23: Answers

The dilated loop:

• Extends above the level of T10 vertebra. Note: the upper abdomen is not included in the
field of view and a repeat film to show the upper abdomen/diaphragm and an erect CXR
(perforation) would also be appropriate.
• Has a ‘coffee bean’ appearance.
• Has visible haustral markings.
• The dilated sigmoid loops converge into the pelvis (convergence sign) and also overlap
(summation line).

The more proximal large bowel is also dilated. These are all features of sigmoid volvulus.
The term volvulus means twisting of the bowel and mesentery on its vascular pedicle.

2 What further imaging would you recommend?


CT abdomen and pelvis with contrast is performed when there is diagnostic doubt or concern
regarding complications of sigmoid volvulus, e.g. bowel ischaemia or perforation:

• CT better demonstrates the degree and level of bowel obstruction and may demonstrate
pneumoperitoneum (intraperitoneal free gas).
• At the transition point (where the bowel changes in calibre) the mesenteric vessels and fat
have a whorled appearance, the site of volvulus (Figure 7.23C).

Fig. 7.23C  Axial CT pelvis showing a grossly dilated loop of sigmoid colon (A) and focal bowel wall
thickening with fat/vessel distortion of the mesentery at the transition point (B) at the site of bowel twist.
There is no evidence of perforation on this image.

301

K30031_Book.indb 301 9/6/17 1:37 PM


7  Abdomen and pelvis cases

3 How should the patient be managed?


The patient should be managed supportively with nil by mouth, IV fluids, and NG tube
(‘drip and suck’):

• Erect CXR should be performed to look for perforation.


• General/colorectal surgery referral urgently.
• Insertion of flatus tube to decompress the large bowel, curative in the short term in 90% of
patients, indicated in the absence of perforation/peritonism.
• If flatus tube fails then surgical management, with fixation of the bowel or sigmoid
colectomy, may be required.

Sigmoid volvulus:

• Sigmoid volvulus is more common in elderly patients and is caused by redundant


mesenteric colonic attachment allowing the bowel to mobilise and twist. It is often
recurrent. There is an association with large bowel pathology, sigmoid carcinoma in
particular.
• The commonest sites of large bowel volvulus are at the sigmoid colon (80% cases of
volvulus) and caecum (20%).
• Complications include bowel infarction and perforation.

LEARNING POINTS: SIGMOID VOLVULUS


▪▪ Sigmoid volvulus is the commonest form of large bowel volvulus.
▪▪ AXR shows a grossly dilated large bowel loop arising from the pelvis with inverted U shape.
▪▪ The dilated loop has a coffee bean appearance.
▪▪ Erect CXR should be performed to look for perforation.
▪▪ Patients are managed supportively and with flatus tube, unless there is evidence of necrosis or
perforation (when surgery is indicated).
▪▪ Complications include bowel ischaemia and perforation.

302

K30031_Book.indb 302 9/6/17 1:37 PM


Case 7.24

Fig. 7.24A  Axial and sagittal CT images postcontrast through the right iliac fossa region.

A 32-year-old male presents to the ED with a 48-hour history of abdominal pain that has localised
to the right iliac fossa (RIF). On examination, there is tenderness and guarding in the RIF, tachy-
cardia at 120 bpm, BP is normal, and he is pyrexial at 38.2°C.
A CT scan of the abdomen and pelvis is arranged (Figure 7.24A). This comprises a selection of
axial and also sagittal CT images through the right iliac fossa.

CASE 7.24: QUESTIONS


1 How would you describe the CT appearances?
2 What is the diagnosis and differential diagnosis?
3 What other imaging modalities may be helpful?
4 How is this condition usually treated?
5 What are the potential complications of this condition?

303

K30031_Book.indb 303 9/6/17 1:37 PM


7  Abdomen and pelvis cases

CASE 7.24: ANSWERS


1 How would you describe the CT appearances?
The CT demonstrates a dilated appendix with a distended lumen and a thickened wall. There is
periappendiceal inflammation (with stranding of the surrounding fat). An appendicolith (focus of
calcification) is noted at the neck of the appendix and there is a locule of free gas in keeping with
localised perforation (Figure 7.24B).

Fig. 7.24B  A selection of CT images demonstrating a dilated and thick-walled appendix (A). There is a
calcified appendicolith (B) seen at the appendix neck. Note: inflammatory changes/fluid in the surrounding
periappendicular fat (C) and bubbles of air secondary to infection (D), caecum (E) and ascending colon (F),
free fluid around the liver (G), gallbladder (H), and liver (L).

2 What is the diagnosis and differential diagnosis?


This patient has acute appendicitis. Conditions that may mimic acute appendicitis include:

• Nonspecific mesenteric lymphadenitis.


• Terminal ileitis (due to Crohn’s disease or Yersinia infection).
• Inflamed Meckel’s diverticulum.

304

K30031_Book.indb 304 9/6/17 1:37 PM


Case 7.24: Answers

In female patients consider:

• Acute salpingitis.
• Ovarian cyst accident.
• Ovarian torsion.

3 What other imaging modalities may be helpful?


AXR is rarely helpful and therefore not routinely performed. Occasionally, however, a calcified
appendicolith can be detected in the RIF, if not obscured by overlying bowel gas. This is therefore
a review area to check in the acute abdomen (Figure 7.24C).

Fig. 7.24C  Supine AXR in another patient with acute appendicitis. A calcified appendicolith is just visible
(arrow) in the right iliac fossa. This finding is subtle and will be readily obscured by overlying bowel gas.

305

K30031_Book.indb 305 9/6/17 1:37 PM


7  Abdomen and pelvis cases

US is a useful diagnostic test in some cases, as it is effective at identifying appendiceal inflam-


mation, particularly in thin patients. US should be considered the initial diagnostic choice for
children and young women as it is nonionizing and is accurate in making the diagnosis of acute
appendicitis if the appendix can be visualised. In patients who are very tender, where there is
bowel gas or in obese/larger patients, US is less accurate and CT is usually needed. Appendicitis
also occurs in elderly patients but many present in a more nonspecific way and CT is usually
needed for more general assessment in older adults and the elderly.

4 How is this condition usually treated?


Treatment is with urgent open or laparoscopic appendicectomy.

5 What are the potential complications of this condition?


The potential complications of acute appendicitis include:

• Gangrene and perforation.


• Localised abscess formation.
• Generalised peritonitis.
• Treatment is usually surgical.

LEARNING POINTS: ACUTE APPENDICITIS


▪▪ Appendicitis classically presents with periumbilical pain that localises to the RIF with associated
fever, nausea, and vomiting. A classical presentation is, however, less common in the elderly.
▪▪ US is the investigation of choice in younger patients although CT has a greater sensitivity and
specificity, and is increasingly used first line, especially in older patients, to confirm the diagnosis
and exclude mimics.
▪▪ Complications include perforation and abscess.
▪▪ Treatment is usually surgical.

306

K30031_Book.indb 306 9/6/17 1:37 PM


Case 7.25

Fig. 7.25A  Axial postcontrast CT image of the abdomen at the level of the umbilicus.

A 52-year-old male presents to his GP with nonspecific symptoms of fatigue and malaise. He
reports a history of recent unintentional weight loss and night sweats. On examination the GP
notes enlarged painless cervical, axillary, and inguinal lymphadenopathy. Observations are
normal.
The GP requests routine bloods and finds that the patient is pancytopenic. He refers to haema-
tology for further investigation. A CT scan is performed (Figure 7.25A).

CASE 7.25: QUESTIONS


1 What does the CT show?
2 What are the causes of generalised lymphadenopathy?
3 How could the diagnosis be made?
4 What are the other causes for the CT abnormality?

307

K30031_Book.indb 307 9/6/17 1:37 PM


7  Abdomen and pelvis cases

CASE 7.25: ANSWERS


1 What does the CT show?
The CT demonstrates a soft tissue retroperitoneal mass that is encasing the aorta. These
appearances are typical for lymphadenopathy as a result of non-Hodgkin lymphoma (NHL)
(Figure 7.25B).

Fig. 7.25B  Axial CT image demonstrating the calcified aorta (A) and inferior vena cava (B) surrounded by
a soft tissue density nodal mass (C). Nodal disease in lymphoma typically encases but does not invade the
vessels. Note the liver (D), spleen tip (not enlarged, E), and a pathological mesenteric node (F).

2 What are the causes of generalised lymphadenopathy?


Causes of generalised lymphadenopathy include:

• Lymphoma.
• Leukaemia (chronic lymphatic leukaemia, acute lymphoblastic leukaemia).
• Glandular fever.
• Acquired immune deficiency syndrome (AIDS).
• Chronic infection (such as TB).
• Connective tissue disorders (systemic lupus erythematosus, sarcoid).

3 How could the diagnosis be made?


As the patient also has palpable cervical, axillary, and inguinal lymphadenopathy, an US-guided
core biopsy of an enlarged peripheral node is likely to give the diagnosis. Bone marrow aspiration
and trephine biopsy will confirm bone marrow involvement if required. CT-guided para-aortic
node biopsy would be feasible but is more invasive.

308

K30031_Book.indb 308 9/6/17 1:37 PM


Case 7.25: Answers

4 What are the other causes for the CT abnormality?


Causes of a retroperitoneal mass are listed in Table 7.25.

Table 7.25  Causes of a retroperitoneal mass

Nodal disease Primary retroperitoneal tumour

Neoplastic
Testicular tumour Liposarcoma
Renal cell carcinoma Leiomyosarcoma
Lymphoma Rhabdomyosarcoma
Post-transplant lymphoproliferative disease

Non-neoplastic
Retroperitoneal fibrosis
Extramedullary haematopoiesis
Lipoma
Peripheral nerve sheath tumour (neurofibroma)
Psoas abscess
Haematoma (aneurysm leak, warfarin complication, trauma)

NHL also typically presents with both mediastinal and abdominal para-aortic lymphadenopathy
(see Figures 7.25C and 7.25D).

LEARNING POINTS: RETROPERITONEAL MASS


▪▪ A thorough clinical examination is essential in a patient presenting with lymphadenopathy and will
expedite the diagnosis.
▪▪ Encasement of the aorta on CT is typical for lymphadenopathy as a result of lymphoma.
▪▪ In a male patient with retroperitoneal nodes, ensure the testicles are examined clinically. US of the
testes and testicular tumour markers may also be helpful.

309

K30031_Book.indb 309 9/6/17 1:37 PM


7  Abdomen and pelvis cases

Fig. 7.25C  Axial postcontrast CT image in another patient with NHL at the level of the aortic arch (A).
Note the widespread enlarged mediastinal lymph nodes (arrows).

Fig. 7.25D  Axial postcontrast CT image in the same patient as in Figure 7.25C at the level of the kidneys.
This demonstrates massive para-aortic (A) and mesenteric (B) lymphadenopathy.

310

K30031_Book.indb 310 9/6/17 1:37 PM


Case 7.26

Fig. 7.26A  Axial postcontrast CT image of the upper abdomen.

A 25-year-old male is brought to the ED by ambulance following a RTA in which he was knocked
off his bicycle. He has a GCS of 13; however, witnesses report loss of consciousness at the scene.
He complains of severe left-sided chest and abdominal pain. He is visibly short of breath. On
examination his HR is 110 bpm, respiratory rate 32 bpm, and BP 100/80 mmHg. Respiration is
asymmetrical, there is hyper-resonance to percussion over the left chest wall, and left-sided
breath sounds are reduced. There is LUQ bruising and guarding to palpation, and bowel sounds
are quiet. He has no spinal tenderness but his cervical spine is immobilised.
A portable CXR shows a moderate left-sided pneumothorax, not under tension, which was
treated initially with pleural aspiration with view to insertion of a chest drain. He then proceeded
to CT of the head, spine, chest, abdomen, and pelvis. An image of the upper abdomen is included
(Figure 7.26A).

CASE 7.26: QUESTIONS


1 What does the CT abdomen show?
2 What is the diagnosis?
3 How should the patient be managed?

311

K30031_Book.indb 311 9/6/17 1:37 PM


7  Abdomen and pelvis cases

CASE 7.26: ANSWERS


1 What does the CT abdomen show?
There is rupture of the spleen with a large intraparenchymal and subcapsular haematoma. Low
density within the pancreatic tail suggests a further site of damage with haematoma and lacera-
tion. There is also haemoperitoneum with blood in the hepatorenal space (Figure 7.26B).

Fig. 7.26B  Axial portal venous (mildly delayed imaging post injection of contrast) enhanced CT image
of the upper abdomen. There is splenic rupture with large intraparenchymal (A) and subcapsular (B)
haematoma. Normal splenic enhancement (C), pancreatic tail haematoma (D), and blood in the hepatorenal
space (E) (Morrison’s pouch). Visualised kidneys and liver appear normal.

2 What is the diagnosis?


Major thoracoabdominal trauma with splenic rupture and pancreatic haematoma with possible
laceration:

• Initial arterial phase contrast imaging showed multiple high-density foci within the
spleen indicative of contrast extravasation owing to active arterial bleeding (Figure 7.26C).
• While this is a serious injury, the splenic hilar vessels remain intact with splenic
perfusion apparent, which is an important radiological finding. There is a grading
system for splenic trauma (American Association for the Surgery of Trauma – AAST)
ranging from grade 1 (subcapsular haematoma <10%, capsular laceration <1 cm) through
to grade 5 (spleen shattered, splenic hilar vessel damage). This patient has a grade 3

312

K30031_Book.indb 312 9/6/17 1:37 PM


Case 7.26: Answers

Fig. 7.26C  Axial arterial phase contrast enhanced CT image of the upper abdomen. High-density
contrast blushes (A and B) indicate sites of active haemorrhage. The residual left lung base pneumothorax
is also seen (C).

injury (large intraparenchymal and subcapsular haematomas but no evidence of major


devascularisation). Further treatment (conservative versus embolisation versus surgery)
is based on the haemodynamic and clinical condition of the patient in combination with
imaging findings.

3 How should the patient be managed?


The patient should be managed initially via an ABCDE approach with early involvement of the
entire trauma team. Remember the patient must be haemodynamically stable and safely self-
ventilating or intubated prior to CT transfer:

• Management of his breathing involves decompressing (draining) the pneumothorax.


Management of circulation involved aggressive fluid resuscitation, ABG, full routine
bloods, and group and save. Early transfusion with group O-negative blood may be
required.

313

K30031_Book.indb 313 9/6/17 1:37 PM


7  Abdomen and pelvis cases

• The patient should be urgently referred to the on-call surgical/interventional radiology


team. Injury severity is graded according to extent/depth of the laceration/haematoma,
and disruption of the vascular pedicle denotes a severe injury. Severe splenic injuries with
active haemorrhage may require urgent embolisation or surgery. Less severe injuries,
however, are managed conservatively owing to the important immunological function of
the spleen.

LEARNING POINTS: MAJOR ABDOMINAL TRAUMA, SPLENIC INJURY


▪▪ Trauma patients should be managed via an ABCDE approach, i.e. pneumothorax before
circulatory shock.
▪▪ CT imaging should be undertaken as soon as the patient is stabilised if immediate surgery is not
warranted.
▪▪ Splenic injury is seen in up to 45% of patients with blunt abdominal trauma and necessitates
urgent surgical referral.

314

K30031_Book.indb 314 9/6/17 1:37 PM


Case 7.27

Fig. 7.27A  Localised low-dose XR of the pelvis to assess the position of the IUCD.

A 37-year-old female has a Mirena IUCD in situ but cannot feel the threads. She is asymptomatic
with no lower abdominal pain. She does not recall the device being expelled vaginally.
She undergoes transvaginal US having been referred by her GP to confirm the coil has not
migrated but the US does not demonstrate an IUCD in the endometrial cavity.
As the device is metal, she undergoes low-dose localised XR of the pelvis to assess its position
(Figure 7.27A).

CASE 7.27: QUESTIONS


1 Can you see the device? What has happened?
2 What other causes of ‘lost threads’ are there?
3 What advice should you give this patient?

315

K30031_Book.indb 315 9/6/17 1:37 PM


7  Abdomen and pelvis cases

CASE 7.27: ANSWERS


1 Can you see the device and what has happened?
The IUCD is projected over the right sacral ala (Figure 7.27B). The device has perforated through
the myometrium and migrated outside of the uterus. This has been clinically ‘silent’ but patients
may present with nonspecific lower abdominal pain and, rarely, peritoneal sepsis.

Fig. 7.27B  The displaced coil is projected over the right sacral ala. Note the ‘T’-shaped configuration of
the device (A) with the stem labelled (B).

2 What other causes of ‘lost threads’ are there?


The threads may be coiled in the cervix or they may have been cut too short. Often threads are lost
after a medical procedure such as a colposcopy

316

K30031_Book.indb 316 9/6/17 1:37 PM


Case 7.27: Answers

3 What advice should you give this patient?


The patient requires referral to gynaecology for consideration of laparoscopic removal of
the device. She should be advised that in the interim she should use alternative methods
of contraception.

LEARNING POINTS: MISPLACED INTRAUTERINE CONTRACEPTIVE DEVICE


▪▪ US is the preferred modality for demonstrating an IUCD.
▪▪ If it cannot be visualised, a pelvic XR is performed initially, and if this is unhelpful full AXR or
sometimes CT may be needed.
▪▪ An IUCD lying lateral to the midline suggests that it is displaced.

317

K30031_Book.indb 317 9/6/17 1:37 PM


K30031_Book.indb 2 9/6/17 1:34 PM
Musculoskeletal cases

8 EDWARD SELLON AND


ANDREW SNODDON

Case 8.1 320 Case 8.10 365


Case 8.2 327 Case 8.11 369
Case 8.3 331 Case 8.12 372
Case 8.4 335 Case 8.13 377
Case 8.5 339 Case 8.14 382
Case 8.6 343 Case 8.15 388
Case 8.7 348 Case 8.16 394
Case 8.8 353 Case 8.17 400
Case 8.9 358

319

K30031_Book.indb 319 9/6/17 1:37 PM


Case 8.1

Fig. 8.1A  Lateral XR cervical spine.

320

K30031_Book.indb 320 9/6/17 1:37 PM


Case 8.1: Questions

A 35-year-old male is brought by ambulance to the regional trauma centre following a fall down
a flight of stairs. The patient reports pain throughout his cervical spine.
On examination the patient has bony tenderness over the upper cervical spine. Neurological
examination is normal. The neck is immobilised in a collar to enable safe transfer to the radiology
department. An XR of the cervical spine is arranged (Figure 8.1A).

CASE 8.1: QUESTIONS


1 What features are seen on the XR?
2 What is your technique for interpreting a cervical spine XR?
3 What further imaging could be performed?
4 How would you manage a suspected cervical spine fracture?

321

K30031_Book.indb 321 9/6/17 1:37 PM


8  Musculoskeletal cases

CASE 8.1: ANSWERS


1 What features are seen on the XR?
There are multiple fractures of the C2 vertebra (Figure 8.1B). These include a displaced fracture
of the anterior inferior corner of the vertebral body and bilateral fractures of the pedicles/pars
interarticularis. This is a so-called ‘Hangman’s’ fracture (a hyperextension injury) with bilateral
pedicular fractures of C2, an unstable and potentially lethal injury. Immobilisation of the cervical
spine and urgent discussion with seniors is required.

Figs. 8.1B, C  Lateral XR of the cervical spine (8.1B) with C2 fractures involving the anterior inferior corner
of the body (E) and through the pedicles (F). Note the normal alignment lines (see lines A–D, 8.1C), normal
odontoid peg (G), and anterior arch atlas (H). Alignment lines are nondisrupted in this patient and only
minor soft tissue swelling is also present. C2 and C7 vertebrae are labelled. Note the C7/T1 junction is not
visualised clearly and also how difficult this serious fracture is to diagnose on XR, with minimal soft tissue
swelling and no malalignment.

322

K30031_Book.indb 322 9/6/17 1:37 PM


Case 8.1: Answers

2 What is your technique for interpreting a cervical spine XR?


Evaluation of the cervical spine XR in trauma involves searching for malalignment, cortical frac-
ture, and soft tissue swelling.

• The standard radiographic views are the lateral, AP, and peg views. Remember to review all
of these. You are very unlikely to be asked to report a cervical spine XR in an exam at finals
but you do need to understand the principles. You would always ask for senior help and
would not ‘clear’ a spine XR on your own (you should not do this as a FY1/FY2 either!).
• Check for adequate spinal coverage. You must be able to see from the craniocervical
junction down to T1 on the lateral view for the XR to be adequate. In this case C7 is labelled
but not well seen. C7/T1 is where most fracture/dislocations occur and must be clearly
visualised on XR to ensure normal alignment.
• Check for vertebral alignment on the lateral view (Figure 8.1C). Review the anterior
vertebral line (A – the line of the anterior longitudinal ligament), the posterior vertebral line
(B – the line of the posterior vertebral ligament), and the spinolaminar line (the line formed
by the anterior margin of the spinous processes – C) plus line D (tips of spinous processes).
These lines should be continuous, without any steps.

C
Fig. 8.1C

323

K30031_Book.indb 323 9/6/17 1:37 PM


8  Musculoskeletal cases

• Check the outline of each bone for fractures.


• Check the disc spaces. These should be approximately equal in height.
• Check the prevertebral soft tissues for haematoma and swelling:
• In adults, above C4, the prevertebral soft tissues should measure less than one-third of
the width of the vertebral body. Below C4 these should measure less than the width of
the vertebral body.
• There are many patterns of cervical spine fracture and these are usually classified according
to the mechanism of injury (i.e. excess flexion, extension, or rotation).

3 What further imaging could be performed?


CT is the best first-line investigation for the cervical spine in the context of trauma, particularly
in the elderly or following high energy trauma. CT can be followed by MRI if there is neural
­compromise. The cervical spine should first be immobilised and then senior help sought to assist
with the patient transfer onto the scanner.

Fig. 8.1D  Sagittal CT cervical spine confirming fracture of anterior C2 vertebral body (A) in the patient
from Figure 8.1B. C2 and C7/T1 normal alignment are labelled.

324

K30031_Book.indb 324 9/6/17 1:37 PM


Case 8.1: Answers

• CT may be required following XR if:


• The XR is inadequate (not showing the top of T1).
• The XR shows an abnormality that requires further evaluation. 
• The XR is normal despite strong clinical suspicion of fracture. 
• CT should be used as the first-line investigation instead of XR if: 
• There is abnormal neurology, (prior to arranging an MRI).
• There is a high energy mechanism of injury.
• The patient is elderly (>65 years).
• The patient has a head injury requiring a CT brain and also has a suspected cervical
spine injury.
• The patient is being scanned for multi-region trauma (polytrauma).
• CT provides excellent spatial resolution and bone detail. It can also help to further evaluate
the soft tissues (Figures 8.1D and 8.1E).
• MRI provides excellent soft tissue contrast resolution, providing a detailed assessment of
the ligamentous structures in the neck and also the spinal cord.

Fig. 8.1E  Axial CT cervical spine (same patient as in Figure 8.1D) confirming bilateral pars interarticularis/
pedicle fracture of C2 (A).

325

K30031_Book.indb 325 9/6/17 1:37 PM


8  Musculoskeletal cases

4 How would you manage a suspected cervical spine fracture?


Management may be conservative or surgical depending on type/stability of fracture and condi-
tion of the patient.

• Conservative management may be chosen for stable cervical spine fractures, and may take
the form of a collar or brace.
• Surgical management may be used in patients with neurological injuries or unstable
cervical spine fractures, to stabilise the cervical spine and improve alignment before bone
healing will occur.

LEARNING POINTS: CERVICAL SPINE FRACTURE


▪▪ Ensure that the spine has been adequately immobilised before you request imaging.
▪▪ Always seek senior advice early and do not feel pressurised to ‘clear’ cervical spine XRs as a
foundation doctor.
▪▪ Make sure that the XR is adequate and that you have seen all the required views before making
comment.
▪▪ Avoid ‘satisfaction of search’. Once you have seen one abnormality, carry on looking for others!
▪▪ If there is high clinical suspicion of fracture, CT should be performed regardless of XR findings.

326

K30031_Book.indb 326 9/6/17 1:37 PM


Case 8.2

Fig. 8.2A  XR of the pelvis.

A 72-year-old female presents to the orthopaedic outpatient clinic with a 12-month history
of ­progressive left hip pain and stiffness. She notes her symptoms are worse after exercise.
No ­significant discomfort is present in her right hip.
On examination there is a reduced range of movement and crepitus in the left hip joint. A nor-
mal range of movement is demonstrated in the right hip.
An XR of the pelvis is obtained (Figure 8.2A).

CASE 8.2: QUESTIONS


1 What are the XR findings?
2 What is the diagnosis?
3 What is the difference between primary and secondary forms of this condition?
4 How should this patient be managed?

327

K30031_Book.indb 327 9/6/17 1:37 PM


8  Musculoskeletal cases

CASE 8.2: ANSWERS


1 What are the XR findings?
XR of the pelvis shows the following changes around the left hip joint (Figure 8.2B):

• Loss of joint space.


• Osteophyte formation.
• Subchondral cyst formation.
• Subchondral sclerosis.

Fig. 8.2B  XR left hip. Loss of joint space (A), marginal osteophyte (B), subchondral cyst formation (C), and
subchondral sclerosis (D).

328

K30031_Book.indb 328 9/6/17 1:37 PM


Case 8.2: Answers

2 What is the diagnosis?


Severe osteoarthritis (OA) of the left hip joint:

• The classic XR features are listed above and form the mnemonic ‘LOSS’.
• Classification is usually subjective using mild, moderate or severe. During clinical
placements take the opportunity to review XRs that demonstrate the full spectrum of
disease.
• OA is usually bilateral and commonly affects the large weight-bearing joints. Always
remember to review the contralateral side.
• The Kellgren-Lawrence scoring tool (Table 8.2) is useful to know about but need not be
learnt for the exam.

Table 8.2  Kellgren-Lawrence scoring tool

Grade 1 Minor joint space narrowing and osteophyte formation


Grade 2 Definite osteophyte formation and possible joint space narrowing
Grade 3 Multiple osteophytes, definite joint space narrowing, sclerosis, and possible bony deformity
Grade 4 Large osteophytes, marked joint space narrowing, severe sclerosis, and definite bony deformity

3 What is the difference between primary and secondary forms of this condition?
Primary (idiopathic) OA mainly affects the hips, knees, and hands (base of thumb and distal
interphalangeal joints). The aetiology is unknown; however, it is more common in women and a
hereditary link has been proposed.
Secondary OA occurs as a result of previous joint damage, for example cartilage injury,
fracture, neuropathy (such as diabetic neuropathy leading to Charcot joint), and congenital or
acquired deformity.

4 How should this patient be managed?


It is important to note that XR findings do not necessarily correlate with clinical findings.
Management should, therefore, be tailored to the level of pain and disability. Management
options are physical, medical, and surgical:

• Physiotherapy to improve strength and mobility.


• Medical therapy includes analgesia and anti-inflammatory medication. Intra-articular
injections are sometimes used.
• Surgical management options include joint replacement once conservative measures have
failed (Figure 8.2C, overleaf).

LEARNING POINTS: OSTEOARTHRITIS


▪▪ The classical features of OA are loss of joint space, osteophyte formation, subchondral cyst
formation, and sclerosis (‘LOSS’).
▪▪ Common joints affected include weight-bearing joints, such as the hip and knee, and small joints
of the hand, e.g. 1st carpometacarpal and distal interphalangeal joints.
▪▪ Remember to comment on both hips if presented with an XR of the pelvis.
▪▪ Symptoms may not correlate well with XR findings. Management options include physical,
medical, and surgical approaches.

329

K30031_Book.indb 329 9/6/17 1:37 PM


8  Musculoskeletal cases

Fig. 8.2C  Cemented total


hip replacement in the same
patient as in Figure 8.2A.

330

K30031_Book.indb 330 9/6/17 1:37 PM


Case 8.3

Fig. 8.3A  XR pelvis/hips.

A 70-year-old female presents to the ED with an acute episode of pain in her left groin, radiating
to the thigh following a fall down four steps. She has a history of hypertension but is otherwise
fit and well.
On examination there is tenderness over the left hip, with a very limited range of movement.
The left leg appears shortened and externally rotated. She is not able to weight bear. An XR of the
pelvis and left hip is arranged (Figure 8.3A).

CASE 8.3: QUESTIONS


1 What is the XR diagnosis?
2 How can this condition be classified?
3 How should this patient be managed?
4 What important complications should be considered?

331

K30031_Book.indb 331 9/6/17 1:37 PM


8  Musculoskeletal cases

CASE 8.3: ANSWERS


1 What is the XR diagnosis?
There is a displaced extracapsular, intertrochanteric fracture of the left neck of femur, with impac-
tion and shortening (Figure 8.3B). Ideally, as with all suspected fractures, the joint should be
demonstrated in two orthogonal planes (AP and lateral), although lateral views may be techni-
cally difficult owing to pain.

2 How can this condition be classified?


Neck of femur fractures are classified according to their anatomical position along the neck but
are also broadly split into intracapsular and extracapsular fractures as this affects management
(Figure 8.3B shows line of capsule insertion).

• The distinction between intracapsular and extracapsular femoral neck fractures is important
because of the impact on blood supply to the femoral head and the risk of subsequent femoral
head avascular necrosis (AVN). Blood supply to the femoral head is from retinacular vessels,
which pass along the femoral neck, receiving blood supply from the circumflex femoral
arteries – some arterial supply is also via the ligamentum teres. These vessels are all at risk

Fig. 8.3B  Cropped XRs of the left hip (right image) and normal right hip (left image). There is an
extracapsular, intertrochanteric fracture of the left neck of the femur (A) with impaction, angulation, and
shortening (B). Normal right hip: greater trochanter (C), lesser trochanter (D), intertrochanteric line (E, site of
hip capsular insertion).

332

K30031_Book.indb 332 9/6/17 1:37 PM


Case 8.3: Answers

from intracapsular fractures. Displaced intracapsular neck fractures require replacement


of the femoral head with hemiarthroplasty, whereas extracapsular and nondisplaced
intracapsular fractures can usually be safely managed with a dynamic hip screw (DHS).
• Garden classification of intracapsular fractures:
• Type I: Incomplete, undisplaced.
• Type II: Complete, undisplaced.
• Type III: Complete, incompletely displaced.
• Type IV: Complete, completely displaced.
• Remember the rhyme ‘1, 2 Dynamic Hip Screw; 3 and 4 Austin Moore’ (the Austin Moore is
an uncemented hemiarthroplasty).

3 How should this patient be managed?


In this patient the extracapsular fracture can be managed with open reduction and internal fixa-
tion using a DHS (Figure 8.3C). If there is evidence of osteoporosis this should also be treated
medically.

Fig. 8.3C  XR left hip


with DHS and satisfactory
alignment, and remodelling
across the fracture site.

333

K30031_Book.indb 333 9/6/17 1:37 PM


8  Musculoskeletal cases

• It is particularly important not to overlook an undisplaced intracapsular fracture, as if


these are missed they will extend and displace, requiring hemiarthroplasty. If caught early,
(Garden type 1 or 2), screw and plate fixation will suffice (Figure 8.3D).
• If the XR appears normal but there is continued clinical suspicion (i.e. the patient is still
unable to weight bear), then cross-sectional imaging is recommended. CT may be sufficient
but MRI is the reference standard investigation.

Fig. 8.3D  XR left hip with a transcervical, intracapsular fracture, before (left) and after surgical fixation with
a DHS (right). This fracture is subtle. Note the medial cortical breach (arrow).

4 What important complications should be considered?


Mortality risk increases after a hip fracture and there is evidence that this correlates with the
length of hospital stay. Long periods of immobility may predispose to venous thromboembo-
lism or pressure sores. Surgical complications include fracture, nonunion or malunion, avascular
necrosis, fat embolus, and infection of the prosthesis metalwork.

LEARNING POINTS: HIP FRACTURES


▪▪ XRs of the pelvis and affected hip are usually sufficient to make the diagnosis.
▪▪ CT or MRI are useful in the context of a normal XR but high clinical suspicion.
▪▪ ‘1, 2 Dynamic Hip Screw; 3 and 4 Austin Moore’ guides the management of intracapsular
fractures.
▪▪ Mortality risk increases following a hip fracture; therefore, careful medical as well as surgical
management is required.
▪▪ Prevention is important: home assessments to reduce risk of fall, hip pads, and osteoporosis
screening and treatment in those at risk.

334

K30031_Book.indb 334 9/6/17 1:37 PM


Case 8.4

Fig. 8.4A  AP XR of left


big toe.

A 72-year-old tablet-controlled diabetic male presents to the ED with a 3-week history of pain
over his left big toe, increasing in severity over the last 2 days. He is feverish and struggling to
weight bear. He lives alone and has had problems with diabetic control.
On examination there is an ulcer adjacent to the medial aspect of the distal interphalangeal
joint of his left big toe, with surrounding soft tissue swelling, erythema, and tenderness. He is
pyrexial at 39.1°C and inflammatory markers are elevated, with blood tests showing:

WCC 15.6 × 109/L (4–11 × 109/L) Neutrophils 12.5 × 109/L (2–7.5 × 109/L) CRP 154 mg/L (<5 mg/L)

An XR of the big toe is arranged (Figure 8.4.A).

CASE 8.4: QUESTIONS


1 What features are seen on the XR? 3 What further investigations may be helpful?
2 What is the diagnosis? 4 How should this patient be managed?

335

K30031_Book.indb 335 9/6/17 1:37 PM


8  Musculoskeletal cases

CASE 8.4: ANSWERS


1 What features are seen on the XR?
The left big toe XR demonstrates multiple sites of bone destruction at the hallux involving the
distal (A) and proximal (B) phalanges (Figure 8.4B).

Fig. 8.4B  XR left hallux (big toe) demonstrating bony destruction of the distal (A) and proximal (B) phalanx
of the left hallux. There is a pathological fracture of the distal proximal phalanx (C). Note: marked overlying
soft tissue swelling and also vascular calcification (D).

336

K30031_Book.indb 336 9/6/17 1:37 PM


Case 8.4: Answers

2 What is the diagnosis?


The history and XR appearances are typical of osteomyelitis, in this case secondary to underlying
diabetes mellitus.

• Osteomyelitis or bone infection can occur secondary to haematogenous or direct spread of


infection. In diabetes it is often secondary to direct spread from a pressure ulcer.
• The most common causative organism is Staphylococcus aureus.
• Salmonella infection is associated with sickle cell anaemia patients.
• In the early stages of osteomyelitis the XR may be normal. Soft tissue swelling and
obliteration of fat planes are early signs. Bone destruction is a late feature (Figure 8.4C).
• Management involves IV antibiotics, debridement of large abscesses, and surgery for
progressive disease.

Also look carefully for evidence of gas in the soft tissues, which may be secondary to gas-
forming organisms, and vascular calcification is common in diabetics (Figure 8.4B).

3 What further investigations may be helpful?


In addition to inflammatory markers, blood cultures and wound swabs can help to isolate an
organism. A bone biopsy may be necessary in some cases. MRI is also useful to further delineate
the extent of bone and soft tissue involvement (Figure 8.4D). The loss of high fat signal intensity
in the bone marrow on T1 imaging is highly specific for infection.

Fig. 8.4C  Lateral calcaneal XR in another patient with an infected heel ulcer. Note: soft tissue swelling
and calcaneal cortical destruction (A).

337

K30031_Book.indb 337 9/6/17 1:37 PM


8  Musculoskeletal cases

Local complications of infection include:


• Adjacent abscess in soft tissue.
• Sinus or fistula.
• Pathological fracture.
• Extension into an adjacent joint, leading to septic arthritis.
• Deformity or subsequent growth disturbance.
• Progression to chronic osteomyelitis.
• Systemic infection, e.g. septicaemia.

4 How should this patient be managed?


The patient will need a combination of surgical debridement of the ulcer and IV antibiotics guided
by the causative organism. Longer term, he will need help with diabetic control and a podiatric
assessment.

Fig. 8.4D  A fat suppressed MR image


(this sequence highlights fluid as bright/white)
in this patient showing high signal oedema
with loss of fat marrow signal in the dorsal
calcaneus (A) consistent with infection deep
to the ulcer. Calcaneus (CA), cuboid (CU).

LEARNING POINTS: DIABETIC FOOT/OSTEOMYELITIS


▪▪ Osteomyelitis may occur secondary to haematogenous or direct spread of infection.
▪▪ XR is usually normal in the early stages. Bone destruction is a late sign.
▪▪ MRI can help to delineate the extent of bone and soft tissue infection.
▪▪ Antibiotic therapy should be guided by the results of bone or tissue biopsy and microbiology
assessment.

338

K30031_Book.indb 338 9/6/17 1:37 PM


Case 8.5

Fig. 8.5A  XR pelvis.

A 25-year-old male presents to the ED with acute on chronic pain in his left hip exacerbated by a
minor fall from standing on the way to work. There is no relevant past medical history.
On examination there is tenderness over the left hip and limited range of movement. XR of the
pelvis and left hip is arranged (Figure 8.5A).

CASE 8.5: QUESTIONS


1 What features are seen on the XR?
2 What is the cause for the hip pain?
3 What conditions can cause lytic bone lesions?
4 What further investigations may be indicated?
5 How should this patient be managed?

339

K30031_Book.indb 339 9/6/17 1:37 PM


8  Musculoskeletal cases

CASE 8.5: ANSWERS


1 What features are seen on the XR?
XR of the pelvis (Figure 8.5B) shows:

• A subcapital fracture through the left neck of femur, passing through a lucent (dark) lesion
within the femoral neck. The fracture is intracapsular and undisplaced.
• Further abnormal lucent lesions are noted within the left and right proximal femoral
diaphyses. Note the inferior extent of the left femoral diaphyseal lesion is not included in the
field of view and additional radiographic views of the more distal femur should be performed.
• A pathological fracture is a fracture occurring through an area of abnormal bone. This can
be due to a focal abnormality or a generalised, diffuse process. The underlying abnormality
may be benign or malignant.
• The term ‘lucent’ refers to a bone lesion where the cortex or marrow is replaced with less
dense material and therefore appears dark or radiolucent; sclerotic implies replacement with
more dense material. Associated bone destruction in relation to a lucent lesion suggests an
aggressive process and these type of lesions are termed ‘lytic’.

Fig. 8.5B  XR pelvis with a pathological impacted, undisplaced subcapital fracture of the left femoral neck
(A) and bilateral lucent bone lesions (B).

2 What is the cause for the hip pain?


In the above case, the multifocal lucent lesions are due to fibrous dysplasia (confirmed on biopsy),
a benign disorder in which normal bone is replaced with fibrous tissue causing deformity and
pain. It produces a ‘ground-glass matrix’ appearance to the bone, expansion, remodelling, and
often endosteal scalloping. The affected bone is weak and, therefore, susceptible to pathological

340

K30031_Book.indb 340 9/6/17 1:37 PM


Case 8.5: Answers

fractures. Fibrous dysplasia is not a condition for finals but pathological fracture and lucent/lytic
lesions might occur in the exam.

3 What conditions can cause lytic bone lesions?


There are many conditions, benign and malignant, that can cause lucent or lytic bone lesions.
The XR appearances should be correlated with the clinical details and the patient’s age. The
important aggressive conditions to consider are metastases and myeloma (in an adult), and infec-
tion (all ages). Examples of lytic lesions and myeloma are included elsewhere in this book.
Factors aiding in the diagnosis of malignant bone tumours from benign lesions are:

• Zone of transition from normal to abnormal bone: a sharp, discrete, and narrow zone
is associated with slow growing benign lesions, whereas less well-defined lesions are
associated with fast growing aggressive processes.
• Location: lesions often arise in specific bones and regions of bones. Say if the lesion is
epiphyseal, metaphyseal or diaphyseal and if it is central, eccentric or cortical.
• Periosteal reaction: a thick, wavy periosteal reaction is more often associated with slow
growth, benign disease. A lamellated (onion-skin), amorphous or sunburst pattern is more
aggressive.
• Age of the patient: specific lesions tend to occur at specific ages. Metastases and myeloma
are rare under 40 years old.

4 What further investigations may be indicated?


A dedicated XR series of the lesion is usually sufficient. CT and MRI are sometimes also used,
however, to further examine the nature of the underlying bone lesion (Figure 8.5C). If metastases
are suspected, a staging CT of the chest, abdomen, and pelvis should be performed to look for
the primary tumour together with a radionuclide bone scan to evaluate the entire skeleton. An
image-guided bone biopsy may also be required. If myeloma is suspected, serum electrophoresis
and urine Bence Jones protein are checked, and either an XR or MRI skeletal survey is performed.

5 How should this patient be managed?


The patient is at risk of further fractures and, given the large volume of bone affected, surgical
fixation with an intramedullary nail is required (Figure 8.5D). Prophylactic internal fixation of
the contralateral side may also be indicated if imaging confirms large volume lesions.

LEARNING POINTS: PATHOLOGICAL FRACTURE, LYTIC LESION


▪▪ A pathological fracture is a fracture through abnormally weakened bone.
▪▪ Suspect an underlying lesion if the fracture does not fit the mechanism of injury.
▪▪ Scrutinise for any abnormality in bone texture around the fracture. Lytic lesions can be single,
multiple or diffuse.
▪▪ In patients over the age of 40 years, think of metastases and myeloma.
▪▪ In younger patients think of Ewing’s sarcoma, osteosarcoma. Also, consider infection in patients
of all ages.
▪▪ Management includes obtaining a dedicated XR series of the lesion, keeping the patient
nonweight bearing with adequate analgesia, and seeking an orthopaedic opinion.

341

K30031_Book.indb 341 9/6/17 1:37 PM


8  Musculoskeletal cases

Fig. 8.5C  Coronal T1-weighted MRI of both femurs in the Fig. 8.5D  XR left hip showing an
same patient as in Figure 8.5A showing loss of the normal intramedullary nail passing through the
bright fat signal in both proximal femurs (A) due to fibrous large lucent area of expanded medulla, and
dysplasia lesions. Normal fat-containing bone marrow is ‘ground-glass matrix’, which is typical for
seen distally (B). fibrous dysplasia.

342

K30031_Book.indb 342 9/6/17 1:37 PM


Case 8.6

Fig. 8.6A  XR right hallux.

A 55-year-old male presents to his GP with an acute episode of pain in his right great toe. There
is no history of trauma. He has self-managed previous similar episodes with over the counter
anti-inflammatories.
On examination the great toe is hot, swollen, and erythematous. He is afebrile. The toe is
exquisitely tender on palpation over the metatarsophalangeal (MTP) and interphalangeal (IP)
joints. No other joint swelling is identified.
An XR of the right hallux is arranged (Figure 8.6A).

CASE 8.6: QUESTIONS


1 What features are seen on the XR? 3 What further investigations may be ordered?
2 What is the diagnosis? 4 How should this patient be managed?

343

K30031_Book.indb 343 9/6/17 1:37 PM


8  Musculoskeletal cases

CASE 8.6: ANSWERS


1 What features are seen on the XR?
XR of the right hallux shows (Figure 8.6B):

• Marked soft tissue swelling around the MTP and IP joints.


• Multiple eccentric periarticular erosions on both sides of the MTP and IP joints, some with
overhanging sclerotic margins.
• Bone density and joint spaces are preserved.

Fig. 8.6B  XR right hallux.


Soft tissue swelling around
the MTP and IP joints.
Periarticular erosions are
present with thin sclerotic
overlying edges (arrows).
Note the preserved joint
spaces and bone density.

2 What is the diagnosis?


The history and XR are typical for gout. When considering any hot and tender joint, however,
the differentials also include septic arthritis and calcium pyrophosphate deposition (CPPD) or
‘pseudogout’.
• Gout is a common arthritis caused by hyperuricaemia and deposition of monosodium urate
crystals within joints and overlying soft tissues.

344

K30031_Book.indb 344 9/6/17 1:37 PM


Case 8.6: answers

• The hands and feet are most commonly involved (particularly the hallux MTP joint)
but knee, hip, and sacroiliac joint involvement is not uncommon. Associated soft tissue
changes that may be seen on XR include bursitis (olecranon or prepatellar) and dense tophi
(soft tissue deposition of urate crystals).
• Septic arthritis is characterised by a joint effusion, juxta-articular osteoporosis, and
destruction of cartilage and bone on both sides of the joint. Early in the disease the XR is
normal but later you may see bone and joint destruction.
• CPPD (often referred to as pseudogout when it results in acute pain and swelling) shares
many XR features with OA, including loss of joint space and subchondral sclerosis. A key
feature is chondrocalcinosis (calcification within cartilaginous structures). Commonly
affected areas are the 1st and 2nd metacarpophalangeal (MCP) joints of the hand and the
triangular fibrocartilage complex (TFCC) of the wrist (Figure 8.6C).

Fig. 8.6C  XR right hand in a patient with pseudogout. There is joint space loss in the 1st and 2nd
MCP joints and at the radiocarpal joint (A). Chondrocalcinosis is present in the 3rd MCP joint (B) and the
triangular fibrocartilage complex of the wrist (C).

345

K30031_Book.indb 345 9/6/17 1:37 PM


8  Musculoskeletal cases

3 What further investigations may be ordered?


Other helpful investigations in suspected gout include:

• Serum urate levels and inflammatory markers (e.g. CRP), all of which are usually elevated.
• Microscopic examination of synovial fluid from an affected joint. This will
demonstrate needle-shaped urate crystals with negative birefringence under polarised
light. In CPPD, microscopy demonstrates rhomboid-shaped crystals with positive
birefringence.
• A recent alternative to joint aspiration (usually under US guidance) and microscopy is
dual-energy CT. This subtracts the soft tissues and leaves behind only residue of calcific or
monosodium urate attenuation (density) (Figure 8.6D).

Fig. 8.6D  Dual-energy CT showing urate crystal deposition around the MTP and IP joints of the right
foot (A). Further deposition can be seen in the ankle joint (B) and Achilles tendon (C).

4 How should this patient be managed?


Management of an acute attack of gout is symptomatic with cool packs, NSAIDs, and either col-
chicine or steroids.
Preventive intervention aimed at lowering serum urate levels includes dietary advice
(avoidance of beer, spirits, red meat, and seafood), weight loss, and allopurinol therapy
(remember starting allopurinol therapy may precipitate a further acute episode). Strict man-
agement of comorbidities, such as renal disease, cardiovascular disease, and diabetes mel-
litus, is also important.

346

K30031_Book.indb 346 9/6/17 1:37 PM


Case 8.6: answers

LEARNING POINTS: GOUT


▪▪ XR findings in gout include demarcated periarticular erosions with sclerotic, overhanging edges.
Also, preserved joint space and bone mineral density.
▪▪ Soft tissue changes include calcified periarticular tophi and bursitis.
▪▪ In any hot swollen joint also consider septic arthritis and CPPD.
▪▪ US-guided aspiration of joint fluid for microscopy or dual energy CT can confirm the
diagnosis.
▪▪ In discussing management, think: symptom relief, prevention strategy, and comorbidities.

347

K30031_Book.indb 347 9/6/17 1:37 PM


Case 8.7

Fig. 8.7A  AP view of the left foot.

A 79-year-old male visits his diabetic outpatient clinic with a 6-month history of swelling and
pain in the left foot. For the last week he has been unable to weight bear on this side. He is
reviewed by the Specialist Registrar who makes note of a long history of diabetes and several
related complications including severe renal impairment and diabetic retinopathy.
Examination reveals a swollen left foot, the overlying skin is erythematous, and it feels hot.
The skin is intact. There is loss of sensation to light touch around the ankle and foot, and the
tarsal joint feels loose when moved. The patient is apyrexial. Reviewing the patient’s blood tests

348

K30031_Book.indb 348 9/6/17 1:37 PM


Case 8.7: Questions

Fig. 8.7B  Oblique view of the left foot.

you notice that the last HbA1c was 50 mmol/mol (target range in diabetes mellitus 42–48 mmol/
mol). You arrange a new set of bloods including a FBC, renal profile, and repeat HbA1c. You also
request an XR of the foot (Figures 8.7A and 8.7B).

CASE 8.7: QUESTIONS


1 What is the most likely diagnosis and 3 Is any further imaging indicated?
differential diagnosis? 4 How might this patient be managed?
2 What do the XRs show?

349

K30031_Book.indb 349 9/6/17 1:37 PM


8  Musculoskeletal cases

CASE 8.7: ANSWERS


1 What is the most likely diagnosis and differential diagnosis?
A neuropathic (Charcot) joint best fits the clinical findings. It is important, however, to exclude
infection (i.e. septic arthritis and osteomyelitis).

2 What do the XRs show?


They show joint destruction. There are findings of bone sclerosis and fragmentation in the tarsus
with subluxations evident. There are small foci of calcification projected over the joint in keeping
with loose joint debris. There is also soft tissue swelling (Figure 8.7C).
These findings are characteristic of a neuropathic joint (Charcot joint). The neuropathy causes
repetitive joint damage and is usually secondary to diabetes.
The D’s of a neuropathic (Charcot) joint:

• Destruction of the articular cartilage.


• Degeneration (joint space loss).
• Debris (loose bodies).

Fig. 8.7C  Magnified XR of the left tarsus demonstrating the classical features of a neuropathic Charcot
joint. There is soft tissue swelling overlying the tarsus with extensive destruction, fragmentation, and
subluxation of the tarsal bones. The normal anatomy is disrupted with the normal tarsal bones hard to
identify. The calcaneus is seen (C) and talus outline is shown (T). The normal cuboid (lateral), cuneiforms,
and navicular are not well delineated and are fragmented and subluxed – the outline of these bones is
traced by a dotted line. Note: subchondral cyst formation (A) and also evidence of bone erosion (B). Tip of
medial malleolus is just seen (D).

350

K30031_Book.indb 350 9/6/17 1:37 PM


Case 8.7: Answers

• Dislocation.
• Distension of the joint (i.e. effusion).
• Density of bone normal for the patient.

Diabetic neuropathy affects weight-bearing joints, most commonly the ankle or foot. Other
causes of neuropathic joint include neurosyphilis (tend to have knee involvement) and syringo-
myelia of the spinal cord (may demonstrate shoulder deformity).

3 Is any further imaging indicated?


A MRI scan may be helpful if there is clinical suspicion of infection. It is not always possible to
differentiate between the two diagnoses on imaging findings alone. In osteomyelitis, however,
there is usually a more focal pattern of bone destruction that often relates to an overlying skin
ulcer. There may also be fluid collection or air in soft tissues.
CT is used to further characterise the extent of joint destruction and to help plan surgical
management (Figure 8.7D). Subchondral cysts and loose bodies make a neuropathic joint a more
probable diagnosis in this case than septic arthritis and osteomyelitis.

Fig. 8.7D  Sagittal CT image of the right ankle in another patient illustrating clearly the extent of the
abnormality. The articular surface of the distal tibia is fragmented (A) and there are numerous subchondral
cysts (B). Note: also sclerosis and cyst formation of articulating talus (C). Calcaneus (D).

351

K30031_Book.indb 351 9/6/17 1:37 PM


8  Musculoskeletal cases

4 How might this patient be managed?


In most cases management is conservative once infection is excluded. However, surgery may be
considered in order to facilitate ambulation (for example if adapted footwear is otherwise impos-
sible) or to reduce a deformity that is at risk of ulceration and infection. Addressing diabetic con-
trol is also important.

LEARNING POINTS: NEUROPATHIC (CHARCOT) JOINT


▪▪ A neuropathic (Charcot) joint is most commonly seen as a complication of chronic and poorly
controlled diabetes.
▪▪ The location of the neuropathic joint gives a clue as to the underlying pathology. The ankle and
foot are most commonly affected by diabetes.
▪▪ Remember the ‘6 × Ds’ of the Charcot joint.
▪▪ Coexistence of osteomyelitis is common.

352

K30031_Book.indb 352 9/6/17 1:37 PM


Case 8.8

Fig. 8.8A  XR of both hands and wrists.

A 47-year-old female presents to her GP with longstanding pain, morning stiffness, and swell-
ing in the joints of the wrists and hands. On examination, there is muscle wastage of the small
muscles of the hand and soft tissue swelling over the wrists, MCP, and proximal interphalan-
geal (PIP) joints. There is reduced movement at the wrist and a deformity in alignment of the
little fingers.
An XR of both hands and wrists is obtained (Figure 8.8A).

CASE 8.8: QUESTIONS


1 What does the XR show?
2 What is the diagnosis?
3 What are the typical radiological features of this condition?
4 What are the diagnostic criteria?
5 How would you manage this patient?

353

K30031_Book.indb 353 9/6/17 1:37 PM


8  Musculoskeletal cases

CASE 8.8: ANSWERS


1 What does the XR show?
There is a bilateral symmetrical, polyarthropathy of the wrists and hands. There is periarticu-
lar soft tissue swelling over the wrists, MCP, and PIP joints (particularly in the 2–4th fingers).
These joints display joint space narrowing, marginal erosions, and juxta-articular osteoporosis.
There are ulnar subluxations at the 5th MCP joints. Severe secondary degenerative change is
noted at the radiocarpal and thumb carpometacarpal (CMC) joints (Figure 8.8B).

Fig. 8.8B  Cropped XR of the left hand with advanced features of rheumatoid arthritis (RhA). Periarticular
soft tissue swelling (A), marginal erosions (B), juxta-articular osteoporosis (C: ringed dotted line), joint space
loss (D), ulnar subluxation (E), and secondary radiocarpal degenerative change (F).

354

K30031_Book.indb 354 9/6/17 1:37 PM


Case 8.8: Answers

2 What is the diagnosis?


The XR shows a bilateral, symmetrical, polyarthropathy in keeping with rheumatoid arthritis
(RhA). The key radiological features include:

• Joint space narrowing.


• Marginal erosions.
• Juxta-articular osteoporosis.
• Periarticular soft tissue swelling.
• Ulnar subluxations.
• Secondary degenerative change at the wrists and base of thumbs.
• Proximal symmetrical distribution across the wrists and MCP joints.

RhA has a female predominance and tends to involve young and middle-aged individuals. It is
characterised by joint pain and morning stiffness with overlying soft tissue swelling. Rheumatoid
factor (RhF) is positive in the majority of patients (seropositive) but also present in about 6% of
the normal population. HLA B27 is positive in <10% of patients with RhA.

3 What are the typical radiological


features of this condition?
Periarticular soft tissue swelling,
juxta-articular osteoporosis, marginal
erosions, and uniform joint space
narrowing are typical early features.
A lack of reactive proliferative bone
formation is typical.

• In the hands and feet the


distribution is proximal, involving
the MCP joints before the PIP
joints. The distal interphalangeal
(DIP) joints are relatively spared
(Figure 8.8C).
• These features are usually bilateral
and symmetrical.
• Any joint can be affected.
The appendicular skeleton is
commonly involved, as are the
distal clavicles (Figure 8.8D)
and the atlantoaxial joint of the
cervical spine.
• Features of RhA are included in
Tables 8.8A and 8.8B.

Fig. 8.8C  Magnified XR of the right


forefoot in the same patient as 8.8A with
marginal periarticular erosions at the
2nd–4th metatarsal heads (arrows).

355

K30031_Book.indb 355 9/6/17 1:37 PM


8  Musculoskeletal cases

Fig. 8.8D  Coned-in view of the left shoulder in a patient with RhA. The humerus (H) is ‘high riding’ with
reduced glenohumeral (GH) joint space (A) and subacromial joint space (B). The latter is due to associated
rotator cuff damage. There are marginal erosions of the humeral head (C) and distal end of the clavicle (D)
with widening of the acromioclavicular joint space.

Table 8.8A  Nonmusculoskeletal features of RhA

Pulmonary Interstitial fibrosis (lower zone), rheumatoid nodules, pleural thickening, and pleural
effusions
Cardiovascular Accelerated coronary and cerebral artery atherosclerosis, pericarditis, and vasculitis
Cutaneous Rheumatoid nodules in pressure areas such as elbows, occiput, and lumbosacral
region in RhF-positive patients
Ocular Uveitis, episcleritis, and keratoconjunctivitis sicca

356

K30031_Book.indb 356 9/6/17 1:37 PM


Case 8.8: Answers

Table 8.8B  Clinical features of RhA

Mnemonic: RHEUMATISM
R RhF +ve in 80%/Radial deviation of wrist
H HLA-DR1 and DR-4
E ESR/Extra-articular features
U Ulnar deviation of fingers
M Morning stiffness/MCP + PIP joint swelling
A Ankylosis/Atlanto–axial joint subluxation/Autoimmune/ANA +ve in 30%
T T-cells (CD4)/TNF
I Inflammatory synovial tissue (pannus)/interleukin-1
S Swan-neck deformity, Boutonniere deformity, Z-deformity of thumb
M Muscle wastage of small muscles of the hand

4 What are the diagnostic criteria?


The American College of Rheumatology revised criteria (1988) require four out of the following
seven diagnostic criteria to be present for diagnosis of RhA:

• Morning stiffness lasting >1 hour.


• Soft tissue swelling of three or more joints observed by a physician.
• Soft tissue swelling of either the wrist, PIP or MCP joints on imaging.
• Symmetric swelling.
• Subcutaneous rheumatoid nodules.
• RhF (seropositive).
• Radiographic signs: erosions or periarticular osteopenia in the hand/wrist.

5 How would you manage this patient?


Treatment aims to improve symptoms and delay disease progression. Therapy is with a combi-
nation of corticosteroids, NSAIDs, disease modifying antirheumatic drugs (DMARDs) such as
methotrexate, sulfasalazine, ciclosporin, and anti-tumour necrosis factor (TNF) agents. The anti-
TNF treatments and variants of, which suppress the immune system, are known collectively as
biological therapies.

LEARNING POINTS: RHEUMATOID ARTHRITIS


▪▪ Look for a bilateral, symmetrical and proximal polyarthropathy and periarticular erosions.
▪▪ Periarticular soft tissue swelling is an early sign.
▪▪ Rheumatoid factor is positive in the majority of cases.
▪▪ Early diagnosis and initiation of therapy is the key to altering the course of the disease.

357

K30031_Book.indb 357 9/6/17 1:37 PM


Case 8.9

Fig. 8.9A  AP XR of the right shoulder.

A 28-year-old female sustained a fall while playing volley-ball. She had extremely limited and
painful movement in the right shoulder and was removed from the court. She attends the ED
supporting her right arm with her other hand. You arrange an urgent XR (Figures 8.9A and 8.9B).

358

K30031_Book.indb 358 9/6/17 1:37 PM


Case 8.9: Questions

Fig. 8.9B  Axial XR of the right shoulder.

CASE 8.9: QUESTIONS


1 What injury has this patient sustained?
2 How is this type of injury classified, and what other types are there?
3 How would you manage this patient?

359

K30031_Book.indb 359 9/6/17 1:37 PM


8  Musculoskeletal cases

CASE 8.9: ANSWERS


1 What injury has this patient sustained?
The XR shows an anterior dislocation of the right humeral head with an associated Hill–Sachs
defect of the humeral head (Figure 8.9C). On the AP view the humeral head lies inferomedially
in relation to the glenoid. An axial XR of the shoulder is obtained with the patient’s upper arm
elevated (may be difficult owing to pain) with the XR beam passing down in a head to foot direc-
tion (craniocaudal).
• Hill–Sachs lesions are compression fractures of the posterolateral humeral head, typically
secondary to recurrent anterior shoulder dislocations as the humeral head comes to rest
on the anteroinferior glenoid rim. If large, the wedge defect can also be seen on the post-
relocation XR (Figure 8.9D).

Fig. 8.9C  Axial XR of the right shoulder showing anterior dislocation of the humeral head (A) with a Hill–
Sachs defect (B) where the humeral head is perched on the anterior inferior glenoid rim (C). Glenoid (D),
acromion (E). The humeral head lies anteriorly and has lost its normal glenoid articulation.

360

K30031_Book.indb 360 9/6/17 1:38 PM


Case 8.9: Answers

Fig. 8.9D  AP XR of the right shoulder in a different patient following relocation of an anterior dislocation
in the ED. A Hill–Sachs lesion (dotted line) is seen as a wedge defect in the humeral head. Normal glenoid
(A) and humeral head (B) alignment is demonstrated by the tracing of a smooth continuous line along the
scapulohumeral arch (solid line). This is also called the Bandi line.

• Bankart lesions are often seen in combination with Hill–Sachs lesions and are caused by
humeral head impaction on the anteroinferior glenoid. They may constitute glenoid labral
damage only (‘soft Bankart’) or an impaction fracture of the glenoid (‘bony Bankart’).

2 How is this type of injury classified, and what other types are there?
Shoulder dislocation may be anterior (most common), posterior (uncommon) or inferior (least
common).

• Posterior dislocation makes up 2–4% of cases and is classically associated with a convulsive
disorder, electrocution or high energy RTA, causing forced posterior displacement with the
arm in abduction and internal rotation.

361

K30031_Book.indb 361 9/6/17 1:38 PM


8  Musculoskeletal cases

Fig. 8.9E  Y-view (lateral) of the right shoulder showing a posterior dislocation. The central aspect of the
humeral head (A) does not overlie the glenoid (B: outlined by dotted line) but is positioned posteriorly. The
coracoid (C) is a useful anterior anatomical landmark.

• Posterior dislocation on frontal XR shows an internally rotated humeral head (‘lightbulb


sign’), loss of normal glenohumeral (GH) overlap with widening of the joint >6 mm, and
disruption of the scapulohumeral arch. The axillary view is preferred as it directly shows
displacement in the posterior direction (Figures 8.9E and 8.9F).

3 How would you manage this patient?


Anterior dislocation is usually managed with closed reduction and a period of immobilisation to
allow capsular healing. It is important that the patient undergoes a programme of physiotherapy
to restore range of movement and muscle strength.

362

K30031_Book.indb 362 9/6/17 1:38 PM


Case 8.9: Answers

Fig. 8.9F  AP view of the right shoulder showing a posterior dislocation. There is a rounded appearance
of the humeral head (‘light bulb sign’) and absent GH overlap indicating widening of the GH joint. Humeral
head (A), glenoid (B), and coracoid (C: dotted line).

SHOULDER RELOCATION PROCEDURE


▪▪ Wash hands, Introduce, ask the Patient’s name and date of birth, Explain the procedure and
risks, and gain consent.
▪▪ Review the XRs (both views) before and after to determine pattern of dislocation/confirm
relocation/exclude fractures (do not reduce if there is a fracture).
▪▪ Examine neurovascular status and document it. In particular axillary nerve sensation over the
lateral deltoid.
▪▪ Analgesia and sedation with IV morphine, midazolam, and Entonox.
▪▪ Technique depends on pattern of dislocation:
– Anterior: use Kocker’s, Stimpson’s or Modified Milch technique (look these up).
– Posterior: pull arm gently forward and externally rotate.
▪▪ Aftercare with broad arm sling or polysling for 3 weeks and appointment at the fracture clinic.

363

K30031_Book.indb 363 9/6/17 1:38 PM


8  Musculoskeletal cases

LEARNING POINTS: SHOULDER DISLOCATION


▪▪ Always check the XR for associated fractures of the glenoid or humeral head.
▪▪ A Hill–Sachs lesion is a compression fracture of the posterolateral humeral head caused by
impaction against the anterior glenoid rim.
▪▪ A bony-Bankart lesion is a fracture of the anteroinferior glenoid rim.
▪▪ Dislocations can usually be reduced in the ED with sufficient analgesia and sedation. Make sure
to examine the neurovascular status of the arm before and after reduction, and document it in the
notes.

364

K30031_Book.indb 364 9/6/17 1:38 PM


Case 8.10

Fig. 8.10A  XR of hips.

A 68-year-old female attends the orthopaedic clinic 4 years following revision of her left total hip
replacement (THR), with worsening left groin and thigh pain. She is otherwise fit and well. On
examination, there is a limited range of movement in the left hip owing to pain and there is some
shortening of the left leg.
An XR of her pelvis and left hip is arranged (Figure 8.10A).

CASE 8.10: QUESTIONS


1 What does the XR show?
2 What is the cause of the pain?
3 What are the differential diagnoses?
4 What is stress shielding?
5 How should this patient be managed?

365

K30031_Book.indb 365 9/6/17 1:38 PM


8  Musculoskeletal cases

CASE 8.10: ANSWERS


1 What does the XR show?
The XR shows a left total hip arthroplasty (THA or THR [total hip replacement]). There is proxi-
mal osteolysis (bone resportion) of the calcar (ridge of dense bone in the posteromedial femoral
neck that is important for mechanical support). There is also cortical thinning and bone loss
within the nonweight-bearing greater trochanter and lateral femoral cortex in keeping with stress
shielding (Figure 8.10B).

Fig. 8.10B  Left THR with


osteolysis of the calcar (A)
and greater trochanter (B)
owing to stress shielding.
The calcar is a vertical
plate of bone deep to
the lesser trochanter.
A large, concentric, linear,
periprosthetic lucency
(C) indicates loosening.
Multifocal distal femoral
lucencies with endosteal
scalloping (D) suggest
further foci of loosening
secondary to particle
disease.

366

K30031_Book.indb 366 9/6/17 1:38 PM


Case 8.10: Answers

A tension band wire has been used to stabilise the middle zones of the femoral prosthesis
where there is a long, linear, concentric periprosthetic lucency measuring >2 mm depth. This is
indicative of mechanical loosening.
Multiple discrete and well-defined lucencies surround the distal femoral prosthesis with
smooth endosteal scalloping. This is the appearance of osteolysis caused by particle disease.
There are no signs of infection.

2 What is the cause of the pain?


The pain is caused by loosening of the femoral prosthesis. This is likely caused by a combination
of mechanical loosening and particle disease. Importantly, there are no signs of infection.

• Particle disease is an aggressive granulomatous reaction that is caused by shedding of


particles from the prosthesis, polyethylene cup, and/or cement. The submicron sized
particles can migrate along the whole course of the prosthesis causing multiple foci of
osteolysis and eventually prosthetic loosening.
• Clinical features: patients tend to be asymptomatic until there is substantial bone loss,
then in severe pain. There may be limb shortening (as seen here) and limitation of
movement.

3 What are the differential diagnoses?


Lucencies at the metal–cement or metal–bone interface >2 mm may indicate particle disease,
loosening or infection, or all three.

• Mechanical loosening is the most common indication for revision surgery. It is characterised
by a linear periprosthetic lucency, >2 mm depth. Component migration is a late but
diagnostic sign.
• Particle disease usually produces multifocal lucencies with endosteal scalloping, which do
not conform to the shape of the prosthesis. It tends to occur 1–5 years after surgery.
• Infection causes ill-defined, irregular, and eccentric bone resorption. There is also usually a
periosteal reaction. It is important to check the inflammatory markers.

KEY POINTS: RISK OF INFECTION


▪▪ Infections <1 year after surgery are acquired during surgery. The risk of intraoperative infection
is less than 1% owing to the use of antimicrobial prophylaxis and laminar airflow in the operating
theatre.
▪▪ Infections later than this are acquired by haematogenous seeding from respiratory tract, dental,
and urinary tract infections.

4 What is stress shielding?


Stress shielding is reduction in bone density, osteopenia, as a result of the removal of normal
stress on the bone owing to a prosthesis. If bone loading decreases, the bone becomes less dense
and weaker as there is no longer a stimulus to normal bone remodelling. First there is osteope-
nia, with thinning of the cortex, and then bone resorption. This is usually seen medially, deep
to the lesser trochanter (at the calcar), where it can cause limb shortening. Stress shielding and
calcar resorption are normal findings on follow-up and are not associated with loosening of the
prosthesis.

367

K30031_Book.indb 367 9/6/17 1:38 PM


8  Musculoskeletal cases

5 How should this patient be managed?


Surgical revision is almost always necessary.

LEARNING POINTS: PROSTHETIC LOOSENING


▪▪ If you see a THA remember to check for signs of mechanical loosening, particle disease, and
infection.
▪▪ A lucent zone >2 mm at the bone–prosthesis or bone–cement interface is indicative of loosening.
A thinner zone of lucency is likely to be normal but take senior advice if you are unsure.
▪▪ Ask to see previous postoperative XRs for comparison.
▪▪ Other complications of THA include fracture and dislocation.

368

K30031_Book.indb 368 9/6/17 1:38 PM


Case 8.11

Fig. 8.11A  Lateral skull XR.

A 72-year-old female has visited her GP several times in recent months complaining of tiredness
and lower back pain, with an initial lumbar spine XR showing mild OA only. Her pain worsens
and is not controlled by her prescribed analgesia (paracetamol and codeine). In addition to the
back pain, she also now complains of head and arm pain.
On examination there is bony tenderness over the proximal left humerus. There is a palpable
liver edge and the spleen also feels enlarged. Blood tests show:
Hb 86 g/L (125–165 g/L)
Total serum calcium 2.75 mmol/L (2.15–2.55 mmol/L)

There is also evidence of acute renal impairment. She is referred to a haematologist who
requests an XR series (Figure 8.11A).

CASE 8.11: QUESTIONS


1 What is shown on the XR? 3 Is any other imaging required?
2 What is the most likely diagnosis? 4 How might this patient be managed?

369

K30031_Book.indb 369 9/6/17 1:38 PM


8  Musculoskeletal cases

CASE 8.11: ANSWERS


1 What is shown on the XR?
This is a selected image from a skeletal survey (a collection of XRs taken to demonstrate the
whole skeleton). It is commonly performed in adults to investigate myeloma. Similar skeletal
surveys are also used in children to look for fractures in cases of suspected nonaccidental injury
(NAI).
The lateral skull XR shows multiple small lucencies, the so-called ‘pepper pot’ skull. This find-
ing is most suggestive of myeloma. Bone metastases could give a similar appearance but are
less likely given the patient’s symptoms (Figure 8.11B). Hyperparathyroidism may also cause a
‘pepper-pot skull’.

Fig. 8.11B  Lateral skull XR with multiple small lucencies (arrows).

370

K30031_Book.indb 370 9/6/17 1:38 PM


Case 8.11: Answers

2 What is the most likely diagnosis?


The most likely underlying diagnosis is multiple myeloma.

• Multiple myeloma is a cancer of the plasma cells found in bone marrow. Plasma cell
proliferation destroys the bone causing pain and releasing calcium.
• Bone destruction causes characteristic ‘punched out’ lytic lesions and increases the risk of
pathological fracture.
• Erythropoiesis is hampered as bone marrow is replaced by plasma cells resulting in
anaemia. Patients may present with marrow failure, also renal failure and hypercalcaemia.
• A plasmacytoma (not shown) is a larger lytic/expansile lesion associated with multiple
myeloma usually found in the spine, pelvis or ribs.
• The diagnosis is made using a combination of biopsy (including bone marrow with >30%
plasma cells), lytic bone lesions, and raised immunoglobulin levels (elevated M protein is
the hallmark).
• XRs are frequently normal in myeloma: the commonest XR finding is actually osteopenia.
Lytic bone lesions, when they occur, are usually pathognomonic. Bone scans are often
negative and are of limited use in this condition.

3 Is any other imaging required?


No further XR imaging is required.

• Whole body MRI using diffusion-weighted techniques, however, is gaining popularity


for the detection of bone lesions in multiple myeloma as it is more sensitive than XR. It is
especially useful for assessment of the spine.

4 How might this patient be managed?


The patient should be managed by a haematologist specialising in myeloma.

• Multiple myeloma is an incurable disease with a variable prognosis depending on disease


severity. The majority of patients have ‘standard risk’ disease with a median survival of
approximately 10 years with modern treatment strategies.
• Treatment is complex and ranges from watchful waiting in early disease to chemotherapy or
stem cell transplantation for eligible patients.

LEARNING POINTS: MULTIPLE MYELOMA


▪▪ Multiple myeloma is a haematological malignancy affecting the plasma cells.
▪▪ As plasma cells reside in the bone marrow, the disease may be diagnosed on XR or MRI.
▪▪ Bone lesions have a lucent, punched out appearance on XR.
▪▪ Whole body diffusion weighted MRI is gaining popularity for myeloma screening as it is more
sensitive than XR for demonstrating bone disease.

371

K30031_Book.indb 371 9/6/17 1:38 PM


Case 8.12

Fig. 8.12A
Lateral XR of the
thoracic spine.

372

K30031_Book.indb 372 9/6/17 1:38 PM


Case 8.12: Questions

A 68-year-old female presents to the orthopaedic clinic with a long history of back pain,
­exacerbated recently by lifting a heavy suitcase. Lying in the supine position relieves some
of the discomfort but it is exacerbated by standing or walking. She reports having had a bone
density scan carried out the previous week, with a T score of -2.9 (low). She denies any weight
loss. She has a sedentary lifestyle, smokes 20 cigarettes per day, and drinks an excessive
amount of alcohol.
On examination the thoracic spine is kyphotic and there is mid-thoracic tenderness on per-
cussion. There is pain with forward and lateral flexion. The straight leg raise test is negative and
neurological exam normal. Routine blood tests are unremarkable; in particular, serum calcium,
phosphate, and ALP are normal.
An XR series of the thoracic spine is arranged. The lateral view is included (Figure 8.12A).

CASE 8.12: QUESTIONS


1 What are the XR findings?
2 What is the diagnosis?
3 What can you say about bone mineral density in this patient?
4 How would you manage this patient?

373

K30031_Book.indb 373 9/6/17 1:38 PM


8  Musculoskeletal cases

CASE 8.12: ANSWERS


1 What are the XR findings?
The XR shows severe compression fractures of the anterior aspects of the T7 and T8 vertebral
bodies and a mild compression fracture of the central aspect of the T9 vertebral body. The
posterior vertebral line is intact, as are the pedicles. There are no visible retropulsed fragments
(Figure 8.12B).

Fig. 8.12B  Lateral XR


of the thoracic spine
showing severe anterior
wedge compression
of T7 and T8 vertebral
bodies. Also mild
biconcave compression
fracture of T9. Intact
posterior vertebral
line (arrows). There is
kyphotic deformity with
reduced bone mineral
density.

374

K30031_Book.indb 374 9/6/17 1:38 PM


Case 8.12: Answers

• Vertebral compression fractures (VCFs) are also called wedge fractures when either the
anterior or posterior part of the vertebral body is compressed, and biconcave fractures
where the central endplates are compressed.
• The most common causes of VCF:
• Osteoporosis (also osteomalacia and hyperparathyroidism): consider a DEXA scan if not
already done.
• Trauma: consider a CT scan if not already done.
• Metastasis/myeloma: consider MRI. Also a myeloma screen or CT body to look for a
primary tumour.
• It is important to assess and comment on the neurological status of the patient as a
retropulsed bony fracture fragment might narrow the spinal canal and cause cord/cauda
equina compression. This may only be visible on CT. Look for posterior vertebral line
alignment. Neurological symptoms would include:
• Muscular weakness.
• Abnormal sensation in the dermatomes below the vertebral level.
• Increased tone in the lower limbs.
• Increased reflexes in the lower limbs.
• Cauda equina syndrome (paraparesis, urinary incontinence, saddle-like dermatome
anaesthesia, and areflexia).
• If there are neurological signs, concerns over fracture stability or suspicion that the fracture
may be pathological, a CT or MRI is indicated. MRI is particularly useful in distinguishing
acute from chronic fractures, and tumour from infection.
• If there is concern over malignancy, FBC, ALP, LFTs, and CRP are indicated. If infection
is suspected, FBC, inflammatory markers, and blood cultures should be obtained. For
metabolic causes, serum calcium, albumin, PTH, phosphate, ALP, magnesium, creatinine,
TFTs, and vitamin D should be tested.

2 What is the diagnosis?


Osteoporotic VCF.
• There are no features on the XRs to suggest pathology other than osteoporosis, although
metastases and myeloma should be considered and excluded in the absence of trauma. It is
important to note blood bone profile (calcium, phosphate, ALP) is normal, pointing to a
nonmalignant aetiology.
• VCFs are often insidious and may produce only modest back pain early in the course of
progressive disease. They are common and present in up to 14% of women over 60 years
old. Modifiable risk factors are smoking, alcohol, insufficient physical activity, and
oestrogen, calcium or vitamin D deficiency.
• VCFs can occur anywhere in the spine but are most common at the thoracolumbar junction.
They are usually multiple and may be consecutive or at different levels. Therefore, ensure
imaging of the whole spine.
• VCFs can be graded based on the degree of vertebral height loss:
• Mild: 20–25%.
• Moderate: 25–40%. 
• Severe: >40%.

TOP TIP
Remember! An osteoporotic spine is the most common XR abnormality in myeloma.

375

K30031_Book.indb 375 9/6/17 1:38 PM


8  Musculoskeletal cases

3 What can you say about bone mineral density in this patient?
There is increased radiolucency of the vertebral bodies with thinning of the cortices in keeping
with low bone density. This is supported by the results of the preceding DEXA scan.

• DEXA scans assess bone mineral density in the femoral neck and lumbar spine. A T-score
more than 2.5 standard deviations below the young adult female reference range (20–40
years) is diagnostic for osteoporosis (WHO guideline). A Z-score uses the same threshold
but is matched for the patient’s age and ethnicity, making it more accurate. This patient had
a Z-score of -2.9 in the spine, which is diagnostic for osteoporosis.

DEXA SCAN REFERRAL CRITERIA


▪▪ All patients >50 years with a low trauma fracture.
▪▪ All patients on oral prednisolone >2.5 mg for >3 months.
▪▪ All >50 years with 1 risk factor for osteoporosis.
▪▪ All <50 years with 1 major risk factor for osteoporosis.

Major risk factors: malabsorption syndromes, rheumatic disorders, glucocorticoids, organ transplanta-
tion, chronic liver disease, thyrotoxicosis, primary hyperparathyroidism, prolonged immobilisation, alco-
hol excess, low BMI <20.
Minor risk factors: renal disease, diabetes, early menopause <45 years, anticonvulsants, antipsychotics.

4 How should this patient be managed?


VCFs are usually stable and are treated conservatively with analgesia and a short period of rest.
Muscle relaxants, use of a back-brace, and physiotherapy may also help. Risk factor modification
would include smoking and alcohol cessation, and an exercise programme with a home visit
and fall prevention advice. Patients who do not respond to conservative treatment and continue
to have severe pain may be candidates for percutaneous vertebroplasty. This involves injecting
acrylic cement into the collapsed vertebra to stabilise and strengthen the vertebral body.

LEARNING POINTS: OSTEOPOROTIC SPINAL FRACTURES


▪▪ VCFs are usually due to osteoporosis but may also be due to trauma or metastatic infiltration.
▪▪ Request imaging (usually XR initially) of the whole spine, as fractures are often multiple.
▪▪ Always assess the posterior vertebral body line for alignment.
▪▪ Remember to check and comment on neurological status.

376

K30031_Book.indb 376 9/6/17 1:38 PM


Case 8.13

Fig. 8.13A  XR of the pelvis.

A 67-year-old male visits his GP with a history of chronic pain in the left hip. On examination the
GP elicits pain on external rotation of the hip and suspects OA.
An XR of the pelvis is performed (Figure 8.13A).

CASE 8.13: QUESTIONS


1 What are the XR findings?
2 What is the likely diagnosis?
3 Are there any risks associated with this disease?
4 Is any further imaging required?

377

K30031_Book.indb 377 9/6/17 1:38 PM


8  Musculoskeletal cases

CASE 8.13: ANSWERS


1 What are the XR findings?
There is cortical thickening, bone expansion, trabecular coarsening, and sclerosis of the left
hemipelvis. This includes thickening of the iliopubic and ilioischial lines (Figure 8.13B).
The bone cortex is intact and there is no periosteal reaction or overlying soft tissue swelling
to suggest an underlying aggressive process. There is no bone destruction. Note also the severe
left hip OA.

Fig. 8.13B  AP XR of the pelvis. There is cortical thickening of the left hemipelvis with sclerosis and
coarsening of the trabeculae. The iliopubic (A) and ilioischial (B) lines are also thickened. There is joint
space narrowing and marginal osteophyte formation in the left hip joint, consistent with OA.

2 What is the likely diagnosis?


These are the characteristic appearances of Paget’s disease of the bone, a common benign disor-
der, which results in disordered and excessive bone remodelling.

• Bone marrow is replaced by fibrous tissue with numerous vascular channels.


• It is rare under the age of 40 years and common over 80 years. The most common sites of
involvement are the pelvis, spine, and skull.

378

K30031_Book.indb 378 9/6/17 1:38 PM


Case 8.13: Answers

• There are several stages of the disease ranging from initial active osteolysis (osteoclasts
predominate), then a mixed phase, and then a chronic healing phase (osteoblasts
predominate) with sclerosis.
• The inactive phase gives the classic XR appearance seen here, while the active phase
produces a lytic appearance (Figures 8.13C and 8.13D).
• Paget’s disease usually affects more than one site (polyostotic) and is asymmetric in
its distribution. In chronic Paget’s disease the bones soften and long bones may bow
(Figure 8.13E).
• There is often associated OA of the hips, with findings such as joint space narrowing and
subchondral sclerosis.

Fig. 8.13C  AP skull XR showing geographic lysis (arrows) in the active phase of Paget’s, also known as
osteoporosis circumscripta.

379

K30031_Book.indb 379 9/6/17 1:38 PM


8  Musculoskeletal cases

Fig. 8.13D  Lateral skull XR in chronic Paget’s disease with marked thickening of the calvarium.

3 Are there any risks associated with this disease?


The most serious complication is malignant transformation to osteosarcoma (rare).

• Concerning XR, features of malignancy would include cortical destruction, a ‘moth-eaten’


or ‘permeative’ pattern of lucency in the bone, periosteal reaction (fluffy edge to the bone
cortex) or a soft tissue mass.
• Other potential complications of Paget’s disease include insufficiency fractures (typically
in a long bone) and nerve entrapment owing to bone softening and expansion (this
might cause, for example, a radiculopathy or a cranial nerve palsy). Patients may be
hypercalcaemic (rare, associated with immobilisation) and high output cardiac failure is
described owing to new bone blood vessel formation.

4 Is any further imaging required?


Usually no further imaging is required. MRI may be used if there is ongoing concern regarding
malignancy. A whole body isotope bone scan can be used to demonstrate the full extent of Paget’s
disease in the bone.

380

K30031_Book.indb 380 9/6/17 1:38 PM


Case 8.13: Answers

Fig. 8.13E  AP view


of the right femur in
chronic Paget’s, with
trabecular coarsening,
cortical thickening, and
bowing of an expanded
femur. No fractures are
seen. Note: hip joint
degenerative changes.

LEARNING POINTS: PAGET’S DISEASE


▪▪ Paget’s disease is a benign disease of uncertain aetiology, more common in the elderly, involving
abnormal bone remodelling.
▪▪ The pelvis or spine is affected in 75% of cases.
▪▪ There are different phases of the disease, which are reflected in the radiological appearance.
Broadly speaking there is an active lytic phase followed by a chronic sclerotic phase.
▪▪ Complications include pathological fracture, nerve entrapment, and malignant conversion (rare).

381

K30031_Book.indb 381 9/6/17 1:38 PM


Case 8.14

Fig. 8.14A  CXR.

A 55-year-old female with a prior history of breast cancer treated with left wide local excision and
axillary node clearance presents to her GP with several months of gradually worsening severe
intractable pelvic, back, and chest wall pain. She delayed presentation having previously attrib-
uted the pain to a muscle strain. Today she presents with acute, severe worsening of pain in her
right hip, which occurred while walking upstairs at home. A blood screen is taken, which shows
a serum calcium of 3 mmol/L (2.15–2.55 mmol/L).
A CXR is arranged (Figure 8.14A), together with XRs of the pelvis and right hip (Figure 8.14B).

382

K30031_Book.indb 382 9/6/17 1:38 PM


Case 8.14: Questions

Fig. 8.14B  AP XR of pelvis and lateral XR of the right hip.

CASE 8.14: QUESTIONS


1 Describe the XR appearances.
2 What is the diagnosis?
3 What other imaging tests would be justified?
4 How would you manage this patient?

383

K30031_Book.indb 383 9/6/17 1:38 PM


8  Musculoskeletal cases

CASE 8.14: ANSWERS


1 Describe the XR appearances.
The chest and pelvis XRs show multiple, sclerotic, osteoblastic bone lesions. These are best seen
within the ribs and pubic rami. The bones appear generally dense and white (compare with other
normal bones in this book). The lungs, hila, and pleural spaces are clear. There is also an angulated
pathological subtrochanteric fracture of the right femoral neck, with healed right pubic rami frac-
tures (Figure 8.14C). With the given history, these sclerotic lesions are likely to represent breast
cancer metastases. Note: deformity of the left breast shadow on CXR owing to previous surgery.

• Sclerotic lesions are areas of increased bone density that result from overactivation of
osteoblast cells. The differential diagnosis of multiple sclerotic lesions includes osteoblastic
metastases, congenital bone islands, metabolic disorders, infection, and bone infarction.
• Multiple sclerotic or osteoblastic metastases can arise from a number of primary
malignancies but are most commonly related to breast or prostate carcinoma. In breast
cancer, lytic, sclerotic or mixed lesions may occur, and lesions may also become sclerotic as
they heal post treatment.

Always suspect a pathological fracture, especially if the patient has a history of malignancy,
when a fracture occurs that does not fit with the mechanism of injury. This area is covered in more
detail elsewhere in the book.

Fig. 8.14C  The bone texture of the pelvis is generally abnormal. There is an angulated fracture of the
subtrochanteric right femoral neck (A). On both AP and lateral views there is an area of ill-defined bone lysis
(destruction) in the adjacent femur (B) with periosteal reaction seen (C). The XR findings and clinical history
suggest fracture through an underlying lesion, which looks aggressive on radiology. With the patient history
a lytic metastasis is most likely. There are old and healed pathological fractures of the right pubic rami (D).
Compare the sclerotic density of the right pubic rami with normal bone density on the left side (E). Note the
‘o’ placed next to the R marker on the AP XR. Radiographers will place this if they suspect an abnormality/
fracture. This is a sign worth noting and discussing with the radiographer if necessary.

384

K30031_Book.indb 384 9/6/17 1:38 PM


Case 8.14: Answers

2 What is the diagnosis?


Metastatic breast cancer is the likely diagnosis, with associated pathological fracture of the
right hip.

• Breast cancer is the most common site of origin of metastatic deposits in the female
skeleton (comparable with prostate cancer in males). Bone is the most common site of
recurrence of breast cancer.
• Breast cancer metastases most commonly affect the spine, ribs, pelvis, sternum, and
proximal long bones.
• Systemic symptoms may occur, in particular those relating to hypercalcaemia such as bone
pain, abdominal pain, renal or biliary stones, and depression (remember the mnemonic,
‘stones, bones, groans and psychiatric overtones’).

3 What other imaging tests would be justified?


This patient requires urgent orthopaedic referral for management and internal fixation of the
right hip fracture, and discussion at the breast MDT meeting. She will need staging. This usu-
ally comprises CT chest/abdomen and pelvis, and a radionuclide bone scan. As always adopt an
ABCDE approach initially, check bloods (exclude hypercalcaemia from widespread bone metas-
tases, as in this patient) and manage the patient’s pain.

• XR is the best first option usually for focal pain. Multiple, confluent, sclerotic, blastic bony
lesions are typical of metastatic breast cancer. However, metastases may also present as
purely lytic, aggressive, and destructive lesions. You may also see evidence of previous
breast surgery on the CXR. It is important to identify both breast shadows in all female
patients on CXR. XRs of the femoral necks are also sometimes performed in patients
with bony metastatic disease to identify lesions at risk of fracture (these may undergo
prophylactic nailing).
• Radionuclide bone scans are used to identify blastic lesions scattered throughout the
skeleton, with these appearing as foci of increased radionuclide uptake (Figure 8.14D).
• CT is used to further stage the cancer, looking at the lungs, liver, and brain (brain only
if symptomatic) particularly for soft tissue metastases, and also axillary and mediastinal
nodes.
• MRI of the spine is used if there is clinical evidence to suggest nerve root or spinal cord
compression (Figure 8.14E).
• PET/CT is useful in some cases for equivocal lesions.

4 How would you manage this patient?


Urgent orthopaedic, breast, and oncology team review with discussion at the breast cancer MDT
meeting is required.

• Bone lesions of breast cancer may dramatically improve following bisphosphonate


medications and chemotherapy or radiotherapy. Bisphosphonates reduce malignant bone
pain and manage hypercalcaemia; they may also delay progressive bone disease. Urgent
surgical decompression may be appropriate in some cases. Pre-emptive surgical fixation
of the femoral necks should be considered if there are large lesions weakening the weight-
bearing bone.

385

K30031_Book.indb 385 9/6/17 1:38 PM


8  Musculoskeletal cases

Fig. 8.14D  Images from a radionuclide bone scan in another patient demonstrating multiple areas of
metastatic increased uptake throughout the pelvis, femora, spine, ribs, and skull.

386

K30031_Book.indb 386 9/6/17 1:38 PM


Case 8.14: Answers

Fig. 8.14E  Sagittal T1-weighted


MR image of the lumbar spine. Note:
the vertebrae are all markedly low
signal – black – in keeping with sclerotic
metastatic infiltration.

LEARNING POINTS: METASTATIC BREAST CANCER,


SCLEROTIC BONE LESIONS
▪▪ When you see a woman aged over 40 years with a history of breast cancer and multiple bone
lesions, think metastatic breast cancer.
▪▪ Potential complications include pain, pathological fracture, neural impingement, and symptoms
relating to hypercalcaemia.
▪▪ A radionuclide bone scan, staging CT, and MRI spine may also be useful.
▪▪ In the exam (and real life!) remember to recommend urgent discussion at the relevant cancer MDT
meeting.

387

K30031_Book.indb 387 9/6/17 1:38 PM


Case 8.15

Fig. 8.15A  AP XR of the


right wrist.

A 68-year-old female presents to the ED having slipped on an icy pavement. She describes having
fallen on her outstretched right hand with subsequent pain and swelling at the wrist.
On examination there is soft tissue swelling over the dorsum of the wrist with focal bony
tenderness over the distal radius. Careful palpation of the carpal bones and proximal forearm and
elbow reveal no further areas of tenderness. There are no signs of neurovascular compromise.
An XR of the wrist is arranged (Figures 8.15A and 8.15B).

388

K30031_Book.indb 388 9/6/17 1:38 PM


Case 8.15: Questions

Fig. 8.15B  Lateral XR of the right wrist.

CASE 8.15: QUESTIONS


1 Describe the XR findings.
2 What is the diagnosis?
3 What do you think of her bone mineral density?
4 What is happening at the base of the thumb?
5 How would you manage this patient?

389

K30031_Book.indb 389 9/6/17 1:38 PM


8  Musculoskeletal cases

CASE 8.15: ANSWERS


1 Describe the XR findings.
There is an extra-articular fracture of the distal radius with dorsal displacement and angulation.
The medial fracture line passes proximal to the distal radial ulnar joint (DRUJ). There is also a
minimally displaced avulsion fracture of the ulnar styloid tip (Figures 8.15C and 8.15D).

• As with any fracture, it is worth commenting on degree of displacement, angulation,


impaction, and shortening.

Fig. 8.15C  AP XR of the


wrist with a Colles fracture
of the distal radius (line A)
and additional fracture of the
ulnar styloid tip (line B). OA
is present at the base of the
thumb (C).

390

K30031_Book.indb 390 9/6/17 1:38 PM


Case 8.15: Answers

Fig. 8.15D  Lateral XR of the wrist


with a Colles fracture (C) showing
dorsal angulation of the distal radial
fracture fragment and some loss
of palmar tilt. The palmar tilt angle
is formed between a line drawn
perpendicular to the long axis of the
radius (B) and a second line drawn
tangentially to the radial articular
surface (A). In women the palmar
tilt angle is about 12°; this will be
reduced in compression fractures
with dorsal displacement of the
distal radial fragment. Loss of palmar
tilt is also indicative of likely loss of
function later on.

391

K30031_Book.indb 391 9/6/17 1:38 PM


8  Musculoskeletal cases

• Assessment of a wrist fracture must also include a description of the distal ulna and
distal radioulnar joint (DRUJ). It is important to state whether or not there is articular
communication of the fracture as this may indicate instability (high risk of secondary
displacement). Subluxation or dislocation of the ulnar head as a result of avulsion of the
base of the ulnar styloid, or tear of the TFCC or capsular ligaments will need reduction to
avoid chronic instability. Avulsions of the tip of the ulnar styloid, as demonstrated in this
case, however, are stable. Ulnar styloid fractures are present in up to 50% of cases.
• It is also important to scrutinise for evidence of intra-articular radiocarpal involvement
as this may also indicate instability and lead to post-traumatic OA of the wrist. If you are
unsure you could recommend a CT scan to look for intra-articular (radiocarpal or DRUJ)
involvement and subluxation.
• Other signs of instability include radial shortening (where the distal radius is more proximal
than the distal ulna), loss of radial height, and loss of radial inclination (Figure 8.15E).

Radial height: two lines are drawn perpendicular to the radial shaft, one along the articular surface
and one along the radial styloid tip (T). The distance between them should measure >12 mm and
is reduced to <9 mm in distal radial fractures.

Radial inclination: is the angle of the distal radial surface with respect to a line perpendicular to
the shaft (A). A normal slope should be 15–25° and is also reduced when fractured.

Fig. 8.15E  AP XR of the wrist with annotations to illustrate radial height and radial inclination measurements.

392

K30031_Book.indb 392 9/6/17 1:38 PM


Case 8.15: Answers

2 What is the diagnosis?


These features are diagnostic of a Colles fracture.
Classic features of a Colles fracture are:

• Transverse fracture of the distal metaphysis of the radius.


• Fracture is proximal to the DRUJ.
• Dorsal displacement and dorsal angulation (‘dinner fork deformity’).

A Smith’s fracture is a reverse Colles; namely there is palmar angulation of the distal radial
fracture fragment (caused by a fall on a flexed wrist).

3 What do you think of her bone mineral density?


The bone density appears normal.

• This is important to comment on, even if normal as in this case, as it demonstrates an


understanding of the pathogenesis. Colles fracture usually occurs following a fall on an
outstretched hand (FOOSH) in elderly postmenopausal women owing to decreased bone
mineral density.
• A low energy fracture in the elderly may reflect previously undiagnosed osteoporosis or
other metabolic bone disease. Take the opportunity to recommend a DEXA scan to assess
bone fragility.

4 What is happening at the base of the thumb?


There is joint space loss and subchondral sclerosis at the thumb CMC joint, in keeping with OA.
This is a common location for OA in the hand.

5 How would you manage this patient?


First check that the patient has been examined for:

• Acute carpal tunnel syndrome (weakness or altered sensation in the thumb or index finger).
• Vascular injury (rare).
• Concomitant injury to the ulnar side of the wrist, the elbow, and the carpal bones (‘Always
assess the joints above and below the known injury’.)

The fracture can then be managed in the ED with closed reduction and cast immobilisation
using a ‘backslab’. Analgesia and sedation will be required.
If there are signs of instability (fractures that are displaced, comminuted, or articular), special-
ist orthopaedic opinion is required with a view to open reduction and internal fixation (ORIF)
surgery.

LEARNING POINTS: COLLES FRACTURES


▪▪ Colles fractures are extra-articular fractures of the distal radius with dorsal angulation.
▪▪ Check for fracture extension into the radiocarpal joint and/or distal radioulnar joints as these will
effect stability.
▪▪ Check for an ulnar styloid fracture.
▪▪ Always comment on bone mineral density.
▪▪ Remember to also assess the elbow and the carpal bones for injury.

393

K30031_Book.indb 393 9/6/17 1:38 PM


Case 8.16

Fig. 8.16A  AP view of


the wrist.

A 45-year-old female attends the ED having had a FOOSH that day. She has pain and swelling of
the right wrist, which was exacerbated when she attempted to grip the steering wheel of her car.
Examination confirms marked swelling of the radial aspect of the right wrist. The patient com-
plains of pain on deep palpation of the anatomical snuff-box.
XRs of the wrist are requested (Figures 8.16A and 8.16B).

394

K30031_Book.indb 394 9/6/17 1:38 PM


Case 8.16: Questions

Fig. 8.16B  Lateral view of


the wrist.

CASE 8.16: QUESTIONS


1 What abnormality is demonstrated?
2 How should this patient be managed?
3 What complications may arise from this injury?
4 Is any further imaging required?
5 Can you identify the carpal bones?

395

K30031_Book.indb 395 9/6/17 1:38 PM


8  Musculoskeletal cases

CASE 8.16: ANSWERS


1 What abnormality is demonstrated?
There is a displaced fracture of the waist of the scaphoid bone. The other carpal bones are intact
and normally aligned (Figure 8.16C).

• Usually when a scaphoid fracture is suspected a dedicated ‘scaphoid series’ is performed:


this comprises AP, lateral, pronated oblique, and supinated oblique views.
• Up to 25% of scaphoid fractures are occult on initial XR. Therefore, if no fracture is seen the
patient should still be managed as if a fracture is present and an MRI arranged.

Fig. 8.16C  AP view of the wrist with scaphoid waist fracture (A). Note the distal (B) and proximal (C)
fracture fragments.

396

K30031_Book.indb 396 9/6/17 1:38 PM


Case 8.16: Answers

• Transverse fractures through the waist (middle one-third) of the scaphoid are most
common, and result from compression of the scaphoid against the distal radius (such as
occurs with FOOSH).
• A minority of patients with a scaphoid fracture have either a dislocation or fracture
elsewhere. Figures 8.16D and 8.16E show a scaphoid waist fracture with lunate dislocation.
This is a rare but serious injury requiring surgical management (trans-scaphoid perilunate
dislocation).

2 How should this patient be managed?


The patient should be given adequate analgesia, the wrist immobilised, and an appointment
made for review in the fracture clinic.

• The thumb and wrist is immobilised with a below-elbow cast.


• The patient may be reviewed in clinic 2 weeks later with repeat XR. If the fracture is
confirmed the cast is reapplied for a further 6–8 weeks. A further XR is taken to check for
bone union.

3 What complications may arise from this injury?


Avascular necrosis, nonunion, and osteoarthritis are potential complications of scaphoid fracture.

• The blood supply to the scaphoid enters the bone at the distal one-third. Displaced fractures
of the scaphoid waist risk interrupting the blood supply to the proximal fragment and may
result in avascular necrosis. The proximal fragment may appear sclerotic on follow-up XR.
This is managed surgically with a bone graft.
• Nonunion is the failure of a fracture to heal. OA, pain, and limitation of movement may
result.

4 Is any further imaging required?


Further imaging is used in equivocal fractures, to demonstrate avascular necrosis not demon-
strated on XR, or to plan for surgery.

• MRI is the most sensitive tool for diagnosing scaphoid fractures and complications and is
increasingly being used in place of XR.
• Bone fractures appear as a low signal (dark) line on T1 imaging.
• A bone affected by avascular necrosis will appear as diffuse low T1 signal (dark) owing to
replacement of the normal fatty bone marrow.

5 Can you identify the carpal bones?


For nomenclature of the carpal bones see Figure 8.16D.

397

K30031_Book.indb 397 9/6/17 1:38 PM


8  Musculoskeletal cases

Figs. 8.16D, E  AP (8.16D) and lateral (8.16E) XRs of the wrist in another patient showing a displaced
fracture of the scaphoid (A) with palmar rotation of the proximal fragment and lunate and dorsal migration
of the capitate. On the lateral view the capitate can be seen to have slipped posteriorly (F) and the lunate
has tipped forwards (B). This is a serious injury and requires urgent surgical management. This is a trans-
scaphoid perilunate dislocation – on the lateral view the carpus can be seen to have dislocated posteriorly
in relation to the lunate/radius. The lunate has slipped/rotated a little and looks triangular on the AP view.
Carpal bones: scaphoid (+ fracture) (A), lunate (B), triquetral (C), pisiform (D), hamate (+ hook) (E), capitate
(F), trapezoid (G), trapezium (base of the thumb) (H).

398

K30031_Book.indb 398 9/6/17 1:38 PM


Case 8.16: Answers

Fig. 8.16E

LEARNING POINTS: SCAPHOID FRACTURE


▪▪ Scaphoid fracture is a common injury resulting from a FOOSH – it is frequently occult on the
standard XR series.
▪▪ Scaphoid fracture should be suspected where there is an appropriate clinical history and
anatomical snuffbox tenderness.
▪▪ If there is adequate clinical suspicion but a normal XR, the patient may either proceed to MRI or
be managed as a fracture, reviewed in fracture clinic in 2 weeks, and a repeat XR taken.
▪▪ Avascular necrosis and nonunion are recognised complications of scaphoid fractures that require
operative management.

399

K30031_Book.indb 399 9/6/17 1:38 PM


Case 8.17

Fig. 8.17A  AP XR of the lumbar spine.

A 65-year-old male with a long history of back pain and stiffness comes to see his GP with wors-
ening lower back pain, which came on as the patient attempted to stand up.
Examination reveals some generalised tenderness over the lumbar spine but neurological
examination is normal. The patient is referred for XR of the lumbar spine (Figures 8.17A and
8.17B).

400

K30031_Book.indb 400 9/6/17 1:38 PM


Case 8.17: Questions

Fig. 8.17B  Lateral XR of the


lumbar spine.

CASE 8.17: QUESTIONS


1 What are the key XR findings?
2 What is the underlying diagnosis?
3 What other imaging may be helpful?
4 What are the complications of this condition?

401

K30031_Book.indb 401 9/6/17 1:38 PM


8  Musculoskeletal cases

CASE 8.17: ANSWERS


1 What are the key XR findings?
The sacroiliac joints are not clearly visible (Figures 8.17C and 8.17D), with a thin sclerotic line
only present. The joints have fused.
Abnormalities are also seen in the lumbar spine:
• Vertebral body squaring.
• Diffuse interspinous ligament calcification, ‘dagger spine’.
• Diffuse syndesmophyte ankyloses, ‘bamboo spine’.
• Diffuse ossification of spinal ligaments.

Figs. 8.17C, D  AP (8.17C) and lateral (8.17D) XRs of the lumbar spine demonstrating sclerosis and fusion
of the sacroiliac joints (A). There is interspinous ligamentous calcification (arrows B) ‘dagger spine’ and also
flowing syndesmophyte calcification and ankylosis seen on both views (arrows C, ‘bamboo spine’). No
fractures are visible.

402

K30031_Book.indb 402 9/6/17 1:38 PM


Case 8.17: Answers

Fig. 8.17D

2 What is the underlying diagnosis?


The diagnosis is ankylosing spondylitis. The fused sacroiliac joints indicate endstage disease.
Earlier in the disease process the joints may widen secondary to active inflammation but will then
become progressively more erosive and sclerotic.
Ankylosing spondylitis is a seronegative spondyloarthropathy with a male predilection
(male:female = 3:1), which usually manifests in the third decade; 90% of Caucasian patients are
HLA-B27 positive. The disease commonly involves the spine and sacroiliac joints but may also

403

K30031_Book.indb 403 9/6/17 1:38 PM


8  Musculoskeletal cases

affect other small and large joints. Sacroiliitis, symmetrical and bilateral, is the commonest first
manifestation. Early spondylitis in the spine is associated with small erosions at the corners of
vertebral bodies, with sclerosis as they heal (Romanus lesions). Progressive erosion leads to scle-
rosis, fusion, and ankylosis.

3 What other imaging may be helpful?


MRI is the technique of choice for demonstrating the early changes of sacroiliitis (Figure 8.17E).
CT is useful in the assessment of spinal fractures to which these patients are prone. Bone scintig-
raphy has a role in equivocal cases.

Fig. 8.17E  MRI of the sacrum in another patient with early stage ankylosing spondylitis. This is a STIR
sequence (short tau inversion recovery): do not worry overly about the physics or nature of the sequence, it
is designed to highlight fluid/oedema and is very useful in assessing early joint inflammatory changes. This
image is in the coronal plane and shows the sacroiliac joints (A). The joints have rather irregular margins
and there is significant adjacent high signal (white) oedema change adjacent to the joints, indicative of
active inflammation (B).

404

K30031_Book.indb 404 9/6/17 1:38 PM


Case 8.17: Answers

4 What are the complications of this condition?


Ankylosing spondylitis is associated with a number of potentially serious complications.

• The diffusely ossified spine is brittle and highly susceptible to pathological fracture.
Fractures are often highly unstable and can occur with only minimal trauma. Cervical
spine fractures are particularly hazardous and can cause cord compression/transection.
• Other associations include:
• Anterior uveitis.
• Inflammatory bowel disease.
• Interstitial lung fibrosis (1% of patients, predilection for upper lobes).
• Aortic valve disease and aortitis.
• Amyloidosis.

LEARNING POINTS: ANKYLOSING SPONDYLITIS


▪▪ Ankylosing spondylitis is a cause of low back pain, which can radiate to the lower limbs or groin.
▪▪ The condition is more common in males and is associated with HLA-B27 positivity.
▪▪ MRI is the imaging technique of choice, it is nonionising, and can detect early stage sacroiliitis,
▪▪ The ankylosed spine is brittle and prone to unstable, pathological fracture.

405

K30031_Book.indb 405 9/6/17 1:38 PM


K30031_Book.indb 2 9/6/17 1:34 PM
Neurology cases

9 VINCENT HELYAR AND EDWARD SELLON

Case 9.1 408 Case 9.6 433


Case 9.2 413 Case 9.7 440
Case 9.3 417 Case 9.8 446
Case 9.4 423 Case 9.9 450
Case 9.5 427 Case 9.10 455

407

K30031_Book.indb 407 9/6/17 1:38 PM


Case 9.1

Fig. 9.1A  Unenhanced (noncontrast) axial CT image of the brain at the level of the frontal horns of the
lateral ventricles.

408

K30031_Book.indb 408 9/6/17 1:38 PM


Case 9.1: Questions

A 76-year-old male presents to the ED with new-onset slurred speech and right-sided facial and
limb weakness. He lives on his own in a warden-controlled flat and has had a series of falls,
according to his daughter.
On examination, there is no sign of a head injury and the patient is alert, although mildly
disorientated. There is, however, a clear motor weakness on the right side of the body affecting
the face, arm, and leg. There is also expressive dysphasia. Routine blood tests are unremarkable.
A CT scan of the head is arranged (Figure 9.1A).

CASE 9.1: QUESTIONS


1 What are your provisional differential diagnoses given the history and examination
findings?
2 What findings are shown in Figure 9.1A?
3 How should this patient be managed?

409

K30031_Book.indb 409 9/6/17 1:38 PM


9  Neurology cases

CASE 9.1: ANSWERS


1 What are your provisional differential diagnoses given the history and examination
findings?
The main differential diagnoses to consider are transient ischaemic attack/stroke and intracranial
haemorrhage. The focal neurological signs are nonspecific.

• In order to determine the diagnosis and to rule out haemorrhage, a CT scan of the head
is required. The presence of focal neurology indicates this scan should be performed
urgently.
• The CT scan will be initially performed without IV contrast (‘unenhanced’) to allow
detection of acute haemorrhage, which may be obscured by contrast.

2 What findings are shown in Figure 9.1A?


CT shows a shallow, hyperdense extra-axial (i.e. not within neuroaxis or brain parenchyma)
collection overlying the left cerebral hemisphere (Figure 9.1B). The appearance is typical for a

Fig. 9.1B  ​
Unenhanced axial
CT of the brain
demonstrating a
thin hyperdense
collection overlying
the left cerebral
hemisphere (A)
and extending
along the posterior
falx cerebri
(B) in keeping
with subdural
haematoma. There
is also some mass
effect, with midline
and ventricular
shift to the right.
Note also the
sulcal effacement
of the left cerebral
hemisphere,
and the normal
right sulci.

410

K30031_Book.indb 410 9/6/17 1:38 PM


Case 9.1: Answers

subdural collection. Note: it has a concave inner margin, not biconvex as is seen in extradural
haematoma, it overlies most of one hemisphere, is not contained by the sutures of the cranium,
and pushes away the brain parenchyma in an undulating fashion.

• Acute intracranial blood appears white, hyperdense on CT. As blood matures


it becomes isodense to brain by 7–10 days, and by about 1 month blood
methaemoglobin converts to haemosiderin and appears dark and hypodense
when compared with the brain. The appearances may be confusing as rebleeds
are common in subdural haematoma and acute blood can mix with older blood
products (Figure 9.1C).
• Subdural haematoma is common and often caused by laceration of the superficial
bridging cortical veins. The risk of haemorrhage is increased in those with reduced brain
volume (e.g. old age, chronic alcohol abuse, dementia) owing to stretching of the bridging
veins.
• Subdural haematoma may be spontaneous; however, it is often precipitated by trauma or
anticoagulation.

Fig. 9.1C  ​
Unenhanced
axial CT in
another patient
showing
maturation
of a subdural
haematoma (A).
This left subdural
haematoma is
also associated
with mass effect
but note the
mixed density;
it is mature,
hypodense but
contains internal
high density
rebleeds (B).

411

K30031_Book.indb 411 9/6/17 1:38 PM


9  Neurology cases

3 How should this patient be managed?


These findings require urgent action. You should seek senior advice and speak to a neurosurgeon
for their opinion.

• The mortality of subdural haematoma in this age group is about 65%.


• The morbidity is also high with many patients being unable to return to their previous level
of function, especially if the haematoma was severe enough to warrant surgery.
• Craniotomy may be considered where the depth of acute blood measures >5 mm or where
there is significant mass effect.

LEARNING POINTS: SUBDURAL HAEMATOMA


▪▪ Subdural haematoma may follow minor head trauma and is more common in the elderly or those
with a history of alcohol abuse.
▪▪ The presence of mass effect and midline shift may require urgent neurosurgical intervention.
▪▪ Rebleed is common with subdural haematoma. Acute and chronic blood products give a mixed
density appearance (i.e. areas of high density acute blood with low density old blood).

412

K30031_Book.indb 412 9/6/17 1:38 PM


Case 9.2

Fig. 9.2A  Axial


unenhanced CT of the
brain at the level of the
bodies of the lateral
ventricles.

A 22-year-old male is brought unconscious to the ED by ambulance in the early hours of a Sunday
morning. The patient smells strongly of alcohol and was found by passers-by at the bottom of a
flight of stairs. No further history is available.
Initial examination reveals a Glasgow Coma Scale (GCS) of 8 (E2 V2 M4) (E = eye opening;
V = verbal response; M = motor response) and signs of an injury to the back of the head. While the
anaesthetic registrar intubates the patient you expose the patient to complete your examination.
You palpate the occiput and find a large swelling, which feels ‘boggy.’ The patient is referred for
CT (Figure 9.2A), intubated, and his cervical spine immobilised prior to imaging.

CASE 9.2: QUESTIONS


1 What is your provisional clinical 3 What is the pathophysiology for this
diagnosis? What additional imaging abnormality?
should you request? 4 What will you do next?
2 What is shown in Figure 9.2A?

413

K30031_Book.indb 413 9/6/17 1:38 PM


9  Neurology cases

CASE 9.2: ANSWERS


1 What is your provisional clinical diagnosis? What additional imaging should you
request?
The history is of a head injury in a young, intoxicated patient with resulting severe impairment of
consciousness. Your first thought should be that of an intracranial haemorrhage.
• The boggy swelling overlying the occiput most likely represents a haematoma. A depressed
skull fracture could also feel boggy.
• An emergency CT scan of the brain and cervical spine are required; these should be
reported urgently.

Fig. 9.2B  ​Unenhanced axial CT scan of the brain showing hyperdense left frontal lobe parenchymal
haemorrhage (A) with associated oedema and mass effect. The bodies of the lateral ventricles are
distorted and displaced to the right (B). There is some compression of the body of the left lateral ventricle
and dilatation of the body of the right lateral ventricle. Shallow left posterior subdural haematoma (C) and
overlying soft-tissue swelling (D) are also demonstrated.

414

K30031_Book.indb 414 9/6/17 1:38 PM


Case 9.2: Answers

• Imaging of the cervical spine is mandated owing to the impaired consciousness and
mechanism of injury. Further CT imaging of the rest of the body would be advised if the
patient had fallen from a significant height (e.g. >5 steps).

2 What is shown in Figure 9.2A?


Figures 9.2A and 9.2B show extensive hyperdense material in the left frontal lobe representing
acute intra-axial (i.e. parenchymal) haemorrhage.

• There is adjacent oedema and significant mass effect, which causes partial effacement of
the anterior horn of the left lateral ventricle.
• There is also hyperdense material posteriorly, which overlies the left occipital lobe,
consistent with an acute subdural haematoma.
• There is superficial soft-tissue swelling overlying the left occipital bone, which is the
swelling felt when palpated (Figure 9.2C).

Fig. 9.2C  ​
Unenhanced
axial CT scan
of the brain
slightly inferior
to that in 9.2B
displayed on a
bone window
setting. Note the
occipital bone
fracture (A).
Normal sutures
(B) should not
to be confused
with fractures.

415

K30031_Book.indb 415 9/6/17 1:38 PM


9  Neurology cases

3 What is the pathophysiology for this abnormality?


The pattern of injury is the result of trauma to the back of the head. This is demonstrated by the
site of the fracture and soft-tissue swelling.

• The pattern of parenchymal injuries is known as ‘coup’ (same side of the brain as the
trauma) and ‘contrecoup’ (on the opposite side of the brain to the trauma).
• Coup injury is caused by the direct effect of the trauma, while contrecoup is usually the
result of brain shaking.
• Parenchymal haemorrhage arising from trauma typically occurs where the brain is thrown
against bone, most commonly seen in the inferior frontal and temporal lobes.

4 What will you do next?


The patient should be discussed urgently with the neurosurgical team with a view to transfer to
a neurosurgical unit for further management. This may involve surgical decompression of the
haematoma and also shunting of the lateral ventricles.

LEARNING POINTS: INTRACEREBRAL HAEMATOMA


▪▪ When you spot an intracranial haemorrhage, comment on any mass effect and search for an
underlying fracture, or possible lesion if no history of trauma.
▪▪ The most common sites of intra-axial haemorrhage following head injury are the inferior frontal
lobes and inferior temporal lobes.
▪▪ The site of the haematoma may be on the same (coup) or opposite (contrecoup) side to the site of
trauma.

416

K30031_Book.indb 416 9/6/17 1:38 PM


Case 9.3

Fig. 9.3A  Precontrast axial CT at the level of the frontal lobes.

A 56-year-old female visits her GP with a long history of headache and visual disturbance. More
recently her son has noticed some short-term loss of memory and erratic behaviour. The patient is
concerned that she might have dementia.
Your examination reveals near blindness in the right eye and diffuse weakness on the left side
of the body. There is evidence of both anterograde and retrograde amnesia.
Routine blood tests are unremarkable. The patient is referred for urgent CT of the brain
(Figures 9.3A–9.3C).

417

K30031_Book.indb 417 9/6/17 1:38 PM


9  Neurology cases

Fig. 9.3B  Postcontrast axial CT at same level as 9.3A.

418

K30031_Book.indb 418 9/6/17 1:38 PM


Case 9.3: Questions

Fig. 9.3C  Precontrast midline brain sagittal CT reconstruction.

CASE 9.3: QUESTIONS


1 What differential diagnoses would you consider from the history and examination?
2 What other imaging tests are indicated?
3 How would you describe the abnormality shown in Figures 9.3A–9.3C? What is the normal
structure ‘C’?
4 Do you think this pathology has developed slowly or quickly?
5 How should this patient be managed?

419

K30031_Book.indb 419 9/6/17 1:38 PM


9  Neurology cases

CASE 9.3: ANSWERS


1 What differential diagnoses would you consider from the history and examination?
The main concerns are those of a brain tumour (primary or secondary), while dementia, stroke,
and chronic subdural haematoma are also possibilities.
• The presence of focal neurology makes dementia less likely.
• The insidious onset of symptoms makes a stroke less likely.
2 What other imaging tests are indicated?
A CT scan of the brain is usually requested in the first instance, especially if stroke or bleed are
in the differential diagnosis. If tumour is suspected CT should be performed before and after an
IV infusion of contrast. A precontrast scan is required to assess for the presence an infarct or, if a

Figs. 9.3D, E  Precontrast (9.3D) and postcontrast (9.3E) CT scans of the brain. There is a large and
rounded lesion in the frontal region to the right of the midline (A); on the sagittal image this is seen to arise
from the floor of the anterior cranial fossa (Figure 9.3C). Precontrast this lesion is mildly hyperdense – note
the adjacent low density tumoural type oedema (B) – and the lesion enhances avidly and homogeneously
postcontrast.
420

K30031_Book.indb 420 9/6/17 1:38 PM


Case 9.3: Answers

lesion is present, to assess lesional haemorrhage/calcification and to compare with the postcon-
trast images to see if there is lesional enhancement.
A contrast-enhanced MRI scan may also be requested (usually following specialist neurol-
ogy review). This would be a useful investigation if the CT scan is normal (as it is more sensitive
for hyperacute strokes and small mass lesions than CT) or to further characterise a lesion if the
CT scan is positive. If CT and MRI demonstrate possible intracranial malignancy, then further
imaging via CT chest/abdomen and pelvis may be undertaken to look for a possible primary
tumour.

3 How would you describe the abnormality shown in Figures 9.3A–C? What is the
normal structure ‘C’?
There is a large, circumscribed mass lesion in the right frontal region with surrounding perile-
sional oedema (Figures 9.3D and 9.3E). There is shift of midline structures to the left but no
significant effacement of the CSF spaces on the left. The mass shows homogeneous enhancement
after IV contrast (it appears more dense in Figure 9.3E than 9.3D).

• The lesion arises from the floor of the anterior cranial fossa. The lesion is therefore likely to
be extra-axial (i.e. not arising from the brain parenchyma itself).

E
Fig. 9.3E

421

K30031_Book.indb 421 9/6/17 1:38 PM


9  Neurology cases

• Structure ‘C’ (Figure 9.3C) is the straight sinus overlying the tentorium cerebelli, below
which lies the cerebellum. Anything above the tentorium cerebelli is described as being
‘supratentorial’ in location.

4 Do you think this pathology has developed slowly or quickly?


The extent of oedema (relatively small amount) in relation to the size of the lesion may suggest
that this is a low-grade tumour (i.e. slow growing). The patient history supports this.

• Homogeneous contrast enhancement is also more often a feature of slow growing lesions.
Fast growing lesions tend to outstrip neovascularisation, so parts of the tumour become
ischaemic and then necrotic. This produces a heterogeneous pattern of enhancement
postcontrast.
• Areas of necrosis are low density (dark) on CT.
• Overall, the appearance of this lesion is typical for a meningioma.

Meningiomas are the second most common primary brain tumour after glioma. They are more
common in women aged 50–60 years.

• 95% of meningiomas are low-grade (benign) tumours and 90% are supratentorial.
• They may cause thickening of the adjacent bone (hyperostosis) and a dural tail of
enhancement is a typical feature.

5 How should this patient be managed?


The patient should be referred urgently to a specialist neurosurgical centre for discussion at a
neuro-oncology MDT meeting.

• The MDT meeting would include a neuroradiologist, a neurosurgeon, a neurooncologist,


a neuropathologist, and specialist nurses.
• The purpose of the MDT meeting is to agree a management plan. This might include
conservative measures (e.g. follow-up imaging) or active treatment (e.g. surgery to remove
the tumour) depending on the age, condition, and wishes of the patient.

LEARNING POINTS: BRAIN TUMOUR, MENINGIOMA


▪▪ Postcontrast CT scans of the brain are useful to diagnose brain tumours and brain abscesses.
A precontrast scan is also performed for comparison and to exclude stroke or acute blood.
▪▪ Low-grade tumours tend to have a more homogeneous enhancement pattern while high-grade
tumours enhance heterogeneously.
▪▪ A meningioma is a relatively common, low-grade, extra-axial tumour associated with hyperostosis
and a dural tail of enhancement.

422

K30031_Book.indb 422 9/6/17 1:38 PM


Case 9.4

Fig. 9.4A  ​
Unenhanced
CT brain at
the level of the
basal cisterns
and frontal
horns of the
lateral ventricles
performed
at the time of
presentation to
hospital.

A 45-year-old male arrives in an ambulance intubated to the ED with a history (from his wife) of
prior sudden-onset ‘thunderclap’ headache. This had been followed by several episodes of vom-
iting, a seizure, and loss of consciousness. A collateral history reveals that the patient receives
haemodialysis three times a week owing to end stage renal failure caused by ‘kidney cysts’. There
is also a history of hypertension.
On examination you find brisk reflexes and severe impairment of consciousness with GCS 6
(E2 V2 M2). Baseline observations are stable and he is apyrexial. There is neck rigidity and up-
going plantar reflexes. You request a CT scan of the brain (Figure 9.4A).

CASE 9.4: QUESTIONS


1 What does the clinical scenario suggest? 3 What further investigations might you
2 What is the diagnosis as shown in consider?
Figure 9.4A? 4 How should this patient be managed?

423

K30031_Book.indb 423 9/6/17 1:38 PM


9  Neurology cases

CASE 9.4: ANSWERS


1 What does the clinical scenario suggest?
Sudden onset of severe headache is suggestive of a subarachnoid haemorrhage in this patient.
This provisional diagnosis is supported by subsequent events (collapse, vomiting, and seizures),
which suggest raised intracranial pressure (ICP).

• ‘Kidney cysts’ are also a clue – these are clearly severe as the patient is on dialysis and likely
represent polycystic kidney disease (PKD), which is associated with intracerebral ‘berry’
aneurysms in up to 7% of cases.
• 85% of saccular aneurysms occur in the anterior circulation. Saccular aneurysms (berry
aneurysms) are the most common type of cerebral aneurysm and appear as a round
outpouching of the vessel.
• Only 50% of patients with an aneurysmal subarachnoid haemorrhage are alive at 6
months.

2 What is the diagnosis as shown in Figure 9.4A?


CT shows an extensive subarachnoid haemorrhage (hyperdense fresh blood) lying within the
basal cisterns and the sulcal spaces of both hemispheres (Figure 9.4B).

• Also, the CT section shows dilatation of the temporal horns of the lateral ventricles
indicative of early hydrocephalus. Hydrocephalus in this instance is most likely caused by
obstruction of CSF flow by blood products.
• Smaller subarachnoid bleeds may be occult on CT (10%). Such cases should be considered
for lumbar puncture and CSF examination for xanthochromia where appropriate, assuming
no clinical evidence of raised ICP, which may be underestimated on CT.

3 What further investigations might you consider?


Further imaging is required to look for the underlying cause of haemorrhage. The history of PKD
suggests that the likely cause here is an underlying intracranial aneurysm.

• A CT angiogram of the intracerebral vessels should be considered in order to demonstrate


the location of the aneurysm.
• MR angiography is also used in some cases.
• Further investigations should be performed in a centre that offers interventional
neuroradiology and neurosurgery, as part of the work-up prior to potential endovascular
coiling or surgical clipping.

Figure 9.5C shows an MR image of a cerebral aneurysm in another patient.

4 How should this patient be managed?


You should expect to receive an urgent verbal and then written report from the radiologist. Urgent
transfer to a neurosurgical centre is required.

• The patient is at risk of brain stem herniation (so called ‘coning’) due to cerebral oedema
and obstructive hydrocephalus. Hydrocephalus may be managed by placement of an
intraventricular shunt.

424

K30031_Book.indb 424 9/6/17 1:38 PM


Case 9.4: Answers

Fig. 9.4B  Unenhanced axial CT scan of the brain. There is extensive subarachnoid haemorrhage
with blood seen in the basal cisterns (B), Sylvian fissures (A), and within the sulci (C). Note: there is
also dilatation of the temporal horns of the lateral ventricles indicative of early hydrocephalus (D).
The third ventricle also appears dilated (usually slit-like) and contains a small focus of intraventricular
haemorrhage (E).

425

K30031_Book.indb 425 9/6/17 1:38 PM


9  Neurology cases

Fig. 9.4C  Axial MR angiogram image of the circle of Willis in another patient. Internal carotid (A), middle
cerebral (B), and posterior cerebral (C) arteries are shown, together with the left posterior communicating
artery (D) and anterior cerebral arteries (E). Note: aneurysm (F) arising from the right posterior communicating
artery (G).

• The risk of rebleed is nearly 20% in the first 14 days post haemorrhage and carries a high
mortality (80%). The risk of rebleeding is reduced by medical measures and by aneurysm
coiling or clipping.
• Aneurysm coiling is often preferred to open surgery. It is a procedure performed by
interventional neuroradiologists with access to the arterial vasculature via the common
femoral artery.
• 25% of those surviving a subarachnoid haemorrhage have a permanent neurological deficit.

LEARNING POINTS: SUBARACHNOID HAEMORRHAGE


▪▪ Subarachnoid haemorrhage is characterised by haemorrhage in the subarachnoid space (sulci
and CSF spaces).
▪▪ Small subarachnoid haemorrhages may be occult on CT. Check the posterior aspects of the
lateral ventricles for a small dense fluid level.
▪▪ Subarachnoid haemorrhage carries a high mortality and morbidity with a minority of patients able
to return to their previous level of functioning.

426

K30031_Book.indb 426 9/6/17 1:38 PM


Case 9.5

Fig. 9.5A  Sagittal T2-weighted image through the brain.

A 29-year-old female is referred to the neurology clinic complaining of several weeks of muscle
cramps and episodes of urinary incontinence. Your history reveals that the patient has also been
feeling more tired than usual recently. You notice that the patient suffered an episode of optic
neuritis 18 months ago but was lost to follow-up.
A neurological examination reveals mild loss of power in the lower limbs, more pronounced
on the left. There is increased tone on the left and plantars are up-going bilaterally. The reflexes
are brisk. The retinas have a slightly pale appearance on ophthalmoscopy but are otherwise
­unremarkable. Routine bloods performed by the GP are normal.

427

K30031_Book.indb 427 9/6/17 1:38 PM


9  Neurology cases

Fig. 9.5B  Axial FLAIR sequence through the brain at the level of the bodies of the lateral ventricles.

428

K30031_Book.indb 428 9/6/17 1:38 PM


Case 9.5: Questions

Fig. 9.5C  S​ agittal


T2-weighted image
of the cervical
spine.

CASE 9.5: QUESTIONS


1 What is your working diagnosis? What other investigations would you consider?
2 What do the MR images demonstrate (Figures 9.5A–9.5C)?
3 Where are the abnormalities in Figures 9.5A and 9.5B located? How would you describe these?
4 What additional abnormality is demonstrated in Figure 9.5C?
5 What is the likely course of this disease?

429

K30031_Book.indb 429 9/6/17 1:38 PM


9  Neurology cases

CASE 9.5: ANSWERS


1 What is your working diagnosis? What other investigations would you consider?
The clinical presentation is typical for demyelination as occurring in multiple sclerosis (MS),
an immune-mediated inflammatory disease that attacks myelinated axons in the CNS.
Characteristically the disease produces symptomatic episodes that occur months or years apart
and in varying anatomical locations. MRI is the imaging technique of choice for diagnosing
demyelinating lesions in the brain and spine, and for monitoring response to treatment.
A variety of other tests will be performed including visual evolved potentials and CSF exami-
nation on lumbar puncture for oligoclonal bands of IgG production. The diagnosis ultimately is
made clinically.

2 What do the MR images demonstrate (Figures 9.5A–9.5C)?


The MR images demonstrate multiple high signal lesions in the supratentorial white matter in a
periventricular distribution with further high signal lesions seen in the cervical cord (Figures 9.5D
and 9.5E). These appearances are typical for plaques of demyelination.

Fig. 9.5D  Lesions are shown in the corpus callosum (A) and thalamus (B). High signal CSF is seen in the
lateral ventricle (C) and around the cerebellum (D).

430

K30031_Book.indb 430 9/6/17 1:38 PM


Case 9.5: Answers

3 Where are the abnormalities in Figures 9.5A and 9.5B located? How would you
describe these?
The intracranial lesions labelled in this patient (Figure 9.5D) are found in a classic location for MS
on the underneath of the corpus callosum, known as the ‘callosal septal interface.’

• Typical for MS, there are multiple high T2 signal white matter lesions in a periventricular
distribution.
• The lesions are predominantly round, oval or flame shaped.

Fig. 9.5E  Magnified image of a T2-weighted section of the cervical cord showing a mildly expanded
cervical cord and a high signal focus of demyelination in the cord (A). Note: CSF around the cord appears
high signal, white on T2 weighting.

431

K30031_Book.indb 431 9/6/17 1:38 PM


9  Neurology cases

• The arrangement of multiple lesions along the underside of the corpus callosum is a classic
feature of MS known as ‘Dawson’s fingers’.
• The lesions represent foci of demyelination (destruction of the myelin sheath), known as
plaques. Foci of active inflammation may enhance after contrast administration.
• Aggressive MS, known as tumefactive MS, can resemble a primary brain tumour.

4 What additional abnormality is demonstrated in Figure 9.5C?


Figures 9.5C and 9.5E show focal high signal on T2 weighting located in the cervical cord, con-
sistent with a plaque of demyelination. The cervical cord is mildly expanded at this level owing
to oedema.

5 What is the likely course of this disease?


Modern diagnostic criteria for MS require the following:

• Objective evidence of two separate CNS lesions compatible with MS and separated by both
space and time.
• Other causes for CNS lesions to have been excluded.

A unilateral and painful optic neuritis is a common first presentation, as in this patient.
MS is of unknown aetiology, more common in women with increasing prevalence. It is thought
to be autoimmune with immune system malfunction destroying myelin.
The most common subtype of the disease is known as relapsing remitting MS, which accounts
for about 85% of cases.

• Treatment is aimed both at the underlying autoimmune disease involving the myelin
sheaths and at symptom control.
• Life expectancy is not shortened significantly by most forms of MS. However, rare
fulminant subtypes can be rapidly fatal.

LEARNING POINTS: MULTIPLE SCLEROSIS


▪▪ MS is a diagnosis made by combining clinical findings, imaging, and other tests (e.g. CSF
analysis, evoked potentials).
▪▪ MRI is also used for monitoring the disease and assessing response to treatment.
▪▪ The periventricular white matter is affected most commonly by MS plaques: the characteristic
MRI appearance is known as Dawson’s fingers.

432

K30031_Book.indb 432 9/6/17 1:38 PM


Case 9.6

Fig. 9.6A  Axial unenhanced CT image of the brain at the level of the frontal lobes.

433

K30031_Book.indb 433 9/6/17 1:38 PM


9  Neurology cases

Fig. 9.6B  Axial unenhanced CT image of the brain in the same patient at a more cranial (superior) section
(at the level of the frontal horns of the lateral ventricles).

434

K30031_Book.indb 434 9/6/17 1:38 PM


Case 9.6: Questions

A 26-year-old female is brought to the ED by helicopter following a head injury sustained by a


fall from a bicycle after a low speed collision with a car. Bystanders saw the patient fall on to the
right side of her head. She lost consciousness and a helicopter was dispatched owing to the remote
location.
The patient was awake by the time the helicopter arrived 20 minutes later, although slightly
disorientated (GCS 13). Initial assessment showed no cervical spine tenderness or focal neurology
and a small laceration to the right frontal region.
As the patient was being immobilised for transfer to the local trauma unit she became unre-
sponsive. Eye opening and limb flexion could be elicited only with a painful stimulus (GCS now 7).
She was intubated and transferred to hospital for further management. Urgent cranial CT is
undertaken (Figures 9.6A and 9.6B).

CASE 9.6: QUESTIONS


1 Assuming the patient is currently stable, what additional imaging would you consider?
What is your working diagnosis?
2 What key imaging findings are demonstrated in Figure 9.6A?
3 What important additional finding is illustrated in Figure 9.6B?
4 How should this patient be managed?

435

K30031_Book.indb 435 9/6/17 1:38 PM


9  Neurology cases

CASE 9.6: ANSWERS


1 Assuming the patient is currently stable, what additional imaging would you
consider? What is your working diagnosis?
The patient should undergo additional CT imaging at the same sitting as CT head. In the context
of major trauma, this would involve CT of the cervical spine and also the chest, abdomen, and
pelvis owing to the mechanism of injury (so-called ‘traumagram’).

• The principal concerns are those of intracranial haemorrhage and possible cervical spine
fracture.
• The lucid interval, which preceded significant drop in consciousness, is particularly
suggestive of an extradural haematoma.

2 What key imaging findings are demonstrated in Figure 9.6A?


CT shows a lenticular, biconvex-shaped hyperdense extra-axial (i.e. outside in the brain paren-
chyma) collection overlying the right frontal lobe (Figure 9.6C). The appearance is in keeping
with an acute extradural haematoma.
The hyperdense (hyperattenuating) nature of the haemorrhage means it is acute. As blood
products age they become less dense on CT and eventually become the same density as water or
CSF when chronic (weeks old). See Table 9.6.

• There is ‘mass-effect’ and sulcal effacement (the sulci are compressed and no longer
visualised) and the temporal horn of the right lateral ventricle is also effaced.
• 85% of extradural haematomas are associated with a fracture. Temporal bone fractures
classically tear the middle meningeal artery and cause arterial pressure haemorrhage
between the dura mater and the periosteum.

3 What important additional finding is illustrated in Figure 9.6B?


Figure 9.6B shows the ‘mass effect’ from the right frontal haematoma with a clear midline shift.
There is displacement of the anterior horns of the lateral ventricles to the left and the frontal horn
of the right lateral ventricle is partly effaced (Figure 9.6D).

• The haematoma displaces the entire right cerebral hemisphere. The cranium is essentially
a closed box (see the Monro–Kellie hypothesis), and the extra volume caused by the
haematoma inevitably effaces the CSF spaces (sulci, ventricles) and displaces structures,
initially across the midline (subfalcine herniation).
• As intracranial pressure increases further, the brain is pushed inferiorly through the
tentorium cerebelli (tentorial herniation). The uncus may also be displaced inferiorly (uncal
herniation).
• Uncal herniation is typically a preterminal event as vital brainstem function is
compromised.

4 How should this patient be managed?


These findings are a neurosurgical emergency. The reporting radiologist should telephone a
report to the requesting clinician and advise that an urgent neurosurgical opinion be sought.

• This patient is already intubated. All patients with head injury will need urgent ABCDE
assessment, IV access, bloods and monitoring, and urgent discussion with seniors.

436

K30031_Book.indb 436 9/6/17 1:38 PM


Case 9.6: Answers

Fig. 9.6C  ​A xial CT image of the brain without contrast showing a large right frontal (biconvex) lenticular-
shaped, acute extra-axial haemorrhage (A). There is mass effect with effacement of the temporal horn
of the right lateral ventricle. Note the mildly dilated temporal horn of the left lateral ventricle (B); the right
temporal horn is effaced.

Table 9.6  Hounsfield units (HU) – CT density


The inventor of CT, Sir Godfrey Hounsfield, gave his name to the CT unit of measurement of tissue
radiodensity
• The more a tissue attenuates the CT XR beam, the more dense and whiter it appears on CT
(e.g. bone, typically 700–3000 HU)
• The more easily the CT XR beam passes through tissue, the less dense and darker the tissue
appears (e.g. lung, typically –500 HU)
• The HU scale is centred on water, defined as 0 HU
Acute intracranial blood is hyperdense (i.e. more dense than water) and ranges from 40 to 80 HU

437

K30031_Book.indb 437 9/6/17 1:38 PM


9  Neurology cases

Fig. 9.6D  Axial CT image of the brain without contrast at a level slightly more superior than Figure 9.6C.
There is mass effect from the right frontal haematoma causing midline shift of the anterior horns of the
lateral ventricles to the left (A).

438

K30031_Book.indb 438 9/6/17 1:38 PM


Case 9.6: Answers

If a CT scan is indicated then there needs to be discussion with radiology and then the
radiographers in CT. It is essential that patients who go to CT are haemodynamically stable
and can maintain their airway – if the airway is, or may be, compromised, then urgent
anaesthetic involvement is needed.
• The patient must be transferred urgently to a neurosurgical centre for further management
(e.g. Burr hole to relieve intracranial pressure).

LEARNING POINTS: EXTRADURAL HAEMATOMA


▪▪ Extradural haematoma is life threatening and requires urgent neurosurgical intervention.
▪▪ It is typically a complication of a skull fracture and is characterised clinically by a lucid interval
before loss of consciousness.
▪▪ Acute blood is hyperdense or hyperattenuating on unenhanced CT: as a haematoma ages it
becomes less dense.
▪▪ Midline shift of ≥ 3 mm on CT is significant.

439

K30031_Book.indb 439 9/6/17 1:38 PM


Case 9.7

Fig. 9.7A  Unenhanced axial CT of the brain at the level of the pons.

A 67-year-old male presents to the ED with a history of sudden-onset left-sided weakness 3 hours
previously and altered sensation to the left side of his face. He gives a longstanding history of
hypertension and type 2 diabetes, and a 30 pack-year history of smoking.
On examination, there is a dense left hemiparesis and facial droop, also paraesthesia,
particularly on the left side of the face. The plantar reflex is up-going on the left. His BP is
180/110 mmHg, pulse 100 bpm and regular, and he is apyrexial. Urgent cranial CT is arranged
(Figures 9.7A and 9.7B).

440

K30031_Book.indb 440 9/6/17 1:39 PM


Case 9.7: Questions

Fig. 9.7B  Accompanying image from the same CT examination at the level of the frontal horns of the
lateral ventricles.

CASE 9.7: QUESTIONS


1 What is your provisional diagnosis?
2 How will you manage this patient?
3 What findings are demonstrated in Figures 9.7A and 9.7B?
4 What are the treatment options?

441

K30031_Book.indb 441 9/6/17 1:39 PM


9  Neurology cases

CASE 9.7: ANSWERS


1 What is your provisional diagnosis?
The symptoms and examination findings are suggestive of an ischaemic stroke (or transient
­ischaemic attack: TIA). Hypertension, smoking, and type 2 diabetes are all risk factors. The
differential diagnosis is intracerebral haematoma, although this is less likely given the lack of
­predisposing factors such as recent head injury or anticoagulant therapy.
2 How will you manage this patient?
The patient should be referred urgently to the stroke team. Providing there are no contraindica-
tions, the patient may be treated with IV thrombolysis. The stroke team will request an urgent
unenhanced CT scan of the brain.

• The purpose of the CT scan is principally to exclude intracerebral haematoma


(or haemorrhagic stroke) or a focal intracranial mass lesion. If there is no evidence of
haemorrhage or other contraindication, the patient may be eligible for thrombolysis.

Fig. 9.7C  Axial


unenhanced
CT showing
increased
density of the
right middle
cerebral artery
(MCA) (arrows A),
highly suggestive
for acute
thrombosis.
Note the normal
appearing left
MCA (B).

442

K30031_Book.indb 442 9/6/17 1:39 PM


Case 9.7: Answers

• The initial CT scan may show no signs of ischaemic stroke; indeed at 3–4 hours after a stroke
40% of patients have a normal CT scan. A normal scan, therefore, should not deter thrombolysis.
3 What findings are demonstrated in Figures 9.7A and 9.7B?
The CT scans show a hyperdense right middle cerebral artery (MCA). The hyperdensity is caused
by occlusive thrombus within the artery and is an early sign of ischaemic stroke (Figure 9.7C).
• The MCA is involved in 75% of infarcts, mostly due to atheroma rupture at or near the
bifurcation of the common carotid artery.

The second CT image (Figure 9.7B) does not show a definite focal lesion at this stage in MCA
distribution.
The infarcted brain parenchyma gradually reduces in density on CT over time owing to cell
death and liquefaction. Figure 9.7D shows the same patient 3 days after the infarction. The low
density corresponds to part of the vascular territory supplied by the MCA.
• Eventually (months after infarction), the infarcted brain tissue is absorbed by macrophages
and the affected area adopts a density similar to water or CSF. The loss of brain volume may
cause dilatation of an adjacent ventricle, known as ex-vacuo dilatation.

Fig. 9.7D  ​
Follow-up CT
performed
3 days later
showing diffuse
low density (A)
in keeping with
acute right MCA
infarction with
mass effect and
effacement of
the frontal horn
of the right lateral
ventricle (B).

443

K30031_Book.indb 443 9/6/17 1:39 PM


9  Neurology cases

Figs. 9.7E, F  A ​ xial


CT (9.7E) in another
patient at the level of
the lateral ventricles/
basal ganglia in a
patient with acute
left-sided weakness:
no definite lesion
is seen on CT. The
patient proceeded
urgently to MRI. A DW
image (9.7F) at the
same level as the CT
confirms an area of
high signal, restricted
diffusion, consistent
with an acute infarct
in the right basal
ganglia (A).
E

• MRI has superior sensitivity to CT for diagnosing hyperacute stroke and can add value in
this setting. Diffusion-weighted imaging (DWI) is the most sensitive and can be positive
within minutes of an infarct occurring (Figures 9.7E and 9.7F). Infarcted tissue with
restricted diffusion appears as high signal on DWI and normalises after about 4 days.

4 What are the treatment options?


Suspected strokes are best managed at hospitals with a hyperacute stroke service.

• IV thrombolysis (recombinant tissue plasminogen activator, r-TPA) is currently licensed for


patients with symptom onset of <4.5 hours, having been reviewed by a stroke specialist.
• After 4.5 hours, thrombolysis is associated with an increased mortality.
• CT or MR cerebral angiography can also be performed in hospitals with an interventional
neuroradiology service with a view to clot retrieval if clinically appropriate.
• Following confirmation of a stroke and initial management, the patient should be
transferred to a specialist stroke unit for further assessment and rehabilitation.

444

K30031_Book.indb 444 9/6/17 1:39 PM


Case 9.7: Answers

F
Fig. 9.7F

• If there is further deterioration in neurological status a follow-up CT scan can be performed


to look for haemorrhage, either as a complication of thrombolysis or from haemorrhagic
conversion of an ischaemic stroke.

LEARNING POINTS: STROKE


▪▪ Patients with suspected acute stroke must be referred urgently to the stroke team for
consideration of thrombolysis.
▪▪ CT is performed in the acute setting to exclude haemorrhage (an absolute contraindication to
thrombolysis) but is often normal early on.
▪▪ Diffusion-weighted MRI is the most sensitive imaging modality for hyperacute stroke. MRI is not
widely available for hyperacute stroke assessment and some stroke patients find it difficult to
tolerate.
▪▪ Review the CT for an area of low density (this may be very subtle) corresponding to a vascular
territory. Additional signs include mass effect or a hyperdense MCA.

445

K30031_Book.indb 445 9/6/17 1:39 PM


Case 9.8

Fig. 9.8A 

A 19-year-old female presents to the ED with a 2-day history of constant severe headache, nau-
sea, and several episodes of vomiting. The patient also complains of unsteadiness on her feet and
double vision. She has tried paracetamol to no avail. The patient is overweight (BMI 30) although
otherwise fit and well. She takes no medication other than the oral contraceptive pill.

446

K30031_Book.indb 446 9/6/17 1:39 PM


Case 9.8: Questions

Fig. 9.8B

Observations are stable and the patient is apyrexial. Neurological examination shows
a palsy of the left VI cranial nerve and cerebellar signs (Rhomberg test is positive).
Ophthalmoscopy shows early papilloedema. There is a mild impairment of consciousness
with GCS 14 (E4 V4 M6).

CASE 9.8: QUESTIONS


1 What is your working differential diagnosis? What imaging would you consider in this
situation?
2 What imaging has been performed in Figures 9.8A and 9.8B?
3 What normal structure is labelled ‘A’ and what abnormality is shown by ‘B’ (Figure 9.8A)?
4 How would you describe the abnormality labelled ‘C’? What normal structures are shown
by arrows D, E, and F (Figure 9.8B)?

447

K30031_Book.indb 447 9/6/17 1:39 PM


9  Neurology cases

CASE 9.8: ANSWERS


1 What is your working differential diagnosis? What imaging would you consider in
this situation?
The symptoms and signs are nonspecific. Differential diagnoses to be considered include menin-
goencephalitis, an intracranial mass, and venous sinus thrombosis.

• A CT scan of the brain pre and post-IV contrast is the most useful initial investigation
in the acute setting. It is more readily accessible than MRI and a normal CT scan would
rapidly and effectively exclude an intracranial mass.
• A CT venogram (postcontrast) can be performed easily at the same sitting if venous sinus
thrombosis is suspected. MRI and MR venography are also highly accurate for diagnosing
tumours and sinus thrombosis but are more time consuming and less well tolerated.

2 What imaging has been performed in Figures 9.8A and 9.8B?


Figure 9.8A is an axial CT slice of the brain from a postcontrast venogram examination.
Figure 9.8B is a selected sagittal image from a postcontrast MR venogram.

• CT is often performed first as it is readily accessible and helps to exclude other differential
diagnoses. CT can also be performed rapidly and may be easier for an ill patient to tolerate.
• Hyperdense dural sinuses on an unenhanced CT scan of the brain can suggest a dural
venous sinus thrombosis. Other features to support the diagnosis include cerebral oedema
or venous infarctions (these do not correspond to a vascular territory). A postcontrast study
will confirm a sinus filling defect/thrombus.
• There is little difference in diagnostic performance between a CT venogram and an MR
venogram for the investigation of sinus thrombosis, with sensitivities of approximately 95%
with each.
• MR venogram may, however, have added value where there are equivocal findings on CT or
in looking for subtle complications of a sinus thrombosis (i.e. venous infarction).

3 What normal structure is labelled ‘A’ and what abnormality is shown by ‘B’ (Figure 9.8A)?
Arrow A points to the anterior portion of the superior sagittal sinus. It is filled with contrast as expected
on this CT venogram and appears normal. Arrow B points to the posterior portion of the superior
­sagittal sinus. There is no contrast filling the sinus (the ‘empty delta sign’) because it is obstructed by
thrombus. The diagnosis is, therefore, venous sinus thrombosis: this appears as low density material
within the sinus. A thin layer of contrast can be seen to outline the low density, dark, thrombus.

• Venous sinus thrombosis is a poorly understood condition associated with risk factors
including the oral contraceptive pill, pregnancy, prothrombotic states, and malignancy.
The presentation often is nonspecific.
• Sinus thrombosis may lead to venous hypertension, cerebral oedema, and venous infarction.

4 How would you describe the abnormality labelled ‘C’? What normal structures are
shown by arrows D, E, and F (Figure 9.8B)?
The image demonstrates extensive low signal filling defect (C) within the superior sagittal sinus
posteriorly in keeping with thrombus. The filling defect extends posteriorly to the confluence of
sinuses (at the level of the tentorium cerebelli). Label D points to the normal bony cranium, which
has low signal. E is the cerebellum and F is the sinus confluence.

448

K30031_Book.indb 448 9/6/17 1:39 PM


Case 9.8: Answers

Fig. 9.8C  ​
Axial
T2-weighted
MR image
showing a
mixed signal
filling defect in
the superior
sagittal sinus
(A) posteriorly.
On this
sequence the
sinus should
appear black
owing to
flowing blood.

Figure 9.8C is a single axial slice of a T2-weighted MR image in this same patient. A filling
defect is again demonstrated, in keeping with thrombus.
• The signal characteristics of thrombus change over time on T1 and T2-weighted imaging so
that it is possible to age thrombus. You do not need to know details of this for finals.

LEARNING POINTS: SAGITTAL SINUS THROMBOSIS


▪▪ The presentation of dural venous sinus thrombosis is nonspecific and the differential is broad.
▪▪ There are numerous risk factors, more commonly the oral contraceptive pill, pregnancy,
prothrombotic states, and malignancy.
▪▪ A CT venogram and/or MR venogram are most useful for diagnosis. Which investigation is
performed will depend on availability, contraindications to iodinated CT contrast, pregnancy
(ionising radiation in CT), and patient cooperation.
▪▪ Management is with systemic anticoagulation with LMWH or warfarin.

449

K30031_Book.indb 449 9/6/17 1:39 PM


Case 9.9

Fig. 9.9A  Sagittal T2-weighted MR image of the lumbar spine.

450

K30031_Book.indb 450 9/6/17 1:39 PM


Case 9.9: Questions

Fig. 9.9B  Axial T1-weighted MR image through the L5 level of the lumbar spine.

A 72-year-old female is referred to the oncology team having presented to the ED with a
24-hour history of increasing lower back pain on a background of chronic back pain. There
is no history of trauma. The patient denies any urinary or faecal incontinence. There is a past
medical history of breast cancer, and this was treated with a mastectomy, radiotherapy, and
chemotherapy 3 years ago.
On examination, there is 4/5 power in the lower limbs and reflexes cannot be elicited. There is
loss of perianal sensation but anal tone is maintained. There is diffuse tenderness on palpation of
the lumbar spine, most severe at L5.

CASE 9.9: QUESTIONS


1 What is your provisional diagnosis?
2 What priority would you give to this patient’s problem and what investigations are
indicated?
3 What do the images show (Figures 9.9A and 9.9B)?
4 How should the patient be treated?

451

K30031_Book.indb 451 9/6/17 1:39 PM


9  Neurology cases

CASE 9.9: ANSWERS


1 What is your provisional diagnosis?
The history of chronic lower back pain with acute deterioration in pain and associated lower limb
neurology in the last 24 hours with focal neurology is indicative of cauda equina compression
syndrome. Cauda equina syndrome is said to be ‘complete’ once there is urinary dysfunction.

• Common causes for this presentation include disc prolapse or vertebral collapse causing
nerve root compression. Pathological vertebral collapse is to be excluded given the history
of breast cancer.
• The patient’s presentation is likely to represent a recurrence of breast cancer.
• There are recognised ‘red-flag’ symptoms that should alert the clinician to a possible
serious underlying cause for low back pain and these include:
• Unrelenting pain, worse at night.
• Pain in patients <18 years, >50 years of age.
• Previous history of cancer.
• Associated systemic symptoms/signs, e.g. weight loss, fever, malaise.
• Radiation of pain.
• Change in perianal sensation, incontinence.

2 What priority would you give to this patient’s problem and what investigations are
indicated?
Cauda equina syndrome is a radiological and surgical emergency (Table 9.9):

• This patient requires urgent MRI of the spine in the first instance – liaise with seniors and
then radiology to arrange the scan. This must be done as soon as possible after diagnosis.
Once suspected cord compression is suspected clinically, MRI must not wait until the next
day or after the weekend – if local MRI facilities do not exist out of hours then the patient
should be transferred urgently to a neurosurgical unit for imaging.
• Emergency surgical decompression is generally the preferred approach to decompressing
the lumbosacral nerve roots in patients with no history of cancer and needs to be
performed, if appropriate, urgently after the diagnosis is made. In patients with known
cancer and likely metastatic compression, it may be more appropriate to arrange urgent
radiotherapy to the affected area having started the patient on high-dose steroids to reduce
swelling.
• The urgency of surgery is greatest with incomplete cauda equina syndrome, and
neurological/urological outcomes are best when there is no progression to complete cauda
equina.
• Remember also to check bloods, address patient pain, check urine output if urinary
catheterisation is needed, and exclude hypercalcaemia in likely malignancy.

Table 9.9  Cauda equina syndrome


• The cauda equina refers to the nerve roots originating from the conus (the cord termination)
• These lumbosacral roots provide sensory innervation to the saddle area, motor innervation to the
lower limbs, and voluntary control of the anal and urinary sphincters
• The most common cause of cauda equina syndrome is a disc herniation at L4–5 or L5–S1

452

K30031_Book.indb 452 9/6/17 1:39 PM


Case 9.9: Answers

3 What do the images show (Figures 9.9A and 9.9B)?


They are MR images of the spine and confirm bony metastases and malignant compression of the
cauda equina at L5 (Figures 9.9C and 9.9D).

• A typical MRI of the spine for suspected cauda equina syndrome includes sagittal and axial
T1-weighted imaging (to assess the bone marrow), sagittal and axial T2-weighted imaging
(for assessment of the cord), and postcontrast or STIR imaging if there is suspicion of
infection (spondylodiscitis).

Fig. 9.9C  ​Sagittal


T2-weighted
MR image of the
lumbar spine.
A low signal (dark)
lesion consistent
with vertebral
metastasis is seen
at the L5 vertebral
level (A). The
tumour extends
posterior to L5
causing significant
canal compression
and cauda equina
compromise (B).
The L2 vertebra
and S1 segment
are labelled. The
cauda equina nerve
roots are shown (C)
within a CSF filled
thecal sac.

453

K30031_Book.indb 453 9/6/17 1:39 PM


9  Neurology cases

Fig. 9.9D  Axial T1-weighted MR image at L5 level. Note the tumour extending from the posterior
vertebra (A) and compressed thecal sac (B).

4 How should the patient be treated?


This patient requires urgent MDT discussion between radiology, oncology, and neurosurgery.
She should be started on high-dose dexamethasone by mouth to reduce oedema and then be
considered for further treatment, most likely spinal radiotherapy in view of her previous history:
neurosurgical decompression is less likely in this case. The patient also needs urgent restaging
(CT chest, abdomen, pelvis, and radionuclide bone scan) of her breast cancer prior to consider-
ation of any further treatment (chemotherapy).

LEARNING POINTS: MALIGNANT SPINAL CORD/CAUDA EQUINA COMPRESSION


▪▪ Think of bone metastases in patients over 40 years with bone pain, especially if there is a given
past history of malignancy.
▪▪ Cauda equina syndrome (and cord compression) is a diagnostic and surgical emergency. The
evidence regarding optimal time for surgical decompression is mixed; however, most surgeons
intervene within 24–48 hours.
▪▪ Patients with incomplete cauda equina syndrome are the highest priority for intervention.
▪▪ Cauda equina syndrome is uncommon and mostly caused by disc prolapse.

454

K30031_Book.indb 454 9/6/17 1:39 PM


Case 9.10

Fig. 9.10A

455

K30031_Book.indb 455 9/6/17 1:39 PM


9  Neurology cases

Fig. 9.10B

456

K30031_Book.indb 456 9/6/17 1:39 PM


Case 9.10: Questions

A 45-year-old male lorry driver attends the ED with his wife. Although the patient is slightly
drowsy, you are able to take a brief history. He describes several weeks of feeling unwell with a
productive cough and worsening headache. More recently he has felt tired and irritable. There is
a 30 pack-year history of smoking.
On clinical examination his abdomen is soft and nontender. On auscultation of the chest there
are crackles at the right base. There is a mild impairment of consciousness with GCS 13 (E3 V5 M5).
There is reduced power in all limbs (slightly worse on the left) and hyperreflexia.
His BP is 150/90 mmHg, HR 115 bpm, and respiratory rate 22 bpm. Oxygen saturation is 92%
on room air and he is pyrexial at 37.9°C. Routine bloods show:

Elevated serum glucose 18 mmol/L (normal fasting level 3.9–5.5 mmol/L)


WCC 22 × 109/L (4–11 × 109/L)
CRP 220 mg/L (<5 mg/L)

Urine dipstick is positive for glucose only. You request a CXR, which reports ‘Focal airspace
opacification in the right lower lobe in keeping with infection.’

CASE 9.10: QUESTIONS


1 Does this patient require any immediate further imaging?
2 What studies are shown in Figures 9.10A and 9.10B and what do they show?
3 What further steps should you take in this patient’s management?

457

K30031_Book.indb 457 9/6/17 1:39 PM


9  Neurology cases

CASE 9.10: ANSWERS


1 Does this patient require any immediate further imaging?
You have already established a probable diagnosis of pneumonia. A follow-up CXR at 6 weeks
after treatment is advisable to ensure that the consolidation has resolved and that there is no
underlying malignancy.
Pneumonia alone, however, does not account for the patient’s neurological symptoms and
collapse. Urgent cross-sectional imaging of the brain is required. CT is usually used in the acute
situation, as it is quick to undertake and is sensitive for stroke and blood and tumour. IV contrast
is used following the initial precontrast scan in certain situations, for example suspected tumour
or mass lesion or venous sinus thrombosis.

• The differential for the neurological symptoms include tumour (primary or secondary),
infection, and stroke (ischaemic or haemorrhagic).

Fig. 9.10C  Precontrast axial CT section of the brain at the level of the bodies of the lateral ventricles
showing extensive oedema (A) and an ill-defined periventricular lesion (B).

458

K30031_Book.indb 458 9/6/17 1:39 PM


Case 9.10: Answers

• Given the impaired consciousness, an anaesthetic assessment prior to CT would be appropriate.


Remember in the exam to mention urgent discussion with seniors and radiology. If the patient
has altered conscious level or if his airway is at risk then he will need intubating prior to CT.
• Given that the patient is septic and an undiagnosed diabetic, acute kidney injury should
also be excluded. If the estimated glomerular filtration rate (eGFR) is <30 mL/min there is
increased risk of contrast-induced nephropathy (the patient may require prehydration).

2 What studies are shown in Figures 9.10A and 9.10B and what do they show?
They are CT images of the brain and are repeated here as Figures 9.10C and 9.10D. Figure 9.10C
is a precontrast axial CT image of the brain. After reviewing the images, the on-call radiology reg-
istrar asked the radiographers to repeat the scan after IV contrast administration (Figure 9.10D).
Pre-IV contrast there are large areas of low density in the right frontal and parietal lobes and
left frontal lobe. The appearance is in keeping with vasogenic oedema, and the distribution and
appearance is not typical for stroke (Table 9.10).

Fig. 9.10D  Postcontrast scan performed at almost the same level as in Figure 9.10C showing ring-
enhancing lesions (C) and associated vasogenic oedema.

459

K30031_Book.indb 459 9/6/17 1:39 PM


9  Neurology cases

Table 9.10  CT findings


Cytotoxic oedema
Appears following ischaemic infarction. It is an area of low density involving the white matter and the
cortex where extracellular water passes into cells with swelling and no disruption of the blood/brain
barrier
Vasogenic oedema
Associated with brain tumours and abscesses. Mainly involves the white matter and associated with
disruption of the blood/brain barrier. Irregular low density change extends towards the cortex with
finger-like projections

Following IV contrast, ring-enhancing lesions are clearly seen within the oedema. There is
mass effect with effacement of cerebral sulci and distortion/compression of the frontal horns of
the lateral ventricles.
• This appearance could represent either multiple malignant tumours (metastases) or
intracerebral abscesses. The history of chest sepsis, however, favours abscesses. It is
characteristic of brain abscesses to have a thinned enhancing rim on their medial side
(owing to the relatively avascular white matter medially). Eventually the abscess may
rupture into the ventricle and cause ventriculitis.
• The presence of a smooth, thin enhancing rim and central low density (necrosis) is typical
for the ‘early capsule’ abscess phase. This occurs about 1–2 weeks after onset.
• Most abscesses in the CNS arise owing to bacteraemia typically arising from lung or cardiac
infection. Immunocompromise, congenital heart disease, skull fractures, and neurosurgery
are key risk factors.
• MRI can be helpful in some cases to potentially differentiate abscess from tumour using
DWI techniques.

3 What further steps should you take in this patient’s management?


This is a neurosurgical emergency. The reporting radiologist should telephone a report to the
requesting clinician and advise a neurosurgical opinion.
• Further urgent actions would include prompt broad-spectrum antibiotics following blood
cultures, fluid resuscitation, prophylactic anticonvulsants, and possible surgical excision/
drainage. Lumbar puncture would be contraindicated in a case such as this owing to risk
of coning caused by raised intracranial pressure. He will also need careful observation and
anaesthetic review if his airway is at risk.
• The patient should be transferred to a neurosurgical unit where he will be managed by a
MDT, including neurosurgery, neurology, microbiology, and neuroradiology.

LEARNING POINTS: CEREBRAL ABSCESS


▪▪ A brain abscess is a neurosurgical emergency.
▪▪ The appearance of a brain abscess is nonspecific on CT without corroborative clinical findings.
It is important, therefore, that you provide the radiologist with accurate clinical information.
▪▪ Most brain abscesses arise due to sepsis elsewhere in the body and are more likely in the
presence of immunocompromise, congenital heart disease, skull fracture or recent neurosurgery.
▪▪ Rarely, brain abscesses arise owing to direct spread from an infected facial sinus.

460

K30031_Book.indb 460 9/6/17 1:39 PM


Paediatric cases

10 UDAY MANDALIA AND LUCY SHIMWELL

Case 10.1 462 Case 10.9 510


Case 10.2 468 Case 10.10 516
Case 10.3 474 Case 10.11 523
Case 10.4 480 Case 10.12 529
Case 10.5 486 Case 10.13 535
Case 10.6 495 Case 10.14 540
Case 10.7 501 Case 10.15 545
Case 10.8 505

461

K30031_Book.indb 461 9/6/17 1:39 PM


Case 10.1

Fig. 10.1A
Sagittal
T2-weighted
MR image of
the brain.

A 6-year-old male presents with a 2-week history of nausea, headache, and double vision. His
mother has also noticed that he has become increasingly clumsy and his speech has become
slurred.
On examination the child appears lethargic. He is mildly hypertensive with a systolic blood
pressure of 115 mmHg (90–110 mmHg). He has papilloedema and signs of cerebellar dysfunction
with ataxia and poor balance.
He is seen by the paediatricians and an urgent MRI of the brain is requested (Figures 10.1A
and 10.1B).

CASE 10.1: QUESTIONS


1 What are the abnormal findings?
2 What is the likely diagnosis?
3 What is the investigation of choice for children who present in this way?
4 How should this patient be managed?

462

K30031_Book.indb 462 9/6/17 1:39 PM


Case 10.1: Questions

Fig. 10.1B  Axial T1-weighted MR image through the posterior fossa before (top) and after (bottom) IV contrast.

463

K30031_Book.indb 463 9/6/17 1:39 PM


10   Paediatric cases

CASE 10.1: ANSWERS


1 What are the abnormal findings?
There is a large midline mass in the posterior fossa, centred within the 4th ventricle (Figures 10.1C
and 10.1D). The lesion is obstructing the flow of CSF and is causing obstructive hydrocephalus with
distension of the third ventricle.
The child presents with red flag features of raised intracranial pressure (ICP). Signs of raised
ICP include headaches, which are worse on lying down or in the morning, vomiting, papilloe-
dema, seizures, speech disturbance, and visual disturbance. Cranial nerve palsies are also com-
mon in posterior fossa tumours owing to compression or infiltration of the brain stem.

Fig. 10.1C  Sagittal T2-weighted MR image showing a large mass (A) compressing the 4th ventricle with
consequent distension of the 3rd (B) and lateral (C) ventricles. There is also mass effect on the pons (P)
anteriorly and crowding of the foramen magnum (D) inferiorly owing to inferior extension of the mass.
Note the impingement on the spinal cord/medulla.

464

K30031_Book.indb 464 9/6/17 1:39 PM


Case 10.1: Answers

2 What is the likely diagnosis?


This lesion is likely to represent a medulloblastoma. This is the commonest posterior fossa
tumour of childhood and accounts for 15–20% of all paediatric brain tumours. These tumours
commonly arise from the roof of the 4th ventricle and are made of densely packed cells with
a high nuclear to cytoplasmic ratio giving them a characteristically dense appearance on CT
(Figure 10.1E). On MRI they are typically heterogeneous but bright on T2 and enhance avidly
with contrast.
Other posterior fossa tumours of childhood include brainstem gliomas, ependymomas, cer-
ebellar astrocytomas, and atypical teratoid rhabdoid tumours.

Fig. 10.1D  Axial T1 with contrast MR image showing a large mass (A) filling the 4th ventricle. The pons (P)
is compressed anteriorly and the cerebellar hemispheres (C) are compressed laterally. Note: the mass is
low signal precontrast (see Figure 10.1B, top) and enhances in this figure (the degree of enhancement can
be variable).

465

K30031_Book.indb 465 9/6/17 1:39 PM


10   Paediatric cases

Fig. 10.1E  Axial unenhanced CT of a posterior fossa medulloblastoma (A) in another child with a
characteristically dense appearance.

3 What is the investigation of choice for children who present in this way?
In a child with suspected brain tumour an MRI scan with contrast would be the initial imaging
investigation of choice, as this will best demonstrate the surrounding anatomy and origin of the
tumour. A CT scan may help in the first instance if MRI is not available or the child is unable to
lie sufficiently still. MRI can be performed on young/uncooperative children using general anaes-
thesia. MRI brain is combined with MRI spine also in suspected malignant tumours to check for
‘drop’ spinal metastases from the cranial primary.

466

K30031_Book.indb 466 9/6/17 1:39 PM


Case 10.1: Answers

4 How should this patient be managed?


The child needs to be managed initially using an ABCDE approach. It is important to assess the
child’s neurological status including GCS and cranial nerve assessment. The child should also be
examined for evidence of raised ICP.
Urgent discussion with paediatricians and neurosurgeons is needed.
If there are signs of raised ICP then neuroprotective measures should be undertaken. These
include:

• Nursing the child with the head up to increase cerebral venous drainage.
• Administration of hypertonic solutions, such as mannitol, and also steroids to reduce
cerebral oedema.

Once the child is stabilised they will require emergency neurosurgical intervention in order to
decompress the ventricles and relieve the hydrocephalus. This is usually achieved by inserting a
ventricular shunt. Further definitive treatment in a specialist paediatric neuro-oncology centre is
recommended to optimise the patient’s care. Treatment will be with a combination of neoadju-
vant chemoradiotherapy and surgery. Prognosis will depend on the amount of tumour left after
resection and the presence of any metastases.

LEARNING POINTS: PAEDIATRIC BRAIN TUMOURS


▪▪ Red flag signs of raised ICP include decreased conscious level, seizures, change in behaviour,
persistent and progressive headaches, vomiting, papilloedema, speech and visual disturbance,
and cranial nerve palsies.
▪▪ Posterior fossa tumours can obstruct the 4th ventricle and the flow of CSF into the subarachnoid
space leading to obstructive hydrocephalus.
▪▪ Differential diagnoses for a posterior fossa brain tumour in children include medulloblastoma,
astrocytoma, ependymoma, and atypical teratoid/rhabdoid tumour.
▪▪ Immediate management of raised ICP may include intubation and ventilation if the child is
comatosed, urgent neuroimaging, administration of hypertonic solutions and steroids, and
transfer to a neurosurgical unit.

467

K30031_Book.indb 467 9/6/17 1:39 PM


Case 10.2

Fig. 10.2A  Erect AP CXR.

A 7-year-old male presents to his GP with fever, cough, pleuritic chest pain, and lethargy follow-
ing a upper respiratory tract infection. He has a fever of 39°C. He has some dullness to percussion
and decreased breath sounds on the left.
The GP requests a CXR (Figure 10.2A), suspecting a chest infection, and commences antibi-
otic treatment empirically.
The reporting radiologist is concerned by the XR findings and recommends a repeat XR in 6
weeks after treatment (Figure 10.2B).

468

K30031_Book.indb 468 9/6/17 1:39 PM


Case 10.2: Questions

Fig. 10.2B  Repeat CXR 6 weeks after treatment.

CASE 10.2: QUESTIONS


1 What do the CXRs show and what is the diagnosis?
2 Why was the radiologist concerned?
3 What are the common XR appearances of lower respiratory tract infections in children?
4 What normal structure is often mistaken for a mediastinal tumour in children?

469

K30031_Book.indb 469 9/6/17 1:39 PM


10   Paediatric cases

CASE 10.2: ANSWERS


1 What do the CXRs show and what is the diagnosis?
The initial XR shows an ill-defined opacity in the left mid-zone with additional left lower zone
opacity (Figure 10.2C). The opacity is rounded with poorly defined margins. There is no cavita-
tion or calcification. On the subsequent CXR the abnormality has completely resolved. The clini-
cal features and rapid resolution with antibiotics are consistent with a diagnosis of p­ neumonia.
This XR pattern in children is known as a round pneumonia.
Round pneumonia is a pattern of infection typically seen in children under the age of 8 years.
It is thought to occur because children have underdeveloped collateral air circulation through the
channels of Lambert and the pores of Kohn, which limits the spread of infection and allows the
formation of a contained rounded area of pneumonia.

Fig. 10.2C  The rounded opacity in the left mid-zone (arrows) can be seen. There is further additional
opacity in the left lower zone extending behind the left heart. Note: the left heart border (LHB) is clearly
seen so this additional density must lie within the left lower lobe posterior to the heart: silhouette sign.

470

K30031_Book.indb 470 9/6/17 1:39 PM


Case 10.2: Answers

Round pneumonia has a predisposition for the lower lobes and is more common in a posterior
location with no peripheral or central predisposition. The vast majority of these lesions resolve
on follow-up imaging; however, they can progress to involve the whole lobe. Streptococcus pneu-
moniae is a common causative organism.

2 Why was the radiologist concerned?


Not uncommonly pneumonic consolidation, which assumes this spherical shape, can be mis-
taken for a pulmonary neoplasm and is known as a pseudotumour of the lung. If there is concern
regarding a mass on the CXR then a follow-up XR several weeks after completion of antibiotic
treatment will help to resolve any diagnostic dilemma, as infection will show resolution.
If the opacity is persistent or if there is suspicion of a lung tumour then a CT scan should be
performed after paediatric review. A differential diagnosis for paediatric CXR masses is included
in Table 10.2.

Table 10.2  Solitary pulmonary masses on a paediatric CXR


Pseudotumours Round pneumonia
Encysted pleural effusion
Mucous plug
Non-neoplastic lesions Pulmonary sequestration
Congenital pulmonary airway malformation
Intrapulmonary bronchogenic cyst
Granuloma
Inflammatory pseudotumour
Pulmonary arteriovenous malformation
Neoplastic lesions Solitary metastasis (usually Wilm’s tumours and sarcomas)
Bronchial carcinoid/adenoma
Pleuropulmonary blastoma
Hamartoma
Thoracic neuroblastoma

3 What are the common XR appearances of lower respiratory tract infections


in children?
The radiographic appearances of pneumonia vary according to the age of the patient and the
pathogen. Several factors contribute to the differences in infection patterns seen in infants and
adults:

• The peripheral airways are smaller and have less conductance.


• The collateral pathways of circulation are less well developed.
• There are more mucous glands.
• The airways are more collapsible.

These factors make the smaller airways more susceptible to narrowing, and this predisposes to
air trapping and hyperexpansion or segmental collapse. The XR may demonstrate asymmetrical
appearances. Thickening of the bronchial wall is a further feature seen in paediatric lower respira-
tory infections. These changes are secondary to oedema and inflammation in the bronchial walls.

471

K30031_Book.indb 471 9/6/17 1:39 PM


10   Paediatric cases

The majority of pneumonias in children are viral and these radiographic changes are mostly
associated with viral infections (Figure 10.2D). The classic lobar changes associated with bacte-
rial infection are less common.

Fig. 10.2D  CXR of a 1-year-old infant with a viral bronchopneumonia. The lung appearances are
asymmetrical. There is hyperexpansion in the left lower zone with flattened left hemidiaphragm (A) and
splaying of the ribs (arrows B). On the right there is an area of opacification (C) adjacent to and obscuring
the RHB (border is seen as a dotted line) consistent with collapse and consolidation in the middle lobe
(silhouette sign – opacified lung obliterates the normal air/soft tissue interface of aerated lung and right
heart). Further consolidation is seen in the left mid and lower zone.

4 What normal structure is often mistaken for a mediastinal tumour in children?


The mediastinum in infants can appear abnormally wide owing to the variable appearances of
the thymus, which can be misinterpreted as pathology particularly in those who do not look at
paediatric CXRs routinely. The normal thymus can have a variable size and shape particularly in
infants. It can extend up to the right or left chest wall, inferiorly to the cardiophrenic angles, and
superiorly into the neck. Larger thymuses tend to be commoner in boys.

472

K30031_Book.indb 472 9/6/17 1:39 PM


Case 10.2: Answers

After the age of 5 years, the thymus becomes less visible on XR with a relative decrease in size
and it should not be visible during the second decade. In infants it is often visible as a triangular
extension, usually on the right, known as the ‘sail sign’ (Figure 10.2E).

Fig. 10.2E  The normal thymus in an infant. The gland extends lateral to the right mediastinal border
forming a sail sign (arrows), not to be confused with a mediastinal mass.

LEARNING POINTS: PAEDIATRIC PNEUMONIA


▪▪ Several factors contribute to the differences in infection patterns seen in infants and adults owing
to smaller and more collapsible airways as well as underdeveloped collateral pathways.
▪▪ Round pneumonia is an unusual appearance of pneumonia in children, which can be
misinterpreted as a tumour. Radiographic follow-up to normality post-treatment is essential.
▪▪ Most paediatric pneumonias are viral and manifest with air trapping, atelectasis, and bronchial
wall thickening.
▪▪ The thymus is prominent in infants and manifests as a widened mediastinum. It becomes less
visible after the age of 5 years. The sail sign is a common appearance of a normal thymus.

473

K30031_Book.indb 473 9/6/17 1:39 PM


Case 10.3

Fig. 10.3A  Supine AXR in a baby boy.

A newborn male on the postnatal ward has a distended abdomen and has not passed meconium
in the first 24 hours of life. He is breastfeeding well and not vomiting. On examination the abdo-
men is tense and distended but not peritonitic. You request an AXR (Figure 10.3A).
The baby is transferred to the neonatal unit for closer monitoring. In view of the AXR appear-
ances, a contrast enema is performed using a water-soluble contrast agent, (Figure 10.3B).

474

K30031_Book.indb 474 9/6/17 1:39 PM


Case 10.3: Questions

Fig. 10.3B  Single image from a water-soluble contrast enema study.

CASE 10.3: QUESTIONS


1 What does the AXR show?
2 What does the contrast enema show and what is the most likely diagnosis?
3 What are the complications of this condition?
4 What are the other features of this condition?

475

K30031_Book.indb 475 9/6/17 1:39 PM


10   Paediatric cases

CASE 10.3: ANSWERS


1 What does the AXR show?
Multiple dilated loops of bowel are visible (Figure 10.3C). It is not possible to differentiate large
and small bowel on a neonatal AXR as both are of a similar calibre and the haustral folds of
the colon have not yet fully developed. As a rule of thumb, however, bowel that is wider than the
width of a lumbar vertebral body is considered dilated. The clinical and radiological features in
this baby are consistent with bowel obstruction.
The causes of neonatal bowel obstruction can be divided into high or low level obstruction
(Table 10.3A). The bowel gas pattern on this AXR demonstrates multiple loops and the pattern is
consistent with a distal ‘low level’ obstruction. High obstructions tend to present with vomiting
whereas low obstructions tend to present with the delayed passage of meconium.

Fig. 10.3C  AXR with multiple dilated loops of bowel wider than a vertebral body (arrows, A), in keeping
with bowel obstruction.

476

K30031_Book.indb 476 9/6/17 1:39 PM


Case 10.3: Answers

Table 10.3A  Causes of high and low level bowel obstruction in neonates

High obstruction Low obstruction


Pyloric stenosis Meconium ileus
Duodenal web Ileal atresia
Duodenal atresia Incarcerated inguinal hernia
Malrotation/volvulus Small left colon
Annular pancreas Hirschsprung’s disease
Jejunal atresia Anorectal malformations/imperforate anus

2 What does the contrast enema show and what is the most likely diagnosis?
A small atretic colon. Contrast refluxes into the terminal ileum where it outlines several pellets of
meconium (Figure 10.3D).
The findings are consistent with meconium ileus. Up to 90% of patients with meconium ileus
have cystic fibrosis (CF) and 10–20% of patients with CF will present in this manner.

Fig. 10.3D  Water-soluble contrast enema demonstrates filling defects in the distal ileum (A) in keeping
with meconium ileus. The colon (B) and rectum (C) are empty and atretic with a microcolon appearance.

477

K30031_Book.indb 477 9/6/17 1:39 PM


10   Paediatric cases

CF is caused (in most cases) by a mutation in the CF transmembrane conductance regulator


(CFTR) gene on chromosome 7. This leads to faulty chloride transport across epithelial mem-
branes. This increases the viscosity of meconium, which becomes thick and inspissated in the
terminal ileum. The meconium can then impact and cause obstruction. The commonest CF muta-
tion is screened for by heel prick blood test in the neonatal period.

3 What are the complications of this condition?


Complications of meconium ileus include ileal atresia, perforation, and volvulus.
Infants should be managed with an ABCDE approach. IV fluid resuscitation will be necessary
as the child is kept nil-by mouth. The contrast enema has therapeutic as well as diagnostic value
as the hyperosmolar contrast draws fluid into the bowel lumen and acts as a lubricant, easing the
passage of meconium. If conservative measures fail, surgery is then considered.

4 What are the other features of this condition?


Other features of CF are included in Table 10.3B. Figure 10.3E is a CXR demonstrating thoracic
complications of CF.

Table 10.3B  Manifestations of CF


Thoracic Peribronchial thickening
Bronchial dilatation (bronchiectasis)
Mucus plugging
Air trapping, which can lead to a pneumothorax
Cystic changes
Pulmonary hypertension
Hilar lymphadenopathy
GI Meconium ileus
Distal intestinal obstruction (equivalent of meconium ileus but in older children)
Rectal prolapse
Hepatobiliary/ Fatty infiltration in the pancreas
pancreatic Chronic pancreatitis
Hepatomegaly
Gallstones
Liver cirrhosis
Skeletal Delayed skeletal maturation
Clubbing and hypertrophic cardiomyopathy
Head and neck Chronic sinusitis
Nasal polyps

478

K30031_Book.indb 478 9/6/17 1:39 PM


Case 10.3: Answers

Fig. 10.3E  CXR of a teenager with CF. The lungs are mildly hyperinflated. There are changes consistent with
bronchiectasis with dilated thick-walled bronchi present. The right hilum appears bulky and dense consistent
with lymphadenopathy (A). Note the patient has developed a right pneumothorax (lung edge – B). There is a
right-sided central line used to administer long-term IV antibiotics (C). In the exam it is important to ascertain
if the central line has just been inserted (so pneumothorax may be a complication of insertion) or if the
pneumothorax is a complication of the disease.

LEARNING POINTS: CYSTIC FIBROSIS/MECONIUM ILEUS


▪▪ CF is an autosomal recessive genetic disease that affects the exocrine function of the lungs, liver,
pancreas, and small bowel.
▪▪ 10–20% of patients present with meconium ileus.
▪▪ Meconium ileus causes ‘low level’ bowel obstruction in neonates and causes a ‘microcolon’
appearance on contrast enema.
▪▪ Treatment of CF usually involves long-term antibiotics, steroids, and supplements of vitamins and
pancreatic enzymes. Physiotherapy and lung transplantation may also be required.

479

K30031_Book.indb 479 9/6/17 1:39 PM


Case 10.4

Fig. 10.4A  Neonatal supine AXR.

480

K30031_Book.indb 480 9/6/17 1:39 PM


Case 10.4: Questions

Fig. 10.4B  Single image from an upper GI tract contrast study.

A 6-week-old neonate presents to the ED with bilious vomiting. On examination the child is in
shock and the abdomen is tender and distended. The baby is stabilised and a NG tube inserted.
An AXR is performed (Figure 10.4A) after which the patient is urgently transferred to a paedi-
atric surgical unit.
On arrival at the specialist surgical unit, an upper GI (UGI) contrast study is arranged
(Figure 10.4B).

CASE 10.4: QUESTIONS


1 What do the AXR and the UGI contrast study show and what is the likely diagnosis?
2 What is the pathophysiology of this condition?
3 What are the complications of this condition?
4 What are the differential diagnoses?

481

K30031_Book.indb 481 9/6/17 1:39 PM


10   Paediatric cases

CASE 10.4: ANSWERS


1 What do the AXR and the UGI contrast study show and what is the likely diagnosis?
A distended gas-filled stomach and duodenum are seen with a virtually gasless abdomen distally
(Figure 10.4C). This ‘double bubble’ appearance is suggestive of a high gastrointestinal obstruc-
tion at the level of the duodenum.
The UGI contrast study shows contrast in the stomach, duodenum, and proximal jejunum. The
purpose of this study is to locate the position of the duodenojejunal (DJ) flexure. In this study a
bolus of the radiopaque contrast agent is injected into the NG tube and followed from the stomach
to the duodenum. Normally the course of the duodenum forms a ‘C’-shaped loop as it leaves the
stomach, such that the DJ flexure is sited in the LUQ of the abdomen to the left of the midline.
However, in this case the DJ flexure and proximal small bowel are positioned inferiorly and to the
right of the proximal duodenum (Figure 10.4D). The findings are consistent with malrotation.

Fig. 10.4C  Neonatal AXR with distension of the gas-filled stomach (A) and duodenum (B) with absent
distal bowel gas.

482

K30031_Book.indb 482 9/6/17 1:39 PM


Case 10.4: Answers

Fig. 10.4D  Single image from an upper GI tract contrast study showing the DJ flexure (A) to the right of
the midline. The proximal jejunal loops (B) are also located on the right side. Note the NG tube within the
stomach (C) and the contour of the duodenum (D). The stomach is now collapsed as it has been drained
with the NG tube. There is narrowing of the bowel lumen at the level of the abnormally orientated DJ
flexure (A).

2 What is the pathophysiology of this condition?


During the 6th week of embryonic development the bowel undergoes a process of elongation,
which leads to a physiological herniation of bowel into the umbilical cord. As the bowel returns
back into the abdominal cavity it undergoes a rotation of approximately 270° anticlockwise. If the
intestines fail to rotate then the DJ flexure comes to lie on the right of the midline and the caecum
floats freely in the upper abdomen. This is known as malrotation.
The bowel becomes fixed in this abnormal position by peritoneal bands called Ladds bands.
These stretch from the caecum to the duodenum and attach to the anterior and posterior abdomi-
nal walls.

3 What are the complications of this condition?


Because the DJ and ileocaecal junctions are not in their normal positions (malrotated), the base of
the small bowel mesentery, which extends between these two points, is narrow. This predisposes
to twisting of the bowel known as midgut volvulus. The bowel twists around the mesenteric ves-
sels, which can lead to fatal midgut strangulation; hence the need for emergency treatment if a
diagnosis of malrotation is being considered.

483

K30031_Book.indb 483 9/6/17 1:39 PM


10   Paediatric cases

4 What are the differential diagnoses?


Malrotation complicated by midgut volvulus is considered a ‘high’ obstruction as the obstruction
is proximal to the jejunum (Table 10.4). These cases often present with vomiting. ‘Low’ obstruc-
tions usually present with delayed passage of meconium.

Table 10.4  Causes of high intestinal obstruction in infants


Malrotation
Duodenal atresia/stenosis, jejunal atresia
Duodenal web
Annular pancreas
Pyloric stenosis – nonbilious vomiting

Fig. 10.4E  AXR of a different patient with a ‘double bubble’ caused by distension of the stomach (A)
and proximal duodenum (B). There is absence of distal bowel gas in keeping with complete duodenal
obstruction. This child had duodenal atresia, which has a 30% association with Down’s syndrome.

484

K30031_Book.indb 484 9/6/17 1:39 PM


Case 10.4: Answers

If the volvulus is partial, there may be no evidence of duodenal obstruction and the AXR may
be normal. However, if the obstruction is complete the stomach and duodenum will distend with
gas. The appearances can mimic other forms of high obstruction such as duodenal atresia where
there is often complete duodenal obstruction with absence of distal gas (Figure 10.4E). The dis-
tended stomach and duodenum produce what is known as the ‘double bubble’ sign.
Pyloric stenosis is a common case in finals and you must know the details of this condition.
Following AXR, US is used to diagnose these cases with the hypertrophied pylorus having a
characteristic sonographic appearance. You are unlikely, however, to be shown this type of US
image in finals.

LEARNING POINTS: MALROTATION


▪▪ Bowel obstruction in babies can be divided into ‘high’ and ‘low’ GI obstruction.
▪▪ Causes of a high obstruction include malrotation, duodenal atresia, duodenal web, annular
pancreas, pyloric stenosis, and jejunal atresia.
▪▪ Malrotation is a congenital abnormality of intestines leading to abnormal fixation of the bowel
within the abdomen. This can lead to twisting of the bowel known as midgut volvulus, a potentially
life-threatening complication.
▪▪ Malrotation can often present with bilious vomiting. The diagnosis is made by a UGI contrast
study.
▪▪ Treatment of malrotation includes stabilisation and resuscitation of the sick child followed by
emergency surgery.

485

K30031_Book.indb 485 9/6/17 1:39 PM


Case 10.5

Fig. 10.5A  Supine AXR.

486

K30031_Book.indb 486 9/6/17 1:39 PM


Case 10.5: Questions

Fig. 10.5B  Abdominal US image of the LUQ in longitudinal section.

487

K30031_Book.indb 487 9/6/17 1:39 PM


10   Paediatric cases

Fig. 10.5C  Abdominal US image of the left kidney (arrows) in longitudinal section.

488

K30031_Book.indb 488 9/6/17 1:39 PM


Case 10.5: Questions

Fig. 10.5D  Abdominal US image of the liver in the same child.

A 22-month-old female is taken to her GP with a suspected abdominal mass. She is irritable and
has become less confident on her feet. Her growth has also faltered. On examination she is tachy-
cardic and hypertensive. Her abdomen is distended but not tender, and a tender mass is palpable
in the left loin.
She is referred to the paediatric unit and an AXR and abdominal US are arranged
(Figures 10.5A–10.5D).

CASE 10.5: QUESTIONS


1 What does the AXR show?
2 What do the abdominal US images show?
3 What is the likely diagnosis?
4 How should this patient be managed?
5 What are the common differentials?

489

K30031_Book.indb 489 9/6/17 1:39 PM


10   Paediatric cases

CASE 10.5: ANSWERS


1 What does the abdominal XR show?
The AXR shows a well-circumscribed, soft-tissue density mass in the LUQ with mass effect on
the adjacent splenic flexure of colon, which is displaced inferiorly and medially (Figure 10.5E).

Fig. 10.5E  AXR showing a soft tissue mass in the LUQ (A) with medial displacement of the descending
colon (B). Transverse colon is labelled C.

490

K30031_Book.indb 490 9/6/17 1:39 PM


Case 10.5: Answers

2 What do the abdominal US images show?


A large, left suprarenal mass, measuring almost 8 cm in diameter is present. The echotexture is
heterogeneous (light and dark). The kidney appears distinct from the mass and is displaced infe-
riorly (Figure 10.5C). The images of the liver show multiple hypoechoic (dark) ‘target’ lesions in
keeping with metastases.
The mass can be seen in Figure 10.5B and also at the left edge of the image in Figure 10.5C,
displacing the left kidney inferiorly, to the right of the image.
3 What is the likely diagnosis?
There is a suprarenal neoplasm with liver lesions. The most likely diagnosis is adrenal neuroblas-
toma with liver metastases.
• Neuroblastoma is the most frequent solid neoplasm in childhood (8–10%) and the most
frequent malignancy in the first year of life.
• It is the third most common paediatric malignancy (the first being CNS tumours and the
second leukaemia).
• The median age of diagnosis is 22 months.
• Neuroblastoma can arise anywhere along the neural crest tissues, which give rise to the
adrenal medulla and sympathetic ganglia.
• The clinical features will reflect the size and site of the mass. The majority (45%) arise from
the adrenal medulla and present as an abdominal mass.
• Lesions arising from the sympathetic chain can manifest as a thoracic mass in the posterior
mediastinum.
• Approximately 60% of infants will have metastatic disease at presentation.
• Metastatic spread to the bones, liver, lymph nodes, skin, and brain are common and
children may present with bone and joint pain and/or hepatomegaly.
• Approximately 90% of patients will have elevated levels of hormones and may present
with hypertension secondary to catecholamine secretion or watery diarrhoea owing to the
production of vasoactive intestinal peptide.
• Neuroblastoma in children under 1 year often presents with hepatic, skin, and bone
marrow metastases (stage IV-S disease). This paradoxically has a good prognosis and the
lesions can spontaneously regress.
4 How should this patient be managed?
Neuroblastoma is treated according the stage of the tumour at diagnosis. This is based on the
radiological findings, surgical resectability, and the presence of metastatic disease. The presence
of the n-MYC proto-oncogene is a poor prognostic indicator (n-MYC amplification).
• Neuroblastoma is usually initially identified on US; the tumour appears heterogeneous
and hypervascularised. Hypoechoic (dark) areas are common and caused by haemorrhage
or necrosis. Although the primary tumour may be detected with US, CT and MRI are
required for staging and to help define the local extent of the disease and tissue of
origin (Figure 10.5.F and 10.5G). Tumour calcification is seen on CT in 85% of cases.
Neuroblastoma typically displaces the kidney without distorting the renal collecting
system. It spreads locally by engulfing and encasing the large vessels.
Metaiodobenzylguanidine (MIBG) nuclear medicine scans are used to check for distant spread
of the disease. MIBG is a catecholamine precursor and is taken up by the cells producing catechol-
amine. In children, this uptake is specific for neuroblastoma; 30% of primary neuroblastomas do
not take up MIBG.
491

K30031_Book.indb 491 9/6/17 1:39 PM


10   Paediatric cases

Figs. 10.5F, G  Coronal T2-weighted MR images of a different child with neuroblastoma (without [10.10F]
and with [10.5G] annotations). The primary tumour can be seen in the LUQ (A). The bright high signal
central area represents necrosis (B). The left kidney (C) is displaced inferiorly. The tumour has spread (D)
locally to engulf the aorta (E). Note the normal liver right lobe (F) and spleen (G).

492

K30031_Book.indb 492 9/6/17 1:39 PM


Case 10.5: Answers

Fig. 10.5G

493

K30031_Book.indb 493 9/6/17 1:39 PM


10   Paediatric cases

5 What are the common differentials?


The main differential diagnoses are Wilm’s tumour and adrenal haemorrhage. Wilm’s tumours
arise from the kidneys and tend to displace the adjacent vessels, whereas neuroblastomas tend
to surround and engulf vascular structures. Wilm’s tumours produce a typical claw sign of renal
parenchyma as tumour expands and splays the kidney. Another differential is adrenal haemor-
rhage, which is common in the neonatal period; this is characteristically avascular and will show
regression over time.

LEARNING POINTS: NEUROBLASTOMA


▪▪ Neuroblastoma is the most frequent solid neoplasm in childhood (8–10%) and the most frequent
malignancy in the first year of life.
▪▪ Neuroblastoma can occur anywhere along the neural crest tissue, which gives rise to the
sympathetic chain, and the majority occur in the adrenal glands.
▪▪ The common differential diagnoses would include Wilm’s tumour and adrenal haemorrhage.
▪▪ Approximately 60% of infants will have metastatic disease at presentation. Metastatic spread to
the bones, liver, lymph nodes, skin, and brain is common.
▪▪ Neuroblastoma is treated according the stage of the tumour at diagnosis. This is based on the
radiological findings, surgical resectability, and the presence of metastatic disease.

494

K30031_Book.indb 494 9/6/17 1:39 PM


Case 10.6

Fig. 10.6A  Supine AXR.

An 8-month-old female presents to the ED with vomiting and intermittent abdominal pain. She
appears to be in pain and is intermittently drawing her legs up onto her chest. She was initially
irritable but now has become increasingly lethargic between episodes. Her mother is concerned
as she has refused all feeds and has passed reddish stools. On examination she is pale, lethargic,
and dehydrated. Her abdomen is distended and tender with a right-sided palpable mass. Initial
investigations are arranged, including bloods and an AXR (Figure 10.6A).

CASE 10.6: QUESTIONS


1 What does the AXR show? 4 How should the patient be managed?
2 What is the most likely diagnosis? 5 What are the potential complications of
3 What test should you arrange to confirm this condition?
the diagnosis?

495

K30031_Book.indb 495 9/6/17 1:39 PM


10   Paediatric cases

CASE 10.6: ANSWERS


1 What does the AXR show?
Multiple central dilated bowel loops and a soft-tissue density in the RUQ are seen on AXR
(Figure 10.6B). An NG tube is in situ. There is no evidence of perforation (the child will need an
erect CXR in addition to confirm this).

Fig. 10.6B  AXR shows a rounded soft-tissue density in the RUQ (A) and mildly dilated central small bowel
loops. Note the NG tube (NG) in the stomach.

496

K30031_Book.indb 496 9/6/17 1:39 PM


Case 10.6: Answers

2 What is the most likely diagnosis?


The history and examination findings are typical for intussusception:

• Intussusception is the telescoping of a segment of proximal bowel (known as


the intussusceptum) into a distal segment (the intussuscipiens) leading to bowel
obstruction.
• The bowel mesentery, which contains the lymphatics and vasculature, is drawn in
alongside the intussuscepted bowel. As a consequence there is venous obstruction,
which leads to swelling of the bowel wall; this further compresses the mesentery,
obstructs arterial blood flow, and can cause ischaemia and necrosis of the affected
segment.
• The point of invagination of the bowel is known as the ‘lead point’. This most commonly
occurs with ileum passing through the ileocaecal valve into caecum, referred to as an
ileocaecal intussusception. However, intussusception can occur anywhere in the large or
small bowel.
• Often in children no cause is found. There is some evidence to suggest that it follows viral
gastroenteritis, with mesenteric nodal enlargement acting as the lead point. The majority of
affected children are under 1 year of age, with a peak incidence at 5–9 months. Passage of
‘redcurrant jelly’, bloodstained mucus, in the stool is classical.

Intussusception in adults, however, is almost always pathological. The lead point is commonly
a malignancy, such as a primary bowel cancer or lymphoma.

3 What test should you arrange to confirm the diagnosis?


In children an US scan of the abdomen is required to confirm the diagnosis (Figures 10.6C and
10.6D). In transverse section, the abnormal segment of bowel produces a ‘target’ or ‘doughnut’
sign of bowel within bowel. In longitudinal section one bowel loop can be seen to invaginate into
another along with the mesentery. The appearances are similar to the US appearances of a kidney,
and are referred to as the ‘pseudokidney’ sign. US is also used to detect the presence of blood flow
within the segment, the absence of which is a poor prognostic indicator.

4 How should the patient be managed?


Intussusception is a surgical emergency and a systematic ABCDE approach should be used.

• If shocked, the patient will need resuscitation with IV fluids.


• Blood should be sent for analysis including FBC, U&Es, CRP, and LFTs. A group and save
and cross match are also recommended as the child may go on to have surgery.
• A urine sample should be used to screen for infection.
• A NG tube should be passed to aspirate the stomach contents.
• The child will need an urgent paediatric review and discussion with a specialist paediatric
surgical team.
• Imaging is required in the form of an urgent AXR and US (also erect CXR if suggestion of
perforation).
• If there are signs of peritonitis the child should proceed straight to theatre.

497

K30031_Book.indb 497 9/6/17 1:39 PM


10   Paediatric cases

Figs. 10.6C, D  Abdominal US scan of an intussusception in longitudinal section without annotations (10.6C)
and with annotations (10.6D), showing the intussusceptum (A) invaginating into the intussuscipiens (B).
The bright hyperechoic area in the middle is mesenteric fat (C), which has been drawn in alongside the
intussuscepted bowel. In this case the lead point was enlarged reactive lymph nodes (D).

498

K30031_Book.indb 498 9/6/17 1:39 PM


Case 10.6: Answers

• In the majority of cases, nonoperative management with fluoroscopy or US-guided air


enema reduction is the first line of treatment (Figure 10.6E). Air is insufflated per rectum to
push the intussusceptum back to its normal position. Resolution is confirmed by visualising
gas reflux back into the small intestine. If this fails, the child should be taken to theatre for
laparoscopic or open reduction. An important complication of rectal air enema reduction is
bowel perforation.

Fig. 10.6E  AXR from a fluoroscopic-guided air enema reduction, showing air outlining the
intussusceptum (arrows) in the hepatic flexure.

499

K30031_Book.indb 499 9/6/17 1:39 PM


10   Paediatric cases

5 What are the potential complications of this condition?


If left untreated, the involved segment of bowel can undergo necrosis and subsequent perforation
and peritonitis, which may be fatal.
Recurrence of intussusception is common and can generally be resolved with repeated rectal
air enema reduction.

LEARNING POINTS: INTUSSUSCEPTION


▪▪ Intussusception is the invagination of proximal bowel into a more distal segment, frequently
involving ileum passing into the caecum, via the ileocaecal valve.
▪▪ It often presents with severe, colicky, intermittent abdominal pain and vomiting, and
characteristically ‘redcurrant jelly’ stool containing blood-stained mucus.
▪▪ It occurs most frequently in children under 1 year, with a peak incidence at 5–9 months.
▪▪ There are characteristic diagnostic features on AXR and abdominal US.
▪▪ Treatment is usually nonoperative with rectal air insufflation, unless there is evidence of perforation
or peritonitis. Adequate resuscitation is vital before any attempt at definitive management.
▪▪ If left untreated, bowel perforation, ischaemia, and necrosis can occur, requiring laparotomy and
bowel resection.

500

K30031_Book.indb 500 9/6/17 1:39 PM


Case 10.7

Fig. 10.7A  Supine CXR.

A term baby presents at birth with respiratory difficulty and cyanosis. The mother missed her
antenatal appointments and her 20 week anomaly scan. The neonatal Senior House Officer is
fast bleeped. On examination the baby has reduced air entry on the left with evidence of tracheal
deviation to the contralateral side. The abdomen appears scaphoid. The child is admitted to the
neonatal intensive care unit (NICU).
A CXR is obtained on admission to the unit (Figure 10.7A).

CASE 10.7: QUESTIONS


1 What are the key abnormalities seen on the CXR?
2 What is the most likely diagnosis and the differential diagnosis?
3 What other imaging may help to confirm the diagnosis?
4 How should this child be managed?

501

K30031_Book.indb 501 9/6/17 1:39 PM


10   Paediatric cases

CASE 10.7: ANSWERS


1 What are the key abnormalities seen on the CXR?
There are bubbly lucencies in the left hemithorax, with the appearances of air-filled loops of bowel
(Figure 10.7B).
There is mediastinal shift to the right, with right-sided cardiac and tracheal deviation. In addi-
tion, the NG tube has a right-sided convexity to it.
The left hemidiaphragm is not clearly visualised. The right hemidiaphragm images normally.
Within the left hemithorax, normal lung markings are seen in the left apex only.

Fig. 10.7B  Supine CXR. The trachea (A) and heart (RHB, B) are deviated to the right in keeping with
mediastinal shift. There is also deviation of the NG tube (NG) to the right. There are multiple rounded cystic
lucencies (C) in the left hemithorax. There is a small amount of normal left lung (D) in the left apex. The right
hemidiaphragm (E) is well defined but the left hemidiaphragm is not delineated.

2 What is the most likely diagnosis and the differential diagnosis?


A congenital diaphragmatic hernia (CDH) is most likely. In this condition, bowel loops and
other abdominal viscera herniate into the chest through a congenital defect in the diaphragm.

502

K30031_Book.indb 502 9/6/17 1:39 PM


Case 10.7: Answers

The lung on the side of the hernia will be underdeveloped. The morbidity and mortality associ-
ated with this condition is as a consequence of this pulmonary hypoplasia.
There are two main types of CDH in neonates:

• Bochdalek hernias: most common and occur in the posterolateral corner of the diaphragm,
more commonly on the left (5:1). (BBB = Bochdalek, Back-left, Big)
• Morgagni hernias: much less common in neonates and occur in the anterior and medial
segments of the diaphragm, more commonly on the right. These are seldom associated with
lung hypoplasia and, therefore, have less morbidity. (MMM = Morgagni, Medial-right, Mini)

The differential diagnoses for cystic lung lucencies in a neonate are:

• Diaphragmatic hernia.
• Congenital lobar overinflation (previously known as congenital lobar emphysema).
• Congenital pulmonary airway malformation (previously known as congenital cystic
adenomatoid malformation).
• Pulmonary sequestration.

Diaphragmatic herniation, in which abdominal viscera herniate into the thorax, is usu-
ally detected with antenatal US. It is associated with respiratory distress and cyanosis at birth.
Examination will reveal reduced breath sounds on the side of the hernia and a flat or sunken
(‘scaphoid’) abdomen because of the displaced loops of bowel.

3 What other imaging may help to confirm the diagnosis?


In most cases the CXR is diagnostic. However, if the bowel loops are filled with fluid the chest
will appear opaque, making it difficult to identify individual loops. In such cases, there can be
confusion with other congenital pulmonary lung malformations and US is used to show fluid-
filled thoracic bowel loops. US is less sensitive, however, once the bowel loops become filled with
gas. If there is ongoing uncertainty, a CT scan can help to confirm the diagnosis (Figure 10.7C).

4 How should this child be managed?


If the abnormality is diagnosed antenatally, then an amniocentesis is offered to detect a chromo-
somal abnormality. A termination may be offered if this is found, as the combination of a CDH
and a chromosomal abnormality is usually fatal.
Infants should be delivered in a specialist centre where the necessary obstetric and neonatal
surgical facilities are available.
On delivery, the child should be assessed with an ABCDE approach. Prolonged resuscita-
tion with bag-valve-mask ventilation should be avoided as this will cause gaseous distension of
the intestines and exacerbate the respiratory distress. The child should be intubated and venti-
lated as soon as possible. A large bore NG tube should be inserted to decompress the stomach.
Hypoventilation owing to lung hypoplasia is the main source of morbidity and mortality, and if
severe can be incompatible with life. Occasionally infants with severe ventilatory failure, who do
not respond to conventional ventilator support, may be treated with extracorporeal membrane
oxygenation (ECMO), a form of complete cardiopulmonary bypass.
Definitive treatment is with surgical repair, which can safely be delayed once the child demon-
strates cardiopulmonary stability. The abdominal viscera are returned to the abdomen through a
subcostal approach. An artificial patch may be used to repair the diaphragmatic defect. Survival
rates vary between 47 and 93%.

503

K30031_Book.indb 503 9/6/17 1:39 PM


10   Paediatric cases

Fig. 10.7C  Coronal CT image postcontrast of another neonate with CDH. Bowel loops (A) and the left
kidney (B) have herniated up into the left hemithorax. The left hemidiaphragm is absent but the right (C) is
clearly visualised. Note the normally positioned right kidney.

LEARNING POINTS: CONGENITAL DIAPHRAGMATIC HERNIA


▪▪ Bochdalek and Morgagni are the two main types of congenital diaphragmatic hernia.
The Bochdalek variant is more common and tends to occur in the posterolateral left diaphragm.
▪▪ The morbidity and mortality associated with this condition are related to the degree of pulmonary
hypoplasia.
▪▪ The diagnosis is usually made on antenatal US. Postnatally the diagnosis can be made on CXR,
which will demonstrate bowel loops within the chest and displacement of the NG tube.
▪▪ The differential for cystic lung lesions in neonates includes: cystic pulmonary airway
malformations, congenital lobar emphysema, and pulmonary sequestration.
▪▪ Children with this condition can have severe respiratory failure. Management requires ventilator
support and possibly extracorporeal membrane oxygenation, prior to surgical repair.

504

K30031_Book.indb 504 9/6/17 1:39 PM


Case 10.8

Fig.10.8A  Supine CXR.

A newborn child develops respiratory distress following a prolonged delivery with meconium
staining of the amniotic fluid at 42 weeks gestation. You are crash called with the paediatric reg-
istrar to the delivery room.
On examination there is nasal flaring and intercostal and subcostal recessions. An expiratory
grunt is heard. Auscultation reveals widespread rhonchi and crepitations. The child is intubated and
meconium is seen in the larynx. The child is then transferred to the NICU. Blood gases are taken:

pH 7.1 (7.35–7.45) PaCO2 8 kPa (4.5–6)


PaO2 6 kPa (11–14) BE −15 (−3−+3)

After stabilisation on the NICU a CXR is performed (Figure 10.8A).

CASE 10.8: QUESTIONS


1 What is the diagnosis? 4 What lines and catheters may be inserted
2 How should this patient be managed? to support this baby and what is their
3 What are the complications of this disorder? optimal position?

505

K30031_Book.indb 505 9/6/17 1:39 PM


10   Paediatric cases

CASE 10.8: ANSWERS


1 What is the diagnosis?
The clinical and CXR findings are consistent with meconium aspiration syndrome (MAS).
Placental insufficiency or postmaturity can lead to intrauterine or intrapartum passage of
meconium (the infant’s first stool), aspiration, and respiratory distress. Approximately 10% of
deliveries have meconium stained amniotic fluid at 38 weeks’ gestation, and this rises to 22% by
42 weeks.
Inhaled meconium is extremely toxic to the newborn lungs, acting as a chemical irritant inhib-
iting the production of surfactant. It can also cause partial or complete bronchial obstruction,
leading to peripheral lung collapse or hyperexpansion via a ‘ball-valve’ effect.
The CXR appearances are of asymmetric lung abnormalities (Figure 10.8B) with a mixed pic-
ture of patchy infiltrates, areas of collapse, and segments of hyperexpansion. Hyperexpansion of
the lungs is particularly common in the early stages.
In severe cases the CXR appearances remain abnormal beyond 14 days and produce a similar
pattern to bronchopulmonary dysplasia. The CXR appearance can also mimic congenital infec-
tion and all babies with MAS are commenced on empirical antibiotics after a septic screen.

Fig. 10.8B  The CXR shows asymmetric areas of consolidation (A) with bronchial wall thickening and air
bronchograms. The lungs are hyperexpanded with at least 8 anterior ribs visible (anterior right 8th rib, B)
above the diaphragm. There is a small right pleural effusion (C). Note the NG tube and oxygen tubing.

506

K30031_Book.indb 506 9/6/17 1:39 PM


Case 10.8: Answers

2 How should this patient be managed?


All babies in respiratory distress should be managed using an ABCDE approach. The manage-
ment for MAS is purely supportive, allowing time for macrophage clearance of the inhaled debris.
Intubation and ventilation may be required in severe cases with ECMO support. In addition, the
use of exogenous surfactant has been shown to reduce the duration for respiratory support.
Hypoxia is common and oxygen is required to maintain saturations to greater than 95%.
Ventilation is usually not a problem in mature infants and the PaCO2 is usually normal or only
slightly raised.
The first blood gas taken after birth may show a marked metabolic acidaemia owing to birth
asphyxia. The acidaemia may need to be corrected if the base deficit is greater than −15.
Close monitoring is required on the NICU. This may require insertion of central umbilical
catheters to monitor the baby’s BP and for the administration of drugs, an endotracheal tube
(ETT) and a NG tube to aspirate stomach contents (Figure 10.8C).

Fig. 10.8C  Whole body


XR of a child on the NICU.
This type of XR is often
used to confirm the position
of the support apparatus.
Note: course of the umbilical
catheters. The umbilical
arterial catheter (UAC, A)
curls inferiorly in the umbilical
artery (curved arrow) before
extending upwards through
the iliac artery into the aorta.
The tip of the umbilical
catheter must be confirmed
not to be passing into an
aortic branch (e.g. renal
artery); the tip optimally lies
either high (T6–T10) or low
(L3–L5) and not in-between
to avoid aortic major
branches. The umbilical
venous catheter (UVC, B)
passes towards the liver and
into the inferior vena cava
(IVC) via the ductus venosus:
the tip is optimally placed
at the junction of the IVC
with the right atrium. Note
also the ETT (C).

507

K30031_Book.indb 507 9/6/17 1:39 PM


10   Paediatric cases

3 What are the complications of this disorder?


Potential complications include air leaks such as pneumothorax and pneumomediastinum. These
are caused by the lung hyperexpansion secondary to the ball-valve effect of partially obstructing
viscous meconium, (Figure 10.8D).
Persistent pulmonary hypertension of the newborn (PPHN) is another complication. Babies
with MAS develop hypoxia, and this can lead to pulmonary artery vasoconstriction and a raised
pulmonary artery pressure, which is a feature of PPHN. In this condition the pulmonary arterial
pressure rises above the systemic arterial pressure, resulting in right-to-left shunting across the
ductus arteriosus and foramen ovale, similar to that seen in the fetal circulation.

Fig. 10.8D  Supine CXR with bilateral anterior pneumothoraces in a child with meconium aspiration. Signs
of an anterior pneumothorax include lucent hemithoraces, a deep diaphragmatic sulcus (A), overly well-
defined heart (B) and diaphragmatic (C) borders, and a visible lung edge (D) with absent vascular marking
peripherally. Note: widened superior mediastinum owing to normal thymus (dotted lines, E).

508

K30031_Book.indb 508 9/6/17 1:39 PM


Case 10.8: Answers

The mortality from MAS is between 4 and 12%. The majority of deaths are from respiratory
failure, air leaks, and PPHN.

4 What lines and catheters may be inserted to support this baby and what is their
optimal position?
Lines and catheters required to support the baby include an ETT, UAC, UVC, and a NG tube.
The ETT allows ventilator support. The tube tip should be in the mid-tracheal position about
1 cm above the carina. Paradoxically, when the neck is flexed the tube travels downwards and
when the neck is extended the tube travels upwards. The most common malposition is within the
right main bronchus.
The UAC allows fluids and medication to be administered and can be used to monitor the arte-
rial BP and gases. The catheter passes from the umbilical artery inferiorly into the iliac arteries
before passing up into the aorta. The optimum tip position is at T6–T9 level, well away from the
renal and mesenteric arteries.
The UVC also allows fluids and medication to be administered and is used for blood sampling.
The catheter passes from the umbilical vein up to the left portal vein then courses through the
ductus venosus into the IVC. The optimum tip position is at the right cavoatrial junction.
The NG tube can be used for enteral feeding and to aspirate stomach contents in an intubated
child. The tube tip should lie in the stomach, well below the left hemidiaphragm and beyond the
gastro-oesophageal junction.

LEARNING POINTS: MECONIUM ASPIRATION SYNDROME


▪▪ Meconium is the first neonatal stool and can be passed in utero in response to fetal hypoxic
stress.
▪▪ If inhaled, meconium can obstruct the airways and produce a chemical pneumonitis causing
severe, and sometimes fatal, respiratory distress known as MAS.
▪▪ The CXR findings include collapse and consolidation as well as peripheral hyperexpansion owing
to air trapping.
▪▪ This condition can be complicated by air leaks (such as pneumothorax and pneumomediastinum)
and persistent pulmonary hypertension.

509

K30031_Book.indb 509 9/6/17 1:39 PM


Case 10.9

Fig. 10.9A  AP view of the pelvis.

A 10-year-old male presents to his GP with a 3-month history of right hip pain and a limp.
There is no history of trauma. The pain is worse on activity and improves with rest. He is other-
wise well and his height and weight are between the 50th and 75th centiles. On examination of
his right hip there is no erythema or swelling. He has slightly reduced range of movement but
the remainder of the examination is normal. The GP refers him for an XR series (Figure 10.9A
and 10.9B).

510

K30031_Book.indb 510 9/6/17 1:39 PM


Case 10.9: Questions

Fig. 10.9B  Frog leg lateral view of the hips (gonad protection applied).

CASE 10.9: QUESTIONS


1 What do the XRs show?
2 What is the aetiology of this condition?
3 What are the differential diagnoses for an acutely painful limp in a child?
4 What further imaging would you recommend?
5 What are the management options?
6 What are the potential complications?

511

K30031_Book.indb 511 9/6/17 1:39 PM


10   Paediatric cases

CASE 10.9: ANSWERS


1 What do the XRs show?
They show (Figure 10.9C) a small and fragmented right capital femoral epiphysis. The physis is
widened and the metaphysis irregular in contour. There is an effusion in the right hip joint. These
are the features of Perthes’ disease. The left hip is normal.

Fig. 10.9C  AP magnified view showing features of Perthes’ disease in the right hip. Note the necrotic,
fragmented right capital femoral epiphysis (A), irregularity and widening of the metaphysis (B), and hip
effusion (C).

2 What is the aetiology of this condition?


Perthes’ disease is avascular necrosis of the capital femoral epiphysis. The blood supply to the
femoral head is compromised leading to bone necrosis. Over time, the necrotic bone is removed
and revascularisation occurs with reossification. The aetiology is unknown.

• The quality of new bone growth can be variable and this is where the various treatment
options play a key role.
• The onset is insidious and is usually unilateral. It generally occurs in children aged
4–10 years and is more common in boys.

512

K30031_Book.indb 512 9/6/17 1:39 PM


Case 10.9: Answers

• In the early stages, the XR can be normal or may only show increased density of the capital
femoral epiphysis. As the disease progresses the femoral head becomes necrotic and
fragmented with irregularity of the metaphysis and an effusion.
• It is usually detected and managed in the early stages (Figure 10.9D), with severe
deformity now being rare.

Fig. 10.9D  AP view of the pelvis showing early features of Perthes’ disease in the right hip. Note the
increased density of the right femoral capital epiphysis (A) with some loss of height and normal appearance
on the left (B).

3 What are the differential diagnoses for an acutely painful limp in a child?
The differential diagnoses are listed in Table 10.9 (see page 514).

4 What further imaging would you recommend?


In the early stages of the disease the XR may be normal, and this partially explains why Perthes’
disease may be mistaken for transient synovitis. Therefore, it is important not to be falsely
­reassured by normal imaging. MRI is recommended for detecting very early changes and is par-
ticularly useful where there is a normal XR despite ongoing symptoms.

5 What are the management options?


Management depends on the severity of disease at presentation. Early diagnosis and manage-
ment is the key to preventing early onset OA. The main aims of treatment are to reduce pain,
preserve the hip range of movement, and prevent deformity.

513

K30031_Book.indb 513 9/6/17 1:39 PM


10   Paediatric cases

Table 10.9  Differential diagnosis for a child with an acutely painful limp

Differential How this may present as a limp


Transient synovitis of Also known as irritable hip, this is the most common cause for an acute painful
the hip limp in school age children (2–12 years of age) and often follows a viral illness
Septic arthritis Septic arthritis is an infected joint
This can quickly lead to irreversible cartilage destruction and early onset
osteoarthritis
Blood inflammatory markers, blood cultures, and joint aspiration are required to
confirm the diagnosis followed by early IV antibiotics and joint washout in theatre
Septic arthritis usually presents with a red hot swollen painful joint and is is
more common in children less than 2 years old
Slipped capital This is posteroinferior displacement of the capital femoral epiphysis presenting
femoral epiphysis with acute or gradual onset hip/knee pain and a limp
Most common in obese adolescent boys
Urgent surgical treatment is required to prevent avascular necrosis of the
femoral head
Leukaemia Leukaemia can present with a limp owing to bony involvement
These children tend to appear pale and unwell
There are usually other symptoms such as bruising and bleeding due to low
platelets, and signs of infection due to the compromised immune system
Trauma related injury A history of trauma is usually offered

• Conservative management may be sufficient in young children with early disease and only
minor changes to the femoral head. This can include observation of the hip joint with serial
XRs to ensure there is adequate bone regrowth. Physiotherapy and analgesia (including
NSAIDs) help pain and stiffness.
• Further damage to the femoral head can be prevented by limiting high-impact activities
and sometimes crutches are used to prevent any weight being put through the affected hip
joint.
• Plaster casts and braces may be used to keep the hip in abduction, thereby maintaining the
femoral head within the acetabulum to promote adequate healing. This is usually kept on
for 4–6 weeks but may need to be repeated until healing is complete.
• Surgery is usually reserved for more severe cases, where the child is older (over 8 years
old at diagnosis) or where conservative management has been unsuccessful. This usually
involves osteotomy of the femur or pelvis followed by a cast to hold the hip in position
while healing occurs. Physiotherapy, crutches, and analgesia again play a role.

6 What are the potential complications?


In the shorter term, complications are largely related to the degree of deformity after healing.
These may include pain, stiffness, and reduced range of movement.

• Indicators of poor prognosis in the longer term include late diagnosis, older age at onset,
and severe disease of the femoral head with residual deformity after healing.

514

K30031_Book.indb 514 9/6/17 1:39 PM


Case 10.9: Answers

• Each of these factors increases the potential for OA in adulthood, with a poorer prognosis
related to the development of OA at an earlier age. This may necessitate hip joint
replacement.
• With prompt diagnosis and early intervention, the prognosis is generally very good.

LEARNING POINTS: PERTHES’ DISEASE


▪▪ Perthes’ disease is avascular necrosis of the capital femoral epiphysis.
▪▪ It often presents with a limp and hip or knee pain, in children aged 4–10 years, and is more
common in boys than girls.
▪▪ Initial XRs can appear normal leading to a misdiagnosis of transient synovitis of the hip. Further
imaging is required if symptoms persist.
▪▪ The aims of treatment are to reduce pain, preserve range of movement, and prevent deformity.
Treatment may be conservative or involve surgery.
▪▪ Prognosis depends on the level of residual deformity, age at diagnosis, and severity at diagnosis.
The main consequence in later life is OA.

515

K30031_Book.indb 515 9/6/17 1:39 PM


Case 10.10

Fig. 10.10A  Supine CXR.

A 3-month-old male is taken to the ED with drowsiness and lethargy. His breathing is laboured
and he is floppy. His parents give a history of him becoming quiet after rolling off a couch and
onto the carpeted floor. There is no relevant past medical or family history; however, the family is
known to social services owing to a history of paternal substance abuse.
On examination the child is drowsy and lethargic with a bulging anterior fontanelle. The right
leg appears swollen and red. There are skin bruises overlying his chest.
A CXR and an XR of the right lower leg are obtained (Figures 10.10A and 10.10B).

516

K30031_Book.indb 516 9/6/17 1:39 PM


Case 10.10: Questions

Fig. 10.10B  XR of the right lower leg. The left image is a lateral view; the right image is a magnified lateral
view of the ankle/lower leg.

CASE 10.10: QUESTIONS


1 What do the XRs show?
2 What are your concerns regarding this child?
3 How should the child be managed initially?
4 What further investigations should be performed?

517

K30031_Book.indb 517 9/6/17 1:39 PM


10   Paediatric cases

CASE 10.10: ANSWERS


1 What do the XRs show?
The CXR (Figure 10.10C) shows multiple bilateral posterior and lateral rib fractures with exuber-
ant soft and hard callus formation. The lower leg XR shows a corner fracture of the distal tibial
metaphysis (Figure 10.10D).

• The precise dating of fractures is not possible. Estimates can be made, however, based on
the pattern of XR changes (Table 10.10A).

Fig. 10.10C  CXR showing posterior rib fractures on both sides of the chest (arrows). Soft and hard callus
formation is present indicating subacute fractures of differing ages.

518

K30031_Book.indb 518 9/6/17 1:39 PM


Case 10.10: Answers

Fig. 10.10D  XR of the


right lower leg showing
a triangular fragment of
bone at the corner of
the distal metaphysis of
the right tibia (arrowed).
This is known as a
metaphyseal corner
fracture.

Table 10.10A  Radiographic changes in childhood fractures

Radiological features Early Peak Late


Soft-tissue resolution 2–5 days 4–10 days 10–21 days
Periosteal new bone 4–10 days 10–14 days 14–21 days
Loss of the fracture line definition 10–14 days 14–21 days 42–90 days
Soft callus 10–14 days 14–21 days 2 years
Hard callus 14–21 days 21–42 days
Remodelling 3 months 1 year

519

K30031_Book.indb 519 9/6/17 1:39 PM


10   Paediatric cases

2 What are your concerns regarding this child?


The XR findings of multiple fractures of different ages, at different sites, is suspicious for nonac-
cidental injury (NAI). In particular, metaphyseal corner fractures and posterior rib fractures have
a high specificity for NAI. The clinician should be further alerted by the injuries not correlating
with the given history and the additional social concerns.

• Rib fractures in infants are almost always secondary to abuse. Up to 82% of rib fractures in
children <1 year of age are caused by abuse. The classic location is within the posterior ribs
at the articulation with the transverse process. Accidental rib fractures are extremely rare,
even in the presence of major trauma. The incidence of rib fractures in children with proven
NAI is 5–27%. Fractures are caused by compression of the chest while the baby is being
shaken.
• Metaphyseal corner fractures are also known as bucket handle fractures. These fractures
are highly specific for NAI. They are most commonly seen in nonambulatory infants under
18 months of age. The most frequent locations are around the feet and ankles. The fractures
are caused by twisting of the limb or from acceleration/deceleration forces applied to the limb
as a result of shaking. Other fractures that are associated with NAI are listed in Table 10.10B.

Table 10.10B  Fractures associated with NAI


High specificity Metaphyseal corner fractures
Rib fractures
Scapular fractures
Sternal fractures
Spinous process fractures
Moderate specificity Multiple fractures
Fractures of differing ages
Finger fractures in nonmobile children
Vertebral body fractures
Epiphyseal separation
Complex skull fractures
Low specificity Midclavicular fractures
Simple linear skull fractures
Long bone diaphyseal fractures
Greenstick fractures

3 How should the child be managed initially?


The child should be stabilised and resuscitated with an ABCDE approach. This may entail the
need for invasive ventilation, particularly in the context of an associated head injury and reduced
conscious level. The child may be in haemodynamic shock if there is internal haemorrhage and
will require monitoring of his BP with fluid resuscitation.
The paediatric team should be alerted immediately. A detailed clinical history and examina-
tion is required to gather information of the mechanism of injury and to look for any other signs of
physical abuse. Measures should be taken to ensure the wellbeing of the child. The lead clinician
with a responsibility for child safeguarding should be informed and child protection procedures
initiated. The child should be admitted to hospital both for treatment and as a place of safety. The
social care team and police will need to be informed as there may be siblings who are also at risk.

520

K30031_Book.indb 520 9/6/17 1:39 PM


Case 10.10: Answers

4 What further investigations should be performed?


The child will need further investigation with a CT brain scan and an XR skeletal survey.

• CT is the first-line neuroimaging used to exclude traumatic brain injury (Figure 10.10E).


An MRI may also be used to help date any identified intracranial haemorrhage.
• XR skeletal surveys are used to exclude further fractures anywhere else in the
skeleton. A repeat skeletal survey is also performed after 2 weeks to detect any
previously occult fractures by the identification of callus formation. Occasionally, a
nuclear medicine bone scan is used to detect occult fractures before callus has had a
chance to form.
• Ophthalmological assessment is also recommended to look for retinal haemorrhages,
which are also associated with NAI.

Fig. 10.10E  CT brain of an infant through the posterior fossa and frontal lobe showing bilateral, large, low
density, chronic, subdural haematomas surrounding both cerebral hemispheres (A). There are also two
large, high density, posterior fossa collections in keeping with acute subdural haemorrhage (B).

521

K30031_Book.indb 521 9/6/17 1:39 PM


10   Paediatric cases

The two main mechanisms of intracranial haemorrhage in NAI are impact injuries from a
direct blow or from a shaking injury with acceleration and deceleration forces on the brain. Direct
trauma will result in a skull fracture and contusion to the underlying brain. A shaking injury is
characterised by subdural haemorrhages, often of different ages (Figure 10.10E). There is a high
association with parenchymal ischaemic damage owing to hypoxia. Retinal injury in the presence
of a subdural haemorrhage is very suggestive of an acceleration/deceleration mechanism of NAI.
The term ‘shaken baby syndrome’ describes the association of subdural haemorrhage, massive
cerebral oedema, fractured ribs, and metaphyseal injury in the absence of external signs of head
injury.

LEARNING POINTS: NONACCIDENTAL INJURY


▪▪ Rib fractures and metaphyseal corner injuries have a high specificity for NAI.
▪▪ NAI should be suspected in the context of unexplained trauma, particularly in a nonambulatory
child.
▪▪ Brain injury from NAI may be result from direct impact or from the acceleration/deceleration
forces produced by shaking.
▪▪ The term ‘shaken baby syndrome’ refers to the association of subdural haemorrhages, rib
fractures, and metaphyseal corner injuries in the absence of external signs of head injury.
▪▪ You must promptly communicate any abnormal findings or concerns about NAI to the senior
responsible clinician. If a Social Services referral is not made and you still have concerns, it is your
duty to contact the designated doctor for child protection within the Trust.

522

K30031_Book.indb 522 9/6/17 1:39 PM


Case 10.11

Fig. 10.11A  Supine neonatal CXR.

A woman goes into spontaneous labour at 28 weeks gestation. You are called with the neonatal
registrar to attend the delivery. The baby shows immediate signs of respiratory distress with cya-
nosis and expiratory grunting. Oxygen is administered via a bag, valve, and mask, and the child
is transferred to the NICU for respiratory support.
While on the unit, the baby’s respiratory status deteriorates and necessitates intubation and
ventilation. Arterial blood gases and a CXR are obtained (Figure 10.11A).

pH 7.20 (7.35–7.45) PaCO2 8 kPa (4.5–6 kPa)


PaO2 4 kPa (11–14 kPa) HCO3 35 mmol/L (18–25 mmol/L)

CASE 10.11: QUESTIONS


1 What is the diagnosis? 3 What is the underlying pathophysiology?
2 What are the three main radiographic 4 What are the short- and long-term
features? complications?

523

K30031_Book.indb 523 9/6/17 1:39 PM


10   Paediatric cases

CASE 10.11: ANSWERS


1 What is the diagnosis?
The diagnosis is respiratory distress syndrome (RDS). The blood gases show hypoxia, which is
universal in this condition, and the degree of hypoxia correlates well with the severity. In most
cases the PaCO2 will be raised in keeping with a respiratory acidosis.

2 What are the three main radiographic features?


The main CXR features of RDS are (Figure 10.11B):
• Small lung volumes.
• Air bronchograms.
• Fine granular (reticular nodular) parenchymal lung pattern.

Fig. 10.11B  Magnified


view of the neonatal CXR
(from Figure 10.11A).
Diffuse granular lung
markings with a ‘ground
glass’ appearance
are present. There is
lung volume loss with
only 5 anterior ribs
demonstrated (1–5).

524

K30031_Book.indb 524 9/6/17 1:39 PM


Case 10.11: Answers

The CXR features vary according to the severity of the condition:

• Mild disease: lung markings are uniform, diffuse, linear, and granular. They are often
described as having a ‘ground glass’ pattern. This describes mild diffuse opacification that
is not enough to obscure the pulmonary vasculature.
• Moderate disease: the opacification of the lungs is denser and coarser with the presence of
air bronchograms (Figures 10.11C and 10.11D).
• Severe disease: complete opacification of both lungs. This manifests as a total ‘white out’ on
the CXR.

Figs. 10.11C, D  Supine neonatal CXR (10.11C) and a cropped view of the left lower zone (10.11D) with
signs of moderate-severe RDS. There is bilateral volume loss and lung opacification. As the lungs become
denser and consolidated, air bronchograms appear (arrows).

525

K30031_Book.indb 525 9/6/17 1:39 PM


10   Paediatric cases

Fig. 10.11D

3 What is the underlying pathophysiology?


RDS is primarily a disease of prematurity. Half of all babies born before 30 weeks of gestation
develop RDS compared with 2% of those born after 35 weeks. The underlying pathophysiology is
the absence of lung surfactant, manufactured by type II alveolar pneumocytes, from around week
24 of gestation. Surfactant acts to lower the surface tension of the alveoli, preventing their collapse
on expiration. Without surfactant the alveoli collapse and epithelial cells undergo necrosis.
Treatment is, therefore, based on respiratory support of the immature and fragile lungs and
administration of synthetic surfactant.

4 What are the short- and long-term complications?


Short-term complications include:
• Air leak syndromes (pneumothorax, pneumomediastinum, and pulmonary interstitial
emphysema) owing to barotrauma (Figures 10.11E and 10.11F).
• Persistent patent ductus arteriosus (owing to reduced oxygen stimulus).
• Oxygen toxicity (from treatment).
• Pulmonary haemorrhage.
• Recurrent pulmonary infection.

526

K30031_Book.indb 526 9/6/17 1:39 PM


Case 10.11: Answers

Fig. 10.11E  Supine


neonatal CXR with
pulmonary interstitial
emphysema
in the left lung,
predominantly
perihilar, secondary
to previous
ventilation. Gas leaks
out of the small
airways and tracks
in a radial pattern
towards the hilum.
The condition gives
a cystic ‘bubbly’
appearance. Note
also the right upper
lobe collapse (A).

Fig. 10.11F  Supine


neonatal CXR with
a marked bilateral
zone of lucency
outlining the heart
and mediastinal
structures (dotted
lines). The findings
are in keeping with
pneumomediastinum
on a background of
RDS.

527

K30031_Book.indb 527 9/6/17 1:39 PM


10   Paediatric cases

Fig. 10.11G  Supine neonatal CXR with bilateral coarse reticular lung markings with cyst-like areas
of hyperexpansion. The findings are typical of bronchopulmonary dysplasia. Note also the correctly
positioned ETT and NG tube, also the clipped ductus arteriosus (arrow).

Long-term complications include: bronchopulmonary dysplasia (BPD), also know as chronic


lung disease of prematurity (CLD).
Infants who have a prolonged oxygen requirement beyond 28 days (or a corrected gestational
age of 36 weeks) are described as having BPD/CLD. The lung damage is a manifestation of pres-
sure and volume trauma from artificial ventilation. The CXR characteristically shows coarse inter-
stitial densities, caused by fibrosis and scarring, interspersed with cyst-like areas, which reflect
hyperexpansion, (Figure 10.11G).

LEARNING POINTS: RESPIRATORY DISTRESS SYNDROME


▪▪ RDS is primarily a disease of premature infants caused by the lack of endogenous surfactant,
without which the alveoli collapse and undergo necrosis.
▪▪ Treatment of respiratory distress is based on respiratory support of the immature lungs and
administration of synthetic surfactant.
▪▪ The three main CXR characteristics are small lung volumes, air bronchograms, and a fine granular
(reticular nodular) pattern.
▪▪ Air leak syndromes are a common short-term complication caused by alveolar rupture.
▪▪ As a result of chronic ventilator support the lungs can undergo pressure and volume trauma
leading to BPD.

528

K30031_Book.indb 528 9/6/17 1:39 PM


Case 10.12

Fig. 10.12A  Supine neonatal AXR.

A premature infant born at 24 weeks gestation begins to develop feed intolerance 3 days after
birth while on the NICU. There is bile stained vomiting and increasing volume of NG tube aspi-
rates. The abdomen is tender and distended. Feeding is stopped; however, the infant continues to
remain unstable with apnoeas, tachycardia, tachypnoea, and shock. An arterial blood gas con-
firms a metabolic acidosis and an AXR is performed (Figure 10.12A).

CASE 10.12: QUESTIONS


1 What does the AXR show? 3 What are the complications of this condition?
2 What is the diagnosis? 4 How is the condition treated?

529

K30031_Book.indb 529 9/6/17 1:39 PM


10   Paediatric cases

CASE 10.12: ANSWERS


1 What does the AXR show?
Multiple dilated bowel loops and intramural gas (pneumatosis intestinalis) are visible
(Figure 10.12B). There is no evidence of pneumoperitoneum to indicate bowel wall perforation,
or of any portal venous gas. The NG tube is adequately sited.

Fig. 10.12B  Magnified view of the same neonatal AXR as in Figure 10.12A showing gas within the bowel
wall (arrows, A) known as pneumatosis intestinalis.

2 What is the diagnosis?


In this clinical context the imaging appearances are pathognomonic for necrotising enterocolitis
(NEC). This is a severe enterocolitis, which can affect any part of the large or small bowel. The
commonest sites are the terminal ileum, caecum, and ascending colon. Inflammation starts at

530

K30031_Book.indb 530 9/6/17 1:39 PM


Case 10.12: Answers

the mucosal surface and progresses to transmural necrosis and perforation. Bowel discolouration,
distension, and pneumatosis, (submucosal or subserosal gas) are common findings at surgery.
Although a combination of ischaemic and infective aetiologies has been proposed, NEC is
usually considered idiopathic and multifactorial (Table 10.12A).

Table 10.12A  Risk factors for NEC


Prematurity
Intrauterine growth restriction
Perinatal asphyxia
Umbilical catheterisation
Hypoxia
Patent ductus arteriosus
Nonhuman milk feeds
Rapid introduction of enteral feeding
Polycythemia
Exchange transfusion
Cyanotic congenital heart disease

Neonates with NEC initially demonstrate nonspecific signs such as temperature instability
and increased gastric aspirates. As the disease progresses there is increasing abdominal disten-
sion with bloody stools and vomiting. In severe cases the abdominal wall will appear erythema-
tous and there may even be palpable bowel loops. The blood gases will usually show a metabolic
acidosis, indicative of shock. There may be hypoxia representing respiratory failure owing to
apnoea or diaphragmatic splinting, and this may indicate the need for respiratory support.
AXR is mandatory to confirm the diagnosis. The radiographic findings are listed in Table 10.12B.

Table 10.12B  AXR features of NEC


• Dilated bowel loops – often asymmetrical in distribution. May be fixed over serial AXRs
• Bowel wall thickening – due to oedema, often with thumb printing pattern
• Separation of bowel loops – due to the accumulation of intraperitoneal fluid
• Intramural gas (pneumatosis) – bubble-like lucencies, which can mimic stool (Figures 10.12B, 10.12D)
• Portal venous gas – intramural gas is absorbed into the mesenteric veins and into the portal
venous system. It manifests as linear branching lucencies within the liver (Figure 10.12C)
• Pneumoperitoneum secondary to perforation – air on both sides of the bowel (Rigler’s sign) or air
outlining the falciform ligament (football sign)

3 What are the complications of this condition?


The natural history of NEC is of transmural bowel necrosis leading to perforation and peritonitis.
There is a 30% mortality associated with this condition.
It is not always possible to determine free gas on a supine AXR; indeed, only 50–75% of
patients with proven perforation will have visible free gas. If perforation is suspected then a hori-
zontal shoot-through XR, with the patient lying on their left side, will increase the chance of

531

K30031_Book.indb 531 9/6/17 1:39 PM


10   Paediatric cases

Fig. 10.12C  Magnified view of another neonatal AXR showing linear lucencies across the liver (arrows, A)
indicative of portal venous gas.

detecting free gas gathering alongside the liver. Note: an erect CXR will not be possible in a
ventilated neonate.

4 How is the condition treated?


This is a neonatal emergency and the patient should initially be treated with an ABCDE
approach. This may require assisted ventilation if there is respiratory distress. A NG tube
should be inserted if not already in place. With suspension of enteral nutrition, parenteral nutri-
tion should be commenced. Antibiotic treatment is given empirically to treat any underlying
infection.
Further management can be divided into medical and surgical. The aim of medical man-
agement is to allow the gut to rest and to treat any underlying infection with broad-spectrum
antibiotics.
Surgery is indicated in the following scenarios:

• Intestinal perforation.
• Formation of a mass.
• Failure to respond to medical management.

532

K30031_Book.indb 532 9/6/17 1:39 PM


Case 10.12: Answers

Fig. 10.12D  Whole body XR of a different premature neonate. There are features of RDS with diffuse
bilateral ground glass lung opacification. Also evidence of NEC with distended bowel loops, a mottled
bowel gas pattern, and pneumatosis (A). No evidence of pneumoperitoneum. Satisfactory nasogastric (B)
and endotracheal (C) tube positions.

533

K30031_Book.indb 533 9/6/17 1:39 PM


10   Paediatric cases

Surgery involves the removal of necrotic bowel, usually with the creation of a stoma. Overall,
10–40% of surviving patients go on to develop intestinal strictures, which may require further
surgery. Another complication is malabsorption owing to extensive gut resection, a condition
known as short gut syndrome.

LEARNING POINTS: NECROTISING ENTEROCOLITIS


▪▪ NEC is a multifactorial disease associated with prematurity and low birthweight.
▪▪ The clinical features are nonspecific with temperature instability and increased NG aspirates.
As the disease progresses there is increasing abdominal distension.
▪▪ AXR features include dilated bowel loops, bowel wall thickening, separation of bowel loops,
intramural gas, portal venous gas, and pneumoperitoneum.
▪▪ NEC is a neonatal emergency owing to its high mortality. If it is complicated by perforation, urgent
surgery is indicated.

534

K30031_Book.indb 534 9/6/17 1:39 PM


Case 10.13

Fig. 10.13A  AP view of the pelvis (female gonad protection applied).

A 14-year-old female presents to her GP with a limp and left hip pain. It started when she fell
over at school a few weeks earlier but the limp has not improved and the pain is worsening. On
examination, she has a raised BMI and her mother reports she has recently undergone a growth
spurt. She has restricted and painful abduction and internal rotation of the left hip. The remainder
of the examination is normal. Her GP arranges an XR.

CASE 10.13: QUESTIONS


1 What abnormalities can you see in Figure 10.13A?
2 What other XR view should you request?
3 What are the main clinical features of this condition?
4 What treatment is required?
5 What are the possible complications of this condition?

535

K30031_Book.indb 535 9/6/17 1:39 PM


10   Paediatric cases

CASE 10.13: ANSWERS


1 What abnormalities can you see in Figure 10.13A?
The AP view of the pelvis shows slippage of the left capital femoral epiphysis (Figure 10.13B).
The epiphysis appears relatively dense, the physis is widened, and there is disruption of Shenton’s
line.

Fig. 10.13B  AP view of the pelvis demonstrating on the right side the normal alignment of the
superior border of the femoral neck, which cuts through the lateral aspect of the femoral capital
epiphysis. This is known as Klein’s line (A). On the left, however, the epiphysis has slipped and this
line (B) no longer traverses the femoral capital epiphysis. Also note that Shenton’s line (C), which
follows the contour of the superior pubic ramus and inferomedial border of the femoral neck, is
disrupted on the left (D). The left femoral capital epiphysis has slipped posteromedially, disrupting
the smooth contour (E). The physis is widened on the left (F). Note also the white circle in the bottom
(patient’s right side) of the XR – this is the equivalent of a ‘red dot’ and is placed by the radiographer
if pathology is suspected. Take note of these as they can guide you and ask the radiographer what
concerned them if you are unsure.

2 What other XR view should you request?


It is important to also request a frog leg lateral view in children with hip pain (Figure 10.13C).
This view gives better visualisation of the medial aspect of the hip joint, the growth plate, and
femoral neck. This view is particularly useful where the suspected diagnosis is either slipped
capital femoral epiphysis (SCFE) or Perthes’ disease.

536

K30031_Book.indb 536 9/6/17 1:39 PM


Case 10.13: Answers

Fig. 10.13C  Frog leg lateral view of the pelvis. There is posteromedial slip of the left superior capital
femoral epiphysis (A) with widening of the physis (B). Note the gonadal shield used to protect the female
reproductive organs from radiation exposure. Normal right capital femoral epiphysis (C).

3 What are the main clinical features of this condition?


SCFE (or slipped upper femoral epiphysis, SUFE) is posterior and medial displacement of the
femoral head at the growth plate (physis). There is weakening of the physis, which leads to slip-
page of the femoral head.

• It typically occurs in overweight adolescent boys. The exact cause is unknown but identified
risk factors include a positive family history, raised BMI, certain endocrine disorders
including hypothyroidism, and renal osteodystrophy.
• The typical age of onset is 12–15 years. Presentation in children less than 10 years of
age, particularly if they are of normal weight or short stature, may suggest an underlying
endocrine abnormality.
• SCFE usually presents insidiously over several weeks to months and often during a growth
spurt. Occasionally it can be triggered by minor trauma.
• It presents with intermittent hip or knee pain, which is worse with activity. There may be a
limp owing to the inability to weight bear on the affected side and reduced passive external
rotation of the leg (the leg appears outwardly turned).
• In the majority of patients only one hip joint is affected at presentation, although it presents
with bilateral disease in 20% of cases. Up to 40% of patients with a unilateral diagnosis will
go on to develop the problem in the opposite hip.

4 What treatment is required?


Management for SCFE is always surgical and should happen within 24 hours of diagnosis to
reduce the risk of complications. The surgery required depends on the degree of the slippage of

537

K30031_Book.indb 537 9/6/17 1:39 PM


10   Paediatric cases

Fig. 10.13D  AP view of the pelvis showing an example of in-situ fixation in a child with SCFE in their right
hip. The screw can be seen crossing the nearly fused physis (arrow).

the femoral head. Generally the treatment required is in-situ fixation with a cannulated screw,
which crosses the growth plate (Figure 10.13D). This maintains the position of the femoral head
and prevents further slippage. If the SCFE is unstable with severe slippage, open reduction may
be required to manipulate the femoral head back into its correct position before fixing it in place.

• Physiotherapy and adequate analgesia are an important part of postoperative recovery.


The child is kept partial weight bearing on crutches for 6–8 weeks until pain free with full
strength. Most patients make a good recovery, returning to a normal level of function, but
may be advised to avoid high-impact sports until their growth plate has closed. They will
be followed up with XRs until their growth plate has fused to ensure no further slippage of
the femoral head and to check for involvement of the opposite hip.
• Owing to the risk of involvement in the other hip, prophylactic surgery may be offered. This
would depend on the individual’s risk. For example, if the child was young at presentation
with endocrine abnormalities they would be at a higher risk of disease in the opposite hip.
However, an overweight child presenting in their teens, with no other risk factors, may
purely be monitored with XR follow-up to check for disease in the opposite hip.
• These children should be referred to a paediatrician for investigation of an underlying
cause particularly if the presentation is atypical, i.e. the child is not overweight and younger
than the usual age of presentation (<10 years). Investigations for endocrine abnormalities,
such as thyroid dysfunction and panhypopituitarism, should be considered in these
circumstances.

538

K30031_Book.indb 538 9/6/17 1:40 PM


Case 10.13: Answers

5 What are the possible complications of this condition?


Complications can occur if left untreated.

• The earlier a diagnosis of SCFE is made, the better the outcome for the child.
• If left untreated, SCFE will lead to increasing pain and deformity with reduced range of
movement in the hip. Avascular necrosis of the femoral head could occur if the blood supply
to the femoral head becomes disrupted; this is more likely with unstable SCFEs, which have
more significant slippage. If avascular necrosis has occurred, the child may eventually need
a total hip replacement.
• With in-situ fixation, there is the possibility that the child may outgrow their screw and
require a revision surgery.
• Most children do well with prompt surgery. Those children with more severe slippage and
deformity are more likely to develop problems, such as avascular necrosis, as well as OA in
later life.

LEARNING POINTS: SLIPPED CAPITAL FEMORAL EPIPHYSIS


▪▪ SCFE is posterior and inferomedial displacement of the femoral head around the growth plate
(physis).
▪▪ It frequently occurs in children aged 12–15 years; it is more common in boys and if the child is
overweight.
▪▪ There are certain predisposing conditions such as renal osteodystrophy and some metabolic
endocrine conditions.
▪▪ Diagnosis is with XR, including frog lateral view of the hips, where displacement of the epiphysis is
seen on the affected side(s).
▪▪ Management is surgical with in-situ fixation of the hip. If left untreated, avascular necrosis of the
femoral head may occur and this may cause problems in later life.

539

K30031_Book.indb 539 9/6/17 1:40 PM


Case 10.14

Fig. 10.14A  AP XR of the right upper humerus.

540

K30031_Book.indb 540 9/6/17 1:40 PM


Case 10.14: Questions

A 15-year-old male presents to his GP with 3 months of progressive pain and swelling in his right
shoulder. He is mildly pyrexial with a temperature of 37.5°. The shoulder is swollen and painful,
and there is a palpable soft-tissue mass.
The GP, concerned about osteomyelitis, refers the child urgently for an orthopaedic opinion.
In the hospital, blood tests and an XR of the right shoulder are requested (Figure 10.14A).
The bloods reveal a mild anaemia with a Hb of 9 g/dL (12–16 g/dL) and a leukocytosis with a
WCC of 16 × 109/L (4.5–13.5 × 109/L). The ESR is raised at 50 mm/hr (0–10 mm/hr).

CASE 10.14: QUESTIONS


1 Describe the XR appearances. 3 What is the most likely diagnosis?
2 What are the key differential diagnoses? 4 How should this patient be managed?

541

K30031_Book.indb 541 9/6/17 1:40 PM


10   Paediatric cases

CASE 10.14: ANSWERS


1 Describe the XR appearances.
The upper humerus has an abnormal permeative texture with small rounded lucencies particu-
larly in the metaphysis (Figure 10.14B). The destruction has a broad zone of transition such that
it is difficult to determine the boundary between the abnormal and normal bone. There is also a
periosteal reaction. These are features of an aggressive destructive process.

Fig. 10.14B  XR of the right shoulder with an aggressive destructive lesion in the humeral metaphysis
(circled region) with a permeative or ‘moth-eaten’ appearance extending into the humeral shaft. Note the
periosteal reaction (P) and broad zone of transition.

542

K30031_Book.indb 542 9/6/17 1:40 PM


Case 10.14: Answers

2 What are the key differential diagnoses?


The differential for these aggressive, destructive appearances includes neoplastic and infective
causes (Table 10.14).

• Periosteal reactions are caused by local damage to the periosteum (the connective tissue
membrane around bone). There are benign and aggressive patterns of reaction that help
differentiate the likely cause. The most common causes are trauma, infection, and malignancy.
• In slow, chronic, inflammatory processes, the periosteum has time to remodel and lay down
new bone. The periosteal reaction is wave-like and thickened.
• In rapid, aggressive processes (as in this case), new bone is laid down in a disorganised
manner before the periosteum has time to consolidate. The periosteal reaction is irregular
and can be amorphous, multi-lamellated (onion-skin appearance) or sunburst. Codman’s
triangle describes a triangular area of periosteal new bone created when a tumour raises
periosteum from the bone.
Table 10.14  The differential for ‘moth-eaten bone’
pattern in a child
Neoplastic Neuroblastoma metastases
Leukaemia
Ewing’s sarcoma
Lymphoma
Osteosarcoma
Infective Acute osteomyelitis

3 What is the most likely diagnosis?


The diagnosis in this case is Ewing’s sarcoma, a highly malignant primary bone tumour. This
belongs to a family of small blue round cell tumours, which also includes lymphoma, rhadomyo-
sarcoma, and primitive neuroectodermal tumours. It is the second most common bone tumour
in children after osteosarcoma (osteosarcoma 60%; Ewing’s 30%) and has a peak incidence at
10–15 years of age.
Ewing’s can cause pain, swelling, and limitation of movement, which is sometimes incorrectly
ascribed to a sports injury. There may be systemic symptoms and signs such as lassitude and
fever, and a leukocytosis and raised ESR are common. These features can mimic osteomyelitis,
which has similarly aggressive appearances on XR.

4 How should this patient be managed?


This is a rare tumour and should be managed with a multidisciplinary approach in a specialist bone
tumour centre where the relevant orthopaedic, oncology, and diagnostic expertise is gathered.
The child should have an MRI scan to evaluate the intraosseous extent of the disease and also
the soft-tissue component, which is typically extensive (Figure 10.14C).
A CT scan of the chest, abdomen, and pelvis is performed to assess for distant metastatic
spread. A radioisotope bone scan can also help to identify bone metastases.
Core biopsy of the lesion is necessary and should be undertaken at the specialist unit for histo-
logical confirmation, and carefully planned to avoid compromising the potential for limb-salvage
surgery.
Treatment involves chemotherapy and resection of the tumour with limb-sparing surgery if
possible.

543

K30031_Book.indb 543 9/6/17 1:40 PM


10   Paediatric cases

Fig. 10.14C  MR
T2 fat saturated
coronal image of
the right shoulder
in the same child.
Note: the extent
of the soft-tissue
component of the
tumour (arrows)
surrounding the
bone, and high
signal involvement
of the proximal
humeral epiphysis
(A), metaphysis (B),
and diaphysis (C).

LEARNING POINTS: EWING’S SARCOMA


▪▪ Ewing’s sarcoma is the second most common bone tumour in children, after osteosarcoma, and has
a peak age of incidence at 10–15 years. Urgent referral to a specialist bone tumour unit is essential.
▪▪ The radiological features of an aggressive bone lesion include:
– Cortical destruction.
– A broad zone of transition.
– Irregular, amorphous, multilamellated or sunburst periosteal reaction.
▪▪ MRI can evaluate the intraosseous and significant extraosseous components of the disease.
▪▪ Chemotherapy followed by limb-sparing surgery is the mainstay of treatment.
▪▪ Osteomyelitis can have a similar appearance and should always be considered and excluded.

544

K30031_Book.indb 544 9/6/17 1:40 PM


Case 10.15

Fig. 10.15A  XR of the left wrist with AP (left) and lateral (right) views.

A 9-year-old female presents to accident and emergency after falling off a trampoline and landing
on her outstretched left arm. She cried immediately and now complains of pain in her left wrist
and arm. On examination, her wrist is tender and she is reluctant to move it. The overlying skin is
intact. Her remaining examination is normal. An XR is arranged (Figure 10.15A).

CASE 10.15: QUESTIONS


1 Explain the importance of obtaining more than one XR view in suspected bone trauma.
2 Describe the XR abnormality.
3 What is the physis and how are physeal fractures classified?
4 What complications can arise as a result of this type of injury?
5 Name the common fracture types in children.

545

K30031_Book.indb 545 9/6/17 1:40 PM


10   Paediatric cases

CASE 10.15: ANSWERS


1 Explain the importance of obtaining more than one XR view in suspected
bone trauma.
When obtaining an XR of a suspected bone fracture it is important to review at least two orthogo-
nal plane views at 90° to one other. Generally, AP and lateral are adequate. Some bones and
joints require a third view for further assessment, usually an oblique view, and examples of these
include the metacarpal bones and the paediatric elbow.
It is difficult for very young children to localise their pain. For this reason, it is important to
examine the joints above and below the suspected site of injury. For example, a child with pain in
their forearm should have both their wrist and elbow examined. Additional XRs of these joints
may be necessary.

2 Describe the XR abnormality.


The XR shows a fracture-dislocation of the left distal radial epiphysis (Figure 10.15B). The frac-
ture line passes straight through the growth plate and has caused the epiphysis to dislocate dor-
sally (posteriorly). This pattern of fracture through the growth plate is known as Salter–Harris
(SH) type 1.

Fig. 10.15B  AP (left) and lateral (right) views of left wrist, showing a fracture-dislocation of the left distal
radial epiphysis. On the AP projection, this manifests as overlap of the epiphysis and metaphysis at the
growth plate (A). On the lateral projection, the radial epiphysis (B) and carpus bones have slipped dorsally
off the radial metaphysis (C). Note the normal radial shaft (D), ulna (E), and 1st metacarpal (thumb, F).

546

K30031_Book.indb 546 9/6/17 1:40 PM


Case 10.15: Answers

TOP TIP
If you are given a history of trauma and only one view on XR, tell the examiner that you would like to
review an orthogonal view.

3 What is the physis and how are physeal fractures classified?


The physis is the growth plate in developing bones and it is seen between the metaphysis and
the epiphysis. Tubular bones grow via a process of endochondral bone formation whereby cells
migrate from the physis towards the metaphysis, simultaneously pushing the growth plate in the
opposite direction and leading to bone growth. As the cells migrate they mature and calcify lay-
ing down new bone.
The physis forms a line of weakness in the immature bone and is prone to fracture. The phy-
sis is involved in as many as 18% of paediatric long bone fractures. Physeal injuries are grouped
according to the SH classification, of which there are 5 types. Figures 10.15C–10.15G are illustra-
tions of the SH classification (Types I–V), with real examples.

Direction of
displacement Fracture
E line

Fig. 10.15C  Type 1 fractures involve the physis only (as in the patient at the start of this case);
the epiphysis (E) slips in relation to metaphysis (M).

TOP TIP
A useful mnemonic to remember the Salter–Harris classification is SALTR
S lipped Type I
A bove the growth plate Type II
BeLow the growth plate Type III
T hrough the growth plate Type IV
R ammed growth plate Type V

547

K30031_Book.indb 547 9/6/17 1:40 PM


10   Paediatric cases

Fig. 10.15D
Type II fractures
Fracture involve the
line metaphysis (M)
and the physis
(growth plate).
This example
demonstrates a
fracture of the
distal radius
(small arrows).
These are the
most common
type of physeal
injury.

M
E

Fracture
line

Fig. 10.15E  Type III fractures involve


the growth plate and the epiphysis (E).
This is an XR of the proximal phalanx
of a big toe (inverted); the epiphyseal
fracture is arrowed. These injuries
have a greater risk of growth arrest.
Metaphysis (M).

548

K30031_Book.indb 548 9/6/17 1:40 PM


Case 10.15: Answers

Fig. 10.15F  Type Fracture M


IV fractures extend line
from the metaphysis
(M) through the
growth plate and the
epiphysis (E). This
example shows a
fracture (white arrows)
through the growth
plate of a finger
(inverted). These
injuries also have a
higher tendency to
growth arrest.

Compression
of the physis

Fig. 10.15G  Type V fractures are compression fractures of the


physis owing to an axial crush injury. These are rare and difficult
to detect. They often only become apparent after growth arrest.
Metaphysis (M), epiphysis (E).

549

K30031_Book.indb 549 9/6/17 1:40 PM


10   Paediatric cases

4 What complications can arise as a result of this type of injury?


The most important complication of a SH fracture is growth disturbance owing to premature
fusion of the growth plate. This includes both growth acceleration and retardation, and can
lead to limb length discrepancies and angular deformation. The complications that arise from
these depend on the location, plane of injury, SH type, fracture severity, and the age of the
child. Growth disturbance tends to be more severe with higher grades of SH injury. Older
children in their teens tend to have fewer complications because of their reduced growth
potential.

• Most SH fractures can be managed conservatively with closed reduction and application
of a cast. More severe fractures, particularly those with joint involvement, may require
surgical fixation. The complications that can arise as a result of these fractures may require
further management as the child grows. SH type V fractures are very difficult to detect and
may have very subtle XR changes. They tend to present months or years after the injury
with problems such as leg length discrepancy or gait disturbance. In these cases, MRI can
play an important role in the initial diagnosis.

5 Name the common fracture types in children.


Paediatric bones are elastic and have a greater propensity to bend and bow before fracturing.
This produces certain patterns of fractures, which are unique to the paediatric age group. These
include greenstick, torus, and plastic bowing fractures (Figures 10.15H–10.15J).
• These fractures are incomplete, in that the fracture involves one cortex and does not
extend through the entire diameter of the bone. These fracture types are commonly
seen in the diaphysis of the radius and ulna of the forearm. The mechanism of fracture
is from an angular or longitudinal force exerted on the forearm, for example from a
FOOSH.
• A greenstick fracture results from an angular force on the bone. The bone bends and results
in a break in the cortex on the convex site of the bone, while the opposite cortex remains
intact. The appearances are likened to bending an immature tree branch, hence ‘greenstick’
(Figure 10.15H).
• Sometimes the immature bone may just bend without fracturing. This is known as a
plastic bowing fracture (Figure 10.15H). There is no visible fracture line on XR but
microfractures are seen under the microscope. The diagnosis is easy when an adjacent
bone in the arm or leg is fractured. If there are no associated fractures and there is
diagnostic uncertainty, comparison with the same bone on the other side may be
necessary. Also, repeating the XR after 10–14 days to look for callus formation may help
to confirm the diagnosis.
• Buckle fractures are also known as torus fractures. The cortex at the site of injury
buckles rather than fractures. The cortex can bulge in or out on the side of compression.
The fracture may appear as a subtle bump or undulation of the cortex (Figure 10.15I).
They are commonly seen in the distal metaphyses of the radius, ulna, tibia, fibula,
metacarpals, and phalanges.

550

K30031_Book.indb 550 9/6/17 1:40 PM


Case 10.15: Answers

Fig. 10.15H  Left forearm XR, with magnified view (right) of the distal forearm, showing a greenstick
fracture of the radius (A) and a plastic bowing fracture of the ulna (B).

• Toddler’s fractures are spiral or oblique fractures of the distal tibia without displacement
(Figure 10.15J). They occur in young ambulatory children (from 9 months to 3 years
of age) and result from a rotational force on the limb caused by falling and twisting on
the leg. The periosteum remains intact and the bone is stable. The lack of displacement
in such fractures means that they are easily missed on XR and an additional oblique view
may be required if there is a strong clinical suspicion of injury. A repeat XR at 10–14 days
after the injury will demonstrate a periosteal reaction. These fractures heal well with a
plaster cast.

551

K30031_Book.indb 551 9/6/17 1:40 PM


10   Paediatric cases

Fig. 10.15I  Wrist XR with AP (left) and lateral (right) views showing a buckle fracture of the distal radius.
Note the angular deformity of the radius on the lateral projection with buckling (A) of the cortex rather than
a simple fracture line. The outward bulge seen on the AP projection is also referred to as a torus fracture (B).
There is also a subtle torus fracture of the distal ulna (C).

552

K30031_Book.indb 552 9/6/17 1:40 PM


Case 10.15: Answers

Fig. 10.15J  Lower leg XRs, AP (right) and lateral (left) views, showing an oblique Toddler’s fracture of the
distal tibia (arrows). Note the lack of displacement. The fracture is not seen on the lateral projection, further
demonstrating the importance of reviewing two orthogonal views.

LEARNING POINTS: PAEDIATRIC FRACTURES


▪▪ At least two orthogonal views are required to fully assess a fracture on an XR.
▪▪ The growth plate or physis is the site of new bone formation and bone growth.
▪▪ The physis forms a line of weakness in the immature bone.
▪▪ Physeal fractures are organised according to the Salter–Harris classification.
▪▪ The paediatric skeleton is malleable, and the bones can bend and buckle with trauma rather than
completely fracture.

553

K30031_Book.indb 553 9/6/17 1:40 PM


K30031_Book.indb 2 9/6/17 1:34 PM
Bibliography

Banerjee AK (2006) Radiology Made Easy. Cambridge: Cambridge University Press.


Barrett T, Shaida N, Shaw A, Dixon AK (2010) Radiology for Undergraduate Finals and Foundation
Years: Key Topics and Question Types. Oxford: Radcliffe Publishing.
Begg JD (2006) Abdominal X-rays Made Easy, 2nd edn. Edinburgh: Elsevier Health Sciences.
Chan O (ed.) (2013) ABC of Emergency Radiology, 3rd edn. Oxford: Wiley-Blackwell.
Chowdrey R, Wilson I, Rofe C, Lloyd-Jones G (2010) Radiology at a Glance. Chichester:
John Wiley.
Clarke C (2015) Abdominal X-rays for Medical Students. Chichester: Wiley-Blackwell.
Daffner RH (2007) Clinical Radiology: The Essentials, 3rd edn. Baltimore: Wolters Kluwer/
Lippincott Williams & Wilkins.
Darby MJ, Barron DA, Hyland RE (2012) Oxford Handbook of Medical Imaging. Oxford: Oxford
University Press.
Davies SG (2013) Chapman & Nakielny’s Aids to Radiological Differential Diagnosis, 6th edn.
Edinburgh: Saunders Ltd.
de Lacey G, Morely S, Berman L (2008) The Chest X-ray. A Survival Guide, 2nd edn. Edinburgh:
Elsevier Health Sciences. 
Grainger RG, Adam A, Dixon AK, Allison DJ (2007) Grainger and Allison’s Diagnostic Radiology:
A Textbook of Medical Imaging: Volumes 1–2, 4th edn. London: Churchill Livingstone.
Howlett DC, Ayers B (2004) The Hands-on Guide to Imaging. Oxford: Blackwell Publishing.
Howlett DC, Gainsborough N (eds.) (2013) 100 Cases for Medical Data Interpretation. Boca Raton:
Hodder Education.
Hussain SM, Latif SAA, Hall AD (2010) Rapid Review of Radiology. London: Manson Publishing.
Lisle DA (2007) Imaging for Students. London: Hodder Arnold.
Mettler FA (2013) Essentials of Radiology, 2nd edn. Philadelphia: Elsevier Health Sciences.
Moeller TB, Reif E (1999) Pocket Atlas of Radiographic Anatomy, 2nd edn. Stuttgart: Thieme
Publishing Group.
Patel PR (2010) Lecture Notes, Radiology. Chichester: Wiley-Blackwell.
Qureshi Z (2012) The Unofficial Guide to Passing OSCEs, 3rd edn. London: Zesham Qureshi.
Raby N, Berman L, de Lacey G (2005) Accident and Emergency Radiology. A Survival Guide,
2nd edn. Edinburgh: Elsevier Health Sciences.
Renton P, Butler P (2004) Medical Imaging: An Illustrated Colour Text. Edinburgh: Elsevier
Churchill Livingstone.
Rodrigues M, Qureshi Z (eds.) (2014) The Unofficial Guide to Radiology: Chest, Abdominal and
Orthopaedic X-rays, plus CTs, MRIs and Other Important Modalities. Core Radiology Curriculum
Covered: 100 Annotated X-rays (including how to present them), 300 Multiple Choice Questions
(with detailed explanations). London: Zeshan Qureshi.
Semelka RC, Birchard KR, Busireddy KR (2015) Critical Observations in Radiology for Medical
Students. Chichester: Wiley-Blackwell.

555

K30031_Book.indb 555 9/6/17 1:40 PM


Bibliography

Shaw AS, Godfrey EM, Singh A (2009) Radiology: Clinical Cases Uncovered. Chichester:
Wiley-Blackwell.
Ter Meulen D, Kelly B, Bickle IC (2008) Imaging. Philadelphia: Mosby/Elsevier.
Wasan R, Grundy A, Beese R, Wasan R (2004) Radiology Casebook for Medical Students, 2nd edn.
London: Pastest.
Weill FS, Manco-Johnson ML (1997) Imaging of Abdominal and Pelvic Anatomy. London: Churchill
Livingstone.

556

K30031_Book.indb 556 9/6/17 1:40 PM


List of cases

Case 5.1 Lung cancer 52 Case 7.1 Haematuria/hydronephrosis 206


Case 5.2 Pancoast tumour 58 Case 7.2 Pneumoperitoneum 211
Case 5.3 Cavitating lung abscess 62 Case 7.3 Gallstone ileus 215
Case 5.4 Pulmonary metastases 67 Case 7.4 Small bowel obstruction 219
Case 5.5 Mesothelioma and pleural Case 7.5 Large bowel obstruction 223
plaques 71 Case 7.6 Porcelain gallbladder 227
Case 5.6 Pulmonary fibrosis 76 Case 7.7 Renal colic 230
Case 5.7 Pneumothorax 81 Case 7.8 Emphysematous pyelonephritis 236
Case 5.8 Tuberculosis 86 Case 7.9 Ectopic pregnancy 240
Case 5.9 Pneumonic consolidation 91 Case 7.10 Sickle cell disease 245
Case 5.10 Free intraperitoneal air 95 Case 7.11 Chronic pancreatitis 249
Case 5.11 Subphrenic abscess 100 Case 7.12 Liver metastases 253
Case 5.12 Mycetoma 105 Case 7.13 Renal cell carcinoma 257
Case 5.13 Pleural effusion/opacified Case 7.14 Pneumatosis coli 261
hemithorax 109 Case 7.15 Acute cholecystitis 265
Case 5.14 Left lower lobe collapse 115 Case 7.16 Crohn’s disease 269
Case 5.15 Right upper lobe collapse 119 Case 7.17 Pancreatic cancer 273
Case 5.16 Chronic obstructive Case 7.18 Uterine fibroids 276
pulmonary disease 124 Case 7.19 Acute pancreatitis 281
Case 5.17 Bronchiectasis 130 Case 7.20 Ovarian carcinoma 286
Case 5.18 Nasogastric tube placement Case 7.21 Choledocholithiasis 291
and assessment 135 Case 7.22 The ileocaecal valve/caecal
Case 5.19 Achalasia of the oesophagus 141 volvulus 294
Case 5.20 Oesophageal carcinoma 145 Case 7.23 Sigmoid volvulus 299
Case 5.21 Hiatus hernia 150 Case 7.24 Acute appendicitis 303
Case 5.22 Thoracoplasty 155 Case 7.25 Retroperitoneal mass 307
Case 5.23 Sarcoidosis, bilateral hilar Case 7.26 Major abdominal trauma,
lymphadenopathy 159 splenic injury 311
Case 5.24 Multinodular thyroid/superior Case 7.27 Misplaced intrauterine
mediastinal mass 164 contraceptive device 315
Case 5.25 Breast lumps/breast
carcinoma 169 Case 8.1 Cervical spine fracture 320
Case 8.2 Osteoarthritis 327
Case 6.1 Aortic dissection 176 Case 8.3 Hip fracture 331
Case 6.2 Leaking abdominal aortic Case 8.4 Diabetic foot/osteomyelitis 335
aneurysm 181 Case 8.5 Pathological fracture, lytic lesion 339
Case 6.3 Pulmonary embolism 185 Case 8.6 Gout 343
Case 6.4 Cardiac failure 190 Case 8.7 Neuropathic (Charcot) joint 348
Case 6.5 Left ventricular aneurysm 195 Case 8.8 Rheumatoid arthritis 353
Case 6.6 Ischaemic leg 199 Case 8.9 Shoulder dislocation 358

557

K30031_Book.indb 557 9/6/17 1:40 PM


List of cases

Case 8.10 Prosthetic loosening 365 Case 10.1 Paediatric brain tumours 462
Case 8.11 Multiple myeloma 369 Case 10.2 Paediatric pneumonia 468
Case 8.12 Osteoporotic spinal fractures 372 Case 10.3 Cystic fibrosis/meconium ileus 474
Case 8.13 Paget’s disease 377 Case 10.4 Malrotation 480
Case 8.14 Metastatic breast cancer, Case 10.5 Neuroblastoma 486
sclerotic bone lesions 382 Case 10.6 Intussusception 495
Case 8.15 Colles fracture 388 Case 10.7 Congenital diaphragmatic
Case 8.16 Scaphoid fracture 394 hernia 501
Case 8.17 Ankylosing spondylitis 400 Case 10.8 Meconium aspiration
syndrome 505
Case 9.1 Subdural haematoma 408 Case 10.9 Perthes’disease 510
Case 9.2 Intracerebral haematoma 413 Case 10.10 Nonaccidental injury 516
Case 9.3 Brain tumour, meningioma 417 Case 10.11 Respiratory distress
Case 9.4 Subarachnoid haemorrhage 423 syndrome 523
Case 9.5 Multiple sclerosis 427 Case 10.12 Necrotising enterocolitis 529
Case 9.6 Extradural haematoma 433 Case 10.13 Slipped capital femoral
Case 9.7 Stroke 440 epiphysis 535
Case 9.8 Sagittal sinus thrombosis 446 Case 10.14 Ewing’s sarcoma 540
Case 9.9 Malignant spinal cord/cauda Case 10.15 Paediatric fractures 545
equina compression 450
Case 9.10 Cerebral abscess 455

558

K30031_Book.indb 558 9/6/17 1:40 PM


Index

Note: Page numbers in bold refer to figures; those in italic refer to tables or boxes.

abdomen and pelvis cases 205–317 abdominal X-ray (continued)


acute appendicitis 303–6 masses (solid viscera) 40
acute cholecystitis 265–8 presentation of findings 45–9
acute pancreatitis 281–5 radiation exposure 36
caecal volvulus 294–8 stones (calcifications) 41–2
choledocholithiasis 266, 291–3 technique 36–7
chronic pancreatitis 249–52 see also abdominal X-ray
Crohn’s disease 269–72 achalasia of the oesophagus 141–4
ectopic pregnancy 240–4 acid-and-alcohol fast bacilli (AFB) 89
emphysematous pyelonephritis 236–9 acquired immune deficiency syndrome (AIDS) 308
gallstone ileus 215–8 acromioclavicular (AC) 25
haematuria/hydronephrosis 206–10 acute appendicitis 303–6
ileocaecal valve 294–8 acute cholecystitis 265–8
intrauterine contraceptive device, misplaced acute pancreatitis 281–5
315–17 airway, breathing, circulation, diaphragm,
large bowel obstruction 223–6 everything else (ABCDE) 18, 22–25
liver metastases 253–6 alkaline phosphatase (ALP) 123
major abdominal trauma, splenic injury American Association for the Surgery of Trauma
311–14 (AAST) 312
ovarian carcinoma 286–90 angiotensin-converting enzyme (ACE) 197
pancreatic cancer 273–5 angiotensin receptor blocker (ARB) 193
pneumatosis coli 261–4 ankylosing spondylitis 400–5
pneumoperitoneum 211–14 anteroposterior (view) (AP) 18
porcelain gallbladder 227–9 antinuclear antibodies (ANA) 79
renal cell carcinoma 257–60 aortic arch (AA) 24, 179
renal colic 230–5 aortic dissection 176–80
retroperitoneal mass 307–10 appendicitis, acute 303–6
sickle cell disease 245–8 arterial blood gas (ABG) 523
sigmoid volvulus 299–302 as low as reasonably achievable (ALARA) 16, 36
small bowel obstruction 219–22 atrial fibrillation (AF) 201
uterine fibroids 276–80 avascular necrosis (AVN) 332
abdominal aortic aneurysm (AAA) 3, 181–4
abdominal X-ray (AXR) 1–2 Bacille Calmette–Guerin (BCG) 90
abdominal X-ray (normal) 35–49 bilateral hilar lymphadenopathy 159–63
artefacts 43–4 blood pressure (BP) 11, 28
bones 42 body mass index (BMI) 151
contraindications 35 bone lesions, sclerotic 382–7
exam technique 44–5 brain
gases 38 natriuretic peptide (BNP) 193
interpretation and presentation 37–44 tumour, meningioma 417–22
limitations 35–6 tumour, paediatric 462–7

559

K30031_Book.indb 559 9/6/17 1:40 PM


Index

breast carcinoma 169–73 cholecystitis, acute 265–8


bronchiectasis 130–4 choledocholithiasis 256, 281–2
bronchopulmonary dysplasia (BPD) 528 chronic lung disease of prematurity (CLD) 528
chronic lymphoid leukaemia (CLL) 160
caecal volvulus 294–8 chronic obstructive pulmonary disease (COPD)
calcium pyrophosphate deposition disease 94, 124–9
(CPPD) 344 chronic pancreatitis 249–52
cancer see also carcinoma Colles fractures 388–93
lung 52–7 common bile duct (CBD) 266
pancreatic 273–5 communication stations (for OSCE) 9–10
carcinoma computed tomography (CT) 1, 3–4
breast 169–73, 382–7 intravenous urogram (CT IVU) 4
ductal carcinoma in situ 173 kidneys ureters and bladder (CT KUB) 4
lobular carcinoma in situ 173 pulmonary angiogram (CTPA) 4
oesophageal 145–9 congenital diaphragmatic hernia (CDH) 501–4
ovarian 286–90 coronary artery bypass graft (CABG) 53
renal cell 257–60 craniocaudal (view) (CC) 170
urinary tract 67–70 C-reactive protein (CRP) 12, 181, 295
cardiac failure 190–4 Crohn’s disease 269–72
cardiothoracic ratio (CTR) 24, 196 CSF see cerebrospinal fluid
cardiovascular cases 175–203 cystic fibrosis (CF) 65, 474–9
abdominal aortic aneurysm, leaking 181–4 cystic fibrosis transmembrane conductance
aortic dissection 176–80 regulator (CFTR) 478
cardiac failure 190–4
ischaemic leg 199–203 deep vein thrombosis (DVT) 3, 186
left ventricular aneurysm 195–8 diabetic foot 335–8
pulmonary embolism 185–9 diffusion capacity of the lung for carbon
carpometacarpal (CMC) 354 monoxide (test) (DLCO) 162
cauda equina compression 450–4 diffusion-weighted imaging (DWI) 444
cavitating lung abscesses 62–6 digital subtraction angiography (DSA) 201
central nervous system (CNS) 430 disease modifying antirheumatic drugs
cerebral abscess 455–60 (DMARDs) 357
cerebrospinal fluid (CSF) 4 distal interphalangeal (DIP) 329, 355
chest X-ray (CXR) 1–2 distal radioulnar joint (DRUJ) 390, 392
chest X-ray (normal) 15–33 dual energy X-ray absorptiometry (DEXA)
ABCDE approach 22–5 17, 375
alternative ‘radiologist-review’ approach ductal carcinoma in situ (DCIS) 173
27, 30, 32 duodenojejunal (DJ) 482
anatomical approach 25–6 dynamic hip screw (DHS) 333
cases 26–33
example OSCE stations with CXR ear, nose, and throat (ENT) 168
interpretation 26–33 ectopic pregnancy 240–4
indications for requesting a CXR 15 electrocardiogram (ECG) 12, 161
medical ionising radiation exposure 15–16 emergency department (ED) 26
obvious abnormality, description of 21 emphysematous pyelonephritis 236–9
projection, personal demographics, previous endoscopic retrograde cholangiopancreatography
CXR comparison (PPP) 17–9 (ERCP) 266
reporting technique 16–26 endotracheal (ET) 25
rotation/inspiration/penetration (RIP) endotracheal tube (ETT) 507
17, 19–21 endovascular aneurysm repair (EVAR) 183
systematic review, approaches to 22–6 erythrocyte sedimentation rate (ESR) 12

560

K30031_Book.indb 560 9/6/17 1:40 PM


Index

estimated glomerular filtration rate (eGFR) 459 human leukocyte antigen (HLA) 355, 403
Ewing’s sarcoma 540–4 hydronephrosis 206–10
examination stations (for OSCE) 10–12
extracorporeal membrane oxygenation ideas concerns expectations (ICE) 10
(ECMO) 503 ileocaecal valve 294–8
extracorporeal shock wave lithotripsy imaging modalities, overview of 1–6
(ESWL) 235 computed tomography 3–4
extradural haematoma 433–9 fluoroscopy techniques 6
magnetic resonance imaging 4–5
fall on an outstretched hand (FOOSH) 393 nuclear medicine 5–6
fibroids 276–80 plain films 1–2
fluid-attenuated inversion recovery ultrasound 2–3
(FLAIR) 428 implantable cardiac defibrillator (ICD) 196
fluorodeoxyglucose (FDG) 5 inferior vena cava (IVC) 507, 509
fluoroscopy techniques 6 inflammatory bowel disease 269–72
advantages 6 intensive care unit (ICU) 193
disadvantages 6 intensive therapy unit (ITU) 94, 137
focussed assessment with sonography for trauma international normalised ratio (INR) 249
(FAST) 181 interphalangeal (IP) 343
forced expiratory volume (FEV) 129 intracranial pressure (ICP) 424
fracture, pathological 339–42 intrauterine contraceptive device (IUCD)
free intraperitoneal air 95–9 264, 315–17
full blood count (FBC) 12, 227 intravenous (IV) 3
full field digital mammography (FFDM) 171 intussusception, paediatric 495–500
ischaemic leg 199–203
gallbladder, porcelain 227–9
gallstone ileus 215–18 lactate dehydrogenase (LDH) 112
gamma-glutamyl transferase (GGT) 249 large bowel obstruction (LBO) 223–6
gastrointestinal (GI) 5, 141 last menstrual period (LMP) 241
gastro-oesophageal reflux disease (GORD) 153 left atrium (LA) 24
general practitioner (GP) 10 left heart border (LHB) 24
Glasgow coma scale (GCS) 99, 311 left lower lobe collapse 115–18
glenohumeral (GH) 25, 362 left upper quadrant (LUQ) 38, 96
glomerular filtration rate (GFR) 235 left ventricle (LV) 24, 25
glyceryl trinitrate (GTN) 11 left ventricular aneurysm (LVA) 195–8
gout 343–7 leg, ischaemic 199–203
liver function test (LFT) 253
haematuria/hydronephrosis 206–10 liver metastases 253–6
haemoglobin (Hb) 63, 247 lobar pneumonia 91–4
heart rate (HR) 99, 188 lobular carcinoma in situ (LCIS) 173
hernia low molecular weight heparin (LMWH) 189
congenital diaphragmatic 501–4 lung
hiatus 150–4 cancer 52–7
high-resolution computed tomography cavitating abscess of 62–6
(HRCT) 127 chronic lung disease of prematurity 528
hip fractures 331–4 diffusion capacity of the lung for carbon
history taking stations (for OSCE) 10 monoxide (test) 162
Hounsfield units (HU) 437 fibrosis 76–80
human chorionic gonadotropin (HcG) 241 metastases spread to 67–70
human epidermal growth factor 2 (HER2) 173 lymphadenopathy, bilateral hilar 159–63
human immunodeficiency virus (HIV) 65, 263 lytic lesion 339–42

561

K30031_Book.indb 561 9/6/17 1:40 PM


Index

magnetic resonance (MR) 4 nasogastric (NG) 25, 530


magnetic resonance cholangiopancreatography nasogastric tube placement and assessment
(MRCP) 5, 251 135–40
magnetic resonance imaging (MRI) 2, 4–5, 269 National Patient Safety Agency (NPSA) 139
advantages 4 necrotising enterocolitis (NEC) 529–34
disadvantages 4 neonatal intensive care unit (NICU) 501
indications 5 neuroblastoma 486–94
malignant spinal cord/cauda equina neurology cases 407–460
compression 450–4 brain tumour, meningioma 417–22
malrotation 480–5 cerebral abscess 455–60
maximum intensity projection (MIP) 291 extradural haematoma 433–9
mean cell volume (MCV) 159, 249 intracerebral haematoma 413–16
meconium aspiration syndrome (MAS) 505–9 malignant spinal cord/cauda equina
meconium ileus 474–9 compression 450–4
medial lateral oblique view (MLO) 170 multiple sclerosis 427–32
meningioma 417–22 sagittal sinus thrombosis 446–9
mesothelioma 71–5 stroke 440–5
metacarpophalangeal (MCP) 345 subarachnoid haemorrhage 423–6
metaiodobenzylguanidine (MIBG) 491 subdural haematoma 408–12
metastatic breast cancer 382–7 neuropathic (Charcot) joint 348–52
metatarsophalangeal (MTP) 343 New York Heart Association (NYHA) 193
middle cerebral artery (MCA) 442, 443 nonaccidental injury (NAI) 370, 520
mid-stream urine (MSU) 12, 161 non-Hodgkin lymphoma (NHL) 308, 310
multidisciplinary team (MDT) 30, 173 nonsteroidal anti-inflammatory drug
multinodular thyroid/superior mediastinal (NSAID) 346
mass 164–8 nuclear medicine 5–6
multiple myeloma 369–71 advantages 5
multiple sclerosis (MS) 427–32 disadvantages 6
musculoskeletal cases 319–405 imaging modalities 5
ankylosing spondylitis 400–5
cervical spine fracture 320–6 Objective Structured Clinical Examination
Colles fractures 388–93 (OSCE), hints and tips for finals 7–13
diabetic foot/osteomyelitis 335–8 communication skills 8–9
gout 343–7 communication stations 9–10
hip fractures 331–4 examination stations 10–12
metastatic breast cancer 382–7 history taking stations 10
multiple myeloma 369–71 imaging, blood results, and other test
neuropathic (Charcot) joint 348–52 results 12–13
osteoarthritis 327–30 logistics of preparation and the day
osteoporotic spinal fractures 372–6 itself 7–8
Paget’s disease 377–81 oesophageal carcinoma 145–9
pathological fracture, lytic lesion 339–42 oesophago-gastro-duodenoscopy (OGD) 6
prosthetic loosening 365–8 oesophagus, achalasia of 141–4
rheumatoid arthritis 353–7 open reduction and internal fixation
scaphoid fracture 394–9 (ORIF) 393
sclerotic bone lesions 382–7 orthopaedic bone/joint X-rays 1–2
shoulder dislocation 358–64 osteoarthritis (OA) 327–30
mycetoma 105–8 osteomyelitis 335–8
Mycobacterium avium complex (MAC) 88 osteoporotic spinal fractures 372–6
myocardial infarction (MI) 179 ovarian carcinoma 286–90

562

K30031_Book.indb 562 9/6/17 1:40 PM


Index

overview of imaging modalities 1–6 porcelain gallbladder 227–9


computed tomography 3–4 positron emission tomography (PET) 3
fluoroscopy techniques 6 posteroanterior (view) (PA) 18
magnetic resonance imaging 4–5 pregnancy, ectopic 240–4
nuclear medicine 5–6 primary lung carcinoma 52–7
plain films 1–2 projection, personal demographics, previous
ultrasound 2–3 CXR comparison (PPP) 17–9
prosthetic loosening 365–8
paediatric cases 461–553 proximal interphalangeal (PIP) 353
brain tumours 462–7 pulmonary cavities 62–6
congenital diaphragmatic hernia 501–4 pulmonary embolism (PE) 15, 185–9
cystic fibrosis/meconium ileus 474–9 pulmonary fibrosis 76–80
Ewing’s sarcoma 540–4 pulmonary metastases 67–70
fractures 545–53
intussusception 495–500 radiation doses (main imaging modalities) 17
malrotation 480–5 radiation exposure
meconium aspiration syndrome 505–9 AXR 36
necrotising enterocolitis 529–34 CXR 15–16
neuroblastoma 486–94 recombinant tissue plasminogen activator
nonaccidental injury 516–22 (r-TPA) 444
Perthes’ disease 510–15 renal carcinoma 67–70
pneumonia 468–73 renal cell carcinoma (RCC) 257–60
respiratory distress syndrome 523–8 renal colic 230–5
slipped capital femoral epiphysis 535–9 respiration rate (RR) 99, 190
Paget’s disease 377–81 respiratory distress syndrome (RDS) 523–8
Pancoast (superior sulcus) tumour 58–61 retroperitoneal mass 307–10
pancreatic cancer 273–5 rheumatoid arthritis (RhA) 353–7
pancreatitis rheumatoid factor (RhF) 355
acute 281–5 right atrium (RA) 25
chronic 249–52 right heart border (RHB) 24, 25
parathyroid hormone (PTH) 375 right iliac fossa (RIF) 303
pathological fracture 339–42 right lower quadrant (RLQ) 298
peak expiratory flow rate (PEFR) 117 right upper lobe collapse 119–23
pelvis cases see abdomen and pelvis cases right upper quadrant (RUQ) 25, 228
perforated viscus (gastric or duodenal ulcer) 95–9 right ventricle (RV) 24
peripheral artery occlusive disease (PAOD) 201
per rectum (PR) 101 sagittal sinus thrombosis 446–9
persistent pulmonary hypertension of the Salter–Harris (SH) 546
newborn (PPHN) 508 sarcoidosis 159–63
Perthes’ disease 510–15 saturation pressure of oxygen (SPO2) 117
plain films 1–2 scaphoid fracture 394–9
pleural effusion 109–14, 112 sclerotic bone lesions 382–7
pleural plaques 71–5 senior house officer 501
pneumatosis coli 261–4 short of breath on exertion (SOBOE) 190
pneumonia, paediatric 468–73 short tau inversion recovery (STIR) 404, 453
pneumonic consolidation 91–4 shoulder dislocation 358–64
pneumoperitoneum 211–14 sickle cell disease 245–8
pneumothorax 81–5 sigmoid volvulus 299–302
polycystic kidney disease (PKD) 424 slipped capital femoral epiphysis (SCFE) 535–9
polymerase chain reaction (PCR) 90 slipped upper femoral epiphysis (SUFE) 537

563

K30031_Book.indb 563 9/6/17 1:40 PM


Index

small bowel obstruction (SBO) 219–22 thoracic cases (continued)


spinal cord, malignant 450–4 right upper lobe collapse 119–23
splenic injury 311–14 sarcoidosis, bilateral hilar lymphadenopathy
spontaneous primary pneumothorax 81–5 159–63
stroke 440–5 subphrenic abscess 100–4
subarachnoid haemorrhage 423–6 thoracoplasty 155–8
subdural haematoma 408–12 tuberculosis 86–90
subphrenic abscess 100–104 three-dimensional (3D) 3
syndrome of inappropriate antidiuretic hormone thyroid
(secretion) (SIADH) 55, 123 function tests (TFTs) 12, 167
systemic lupus erythematosus 308 multinodular 164–8
total hip arthroplasty (THA) 366
thoracic cases 51–173 total hip replacement (THR) 365
achalasia of the oesophagus 141–4 transient ischaemic attack (TIA) 442
breast cancer 169–73 triangular fibrocartilage complex (TFCC) 345
bronchiectasis 130–4 tuberculosis (TB) 16, 86–90, 308
cavitating abscess of the right lung 62–6 tumour necrosis factor (TNF) 357
chronic obstructive pulmonary tumour nodes metastases (TNM) 56
disease 124–9 two-dimensional (2D) 2
hiatus hernia 150–4
left lower lobe collapse 115–18 ultrasound (US) 2–3
lobar pneumonia 91–4 umbilical arterial catheter (UAC) 507, 509
lung fibrosis 76–80 umbilical venous catheter (UVC) 507, 509
mesothelioma 71–5 upper gastrointestinal (UGI) 481
multinodular thyroid/superior mediastinal urea and electrolytes (U&Es) 227
mass 164–8 urinary tract carcinoma 67–70
mycetoma 105–8 uterine fibroids 276–80
nasogastric tube placement and assessment
135–40 venous blood gas (VBG) 12, 99
oesophageal carcinoma 145–9 ventilation/perfusion scan (V/Q) 189
Pancoast (superior sulcus) tumour 58–61 vertebral compression fracture (VCF) 372–6
perforated viscus (gastric or duodenal vesicoureteric junction (VUJ) 232
ulcer) 95–9
pleural effusion/opacified hemithorax 109–14 white blood cell (WBC) 101, 181
pneumothorax 81–5 white cell count (WCC) 63, 291, 457
primary lung carcinoma 52–7 World Health Organisation (WHO) 376
renal (or urinary tract) carcinoma with
pulmonary ‘cannonball’ metastases 67–70 Ziehl–Neelsen (ZN) staining 89

564

K30031_Book.indb 564 9/6/17 1:40 PM

You might also like